Simulacro Enarm 3

Descargar como pdf o txt
Descargar como pdf o txt
Está en la página 1de 216

!

1. TODAS LAS SIGUIENTES IMÁGENES APARECIDAS EN UNA MASTOGRAFÍA


REALIZADA DENTRO DE UN PROGRAMA DE SCREENING LE HARÍAN SOSPECHAR
MALIGNIDAD, EXCEPTO:

1. 1. EDEMA ALREDEDOR DE LA AREOLA MAMARIA.


2. 2. PRESENCIA DE 5 MICROCALCIFICACIONES DISEMINADAS ASIMÉTRICAS.
3. 3. RETRACCIÓN DE LA PIEL EN LA MAMA.
4. 4. PÉRDIDA DE LA ESTRUCTURA DEL TEJIDO MAMARIO.
Gráfico de respuestas
Comentario

Concepto muy importante que debe dominar. Se consideran signos mamográficos de posible
malignidad los siguientes:

Microcalcificaciones agrupadas anárquicamente en número igual o superior a cinco, no


diseminadas, lineales o ramificadas, de distribución segmentaria y de tamaño simétrico.

Nódulo denso, espiculado, de bordes imprecisos, con retracción de la piel o con edema cutáneo.

Pérdida de la arquitectura y desestructuración del tejido mamario, y densidades focales


asimétricas.(R2)
!

2. Masculino de 25 años de edad, adicto a drogas por vía parenteral, tiene edemas en
miembros inferiores. En la ecografía renal hay aumento de la ecogenicidad cortical en
ambos riñones. La nefropatía que seguramente padece este enfermo, ¿a qué NO es
secundaria?:

1. 1. Lupus eritematoso sistémico.


2. 2. Reflujo vesicoureteral.
3. 3. Nefropatía tubulointersticial.
4. 4. Nefropatía de cambios mínimos.
Gráfico de respuestas
Comentario

Caso clínico que se puede resumir en ADVP + EDEMAS, que deben hacernos sospechar GN focal
y segmentaria (GNFS). Esta nefropatía, conocida también con el nombre de “hialinosis
segmentaria y focal”, se caracteriza por el hallazgo de áreas de esclerosis glomerular con
depósitos de IgM y C3 en pacientes portadores, en general, de un síndrome nefrótico
corticorresistente.

Debemos conocer las posibles causas de esta enfermedad. En algunos casos, la GNFS aparece
en individuos diagnosticados previamente de nefropatía de cambios mínimos (respuesta 4).
También se han descrito lesiones secundarias en los adictos a la heroína, la nefropatía del reflujo
vesicoureteral (respuesta 2), las nefropatías túbulointersticiales (respuesta 3), la infección por HIV,
la uropatía obstructiva, linfomas, radiaciones ionizantes, obesidad, el rechazo crónico del injerto
renal, la agenesia renal unilateral y la nefrectomía parcial. En estos últimos casos de reducción de
la masa nefronal, las lesiones en los glomérulos residuales sugieren relación con fenómenos de

!
!
!
!
hiperfiltración. Sin embargo, el LES no suele producir GNFS, ya que en su inmunopatogenia
intervienen inmunocomplejos, por lo que causan otro tipo de daño renal.(R1)

3. Respecto al transporte de sustancias transplacentario, ¿cuál de los siguientes


mecanismos NO es el verdadero?

El paso de O2 al feto está facilitado por la menor concentración de hemoglobina en los


1. 1.
eritrocitos fetales.
2. 2. Mayor avidez de hemoglobina fetal por el oxígeno que la materna.
3. 3. La glucosa es el principal sustrato energético del feto para el corazón y el cerebro.
4. 4. El agua pasa al feto por gradiente osmótico.
Gráfico de respuestas
Comentario

Pregunta sobre fisiología placentaria nada fácil. La clave está en que la concentración de
hemoglobina en los eritrocitos fetales (17 g/dL) es mayor que en los eritrocitos maternos (11 g/dL).
Por otra parte, el paso de oxígeno también es facilitado por la mayor avidez de la hemoglobina fetal
al oxígeno que la hemoblobina materna. Por ello, la opción incorrecta es la 1. El principal sustrato
energético del feto es la glucosa, por eso pasa por difusión facilitada (opción 3).(R1)

4. Mujer de 68 años que comienza con aumento del diámetro abdominal desde hace 5
meses, que se acompaña de dolor en los últimos 15 días. Tras realizar las exploraciones
pertinentes, se la diagnostica de cáncer de ovario de tipo cistoadenocarcinoma seroso,
en estadio IIIb. Se le realiza tratamiento quirúrgico, sin dejar ningún resto tumoral. ¿Cómo
completaría el tratamiento?:

1. 1. Radioterapia abdominal.
2. 2. Poliquimioterapia adyuvante.
3. 3. Radioterapia más quimioterapia con cisplatino.
4. 4. Cirugía de rescate si recidiva.
Gráfico de respuestas
Comentario

El tratamiento del cáncer de ovario es fundamentalmente quirúrgico, independientemente del tipo


histológico. La cirugía ha de ser todo lo completa que la extensión tumoral nos permita. Recuerde
la excepción del tratamiento quirúrgico en caso de estadio IA, G1 en pacientes jóvenes con deseos
de descendencia. Generalmente añadimos quimioterapia, salvo en casos muy favorables (IA, IB
con buena o moderada diferenciación); raramente usamos radioterapia como primera línea, como
sugieren la respuestas 1 y 4. La cirugía de rescate se practicaba si tras la primera cirugía no
conseguíamos extirpar por completo el tumor (no, como dice en la respuesta 4, tras recidiva).
Actualmente, en desuso.(R2)

!
!
!
!

5. Recién nacido a término CIR tipo I, presenta


microcefalia llamativa, por lo que se realiza ultrasonido transfontanelar (imagen).
Antecedentes maternos sin interés, la madre tiene otro hijo de 7 años. Periodo neonatal
inmediato sin incidencias salvo que no pasa los potenciales evocados auditivos del oído
derecho. ¿Cuál es su primera impresión?

1. 1. Enfermedad metabólica.
2. 2. Infección congénita por toxoplasma.
3. 3. Infección congénita por CMV.
4. 4. Infección congénita por rubéola.
Gráfico de respuestas
Comentario

Patología preguntada con mucha frecuencia en el ENARM.

Una pregunta de dificultad media alta, porque dependemos bastante de la imagen que nos
aportan. Observe que el niño tiene microcefalia, lo que podría ser compatible con varias
infecciones connatales (salvo con la toxoplasmosis, claro está). Sin embargo, alrededor de los
ventrículos cerebrales vemos áreas hiperdensas, que corresponen a calcificaciones. Y, como ya
sabe, las calficicaciones periventriculares son típicas de la infección congénita por CMV.(R3)

6. ¿Qué otras alteraciones esperaría encontrar?

1. 1. Petequias, ictericia, hepatoesplenomegalia, púrpura, convulsiones.


2. 2. Cataratas congénitas.
3. 3. Hiperamoniemia y acidosis láctica.
4. 4. Deterioro progresivo y muerte.
Gráfico de respuestas
Comentario

Para completar el cuadro “microcefalia + calcificaciones periventriculares”, tendríamos que confiar


en la opción 1. Recuerde que, en cierto modo, la toxoplasmosis es opuesta al CMV:

!
!
!
!
No olvide este concepto para el ENARM.

- CMV: MICROcefalia y calcificaciones PERIVENTRICULARES.

- Toxoplasma: HIDROcefalia y calcificaciones DIFUSAS.(R1)

7. What is the most important examination for the diagnosis of a breast tumor in a 45-
year-old woman with a family history significant for breast cancer?

1. 1. Axial tomography.
2. 2. Ultrasonography.
3. 3. Mammography.
4. 4. Breast self-examination.
Gráfico de respuestas
Comentario

Pregunta que no puede fallar. El screening para la detección de cáncer de mama es con la
mastografía. Respuesta 3 correcta.(R3)

8. Hombre de 3 años de edad que, en la revisión del niño sano, presenta llamativa
dificultad para la retracción del prepucio sin poder visualizar glande ni meato uretral. Los
padres refieren que había sido diagnosticado hace algunos meses, por otro pediatra de
adherencias prepuciales que han sido tratadas con retracciones forzadas diarias. Desde
hace algunos días, el chorro miccional es más débil de lo habitual. ¿Cuál cree que es la
actitud más CORRECTA en este paciente?:

1. 1. Derivar a cirugía pediátrica para circuncisión o plastia de prepucio.


Aplicar pomada de corticoide dos veces al día durante un mes y volver a valorar la
2. 2.
retracción, repitiendo un nuevo ciclo de corticoide tópico si la respuesta es incompleta.
Se trata de un trastorno fisiológico de la infancia, por lo que la actitud correcta es esperar
3. 3.
su resolución espontánea.
Derivar urgentemente a cirugía ya que el prepucio se encuentra estrangulado y necesita
4. 4.
corrección quirúrgica urgente.
Gráfico de respuestas
Comentario

En el momento actual el paciente presenta una fimosis puntiforme: el prepucio es estrecho y cubre
totalmente el glande y el meato uretral, hasta el punto de dificultarle la micción.

Probablemente esta fimosis sea cicatricial, ya que en un primer momento presentaba solamente
adherencias, en esta entidad el glande se ve parcialmente descubierto, con zonas de adhesión al
prepucio. En estos casos no se debe realizar nunca un despegamiento forzado de las adherencias
ya que es un proceso doloroso que irrita la piel y puede crear cicatrices que condicionen un
estrechamiento del prepucio y lo conviertan en fimosis.

El tratamiento de la fimosis puede ser médico (con 1 o 2 ciclos de pomada de corticoide) o


quirúrgico (con circuncisión o plastia), y se suele realizar una vez retirado el pañal, ya que una
herida o el corticoide sobre una zona humedecida y ocluida constantemente por el pañal, puede
ocasionar complicaciones.

!
!
!
!
Las excepciones para tratar antes de esta edad son: la fimosis puntiforme, la parafimosis de
repetición o las infecciones de repetición. En estos casos y en los que no responden al corticoide
tópico, se debe realizar cirugía.

La opción 3 es falsa porque aunque es cierto que la fimosis es fisiológica, en este caso no
podemos esperar su resolución espontánea.

La 4 es incorrecta porque lo que se describe en el caso clínico es un episodio de parafimosis.(R1)

9. Mujer de 60 años, menopaúsica desde los 51, que acude a consulta por presentar
aumento del diámetro abdominal y sensación de pesadez pélvica desde hace unos
meses. No presenta antecedentes familiares ni personales de interés. En la exploración
se detecta una masa pélvica fija a estructuras profundas, dura e indolora; hay líquido
peritoneal libre. Los exámenes de laboratorio generales son normales. En el USG se
encuentra una masa dependiente del ovario derecho que ocupa pelvis parcialmente.
Según su sospecha diagnóstica ¿cuál de las siguientes opciones NO incluiría en su
tratamiento?:

1. 1. Linfadenectomía pélvica, paraórtica e inguinal.


2. 2. Histerectomía total.
3. 3. Toma de líquido ascítico para citología.
4. 4. Extirpación del epiplon.
Gráfico de respuestas
Comentario
Esta pregunta sobre la cirugía del cáncer de ovario es difícil y no tiene mucha relevancia pero no
esta demás saber que la cirugía de elección consiste en: histerectomía total con anexectomía y
salpinguectomía biltateral, omentectomía (extirpación del epiplón), apendicectomía (especialmente
en tumores mucinosos), biopsias múltiples peritoneales, lavado aspiración del liquido peritoneal y
linfadenectomía paraortica (primeros ganglios de drenaje) iliaca y pélvica pero no inguinal. La
linfadenectomía inguinal en ginecología se realiza en el cáncer de vulva.(R1)

10. Mujer de 19 años de edad que hace 6 meses no presenta menstruación, se encuentra
preocupada por aumento de peso, se pone a dieta y baja 10 kg en 8 meses. Actualmente
pesa 41kg (mide 1.65). Test de gestación negativo. La causa más probable es:

1. 1. Disgenesia gonadal
2. 2. Hipogonadismo hipogonadotrofico
3. 3. Sind de ovario poliquístico
4. 4. Himen imperforado
Gráfico de respuestas
Comentario

El caso parece corresponder a una mujer con anorexia nerviosa: pesa demasiado poco para su
talla y aún así decide ponerse a dieta. En estas mujeres, se produce una situación de
hipogonadismo hipogonadotropo, porque el hipotálamos libera menos GnRH. El resto de
respuestas no encajan con este cuadro clínico.(R2)

!
!
!
!
11. Mujer de edad media con historia de asma infantil que, tras una infección respiratoria
de vías altas, es remitida a Urgencias por crisis de broncoespasmo severa. Se realiza una
radiografía de tórax que detecta infiltrados pulmonares bilaterales y exámenes de
laboratorio que demuestra una proteinuria y anemia. La presencia de una intensa
eosinofilia en sangre periférica, junto con el carácter fugaz de los infiltrados en control
radiológico posterior evolutivo, nos orienta a la siguiente entidad diagnóstica:

1. 1. Angeítis alérgica o granulomatosa de Churg-Strauss.


2. 2. Síndrome de Löeffler.
3. 3. Linfangitis carcinomatosa.
4. 4. Granulomatosis de Wegener.
Gráfico de respuestas
Comentario

La eosinofilia podría hacernos pensar en las opciones 1, 2 y 3. Clínicamente, encajaría en una


enfermedad de Churg-Strauss, que es la única que explicaría la anemia y la proteinuria, así como
los infiltrados pulmonares evanescentes.(R1)

12. En relación al tumor de Wilms, ¿cuál de las siguientes afirmaciones NO es cierta?:

1. 1. Suele descubrirse por un aumento del volumen abdominal.


2. 2. En su diagnóstico puede ser útil la urografía intravenosa.
3. 3. Suele sobrepasar la línea media abdominal.
4. 4. Ocasiona metástasis precozmente.
Gráfico de respuestas
Comentario

El tumor de Wilms es el tumor abdominal más frecuente en los niños y, al originarse a partir de las
células epiteliales de los túbulos renales, es también el tumor renal más frecuente.

La mayoría de los casos se manifiestan inicialmente como una masa abdominal que,
característicamente y a diferencia del neuroblastoma, no cruza la línea media (aunque existen
formas familiares que pueden ser bilaterales).

Se descubren metástasis a distancia (fundamentalmente pulmonares) con cierta frecuencia en el


momento del diagnóstico, pero dichas metástasis pueden responder al tratamiento oncológico.

Al ser un tumor renal, deforma los vasos sanguíneos renales y las vías urinarias, por lo que la
arteriografía renal y la urografía pueden ayudar en el diagnóstico diferencial con otros tumores
abdominales.

El diagnóstico definitivo lo da el estudio anatomopatológico. No debe hacerse biopsia, sino


resección del tumor completo (con o sin nefrectomía total asociada), ya que la rotura de la cápsula
tumoral aumenta el riesgo de diseminación.(R3)

13. Con respecto a los reflejos del RN, ¿cuál de las siguientes afirmaciones NO es
correcta?:

1. 1. El reflejo del paracaídas aparece hacia los 9 meses y desaparece al año.

!
!
!
!
La no desaparición de estos reflejos se observa en niños con retraso del desarrollo o con
2. 2.
lesiones motoras centrales.
3. 3. El reflejo de prensión palmar desaparece antes que el de prensión plantar.
La ausencia de respuestas reflejas pone de manifiesto una depresión de las funciones
4. 4.
motoras centrales o periféricas.
Gráfico de respuestas
Comentario

Esta pregunta acerca de los reflejos del recién nacido es importante y es útil para recordarlos. La
presencia de una serie de reflejos primitivos al nacimiento indica integridad funcional del tronco del
encéfalo. Como grupo, son simétrico y desaparecen a los 4-6 meses lo que señala maduración
normal de las influencias cerebrales inhibitorias descendentes.

Los reflejos Landau y paracaídas se manifiestan después del periodo de recién nacido e indican
maduración correcta de las estructuras encefálicas.

El primero desaparece hacia los 15-24 meses y el segundo no lo hace nunca.

Los reflejos de prensión y hociqueo son inhibidos con la maduración del lóbulo frontal (recuerde
que volver a reaparecer en caso de lesiones en este lóbulo).

El de prensión plantar suele desaparecer hacia el 9º- 10º mes, después de que lo haga el palmar
(4º- 6º). Recuerde que el desarrollo psicomotor es céfalo-caudal.(R1)

14. Se ha observado el desarrollo de necrosis cutánea durante el uso de uno de los


siguientes fármacos:

1. 1. Furosemida.
2. 2. Acido nalidíxico.
3. 3. Sales de oro.
4. 4. Warfarina.
Gráfico de respuestas
Comentario

La necrosis cutánea por warfarina ocurre entre 3-5 días después del inicio del tratamiento. Existe
predisposición en las mujeres obesas, postmenopaúsicas. Las lesiones comienzan como placas
dolorosas, eritematosas, que se hacen necróticas y tienden a localizarse en mamas, abdomen y
glúteos. Se asocia a la deficiencia hereditaria o adquirida de proteina C y menos frecuentemente
de la proteína S. Histológicamente se observa trombosis no inflamatoria con fibrina en los vasos
subcutáneos. El tratamiento consiste en suspender la warfarina, administrar vitamina K y proteína
C purificada. Sin tratamiento esta reacción puede producir la muerte.(R4)

15. El protocolo del cáncer de ovario incluye todos, EXCEPTO:

1. 1. Apendicectomía.
2. 2. Omentectomía.
3. 3. Linfadenectomía paraaórtica.
4. 4. Linfadenectomía inguinal.
Gráfico de respuestas
Comentario

!
!
!
!
El tratamiento quirúrgico en el cáncer de ovario tiene una doble utilidad: diagnóstica al permitir
estadificar adecuadamente a las pacientes, y terapéutica, extirpando la mayor cantidad de masa
tumoral posible. La cirugía consiste en:

Lavado y aspiración de líquido peritoneal.

Histerectomía total con anexectomía bilateral.

Linfadenectomía pélvica y paraaórtica.

Omentectomía (extirpación del epiplón).

Exploración de la superficie peritoneal y biopsia de lesiones sospechosas, además de biopsiar el


peritoneo vesical, el fondo de saco de Douglas, los espacios parietocólicos y la cúpula
diafragmática.

Apendicectomía, especialmente en los tumores mucinosos.

Entre estos no se incluye la linfadenectomía inguinal. Respuesta 4.

16. Over a 3-month period, a 55-year-old woman with a history of diabetes mellitus for the
past 30 years develops progressive visual loss. Examination reveals: Visual acuity: Right
eye 4/20, Left eye 8/20. Incipient bilateral cataract. Intraocular pressure: 19/18 mm Hg.
Funduscopy reveals hard exudates at both posterior poles, more pronounced at the
macular area. Which of the following is the most appropriate next step in management?

1. 1. Optical coherence tomography.


2. 2. Visual campimetry.
3. 3. Color test.
4. 4. Corneal topography.
Gráfico de respuestas
Comentario
El edema macular es la causa más importante de pérdida visual en el paciente diabético. Se debe
a la alteración de la permeabilidad vascular va a conducir al acúmulo de líquido en la retina.
Clásicamente se ha tratado con láser focal (fotocoagulando de forma selectiva los puntos de fuga).
En la actualidad se ha difundido mucho el uso de los fármacos antiVEGF en el tratamiento de esta
patología. Sin embargo antes de llevar a cabo cualquier maniobra terapeútica, está indicado
confirmar el diagnóstico. En este sentido la TOC (tomografía de coherencia óptica) resulta de gran
utilidad. Se trata de una técnica de bajo coste y no invasiva que permite obtener cortes con gran
detalle histológico de las capas de la retina. Está técnica permite cuantificar el espesor retiniano. Si
se confirma que existe un aumento de espesor en el área macular, estará indicado un láser focal o
una inyección de fármaco antiVEGF.(R1)

17. De las siguientes, ¿en qué situación NO se encontraría un desdoblamiento invertido


o paradójico del segundo ruido?

1. 1. Ritmos extrasistólicos con foco inicial en el ventrículo derecho.


2. 2. Bloqueo completo de la rama derecha.
3. 3. Hipertensión arterial.
4. 4. Insuficiencia ventricular izquierda.

!
!
!
!
Gráfico de respuestas
Comentario

Pregunta de dificultad moderada sobre un tema bastante habitual: la semiología cardíaca.

El segundo ruido es producido por el cierre de las válvulas semilunares (aórtica y pulmonar, por
este orden). Se produce un pequeño desdoblamiento fisiológico del 2R durante la inspiración, ya
que en esta fase se produce una disminución de la cantidad de sangre que llega al ventrículo
izquierdo. El desdoblamiento del 2R se hace mayor cuando existe un retraso en la eyección del
VD, como en el bloqueo de rama derecha, en la estenosis pulmonar o en la CIA.

Cuando el cierre de la válvula aórtica es más tardío que el de la pulmonar, se produce un


desdoblamiento invertido del 2R. Se observa en el bloqueo de la rama izquierda, en presencia de
extrasístoles ventriculares, en la estenosis aórtica, o cuando el VD tiene un tiempo de eyección
corto, como ocurre en la HTP con fallo ventricular derecho. También podría verse en la HTA, ya
que el aumento de la postcarga hace que el ventrículo izquierdo se vacíe más lentamente.(R2)

18. Femenino de 72 años de edad, con 7 dias de dolor en el cuadrante superior derecho,
ictericia progresiva y temperatura de 39 ºC recibió tratamiento con cefalexina vía oraL,
sin mejoría. Ingresa a urgencias hipotensa, con compromiso del sensorio. Leucocitos:
19,000 x mm2, bilirrubinas, transaminasas y fosfatasa alcalina elevadas, concentración
de amilasa sérica normal. ¿Cuál es su diagnostico?

1. 1. Colelitiasis.
2. 2. Enfermedad de Wilson.
3. 3. Colecistitis aguda alitiásica.
4. 4. Colangitis supurativa aguda.
Gráfico de respuestas
Comentario

Cuadro clínico típico de libro compatible con colangitis.

El paciente presenta la triada de Charcot: fiebre, dolor abdominal e ictericia.

Pentada de Reynolds: hipotensión y cambios en el estado mental + triada de Charcot.

Respuesta 4 correcta.

19. Las siguientes afirmaciones sobre hematomas de la vaina del recto anterior mayor
del abdomen son ciertas, EXCEPTO:

1. 1. Son más frecuentes en hombres que en mujeres.


2. 2. Son raros en niños.
3. 3. Se relacionan con el empleo de anticoagulantes.
4. 4. Se acompañan de enfermedades de la colágena.
Gráfico de respuestas
Comentario

No nos preocupemos mucho por esta pregunta, sólo debemos saber que puede ser útil en caso de
hacer diagnóstico diferencial con apendicitis aguda. El hematoma de la vaina de los rectos puede

!
!
!
!
ser espontáneo o secundario a un esfuerzo. Es más frecuente en pacientes anticoagulados y
mujeres, especialmente en embarazadas y puérperas. Debemos recordar su existencia, puesto
que la clínica puede ser idéntica a una apendicitis en el lado derecho o a una diverticulitis en el
lado izquierdo. El diagnóstico es por ultrasonido y el tratamiento requiere drenaje percutáneo.(R1)

20. En un lactante de 9 meses con lactancia mixta y percentiles adecuados de peso y


talla, sería recomendable indicarle:

1. 1. Suplementos de calcio y hierro.


2. 2. Suplementos de zinc y calcio.
3. 3. Suplementos de hierro y vitaminas.
4. 4. Suplementos de ácidos grasos.
Gráfico de respuestas
Comentario

A partir de los 6 meses el bébé aumenta sus requerimientos de minerales y vitaminas, por lo que
no pueden ser cubiertos exclusivamente por la leche materna, pero no es necesario darlos en
suplementos sino que deben ser proporcionados por alimentos complementarios.

Para asegurar el aporte en calcio tan necesario para la formación de los huesos entre otras cosas,
la dieta del bebé debe incluir lácteos como yogurt y otros productos lácteos, ricos en calcio.

Para garantizar el aporte de hierro deben incluirse en su dieta carnes, ricas en hierro y en cinc así
como los cereales enriquecidos con hierro y frutas con alto contenido en vitamina C, ya que esta
vitamina aumenta la biodisponibilidad de hierro a partir de la dieta y tiene asimismo importancia
como antioxidante y en la síntesis de neurotransmisores.

Únicamente cuando el contenido en flúor del agua es inferior a 0.3 mg/L se recomienda administrar
suplementos de flúor (0.25 mg/día) a partir de los 6 meses de edad.(R3)

21. ¿De qué enfermedad es característico el signo de la impresión digital?:

1. 1. Enfermedad vascular mesentérica.


2. 2. Colitis por irradiación.
3. 3. Enteriris por Yersinia enterocolytica.
4. 4. Enfermedad de Crohn.
Gráfico de respuestas
Comentario
La imagen radiológica de huella digital es la expresión de hematomas submucosos y de edema de
la pared intestinal como consecuencia de procesos vasculares mesentéricos como la isquemia
intestinal y la colitis isquémica.(R1)

22. Con respecto a los quistes y pseudoquistes pancreáticos, señale cuál de las
siguientes afirmaciones es FALSA:

1. 1. Los pseudoquistes no tienen pared propia.


2. 2. Los pseudoquistes se producen en un páncreas no funcionante.
3. 3. Los quistes comunican con el conducto de Wirsung.
Los pseudoquistes son más frecuentes en la pancreatitis crónica que en la pancreatitis
4. 4.
aguda.

!
!
!
!
Gráfico de respuestas
Comentario

Los pseudoquistes son formaciones de contenido líquido no recubiertas del epitelio de los
conductos pancreáticos, que tampoco tienen una cápsula propiamente dicha y que se producen
por el acúmulo de secreciones pancreáticas tras agresiones diversas en el seno del tejido
pancreático, en su mayoría por pancreatitis. Su contenido es muy rico en enzimas pancreáticos.
Así pues, no es habitual la formación de estas colecciones en el seno de un páncreas no
funcionante, por lo que esa es la opción claramente falsa. El resto de las opciones expuestas son
verdaderas.(R2)

23. Hombre de 68 años de edad con diabetes de larga evolución y retinopatía diabética
remitido para estudio de insuficiencia renal con Cr 1.5 mg/dl y proteinuria. Respecto a la
patología renal que probablemente tenga este paciente señale la afirmación FALSA:

1. 1. El control de la proteinuria es lo más importante para el pronóstico renal.


2. 2. El control glucémico no influye en la evolución renal.
3. 3. Aparece en un 30-40% de los pacientes diabéticos.
4. 4. El complemento es normal.
Gráfico de respuestas
Comentario
Lo más probable es que tenga una nefropatía diabética. Para la evolución de la nefropatía
diabética es importante el control de la proteinuria pero también de la glucemia.(R2)

24. Masculino de 68 años de edad, con antecedentes de hipertensión arterial e infarto


agudo de miocardio, en tratamiento con mononitrato de isosorbide, consulta por cuadro
de disfunción eréctil de dos años de evolución. ¿Cuál de las siguientes actitudes
terapéuticas estaría CONTRAINDICADA?

1. 1. Colocación intrauretral de PGE 1.


2. 2. Citrato de sildenafilo vía oral.
3. 3. Utilización de dispositivos de vacío.
4. 4. Colocación de una prótesis peneana.
Gráfico de respuestas
Comentario

El citrato de sildenafilo es considerado, actualmente, como el tratamiento farmacológico de


elección en la disfunción eréctil. Se trata de un inhibidor de la fosfodiesterasa tipo 5. Induce la
relajación del músculo liso del cuerpo cavernoso liberando, óxido nítrico que es el principal
neurotransmisor de la erección. Sus contraindicaciones absolutas son la administración
concomitante de nitratos o fármacos donadores de óxido nítrico (dinitrato de isorbide) y pacientes
en los que esté desaconsejada la actividad sexual como aquellos con angina inestable,
insuficiencia cardíaca o infarto reciente.(R2)

25. Sobre el diagnóstico de un niño VIH+, es FALSO que:

1. 1. Una IgG+ en un lactante de 10 meses indica infección activa por el VIH.


2. 2. La mayoría de los niños infectados presenta hipergammaglobulinemia policlonal precoz.
3. 3. La linfopenia e inversión del cociente CD4/CD8 son menos llamativos que en el adulto.

!
!
!
!
Los resultados positivos en la PCR han mostrado una correlación positiva con el
4. 4.
aislamiento del virus en cultivo.
Gráfico de respuestas
Comentario

Los anticuerpos contra el VIH sólo se pueden valorar a partir de los 18 meses. Antes puede que
sean los de la madre que han pasado la barrera placentaria.

El resto de respuestas son correctas.(R1)

26. Un paciente de 26 años acude a Urgencias tras haber sufrido un accidente de


motocicleta. Presenta dolor e inestabilidad de la articulación MCF del primer dedo de la
mano derecha. ¿Cuál de las siguientes es FALSA?:

1. 1. Probablemente se trate de una artritis traumática.


La inestabilidad orienta a que el proceso sea una rotura del ligamento colateral cubital del
2. 2.
primer dedo.
El tratamiento incluye la reparación quirúrgica del ligamento en los casos con rotura del
3. 3.
ligamento.
La interposición del adductor del primer dedo que impide la curación del ligamento se
4. 4.
conoce como lesión de Stener.
Gráfico de respuestas
Comentario

Lo que nos están describiendo (lesión del ligamento colateral cubital) es lo que también se
denomina “lesión de Stener” o “pulgar del guardabosques”, típico de accidentes donde el sujeto,
institivamente, se agarra a algún objeto cilíndrico (como el manillar de la motocicleta). Cuando los
cabos del ligamento roto se separan lo suficiente, puede quedar interpuesta la aponeurosis del
músculo aproximador corto del pulgar, por lo que precisa tratamiento quirúrgico cuando la lesión es
completa, ya que de lo contrario cicatriza muy difícilmente, o no lo hace en absoluto.(R1)

27. Una paciente de 23 años, sexualmente activa, presenta síntomas compatibles con una
infección de urinaria, la cuarta en el último trimestre. Las tres anteriores se han tratado
adecuadamente con quinolonas, aunque en ningún momento se han realizado
urocultivos. Señale la FALSA:

1. 1. Es posible que se trate de una infección por Staphylococcus saprophyticus.


2. 2. Sería conveniente la obtención de muestras para urocultivo.
3. 3. Sería conveniente tratar a la pareja sexual para eliminar el ciclo infeccioso.
4. 4. Sería conveniente recomendar una serie de medidas higiénico-dietéticas a la paciente.
Gráfico de respuestas
Comentario

Se trata de una IVU que perfectamente puede deberse a Staphylococcus saprophyticus, segundo
germen más frecuente en mujeres sexualmente activas. No tenemos cultivos previos, por lo que no
podemos diagnosticarla de IVU de repetición o recidivas, ya que desconocemos el germen causal,
pero se deberá estar atentos por si se tratara de este cuadro y precisara un estudio más completo.
Estas infecciones no son de transmisión sexual, por lo que no se deberá tratar a la pareja, aunque
sí se deben indicar medidas básicas como la micción antes y después de la relación sexual,
ingesta hídrica adecuada...(R3)

!
!
!
!
28. Uno de los siguientes datos orienta el diagnóstico diferencial entre el taponamiento
cardíaco y la pericarditis constrictiva a favor de la segunda. Señale cuál:

1. 1. Antecedente de pericarditis aguda.


2. 2. Disminución inspiratoria de la presión arterial sistólica de al menos 10 mmHg.
3. 3. Ausencia de disminución de la presión venosa yugular durante la inspiración.
4. 4. Igualación de las presiones diastólicas de ambos ventrículos en el cateterismo cardíaco.
Gráfico de respuestas
Comentario

La respuesta número 3 nos describe el signo de Kussmaul, más frecuente en la pericarditis


constrictiva. La respuesta número 2 describe el pulso paradójico, más frecuente en el
taponamiento (aunque recuerda que ambas pueden estar presentes en las dos patologías).(R3)

29. Señale lo CORRECTO con respecto a los divertículos esofágicos:

1. 1. La disfagia paradójica es característica de los divertículos epifrénicos.


2. 2. La clínica en el divertículo de Zenker está directamente relacionada con el tamaño de éste.
Los divertículos medioesofágicos suelen estar relacionados con procesos inflamatorios
3. 3.
mediastínicos.
4. 4. Los divertículos epifrénicos surgen por el triángulo de Killian.
Gráfico de respuestas
Comentario
La disfagia paradójica es la que se produce en algunas disfagias de tipo motor incipiente, que se
expresan únicamente con disfagia para alimentos líquidos. Los divertículos de Zenker aparecen en
hipofaringe posterior y presenta clínica de disfagia en relación con la asociación de trastonos
motores del cricofaríngeo y, por tanto, en forma independiente al tamaño de los mismos. Los
divertículos del tercio medio del esófago suelen estar relacionados con procesos inflamatorios o
adenopatías que por tracción inducen dichas alteraciones.(R4)

30. Le avisan para investigar un brote de diarrea acuosa y profusa en una guardería
infantil, los agentes etiológicos que usted deberá tener en cuenta son estos. EXCEPTO:

1. 1. Virus Norwalk.
2. 2. Giardia lamblia.
3. 3. Rotavirus.
4. 4. E. coli enterotoxigénica.
Gráfico de respuestas
Comentario

Pregunta lógica. La opción que debe elegir es la 2, ya que Giardia produce diarrea acuosa de
manera subaguda o crónica. Recuerdeque el virus Norwalk y el rotavirus puden causar brotes de
diarreas acuosas; así como la toxina producida por E. coli.(R2)

31. Which of the following is not an antiplatelet agent?

1. 1. Abciximab.
2. 2. Fondaparinux.
3. 3. Eptifibatide.
4. 4. Prasugrel.

!
!
!
!
Gráfico de respuestas
Comentario

Es importante conocer el mecanismo de acción de este tipo de fármacos. El abciximab, y


eptifibatide son 3 antiagregantes muy potentes que sólo se pueden administrar por vía intravenosa.
Pertenecen al grupo de inhibidores de la glucoproteína IIb/IIIa. Prasugrel es un antiagregante oral
nuevo más potente que el clopidogrel.

Sin embargo el Fondaparinux es un análogo sintético que imita la interacción heparina-


antitrombina, tratandose por lo tanto de un anticogaulante, no un antiagregante.(R2)

32. Varón de 33 años que es traído a urgencias tras sufrir un accidente. En la evaluación
inicial se objetivan: rotura esplénica, hematoma perirrenal izquierdo, neumotórax a
tensión, fractura del 3ª vértebra dorsal con lesión medular incompleta y fractura de fémur
izquierdo. La primera lesión a tratar es:

1. 1. La lesión medular.
2. 2. El neumotórax a tensión.
3. 3. El traumatismo renal.
4. 4. La fractura de fémur.
Gráfico de respuestas
Comentario

A: Airway (vía aérea).

B: Breathing (respiración).

C: Circulation (control del choque, de los puntos sangrantes activos…).

D: Disability (lesiones neurológicas).

E: Exposure (exposición). Consiste en la exposición completa del paciente, desvistiéndole y


dándole la vuelta, así como la prevención de la hipotermia.

En este caso, siguiendo este protocolo de actuación, la respuesta correcta sería la 2.

!
!
!
!

33. Mujer de 22 años con diarrea sanguinolenta


de 15 evacuaciones diarias, fiebre de 38.2 ºC y dolor abdominal generalizado. Exámenes
de laboratorio: Hb 9.3 g/dl, VCM 66 fl, 16,500 leucocitos, plaquetas 325,000/mm3, INR 0.9,
bilirrubina 1.2, AST 52, ALT 25, GGT 23, amilasa 46 UI/ml, glucemia 86, urea 77, creatinina
0.8, sodio 137, potasio 3.1. Coprocultivos y parásitos fueron negativos. Serología viral B
y C negativas. Proteinograma e inmunoglobulinas normales. Se realiza rectoscopía,
observándose la imagen adjunta. ¿Cuál de estas actitudes debería evitarse?:

1. 1. Toma de biopsias para estudio histológico.


2. 2. Realizar colonoscopía completa hasta ciego para evaluar la extensión de la lesión.
3. 3. Iniciar tratamiento corticoideo.
4. 4. Toma de muestras para descartar infección por citomegalovirus.
Gráfico de respuestas
Comentario

Teniendo en cuenta la edad de la paciente y el cuadro clínico (diarrea sanguinolenta, pérdida de


peso y negatividad en los estudios microbiológicos), debemos sospechar una enfermedad
inflamatoria intestinal. En la imagen adjunta, observamos hiperemia y sangrado en la mucosa
colónica, afectada de forma continua, por lo que debemos pensar en una posible colitis ulcerosa.
La respuesta correcta sería, por tanto, la 2. Recuerde que la colitis ulcerosa casi siempre afecta a
recto y sigma, extendiéndose más proximalmente en algunos casos. En este caso, no sería
necesario una colonoscopía completa, porque no va a cambiar nuestra actitud, y sin embargo
habría mayor riesgo de perforación que con una sencilla rectosigmoidoscopía.(R2)

34. Señale cuál de las siguientes características NO es típica de la entidad que sospecha:

1. 1. Afectación continua que afecta a todo el colon.


2. 2. Formación de pseudopólipos.
3. 3. Afectación histológica superficial de la pared intestinal.
4. 4. Displasia epitelial.
Gráfico de respuestas
Comentario

Tal como explicábamos en el comentario anterior, el diagnóstico de sospecha es una colitis


ulcerosa. La afectación de la mucosa es continua (a diferencia de la enfermedad de Crohn, donde

!
!
!
!
es parcheada), pero no puede decirse que afecte a TODO el colon (opción 1 falsa). Esto sólo
sucede en los casos más graves. Lo que sí podemos decir es que casi siempre afecta a recto-
sigma, extendiéndose hacia regiones más proximales en un porcentaje variable de casos. La
afectación de todo el colon solamente sucede en los pacientes más graves.(R1)

35. Neonato masculino de 24 días de vida que presenta en los últimos días vómitos
alimentarios proyectivos tras todas las tomas quedando posteriormente hambriento.
Señale la respuesta CORRECTA respecto a la patología de este paciente:

1. 1. La técnica diagnostica de elección sería la realización de una pHmetría.


2. 2. Existe tendencia a la hipopotasemia.
3. 3. El eje renina angiotensina aldosterona se encuentra suprimido.
4. 4. El tratamiento será médico.
Gráfico de respuestas
Comentario

Pregunta que nadie puede fallar en el ENARM, regalada.

Neonato MASCULINO primogénito de 3 semanas de vida con vómitos alimentarios es escopetazo


o proyectivos son las pistas míricas que podemos encontrar refiriéndose a la ESTENOSIS
HIPERTRÓFICA DEL PÍLORO.

Debemos conocer que en esta patología debido a la pérdida de cloro e hidrogeniones por los
vómitos se produce una alcalosis metabólica que a su vez va a condicionar el intercambio de
potasio por hidrogeniones en la célula (el potasio entra a cambio de que salgan hidrogeniones para
compensar la alcalosis). Debido a esta depleción de de volumen por los vómitos se va a activar el
sistema renina-angiotensina-aldosterona que va a producir una mayor pérdida de potasio en el
túbulo distal perpetuando la hipopotasemia. El tratamiento es quirúrgico mediante la piloromiotomía
extramucosa de Ramsted.

El diagnóstico se realiza por medio de USG y te pueden mencionar que se palpa una oliva pilórica
que es característica.(R2)

36. Respecto a la anemia en el período neonatal, señale cuál de las siguientes


afirmaciones es FALSA:

1. 1. La anemia fisiológica es más intensa y más precoz en los RNT que en los RNPT.
En los RNPT está indicada la administración de eritropoyetina recombinante, ya que
2. 2.
disminuye la necesidad de transfusiones durante los primeros meses de vida.
3. 3. La anemia “fisiológica” puede precisar transfusión en algunos casos.
4. 4. La beta-talasemia no se manifestará antes de los 4-6 meses de vida.
Gráfico de respuestas
Comentario

Los RNT, inmediatamente después de nacer, tienen unas cifras de Hb de 14-20 g/dl, y los RNPT,
1-2 g/dl menos. A partir de las 48 horas de vida, comienzan a descender estos valores. En los
RNT, hacia las 8-12 semanas de vida se alcanzan las cifras mínimas (9-11 g/dl); en los RNPT, las
cifras mínimas (7-9 g/dil) se alcanzan a las seis semanas. Respuesta 1 incorrecta.

!
!
!
!
37. ¿Qué factores no indican peor pronóstico en pacientes con glomerulonefritis
membranosa?:

1. 1. Grado de proteinuria mayor de 10 g/día y duración de la misma de más de 6 meses.


2. 2. HTA asociada.
3. 3. Sexo femenino.
4. 4. Función renal afectada en el momento de la presentación.
Gráfico de respuestas
Comentario

Pregunta de dificultad media-alta acerca del pronóstico de la GN membranosa. El pronóstico es


peor en aquellos adultos que pertenecen al sexo masculino (OPCION 3 FALSA), mantienen una
proteinuria intensa (más de 10 g/día) y presentan déficit de la función renal en el momento del
diagnóstico. La HTA y la edad avanzada son también datos que indican peor evolución. Entre las
causas que pueden provocar un deterioro rápido de la función renal se incluyen la trombosis de las
venas renales, el desarrollo de una GNRP sobreañadida o la aparición de una nefritis intersticial
por hipersensibilidad a fármacos.(R3)

38. About ECMO (Extracorporeal


Membrane Oxigenation) what is false?

ECMO is a respiratory assistance that provides properly tissue oxygenation when


1. 1.
irreversible heart failure or respiratory distress occurs.
2. 2. It may be used during a maximum 10 days time.
3. 3. Veno-venous cannulae only allows respiratory support.
Veno-arterial cannulae has a higher rate of complications, mostly related to the vascular
4. 4.
access.

!
!
!
!
Gráfico de respuestas
Comentario

La oxigenación por membrana extracorpórea es un tratamiento que utiliza una bomba para hacer
circular la sangre a través de una membrana artificial, que hace las veces de pulmón, para permitir
el intercambio gaseoso. Suele utilizarse en pacientes muy críticos e implica un circuito de
circulación extracorpórea.

La respuesta incorrecta es la 1, ya que no es obligado que exista un fallo cardiopulmonar


irreversible. Puede emplearse en situaciones que, con el tratamiento adecuado, serían
susceptibles de corregirse, ya sea quirúrgicamente (como en este caso) o incluso con tratamiento
médico (a veces se emplea la ECMO en neumonías de extrema gravedad).

En cualquier caso, no te preocupes si te faltaba seguridad al responder esta pregunta. Hace


referencia a aspectos muy puntuales sobre esta técnica que van más allá de lo que probablemente
te exijan en el examen. La clave estaba en identificar la opción 1 como falsa por el detalle que
hemos citado, más que conocer cada detalle sobre la ECMO.(R1)

39. ¿Cuál de los siguientes regímenes es mejor para el tratamiento preoperatorio de un


paciente con un feocromocitoma conocido?:

1. 1. Sólo propranolol.
2. 2. Propranolol seguido de fenoxibenzamina.
3. 3. Fenoxibenzamina seguida de propranolol.
4. 4. Sólo fentolamina.
Gráfico de respuestas
Comentario

Una de las partes más importantes del feocromocitoma es la que hace referencia al manejo
preoperatorio de estos pacientes. Una vez realizado el diagnóstico, el primer fármaco a administrar
es un alfabloqueante (fenoxibenzamina, prazosín, doxazosina). Tras la administración de estos
fármacos, y nunca antes que ellos, se asociarían los betabloqueantes.

La fentolamina es un fármaco alfabloqueante no selectivo, cuya administración sólo se puede


realizar vía parenteral, de ahí que sea de elección en las crisis hipertensivas en estos
pacientes.(R3)

40. ¿Cuál es la causa MENOS frecuente de eritema nodoso, entre las que se citan en la
lista?:

1. 1. Enfermedad de Behçet.
2. 2. Sarcoidosis.
3. 3. Fármacos.
4. 4. Infección estreptocócica.
Gráfico de respuestas
Comentario
Esta pregunta puede resultar bastante difícil, pero puede razonarse de la siguiente manera: es
cierto que la enfermedad de Behçet se asocia a eritema nudoso (de hecho, la lesiones cutáneas,
como el eritema nodoso o la foliculitis perforante, forman parte de los criterios diagnósticos de esta
enfermedad), pero es un cuadro bastante infrecuente (se da más en Japón y los países más

!
!
!
!
orientales de la cuenca mediterrántea), siendo entre las opciones mencionadas la causa menos
frecuente de eritema nudoso.(R1)

41. En el estudio de un cuadro de deterioro cognitivo en un anciano, ¿cuál de los


siguientes datos orienta hacia el diagnóstico de depresión?:

1. 1. Inicio insidioso.
2. 2. Mejoría con la deprivación de sueño.
3. 3. Interés por las actividades sociales.
4. 4. Atrofia en el TAC craneal.
Gráfico de respuestas
Comentario

La depresión tiene un inicio relativamente brusco (en cuestión de semanas), la demencia es


insidiosa (en cuestión de años). La deprivación de sueño ("agripnia") tiene accidón antidepresiva
pero produce deterioro cognitivo, así que mejora la depresión (y la pseudodemencia, por tanto),
pero agrava la demencia. En la depresión hay conciencia de la angustia, y las quejas maximizan y
exhiben la pérdida de confianza en uno mismo; en la demencia no hay conciencia de la merma
cognitiva, o esta se minimiza y oculta. El interés por las actividades sociales se pierde en ambas.
La atrofia en la TAC craneal es sugestiva de demencia.(R2)

42. En un un USG de control, a las 20 semanas de edad gestacional, en el que hallamos


una malformación cardíaca tipo canal atrioventricular común, CIR simétrico y
acortamiento de las extremidades, con genitales externos masculinos, ¿cuál es la
alteración cromosómica más probable?:

1. 1. Síndrome de Edwards.
2. 2. Síndrome de Klinefelter.
3. 3. Síndrome de Turner.
4. 4. Síndrome de Down.
Gráfico de respuestas
Comentario

Todas las opciones están relacionadas con la aparición de CIR simétrico o tipo I (como todas las
cromosomopatías). Los genitales externos masculinos nos decarta el síndrome de Turner, ya que
el Turner tiene genitales externos femeninos. La opción correcta es la 4, el síndrome de Down, ya
que en el caso clínico aparece el canal atrioventricular común (que es la cardiopatía más frecuente
en el síndrome de Down), y el acortamiento de extremidades (recuerda que en el Down es típico el
acortamiento de la longitud del fémur).(R4)

43. A previously healthy 5-year-old boy is brought to the pediatrician's office presenting
with a new onset rash two weeks after returning from camp. He has been feeling asthenia
and occasional headaches since then. His father says that a tick bit him and he had to
remove the insect with tweezers. The rash is 5 cm in diameter and centered around the
tick bite, with a bull's eye appearance. Topic steroids have not improved the lesion. What
is the best treatment option in this case?

1. 1. Oral ciprofloxacin.
2. 2. Oral terbinafine.
3. 3. Intravenous penicillin.
4. 4. Oral amoxicillin.

!
!
!
!
Gráfico de respuestas
Comentario
Oral amoxicillin. The sequence described in this case is typical of Lyme disease, caused by Borrelia
species. The first stage is erythema migrans, that appears at the site of the inoculation within a
week of the tick bite. Other early symptoms may include fever and general malaise. The treatment
of choice is oral amoxicillin or doxycycline. Doxycycline is contraindicated in children.(R4)

44. ¿Cuál de las siguientes etiologías de amenorrea presenta niveles elevados de


gonadotropinas?:

1. 1. Disgenesia gonadal.
2. 2. Craneofaringioma.
3. 3. Síndrome de Sheehan.
4. 4. Síndrome de Kallman.
Gráfico de respuestas
Comentario

En el estudio de amenorreas cuando se encuentran elevadas las gonadotropinas, el problema se


encuentra a nivel ovárico, por lo que la respuesta correcta es la número 1 o disgenesia
gonadal.(R1)

45. ¿Cuándo se considera taquicardia fetal?

1. 1. Por encima de 150 latidos/minuto.


2. 2. Por encima de 160 latidos/minuto.
3. 3. Por encima de 170 latidos/minuto.
4. 4. Por encima de 180 latidos/minuto.
Gráfico de respuestas
Comentario

Pregunta teórica sobre la monitorización fetal, que no se presta a razonamiento alguno. Para sacar
esta pregunta debemos saber cuál es el rango normal de la frecuencia cardiaca fetal (FCF). La
normalidad está entre 120 y 160 lpm, por lo que la opción correcta es la 2 (taquicardia >160 lpm).
Bradicardia sería menor de 110- 120 lpm. Recuerden que, tanto en la bradicardia como en la
taquicardia, habría que realizar una microtoma de sangre fetal.(R2)

46. Con respecto a los miomas uterinos, señale la afirmación INCORRECTA:

1. 1. Los miomas se pueden hallar en la trompa de Falopio y en la vagina.


2. 2. Los miomas raramente aparecen o crecen después de la menopausia.
3. 3. La degeneración hialina es la forma menos común de degeneración miomatosa.
4. 4. Aunque los miomas parecen encapsulados, ninguna cápsula real existe.
Gráfico de respuestas
Comentario

-La respuesta 1 es verdadera: son muy infrecuentes, pero por poder, se pueden hallar, puesto que
en las paredes de estos órganos hay músculo liso.

-La respuesta 2 es verdadera: suelen aparecer en mujeres en edad fértil.

!
!
!
!
-La respuesta 3 es claramente incorrecta: La degeneración hialina es la forma más común.

-La respuesta 4 es verdadera; están bien delimitados, pero no poseen una cápsula real.(R3)

47. Si encuentra un paciente con eritema y descamación intensa en zonas periungueales


de los dedos de las manos, engrosamiento urqueal, hiperquetosis y grietas en talones,
eritema y descamación en pabellones auriculares y dorso nasal, debe usted pensar en la
existencia concomitante de:

1. 1. Adenocarcinoma gástrico.
2. 2. Linfoma no Hodgkin.
3. 3. Cancer de pulmón de células pequeñas.
4. 4. Carcinoma escamoso aerodigestivo alto.
Gráfico de respuestas
Comentario

El caso clínico que nos presentan es el de una acroqueratosis de Bazex. Las dermatosis
paraneoplásicas son infrecuentes en el examen, por lo que su estudio no es demasiado rentable,
aparte de que el tema es difícil. La acroqueratosis de Bazex se asocia con gran frecuencia a
carcinomas epidermoides de vías aerodigestivas altas (respuesta 4 verdadera), siendo muy
excepcional que no se asocie a cáncer. En ocasiones, precede en años a las primeras
manifestaciones del tumor. Cursa con importantes alteraciones ungueales y una erupción de
placas eritematosas en zonas acras (pulpejos de los dedos, nariz y orejas), cubiertas por escamas
muy adherentes que producen sangrado al ser desprendidas.(R4)

48. De las variedades de presentación de nalgas. ¿Cuál es la más frecuente?

1. 1. Nalgas completas.
2. 2. Nalgas incompletas.
3. 3. Nalgas puras.
4. 4. Rodillas.
Gráfico de respuestas
Comentario

Las nalgas puras es la variedad más frecuente. Recuerda que, junto con las nalgas completas, son
las únicas en las que se permitirá el parto vaginal si el resto de los requisitos se cumplen. Las
presentaciones de pies o de las rodillas se incluyen dentro de las nalgas incompletas y
contraindican la vía vaginal.(R3)

49. Señale la respuesta FALSA respecto al retraso de crecimiento intrauterino:

1. 1. Las causas que producen CIR de tipo simétrico actúan en fases precoces de la gestación.
2. 2. El CIR que se asocia con anomalías fetales más frecuentemente es el tipo I.
3. 3. En el CIR tipo II el tamaño placentario es proporcional al del feto.
4. 4. Los factores más importantes causantes de CIR I son las alteraciones cromosómicas.
Gráfico de respuestas
Comentario

Existen dos tipos de crecimiento intrauterino retardado (CIR):

!
!
!
!
- CIR tipo I o CIR simétrico: Se produce desde el comienzo de la gestación. El crecimiento
longitudinal, los diámetros abdominales y otras medidas guardan entre sí las proporciones
adecuadas. Entre sus causas, destacan las cromosomopatías y las infecciones connatales.

- CIR tipo II o CIR asimétrico: Es mucho más frecuente que el tipo I. Consiste en una reducción
de los diámetros abdominales, quedando preservados los parámetros óseos de medida (diámetro
biparietal y longitud femoral). La causa más frecuente de CIR II son las enfermedades placentarias,
como la preeclampsia u otras condiciones que puedan producir una disminución del intercambio
útero- placentario. En definitiva, el peso de la placenta no guarda la debida proporción con el del
feto.(R3)

50. Respecto al absceso pulmonar, es INCORRECTO:

1. 1. La mayoría de los abscesos pulmonares son infecciones polimicrobianas.


2. 2. Hay abscesos producidos por un solo microorganismo como S. aureus.
3. 3. Streptococcus pneumoniae origina frecuentemente abscesos pulmonares.
4. 4. En pacientes con absceso pulmonar generalmente existe un deterioro del reflejo tusígeno.
Gráfico de respuestas
Comentario

El neumococo no es una bacteria que produzca abscesos pulmonares con frecuencia. Otras
bacterias, como S. aureus, P. aeruginosa o los anaerobios son más habituales en este tipo de
infección.(R3)

51. Un paciente de 23 años es remitido para estudio tras el hallazgo de eosinofilia


persistente. El hallazgo inicial fue en una BH realizado con motivo de un cuadro febril
autolimitado; la eosinofilia persistía en tres controles posteriores (rango 1,000-1,800
eosinófilos/mcl). El paciente se encuentra asintomático y refiere un viaje a la selva 1 año
antes, durante el que permaneció en área rural. Señale cuál de los siguientes agentes NO
se asocia a eosinofilia:

1. 1. Schistosoma mansoni.
2. 2. Ascaris lumbricoides.
3. 3. Loa loa.
4. 4. Trypanosoma brucei rhodesiense.
Gráfico de respuestas
Comentario
La eosinofilia es habitual en los Helmintos con ciclo tisular,
como Schistosoma, Ascaris, Strongyloides y Filarias.
Ascaris lumbricoides (afectación pulmonar) es de distribución cosmopolita. Strongyloides
stercoralis (afectación cutánea y pulmonar), Schistosoma spp (plexos venosos intestinales y
vesicales) y Filarias, se distribuyen por ciertas áreas tropicales y subtropicales. En estas como en
otras infestaciones por Helmintos, los pacientes pueden estar asintomáticos durante tiempo
variable.
Trypanosoma brucei rhodesiense, agente de la forma oriental de la Enfermedad del Sueño, no se
asocia con eosinofilia al no ser un helminto tisular.(R4)

52. Which of the following options is FALSE regarding acute infantile osteomyleitis?

1. 1. The most commonly affected site is the epiphysis.

!
!
!
!
2. 2. It usually presents with pseudoparalysis of the affected limb.
3. 3. X ray exams may show signs of osteoporosis.
4. 4. The dissemination is hematogenous.
Gráfico de respuestas
Comentario

Todas las osteomielitis tienen como origen más frecuente la diseminación de una infección por vía
hematógena, siendo otros orígenes posibles la infección contigua o el traumatismo. En los niños
suele afectar a METAFISIS (la zona de crecimiento óseo por excelencia, bien irrigada) y no a
epífisis; en los adultos es frecuente la espondilodiscitis. El pequeño evitará mover la extremidad, no
porque no pueda, sino porque incrementa su dolor, pero ante una exploración bajo analgesia se
desenmascara la falsa parálisis. Aunque la evidencia radiográfica es tardía, puede mostrar erosión
ósea en su destrucción.(R1)

53. Estructuras intraoculares con inervación adrenérgica incluyen:

1. 1. Músculo dilatador del iris.


2. 2. Fibras circulares del músculo ciliar.
3. 3. Músculo del esfínter del iris.
4. 4. Fibras oblicuas del músculo ciliar.
Gráfico de respuestas
Comentario
En el ojo existen tres estructuras que tienen inervación vegetativa. El músculo encargado de abrir
la pupila (músculo dilatador del iris), que tiene inervación simpática, en tanto que las fibras
encargadas de cerrarla (músculo esfinter de la pupila) reciben inervación parasimpatica. También
recibe inervación colinérgica el músculo ciliar (tanto sus fibras oblícuas como circulares),
encargado de regular la forma del cristalino, y por lo tanto la potencia de este lente. Las fibras
zonulares, son de naturaleza conectiva, anclan el cristalino al músculo ciliar, y por lo tanto no
tienen inervación alguna.(R1)

54. Señale la respuesta FALSA referente a la valoración del recién nacido:

1. 1. Denominamos RN pretermino a aquel cuya edad gestacional es menor de 37 semanas.


2. 2. Denominamos RNT de bajo peso a aquel cuyo peso al nacimiento es menor de 2,500 g.
La patología asociada a un RN pretermino es comparable a la patología asociada a un RN
3. 3.
con bajo peso para la edad gestacional.
Es fundamental valorar la combinación de peso al nacimiento y edad gestacional para
4. 4.
establecer el riesgo patológico del RN.
Gráfico de respuestas
Comentario

La respuesta que debe elegir es la 3 (por ser incorrecta), ya que no es lo mismo un RNPT, que un
RN con bajo peso. El tipo de complicaciones son muy distintas. El resto de respuestas son
correctas.(R3)

55. ¿Cuál de las siguientes afirmaciones acerca de los factores que influyen en el pulso
arterial es INCORRECTA?

1. 1. La amplitud de la curva está fundamentalmente determinada por el volumen de eyección.

!
!
!
!
El pulso normal es regular, aunque puede existir una ligera variación dependiente del
2. 2.
tiempo respiratorio.
3. 3. La pendiente del ascenso del pulso depende de la velocidad de eyección.
4. 4. Es una onda con dos muescas, una en ascenso (anácrota) y otra en descenso (dícrota).
Gráfico de respuestas
Comentario

Una pregunta muy difícil, no se preocupe si la ha fallado. Es la velocidad de eyección, más que el
volumen, lo que determina la amplitud de la onda del pulso.(R1)

56. A 2-day-old infant is brought to the emergency department presenting with alimentary
vomiting after breastfeeding. Physical examination shows a small mouth with a
protruding tongue, slanted eyes, poor muscle tone, a flat nasal bridge and a short neck.
A parasternal systolic murmur with a doubled S2 is heard at cardiac auscultation. What
is the most likely cause of his hyperemesis?

1. 1. Duodenal atresia.
2. 2. Pyloric stenosis.
3. 3. Meconial ileum.
4. 4. Gastroesophageal reflux.
Gráfico de respuestas
Comentario
Duodenal atresia. The physical description of the child in this case is consistent with Down
syndrome. The most common digestive malformation associated is duodenal atresia, which should
be suspected if symptoms of intestinal obstruction with vomiting appear. This child probably suffers
from a heart defect.(R1)

57. En el diagnóstico diferencial de la peritonitis bacteriana espontánea del cirrótico con


respecto a la peritonitis secundaria, ¿cuál de los siguientes datos puede hacer sospechar
una peritonitis secundaria?

1. 1. Presencia de >250 neutrófilos por mm3 de líquido ascítico.


2. 2. Flora monomicrobiana en la tinción de Gram.
3. 3. Nivel de glucosa en líquido ascítico menor que en plasma.
4. 4. Baja concentración de proteínas en líquido ascítico.
Gráfico de respuestas
Comentario

Pregunta de dificultad básica acerca de la peritonitis bacteriana espontánea.

La pregunta nos pide el diagnóstico diferencial entre la peritonitis bacteriana espontánea (PBE) y la
peritonitis secundaria a favor de esta última. La peritonitis bacteriana espontánea (PBE) es una
infección de la ascitis preexistente en ausencia de una fuente intraabdominal obvia. Sucede en
cirróticos y su etiología suelen ser gramnegativos y de ellos, el más frecuente es el E. coli.

La peritonitis secundaria se origina por diseminación bacteriana desde una víscera o desde una
fuente externa por penetración, siendo los gérmenes causales anaerobios.

En cuanto a la clínica, lo más frecuente es que los pacientes refieran dolor abdominal y fiebre. Este
dato es común tanto en la PBE como en la peritonitis secundaria.

!
!
!
!
El diagnóstico definitivo de PBE lo da el cultivo, pero como tarda, nos basamos en otros
parámetros para establecer el diagnóstico de sospecha e iniciar el tratamiento empírico. Lo más útil
es la medición de leucocitos en líquido ascítico: si son >500/mm3; o lo que es mejor, el nivel de
neutrófilos es >250/mm3, establecemos el diagnóstico de sospecha y tratamos empíricamente.

En cuanto a los cultivos, va a favor de peritonitis bacteriana secundaria: leucocitos >10,000/mm3,


proteínas en líquido ascítico >2.5 g/dl, LDH >225, glucosa <50 mg/dl o cuando en los cultivos hay
múltiples patógenos, sobre todo si hay anaerobios. Con esto, la opción 4, que se refiere a la baja
concentración de proteínas en el líquido ascítico, y la opción 2 (flora monomicrobiana al gram) van
a favor de PBE; y así, la opción 3, nivel de glucosa en líquido ascítico menor que en plasma (o <50
mg/dl más en concreto), es la válida, ya que es la que nos habla de peritonitis secundaria.

Por último, recordar que el tratamiento de la PBE se apoya en cefalosporinas de tercera


generación con administración de albúmina intravenosa para prevenir desarrollo de insuficiencia
renal. Para el tratamiento de la peritonitis secundaria tenemos: cefalosporina + metronidazol,
clindamicina + aminoglucósido, imipenem...(R3)

58. ¿Cuál de las siguientes afirmaciones es CORRECTA acerca del análisis cariotípico de
los síndromes mielodisplásicos?:

Las alteraciones más frecuentes son las que implican pérdida total o parcial del cromosoma
1. 1.
5
2. 2. La alteración más frecuente es la de roturas de las cromátides
La trisomía del cromosoma 8 es específica del cuadro de anemia refractaria con exceso de
3. 3.
blastos en transformación
Las alteraciones cariotípicas permiten el diagnóstico diferencial entre síndromes
4. 4.
mielodisplásicos y leucemias mieloblásticas.
Gráfico de respuestas
Comentario

Pregunta de dificultad elevada y poco habitual.

Los pacientes con cariotipo normal, deleción aislada 5q y deleción aislada 20q constituyen un
grupo de mejor pronóstico. Mientras que la presencia aislada de monosomía 7, deleción 7q y las
anomalías complejas tienen un pronóstico desfavorable. Luego es coherente pensar que las
pérdidas totales o parciales de los cromosomas sean las alteraciones más habituales en los SMD.
Las otras opciones no son del todo verdaderas. Aprovecho para recordarles que, la proporción
medular de blastos, la citogenética, el número y grado de las citopenias y la edad son los
principales indicadores pronósticos de los SMD.(R1)

59. Patients with chronic renal failure have multisystemic manifestations due to the
effects of his underlying disease. Which of the following is the most common cause of
death in such patients?

1. 1. Hypokalemia.
2. 2. Acute myocardial infarction.
3. 3. Dialysis dementia.
4. 4. Sepsis.
Gráfico de respuestas
Comentario

!
!
!
!
Se trata de una pregunta con respuesta directa que no le debe hacer vacilar. Las complicaciones
cardiovasculares son la principal causa de muerte en los pacientes con insuficiencia renal crónica.
Recuerde que la demencia dialítica por déposito de aluminio actualmente se puede prevenir y
estabilizar con EDTA. Recuerde también que el potasio se mantiene norma hasta fases avanzadas
de la enfermedad. Luego aparece típicamente hiperpotasemia y no hipopotasemia.(R2)

60. Masculino de 36 años que presenta, desde hace 3 semanas, lesiones asintomáticas
distribuídas por el codo derecho, las rodillas, el ombligo y la región glútea. Hace 2 años
tuvo un episodio similar que desapareció al irse de vacaciones al Caribe. A la exploración
presenta placas redondeadas, de varios centímetros de diámetro, perfectamente
limitadas, con una superficie eritematodescamativa. Al raspado con una cucharilla
dermatológica se desprenden escamas furfuráceas. ¿Qué complicación puede
observarse con frecuencia en estos pacientes?

1. 1. Rinoconjuntivitis.
2. 2. Nefritis.
3. 3. Artritis.
4. 4. Insuficiencia cardíaca de alto gasto.
Gráfico de respuestas
Comentario

Se trata de un caso clínico sencillo. Fíjese en la descripción de las lesiones: - placas asintomáticas
- codos, rodillas y glúteos - eritematodescamativas - con escamas furfuráceas con esta
descripción debemos pensar en psoriasis, cuya complicación puede ser la artritis psoriásica.(R3)

61. En relación con la linfangiectasia intestinal, indique la afirmación INCORRECTA:

1. 1. La biopsia no es diagnóstica, a pesar de ser una lesión difusa.


2. 2. Se trata con dieta pobre en grasas.
3. 3. Se manifiesta con edemas y malabsorción.
4. 4. Los niveles de transferrina y ceruloplasmina están por debajo de lo normal.
Gráfico de respuestas
Comentario
La linfangiectasia intestinal es una de las pocas entidades en que la biopsia intestinal es
patognomónica, y aporta el dignóstico. En el estudio anatomopatológico, se observan dilataciones
tortuosas de los vasos linfáticos. Otras enfermedades en las que la biopsia intestinal es diagnóstica
son: enfermedad de Whipple (macrófagos con inclusiones PAS+), abetalipoproteinemia
(enterocitos cargados de gotas lipídicas), hipogammaglobulinemia (ausencia de células
plasmáticas).(R1)

62. Señale cuál de las siguientes NO es una contraindicación para la colocación de un


dispositivo intrauterino:

1. 1. Enfermedades hematológicas o pacientes con tratamiento anticoagulante.


2. 2. Riesgo de infección de transmisión sexual.
3. 3. Útero miomatoso.
4. 4. Hipertrigliceridemia.
Gráfico de respuestas
Comentario

!
!
!
!
La hipertrigliceridemia no contraindica la colocación del DIU. El resto de opciones lo contraindican,
por el riesgo de complicaciones derviadas.(R4)

63. Señale la respuesta FALSA sobre el síndrome de Boerhaave o rotura esofágica


espontánea:

1. 1. Aparece frecuentemente en hombres que realizan excesos gastronómicos y en alcohólicos.


2. 2. El síntoma más frecuente es el dolor.
3. 3. La rotura se produce típicamente en el esófago cervical.
Uno de los aspectos más importantes de su tratamiento es asegurar un buen drenaje del
4. 4.
mediastino, para evitar la aparición de mediastinitis.
Gráfico de respuestas
Comentario
Este síndrome suele producirse después de vómitos repetidos. Cursa generalmente con intenso
dolor esternal y suele complicarse con mediastinitis, salvo que se coloque un buen drenaje a nivel
mediastínico. La ruptura generalmente es en el esófago medio o distal y es más frecuente en
pacientes con antecedente de alcoholismo.(R3)

64. ¿Cuál de los siguientes nervios es el más frecuentemente lesionado cuando se


obtiene injerto óseo de la región más posterior de la cresta ilíaca?

1. 1. Nervio glúteo superior.


2. 2. Nervio cluneal.
3. 3. Raíz de L5.
4. 4. Nervio obturador.
Gráfico de respuestas
Comentario

La sensación cutánea de la región glútea viene dada por los nervios cluneales superior, medio e
inferior. El N. cluneal superior son ramas laterales de las raíces dorsales de los 3 nervios lumbares
más superiores y penetran en la fascia profunda justo proximal a la cresta iliaca; estos pasan
distalmente a la piel del glúteo y pueden ser lesionados si la exposición de la cresta iliaca se
extiende desde anterolateral más de 8 cm hacia la espina iliaca posterosuperior. El N. fémoro-
cutáneo lateral puede ser lesionado en la región más anterior de la espina iliaca anterosuperior y
se encarga de la inervación sensitiva de la región anterolateral del muslo, siendo también
responsable del cuadro compresivo de meralgia parestésica. El N. glúteo superior e incluso el N.
ciático pueden lesionarse si el hueso se obtiene de la escotadura ciática o la disección no se lleva
a cabo subperiósticamente; sin embargo la tasa de lesiones de estos, está lejos de la del Nervio
Cluneal. Las raíces L5 o S1 son anteriores y pueden ser lesionadas en la tabla interna de la pala
iliaca si se extrae hueso de esa zona y la disección no es mantenida subperiósticamente o es
demasiado medial; sin embargo, la tasa de estas lesiones de nuevo está lejos de la del nervio
cluneal. (R2)

65. ¿Cuál de los siguientes NO es un rasgo típico del síndrome de ovario poliquístico
(SOP)?:

1. 1. Hirsutismo.
2. 2. Oligomenorrea.
3. 3. Menometrorragias.
4. 4. Problemas con el metabolismo hidrocarbonado.
Gráfico de respuestas

!
!
!
!
Comentario

Tema preguntado casi cada año en el ENARM. Los síntomas del SOP son: esterilidad 73%,
trastornos menstraules como oligomenorrea y bache amenorreicos, hirsutismo y resistencia a la
insulina, por lo que la respuesta incorrecta es la 3.(R3)

66. Hombre de 22 años que acude por


diarrea de 8 semanas de evolución, con moco, pero sin sangre ni pus. Fiebre vespertina
de 37.8º C y disminución de 5 kg en los últimos 4 semanas. Destacan leucocitos de 18,000
con 92% de neutrófilos, VSG 66 mm en la primera hora y hemoglobina 10 g/dl. Los
coprocultivos y parásitos fueron negativos. El recuento de inmunoglobulinas,
proteinograma y serología contra enfermedad celíaca fueron normales. Se realizó una
colonoscopía donde observó un colon izquierdo de aspecto normal y la imagen adjunta
en ciego. Acerca de la enfermedad que usted sospecha señale la afirmación CORRECTA:

1. 1. El recto suele encontrase afectado de forma casi constante.


La afectación de la mucosa puede ocurrir en cualquier parte del aparato digestivo de forma
2. 2.
discontinua.
3. 3. El tabaco es factor protector de esta patología.
4. 4. La afectación histológica puede ser transmural.
Gráfico de respuestas
Comentario

Una pregunta muy interesante sobre la enfermedad inflamatoria intestinal. Dada la ausencia de
afectación en colon izquierdo, con lesiones principalmente en la zona del ciego, el diagnóstico que
habría que sospechar es una enfermedad de Crohn. La imagen que nos muestra la pregunta
corresponde a sus típicas úlceras serpinginosas. Respecto a las opciones que nos presentan,
debemos decir:

R1: El recto se afecta de forma casi constante en la colitis ulcerosa, no en el Crohn.

R3: El tabaco es protector en la colitis ulcerosa. En la enfermedad de Crohn, es factor de riesgo.

!
!
!
!
R4: La afectación histológica no es que pueda ser transmural, sino que lo es por definición.(R2)

67. Tras el diagnóstico realizado, ¿qué tratamiento propondría en este paciente de entre
los expuestos a continuación?

1. 1. Mesalazina en enemas.
2. 2. Prednisona oral.
3. 3. Infliximab i.v.
4. 4. Ciclosporina i.v.
Gráfico de respuestas
Comentario

Dado que se trata de una enfermedad inflamatoria intestinal en brote agudo, el tratamiento de
elección serían los corticoides. En este caso, teniendo en cuenta la estabilidad del paciente, no
sería necesario pautar tratamiento intravenoso, al menos de inicio, por lo que bastaría comenzar
con prednisona oral, en dosis altas (0.5- 1 mg/kg/día), que habrá que descender de forma muy
paulatina para evitar el efecto rebote.(R2)

68. Un hombre de 73 años con enfermedad pulmonar obstructiva crónica está ingresado
en medicina interna tras ser atendido en Urgencias por una insuficiencia respiratoria
global, secundaria a una infección respiratoria. Durante su segunda noche en el hospital,
presenta agitación, desorientación temporal y espacial, falsos reconocimientos,
insomnio y agresividad verbal y física hacia el personal cuidador. El paciente se arranca
la mascarilla de oxígeno y las vías de perfusión. Es portador de una prótesis de cadera
derecha. La enfermera de turno le avisa a usted, que es el médico de guardia. ¿Cuál de
los siguientes comportamientos asistenciales es correcto en el contexto clínico descrito?

Invitar al paciente a firmar el alta voluntaria, previa información de los riesgos derivados
1. 1.
del no tratamiento de su condición diagnóstica.
No iniciar ningún procedimiento diagnóstico ni terapéutico al tratarse de un problema
2. 2.
psiquiátrico.
Proceder a la sujección mecánica del paciente para posibilitar su sedación mediante la
3. 3. administración de haloperidol por vía i.v. y la realización de los procedimientos
diagnósticos y terapéuticos que se estimen indicados.
Proceder a la sujección mecánica del paciente, evitando sedación de ningún tipo, aislándolo
4. 4.
en una habitación insonorizada.
Gráfico de respuestas
Comentario

Lo primero es llegar al diagnóstico de DELIRIUM (eso parece fácil) y luego razonar entre las
opciones.

Del delirium hay que saber tres cosas:

•! Su diagnóstico: deterioro agudo de las funciones superiores, que con frecuencia se asocia
a agitación y síntomas psicóticos. La desorientación es típica y se inicia de forma brusca.
•! Todo delirium tiene una causa orgánica que hay que averiguar y tratar.
•! El tratamiento sintomático incluye la sedación con un fármaco de alta seguridad
cardiorrespiratoria (haloperidol) y la sujeción mecánica, si es precisa. Las benzodiazepinas

!
!
!
!
están contraindicadas por el riego de empeorar la confusión y de provocar depresión
respiratoria (salvo en delirium por abstinencia alcohólica o de benzodiacepinas).(R3)

69. A farmer is brought to the emergency department with general malaise, chills,
sweating and fever. Chest auscultation reveals tachycardia with no heart murmurs, and
normal lung auscultation. He has a wound in his left foot, which he sustained three days
ago with one of his tools. The wound appears erythematous, with edema and some
surrounding vesicles. Both palpation of the wound and mobilization of the leg are painful.
Blood tests show: WBC count 13,250, with significant increase in urea and creatinine.
Urinalysis reveals: leukocytes, red blood cells and myoglobin (++/+++). Which of the
following is the most appropriate next step in management?

1. 1. Dialysis and ICU admission.


2. 2. Chloramphenicol + hydration.
3. 3. Penicillin G + hydration + surgical debridement of the wound.
4. 4. Surgical debridement of the wound + hydration + tetanus vaccination + anti-tetanus IgG.
Gráfico de respuestas
Comentario

La gangrena estreptocócica es una infección extraordinariamente grave. Su agente causal es


Streptococcus pyogenes (grupo A). Penetra en los tejidos del huésped a través de pequeñas
heridas. Inicialmente, aparece dolor y eritema local, formándose vesículas. Al poco tiempo, la
infección se extiende por el espacio interfascial, produciendo afectación y necrosis muscular. Suele
acompañarse de insuficiencia renal (por rabdomiolisis), choque y fracaso multiorgánico. Para su
tratamiento, habrá que tener en cuenta:

- Es importante expandir el volumen y aportar factores de coagulación (plasma) para hacer


frente a la CID.

- El desbridamiento quirúrgico del tejido necrótico es imprescindible.

- El tratamiento antibiótico de primera elección es la penicilina (todavía no existen formas


resistentes de S. pyogenes).(R3)

70. ¿Cual de las siguientes es considerado como factor de riesgo en la infección de herida
quirúrgica?

1. 1. Administración de antibióticos previamente.


2. 2. Sexo femenino.
3. 3. Uso de quimioterapia.
4. 4. Asma bronquial.
Gráfico de respuestas
Comentario

La respuesta correcta es la 3, ya que el uso de quimioterapia genera un estado de


inmunosupresión que incrementa el riesgo de infecciones no sólo quirúrgicas si no de cualquier
tipo.

!
!
!
!
71. Un paciente de 45 años consulta por dolor abdominal epigástrico. Una endoscopia
demuestra una úlcera gástrica a nivel del antro. Las biopsias descartan malignidad. Se
inicia tratamiento con 20 miligramos al día de omeprazol. A las cuatro semanas se le hace
un control radiológico, que demuestra que el lecho ulceroso persiste, aunque se ha
reducido en un 75%. Se mide la gastrina sérica, que está claramente elevada. ¿Qué haría
con este paciente?

1. 1. Aumentar omeprazol a 80 miligramos al día.


2. 2. Cambiar tratamiento a un anti-H2.
3. 3. Le haría un TAC pancreático.
4. 4. Seguiría con el mismo tratamiento.
Gráfico de respuestas
Comentario

Algunos tumores gástricos pueden debutar como úlceras gástricas, por lo que es obligatorio, al
hacer un diagnóstico por imagen de una úlcera gástrica, tomar biopsias para descartar malignidad.
Además, las úlceras gástricas, en general, suelen ser más grandes que las duodenales, y por ello
su tiempo necesario para la cicatrización es mayor. Los fármacos que se emplean son los
antisecretores, fundamentalmente inhibidores de la bomba de protones, como el omeprazol. El
tiempo estimado de tratamiento con omeprazol necesario para la cicatrización de una úlcera
gástrica es de unas 12 semanas. En este caso, parece que el tratamiento está siendo correcto,
aunque aún habría que mantenerlo 8 semanas más. En cuanto a la gastrina, es normal que se
eleve, ya que el efecto del fármaco antisecretor disminuye la secreción ácida en el estómago. Y
esta hipoclorhidria es, precisamente, el estímulo más potente para la secreción de gastrina.(R4)

72. Un paciente mascuino de 71 años, alcohólico, en tratamiento con prednisona en dosis


decreciente por una hepatitis aguda alcohólica (actualmente 30 mg diarios), consulta en
el Servicio de Urgencias con un cuadro de tres días de evolución de cefalea, náuseas,
vómitos, sensación distérmica y malestar general. El paciente está febril, somnoliento y
con rigidez de nuca, sin otros hallazgos. Se realiza una TC cerebral, que descarta lesiones
ocupantes de espacio, seguido de una punción lumbar, donde se detecta
hipoglucorraquia, hiperproteinorraquia y linfocitos en LCR. En espera de los resultados
del cultivo, el tratamiento empírico más CORRECTO sería:

1. 1. Ceftriaxona.
2. 2. Ceftriaxona y vancomicina.
3. 3. Ceftriaxona, vancomicina y ampicilina.
4. 4. Cefotaxima y vancomicina.
Gráfico de respuestas
Comentario

Se trata de un cuadro de meningitis subaguda, el tratamiento inicial debe ser: cefalosporina de 3


generación con vancomicina (por el alto % de resistencias a las cefalosporinas de 3 generación) y
ampicilina (por la probabilidad de que la Listeria esté también implicada, dado el antecedente de
alcoholismo y tratamiento con corticoides).(R3)

73. Un niño de 7 años es traido a la consulta por sus padres, los cuales refieren bajo
rendimiento escolar de su hijo. Su maestro comunica que a veces parece preocupado,
presenta episodios de parpadeo y, ocasionalmente, tarda en responder o sus respuestas
son incorrectas. El examen físico es normal. El estudio que más ayuda al diagnóstico es:

!
!
!
!
1. 1. EEG.
2. 2. Calcemia.
3. 3. Glucemia.
4. 4. Examen visual.
Gráfico de respuestas
Comentario

Pregunta típica que nunca falta. Se trata de un caso clínico de crisis de ausencia, por lo que estaría
indicado realizar un EEG. Respuesta 1 correcta.

74. Masculino de 35 años politraumatizado con fractura pélvica que presenta dolor
abdominal bajo, incapacidad para la micción y presencia de sangre en el meato de la
uretra. Seleccione la CORRECTA:

1. 1. La siguiente prueba a realizar sería una uretrografía.


El antecedente de fractura pélvica y uretrorragia nos orienta a afectación del tracto
2. 2.
superior.
3. 3. Se puede realizar una uretroscopia.
4. 4. No es de utilidad un TC si se tratara de un traumatismo del tracto superior.
Gráfico de respuestas
Comentario

Ante la sospecha de un traumatismo urológico debemos distinguir entre traumatismo del tracto
inferior o superior.

En este caso el antecedente de fractura pélvica y la presencia de uretrorragia nos lleva a


declinarnos por la afectación del tracto inferior.

En estos casos la primer prueba diagnósica debe ser una uretrografía.

La cateterización o uretroscopia no deben realizarse porque conllevan un alto riesgo de producir


hematoma e infección.

Si se tratase de un traumatismo del tracto superior deberíamos optar por una TAC o una urografía
intravenosa.(R1)

75. ¿Cuál es la frecuencia de fibrosis quística en la raza blanca?:

1. 1. 1/2,000.
2. 2. 1/10,000.
3. 3. 1/15,000.
4. 4. 1/20,000.
Gráfico de respuestas
Comentario

Concepto poco práctico para fines del ENARM. Esta pregunta sobre la epidemiología de la fibrosis
quística carece de importancia. Simplemente recuerde que es la enfermedad genética letal más
frecuente en la raza blanca.(R1)

76. La osteogénesis imperfecta se caracteriza por la siguiente manifestación ocular:

!
!
!
!
1. 1. Cataratas congénitas.
2. 2. Desprendimiento de retina.
3. 3. Escleras azuladas.
4. 4. Atrofia del nervio óptico.
Gráfico de respuestas
Comentario

Esta pregunta le ayudará a aprender una de las pocas cosas que puede ser interesante saber
respecto a esta enfermedad, que es poco rentable. Entendiendo que la base del trastorno es una
síntesis anómala de procolágeno, y que en el ojo este tejido está presente en la esclera, es fácil
comprender que esta capa estará adelgazada. Lo sorprendente es que la delgadez permite que se
transparente la siguiente capa pigmentada, la coroides, que se percibe como azulada a quien la
observe. Esto no ocurre en todos los pacientes, pero es algo típico. Otras características del
síndrome son: osteopenia generalizada con fracturas frecuentes, sordera, dentinogénesis
imperfecta e historia familiar positiva, ya que es hereditario.(R3)

77. Mujer de 54 años que, en una mastografía de screening, se detecta nódulo espiculado
de 6 mm, en cuadrante superoexterno de la mama izquierda. Biopsia con agua gruesa:
carcinoma ductal infiltrante. Los receptores hormonales resultan ser positivos. Her 2-neu
negativo. Exploración clínica: nódulo no palpable, axila libre. Ultrasonido axilar: no
ganglios sospechosos. Señale la afirmación INCORRECTA respecto a este caso:

Dado el tamaño tumoral, sería preferible una resección conservadora frente a una
1. 1.
mastectomía radical.
2. 2. Sería conveniente añadir tratamiento local con radioterapia.
3. 3. Sería tributaria de recibir hormonoterapia.
4. 4. Sería aconsejable el uso de trastuzumab como parte del tratamiento.
Gráfico de respuestas
Comentario

El trastuzumab sólo tiene uso en el cáncer de mama metastásico que expresa el gen Her2 neu. En
este caso no hay ningún indicio de que se trate de un cáncer diseminado y, sobre todo, el gen Her
2 neu es negativo.(R4)

78. A 41-year-old man is suddenly found in his room unresponsive and his wife, who
works as a nurse, immediately checks his pulse and finds out that he has a
cardiopulmonary arrest. She starts basic cardiopulmonary resusciation until the arrival
of the ambulance. ECG monitoring reveals ventricular fibrillation. After electrical
cardioversion, sinus rhythm is restored and the patient regains his consciousness within
a few minutes. Which of the following options is FALSE?

In our population, acute myocardial infarction is the most common cause of sudden cardiac
1. 1.
death.
Once sinus rhythm is restored, ECG may reveal specific findings that will make us suspect
2. 2. syndromes such as: Brugada syndrome, long QT syndrome or hypertrophic
cardiomyopathy.
Patients with long QT syndrome may present with syncopal episodes or sudden death due
3. 3. to polymorphic ventricular tachycardia known as "torsade de pointes". Beta-blockers are
the first-choice treatment.
In patients with hypertrophic cardiomyopathy, sudden death occurs at rest, and not during
4. 4. physical activity or exertion. The underlying mechanism is an obstruction of the blood flow
from the left ventricle to the aorta.

!
!
!
!
Gráfico de respuestas
Comentario

Una pregunta de dificultad media-alta sobre la muerte súbita, tanto por la extensión como por las
entidades que menciona.

La causa de muerte más frecuente en la miocardiopatía hipertrófica obstructiva es la muerte súbita,


que a veces ocurre sin que exista ningún síntoma previo de cardiopatía. Desde el punto de vista
etiopatogénico, obedece a arritmias ventriculares, no a la obstrucción a la salida de sangre del
ventrículo izquierdo (respuesta 4 falsa).

En relación a este tema, debemos recordar que las arritmias ventriculares son más frecuentes en
pacientes jóvenes con historia familiar de muerte súbita. Por otra parte, no existe relación entre la
probabilidad de muerte súbita y la presencia o gravedad del gradiente o de los síntomas. Como
factores de riesgo, existe clara evidencia a favor de los siguientes:

•! Edad inferior a 30 años, en el momento del diagnóstico.


•! Antecedentes familiares de muerte súbita.
•! Síncope (especialmente en niños).
•! Arritmias ventriculares documentadas en el holter de 48h.
•! Respuesta anormal de la presión arterial con ejercicio.
•! Genotipos desfavorables.

En pacientes con alto riesgo de esta complicación, está indicado el uso de un desfibrilador
automático implantable (DAI).(R4)

79. ¿Cuál de las siguientes afirmaciones es CORRECTA en cuanto a los tumores


epidérmicos?:

Los epiteliomas basocelulares no se presentan en mucosas, ni de forma primaria ni


1. 1.
secundaria.
Se han relacionado con la patogénesis del carcinoma epidermoide infecciones virales,
2. 2.
particularmente por el VHS tipo VIII.
Las metástasis del carcinoma de células escamosas tienen predilección por la piel
3. 3.
deteriorada por el sol.
La eritroplasia de Queirat es el equivalente a la enfermedad de Bowen en las mucosas, pero
4. 4.
su pronóstico es peor por su mayor tendencia metastatizante.
Gráfico de respuestas
Comentario

Los tumores malignos epidérmicos son un tema importante dentro del bloque de dermatología.
Debe conocer muy bien los aspectos diferenciales entre el espinocelular y el basocelular.

El carcinoma espinocelular o epidermoide aparece fundamentalmente sobre piel dañada por el sol.
Los carcinomas basocelulares no aparecen primariamente en mucosas porque no se originan en
ellas, pero sí cabe la posibilidad de que se extiendan a ellas a partir de un origen inicial en la piel,
con lo que podrían afectarlas secundariamente. Si hay un virus implicado en la génesis de los
carcinomas epidermoides, es el del papiloma humano, como sucede en el caso del cáncer de
cérvix. No obstante debe recordar que, en los carcinomas epidermoides cutáneos, el factor más
importante en su génesis es la exposición solar crónica, con gran diferencia. Las metástasis del
carcinoma epidermoide suelen ser linfáticas. Lo que tiene predilección por la piel dañada por el sol

!
!
!
!
es el tumor primario. La enfermedad de Bowen, lo mismo que la eritroplasia de Queyrat, son
carcinomas epidermoides "in situ", es decir, contenidos por encima de la membrana basal, siendo
peor el pronóstico de la eritroplasia porque los cánceres que de ella derivan metastatizan con
mayor frecuencia.(R4)

80. En la reanimación neonatal, si después de realizar la estimulación táctil y


permeabilizar la via aérea el neonato no presenta respiración espontanea, usted
considera:

1. 1. Administrar adrenalina I.V.


2. 2. Administrar oxigeno en casco.
3. 3. Aspirar secreciones.
4. 4. Intubar.
Gráfico de respuestas
Comentario

Pregunta sencilla sobre reanimación neonatal. Respuesta correcta es la 4.

!
!
!
!

81. Durante la adolescencia, el crecimiento se caracteriza por:

1. 1. Iniciarse en el tronco.
2. 2. Ser simétrico.
3. 3. Iniciarse en manos y pies.
4. 4. Iniciarse en brazos y piernas.
Gráfico de respuestas
Comentario

Pregunta que pueden contestar sin la necesidad de conocimiento. La respuesta dos la puede
descartar inmediatamente ya que las otras tres opciones le dicen que el crecimiento se inicia en
lugares específicos. Por puro razonamiento, ¿es posible que el crecimiento empieza en manos,
pies, brazos y piernas? Parece que no. La respuesta correcta es la 1.

82. ¿Cómo son los niveles hormonales en el día 21 del ciclo genital femenino?:

1. 1. LH: baja, FSH: alta, progesterona: alta, estrógenos: altos.


2. 2. LH: baja, FSH: baja, progesterona: alta, estrógenos: altos.
3. 3. LH: alta, FSH: baja, progesterona: alta, estrógenos: bajos.
4. 4. LH: alta, FSH: alta, progesterona: alta, estrógenos: altos.
Gráfico de respuestas
Comentario

A partir de la ovulación (día 14) el folículo sufre una transformación gracias a la LH, y se convierte
en cuerpo lúteo, cuya misión es establecer las condiciones que favorezcan la gestación: para ello
produce progesterona. Parte de esta progesterona producida por el cuerpo lúteo se transforma en
andrógenos y en estrógenos, lo que motiva el pico de estrógenos y progesterona en la mitad de la
fase secretora del ciclo (día 21). La FSH y la LH tiene un pico en la ovulación día 14 pero en el día
21 ambas están bajas.(R2)

Niño de un año de edad cuya madre acaba de ser diagnosticada de tuberculosis pulmonar
bacilífera, habiendo iniciado tratamiento correcto. El niño está asintomático, con una
exploración normal, prueba de tuberculina (PT) negativa, y RX de tórax normal. ¿Cuál
sería la actitud a adoptar en el niño?

1. 1. Sólo separarlo de la madre hasta que ésta concluya su tratamiento (al menos 6 meses).
Realizar una TC de alta resolución pulmonar y si es normal, repetir la PT a las 8-12
2. 2.
semanas.
Quimioprofilaxis con INH, 6-9 meses. Nueva PT y Rx tórax al finalizarlo, para decidir si
3. 3.
concluir, o no, el tratamiento.
Quimioprofilaxis con INH, 8-12 semanas. Repetir PT. Si negativa, cesar el tratamiento. Si
4. 4.
positiva, realizar Rx tórax, para decidir si hay o no enfermedad.
Gráfico de respuestas
Comentario

Pregunta fácil y muy importante sobre el tratamiento de la enfermedad latente en la tuberculosis.

!
!
!
!
La pregunta hace referencia a la actitud a llevar a cabo ante un conviviente, en este caso un niño
cuyo Mantoux es negativo y las pruebas son normales, lo que nos confirma que no está infectado.
Hay que tener claro que al ser menor de 20 años, debemos hacer profilaxis con isoniacida tal y
como dice la respuesta 4, esperando 2-3 meses tras lo cual repetiremos el Mantoux y según sea
este positivo o negativo, repetiremos la radiografía de tórax y baciloscopia o suspenderemos la
isoniacida, respectivamente.(R4)

83. ¿Cuál de los siguientes es el signo más característico de la rubéola o sarampión


alemán?

1. 1. Manchas de Koplik.
2. 2. Lengua en fresa blanca.
3. 3. Amígdalas cubiertas por un exudado blancogrisáceo.
4. 4. Adenopatías retroauriculares y occipitales.
Gráfico de respuestas
Comentario

La rubéola es una enfermedad infecciosa, producida por un Togavirus, cuyo período de incubación
es de 2-3 semanas.

El pródromos consiste en un cuadro catarral leve, con fiebre baja o moderada, conjuntivitis sin
fotofobia y un enantema no patognomónico (manchas de Forcheimer).

El signo más característico de esta fase sonlas adenopatías retroauriculares, cervicales posteriores
y postoccipitales. Poco después, aparece un exantema, que es morbiliforme y confluyente en la
cara. Se resuelve a través de una mínima descamación.

Es posible que aparezcan ciertas complicaciones, como la artritis (la más frecuente), la encefalitis o
la trombopenia.(R4)

84. Mujer de 31 años consulta por una historia de síntomas generales de 4 meses de
evolución, durante los cuales ha tenido 4 episodios sincopales y ha acudido 2 veces al
oftalmólogo por sintomatología ocular. En la exploración destaca una PA=170/105,
ausencia de pulsos radiales y muy ténues en las braquiales, con soplos a nivel de las
arterias subclavia, humeral y carótida. En los exámenes de laboratorio destaca un
aumento de VSG, anemia leve e hipergammaglobulinemia.¿Cúal es la enfermedad más
probable de esta paciente?:

1. 1. Enf. de Kawasaki.
2. 2. Arteritis temporal.
3. 3. Enf. de Takayasu.
4. 4. Ateromatosis precoz.
Gráfico de respuestas
Comentario

Presentan el caso de una paciente joven con cuadro de enfermedad sistémica (por síntomas
generales, anemia y elevación de reactantes de fase aguda), síncopes de repetición y alteraciones
oculares (lo que puede reflejar patología carotídea). Ante un cuadro así, si en la exploración se
encuentra asimetría o ausencia de pulsos como en este caso, debe tener, como primera
posibilidad diagnóstica una arteritis de Takayasu, ya que los otros diagnósticos no son compatibles
por completo: la ateromatosis no es propia de pacientes jóvenes y suele dar otro tipo de clínica,

!
!
!
!
además de no ser muy compatible la existencia de soplos a tantos niveles. La enfermedad de
Kawasaki es una vasculitis que puede producir afectación coronaria además de lesión en piel,
mucosas y adenopatías, pero, además de no ser compatible es una patología de niños, mientras
que la arteritis de la temporal es propia de ancianos y cursa con cefalea y alteraciones visuales por
afectación de la arteria temporal, sin alteración de pulsos. Una gamampatía monoclonal puede
producir un síndrome de hiperviscosidad, pero no lesión a nivel de grandes vasos.(R3)

85. Neonato prematuro de 25 semanas


conectado a ventilación mecánica convencional desde el nacimiento por enfermedad de
membrana hialina recibiendo una dosis de surfactante al nacimiento. Inició nutrición
enteral trófica al segundo día de vida. Hoy presenta aspirado gástrico sanguinolento y
distensión abdominal. Realiza una radiografía de abdomen con los siguientes hallazgos
(imagen nº ##). Con respecto al cuadro que usted sospecha señale la FALSA:

El riesgo es mayor en los recién nacidos expuestos a la cocaína por sus propiedades
1. 1.
vasoconstrictoras y hemodinámicas que favorecen la isquemia intestinal.
Una alimentación hiperosmolar y en pequeñas cantidades parece ser un factor protector del
2. 2.
cuadro.
3. 3. El signo radiológico típico es la presencia de neumatosis intestinal.
4. 4. Ante datos que sugieran perforación (neumoperitoneo) está indicada la cirugía.
Gráfico de respuestas
Comentario

Nos presenta un cuadro típico de neonatología que debemos conocer; la enterocolitis necrotizante.
Se trata de una lesión isquémico-necrótica que suele afectar al íleon distal y colon proximal, y que
se sigue habitualmente de sepsis bacteriana a partir de un foco digestivo. Se consideran factores
de riesgo para su desarrollo: el inicio de la alimentación precoz con elevados volúmenes y
concentraciones, la prematuridad, las situaciones de hipoxia y bajo gasto. El primer signo es la
distensión abdominal, seguido de deposiciones sanguinolentas. En la radiografía de abdomen
simple observaremos patrón de neumatosis intestinal: edema de asas, patrón de migaja de pan,
asas fijas, gas en vena porta y/o neumoperitoneo si existe perforación.(R2)

!
!
!
!
86. ¿Cuál sería el tratamiento más adecuado de un paciente con fiebre y signos clínicos
de oclusión intestinal, que presenta la radiografía mostrada en la imagen?

1. 1. Indicaría dieta absoluta, administración de líquidos intravenosos y antibióticos.


Realizaría exámenes de laboratorio con BH, QS y reactantes de fase aguda y radiológico
2. 2.
seriado cada 6-8 horas o según evolución.
Tras extracción de hemocultivo indicaría antibioticoterapia empírica según los gérmenes
3. 3.
sospechados.
4. 4. Todas las opciones son correctas.
Gráfico de respuestas
Comentario

Como tratamiento realizaremos reposo abdominal absoluto, con administración de líquidos vía
intravenosa, sonda nasogástrica para retirada del alimento ingerido en las horas previas que puede
agravar la isquemia y producir translocación bacteriana, y antibioticoterapia de amplio espectro.
Evidentemente es un cuadro que precisa de estudio y seguimiento estrecho. Recuerden las
indicaciones de cirugía: perforación con neumoperitoneo o sepsis refractaria a tratamiento
médico.(R1)

87. Un día en que la presión atmosférica es de 705 mmHg, un paciente de 40 años se


presenta en la sala de Urgencias con una presión arterial de oxígeno de 37 mmHg, una
presión de anhídrido carbónico de 82 mmHg y un pH de 7,22. Calculamos la presión
alveolar de oxígeno, que resulta ser 39 mmHg. Entre las causas de insuficiencia
respiratoria mencionadas más abajo, ¿cuál será la más probable?

1. 1. Una embolia de pulmón.


2. 2. Tiene una crisis asmática grave.
3. 3. Una neumonía extensa.
4. 4. Una sobredosis de morfina.
Gráfico de respuestas
Comentario

Pregunta fácil (por ser ampliamente explicada en clase) de un tema constante en el MIR.

Una vez más en el MIR nos preguntan sobre las causas de hipoxemia y el modo de diferenciarlas.
Se considera hipoxemia cuando la PaO2 es menor de 80 mmHg e hipercapnia cuando la PaCO2
es mayor de 45 mmHg. La insuficiencia respiratoria se define con una PaO2 menor de 60 mmHg y
es global si, además, añade una PaCO2 mayor de 50 mmHg. Existen varios mecanismos que
ocasionan disminución del O2 en la sangre, que pueden diferenciarse según el valor de la PaCO2,
de la D(A-a) y la respuesta al tratamiento con oxígeno suplementario. (ver los esquemas de la
página siguiente).

En nuestra pregunta, los datos que nos sirven para llegar a la causa más probable de hipoxemia
del paciente son:

•! PaO2 de 37 mmHg (por tanto, está en insuficiencia respiratoria).


•! PaCO2 de 82 mmHg (es decir, tiene retención de carbónico, así que la insuficiencia
respiratoria es global).
•! D(A-a)O2 = 39 mmHg – 37 mmHg = 2 mmHg (normal <15 mmHg).

!
!
!
!
Tenemos, en consecuencia, un paciente que está hipoventilando y cuya D(A-a)O2 es normal (la
causa no está en el pulmón). La única opción de las que nos ofrecen que concuerda con estos
datos es la 5: sobredosis de morfina (produciría hipoventilación por alteración del centro
respiratorio). En el caso de un EPOC reagudizado (opción 2) o una crisis asmática grave (opción 3)
podríamos encontrar retención de carbónico, pero la D(A-a)O2 estaría elevada (el mecanismo
responsable sería la alteración en la relación V/Q).

En el caso de la embolia de pulmón (opción 1), el mecanismo responsable también es la alteración


V/Q, pero la PCO2 estaría disminuida en lugar de aumentada por la hiperventilación del paciente.
Por último, en la neumonía (opción 4) lo que da lugar a hipoxemia es un mecanismo de shunt y
encontraríamos una PCO2 disminuida por la hiperventilación y una D(A-a)O2 aumentada.(R4)

Algoritmo diagnóstico de la hipoxemia

!
!
!
!

88. ¿Cuál es la deficiencia enzimática responsable de la forma clásica de galactosemia?:

1. 1. Galactosa 1 fosfato uridiltransferasa.


2. 2. Uridil difosfato galactosa 4-epimerasa.
3. 3. Glucosa 6-fosfato deshidrogenasa.
4. 4. Glucuroniltransferasa.
Gráfico de respuestas
Comentario

Pregunta dificil. La respuesta es la 1 la galactosa 1 fosfato uridiltransferasa

La galactosemia es una enfermedad hereditaria causada por una deficiencia enzimática y se


manifiesta con incapacidad de utilizar el azúcar simple galactosa, lo cual provoca una acumulación
de éste dentro del organismo, produciendo lesiones en el hígado y el sistema nervioso central.

Durante la digestión de la lactosa, la enzima lactasa degrada la molécula en glucosa y galactosa.


Los individuos que padecen galactosemia, tienen niveles muy bajos o ausencia completa de las
enzimas necesarias para la posterior metabolización de la galactosa, lo que conlleva la
acumulación de galactosa 1- fosfato en diversos tejidos. Esta acumulación genera niveles tóxicos
de galactosa que, tal y como sucede en el tipo clásico de este trastorno, pueden provocar
hepatomegalia, cirrosis, fallo renal, cataratas, daños cerebrales y, en mujeres, disfunción ovárica.
El tratamiento de esta enfermedad es crucial puesto que la mortalidad en niños con galactosemia
sin tratamiento es de un 75%.(R1)

89. Un paciente intervenido por úlcera duodenal con una antrectomía y


gastroyeyunostomía tipo Billroth II presenta una recaída ulcerosa. La gastrina sérica está
elevada basalmente y aumenta de forma importante tras inyección de secretina. ¿Qué
sospecharía?

1.! ! 1.! Adenocarcinoma!gástrico.!


2.! ! 2.! Antro!retenido.!
3.! ! 3.! Gastrinoma.!
4.! ! 4.! Síndrome!de!asa!aferente.!
Gráfico de respuestas
Comentario

En un paciente sano, la respuesta normal consiste en que la gastrina se inhibe cuando


administramos secretina. Sin embargo, en los que padecen un gastrinoma, se produce una
respuesta paradójica que es precisamente lo contrario: se ELEVA la gastrina cuando
administramos la secretina, que normalmente tendría que inhibirla. Cuando vea este tipo de
respuesta, no lo dude, es prácticamente seguro que está ante un gastrinoma (respuesta 3
correcta). En este caso clínico, ya se puede imaginar el motivo de su recidiva ulcerosa…(R3)

!
!
!
!
90. En la infección congénita sintomática por CMV suelen encontrarse los siguientes
hallazgos, EXCEPTO:

1. 1. Coriorretinitis.
2. 2. Calcificaciones intracraneales difusas.
3. 3. Hepatoesplenomegalia.
4. 4. Neumonitis.
Gráfico de respuestas
Comentario

En la infección connatal por CMV puede aparecer una tríada (microcefalia, coriorretinitis,
calcificaciones periventriculares), hipoacusia neurosensorial, neumonitis, hepatomegalia, anemia y
trombopenia. Recuerde que en la infección connatal por Toxoplasma aparecen también
calificaciones intracraneales, pero en este caso están dispersas, y no sólo concentradas alrededor
de los ventrículos.(R2)

91. Paciente de 32 años que consulta por fiebre y artralgias de 1 mes de evolución. A la
exploración física destaca hepatomegalia homogénea de 8 cm. En los exámenes de
laboratorio: 5,300 leucocitos con fórmula normal, GOT 260 uI/L, GPT 300 uI/L, GGT 320
uI/L, Bb 1.8 g/L. En la ecografia abdominal se observa hepatomegalia. Las serologías
fueron: Ag S+, IgM core-, IgG core +, DNA VHB +, Ag e -, anti e+, anti VHC -. ¿Propondría
como la siguiente medida en el manejo de este paciente?:

1. 1. Biopsia hepática.
2. 2. Dosis altas de interferón.
3. 3. Repetir la determinación de Ag e.
4. 4. Solicitar alfafetoproteina.
Gráfico de respuestas
Comentario
Se trata de un paciente con hepatitis crónica B (AgHBs + y anti- HBc de tipo IgG) que está
replicando con AgHBe- y por tanto, cepa mutante precore. Otro dato a tener en cuenta es que tiene
elevación de transaminasas, por lo que podríamos tratarle. Sin embargo, antes de empezar a tratar
con Lamivudina o Adefovir, estaría indicado realizar una biopsia hepática para valorar la afectación
histológica ya que, si se encontrara una histología de cambios mínimos o cirrosis establecida no
estaría indicado tratarla por su escasa repuesta.(R1)

92. Drenaje de líquido sanguinolento por el pezón es característico de:

1. 1. Ginecomastia.
2. 2. Papiloma intraductal.
3. 3. Necrosis grasa de mama.
4. 4. Ninguna de las anteriores.
Gráfico de respuestas
Comentario

Concepto poco preguntado en el ENARM. La secreción serosanguinolenta y hemorrágia cuando es


unilateral o uniorificial se debe al papiloma intraductal, pero también puede observarse en
carcinomas, enfermedad fibroquística o dilatación avanzada de los conductos. Respuesta 2
correcta.

!
!
!
!

93. ¿Qué alteración hematológica se asocia característicamente a la fibrosis


endomiocárdica de Loeffler?:

1. 1. Anemia microcítica.
2. 2. Neutropenia.
3. 3. Reducción del tiempo de protrombina.
4. 4. Eosinofilia.
Gráfico de respuestas
Comentario

La fibrosis endomiocárdica eosinofílica (de Loeffler) es una forma específica de miocardiopatía


restrictiva. Parece ser una forma específica del síndrome hipereosinofílico, con especial daño del
miocardio. Además de la clínica de congestión venosa y pulmonar, en esta enfermedad es muy
frecuente la formación de trombos murales y por lo tanto la existencia de embolismos sistémicos.
La alteración hematológica característica es la eosinofilia.(R4)

94. Una mujer de 40 años nos consulta porque desde hace 9 meses tiene las mejillas
enrojecidas y le salen "granos en la cara". En la exploración física se observa unas placas
eritematosas en mejillas con numerosas pápulas y pústulas. No hay comedones. La
paciente respondió muy bien al metronidazol tópico y a la doxiciclina vía oral. ¿Cuál es
su diagnóstico?:

1. 1. Acné vulgar.
2. 2. Querion.
3. 3. Rosácea.
4. 4. Foliculitis.
Gráfico de respuestas
Comentario

La rosácea es una enfermedad crónica de patogenia desconocida que afecta más frecuentemente
a mujeres de edad media. Aunque la etiología es desconocida, se han implicado diversos factores
como son la infección por Demodex folliculorum o H. pylori, labilidad vasomotora, factores
genéticos e infecciones gastrointestinales. La clínica comienza con episodios de flushing facial en
respuesta a ciertos estímulos (comidas picantes o calientes, alcohol, sol, aumento de temperatura
etc.). Progresivamente aparece eritema persistente así como telangiectasias y papulopústulas. En
las formas más severas pueden aparecer forúnculos, nódulos quísticos, abscesos y cicatrización
queloidea. Sin embargo en ningún momento de la evolución de la enfermedad se forman
comedones. Otras complicaciones que pueden observarse son las lesiones oftálmicas
(conjuntivitis, blefaritis, iritis) e hiperplasia de partes blandas, más frecuente en varones, como son
el rinofima, otofima y blefarofima. El tratamiento de las formas leves se realiza con metronidazol
tópico. Las formas moderadas o leves que no respondan al tratamiento tópico se tratan de elección
con tetraciclinas orales y las formas severas con retinoides sistémicos. Es importante evitar
aquellos estímulos que puedan empeorar o desencadenar las lesiones.

El acné vulgar es una enfermedad que aparece sobretodo en adolescentes, se localiza no solo en
mejillas sino también en frente y barbilla y es muy típica la presencia de comedones.

El lupus discoide presenta de forma característica atrofia y lesiones cicatriciales.

!
!
!
!
El querion es una tiña inflamatoria que se manifiesta como una placa alopécica, supurativa, en el
cuero cabelludo.

La foliculitis aparece como pústulas foliculares y aparece en otras localizaciones como pestañas,
axilas, pubis, muslos.(R3)

95. EXCEPTO una, todas las eventualidades siguientes predisponen a los enfermos a un
mayor riesgo de aparición de un linfoma maligno:

1. 1. Síndrome de Chédiak-Higashi.
2. 2. SIDA.
3. 3. Esclerosis múltiple.
4. 4. Tratamiento con difenilhidantoína.
Gráfico de respuestas
Comentario

Pregunta de dificultad media- alta. Algunas de las opciones que ofrece debería conocerla. La
frecuencia de linfomas en el paciente con SIDA está muy aumentada respecto a la población
general. De hecho, el cáncer más frecuente en el VIH es, actualmente, el linfoma. Respecto al
síndrome de Chédiak- Higashi, no es necesario que lo conozca en profundidad. Como regla
general, debe recordar que la mayoría de las inmunodeficiencias, sean congénitas o adquiridas,
aumentan el riesgo de linfoma. Lo mismo sucede con algunas enfermedades autoinmunes, como
la enfermedad celíaca y la artritis reumatoide. En este último caso, suele tratarse de linfomas B de
célula grande. Sin embargo, en la esclerosis múltiple no está descrita esta complicación.

Respecto a la opción 4, recuerde que la fenitoína puede producir linfadenopatía, sin que
necesariamente se trate de un linfoma (pseudolinfoma por fenitoína). Sin embargo, se han descrito
unos pocos casos en los que se trataba de un auténtico linfoma, aunque esta complicación es
absolutamente excepcional.(R3)

96. Mujer de 37 años que acude a su consulta solicitando método anticonceptivo (no
desea esterilización ). Antecedentes personales: enfermedades propias de la infancia. No
refiere intervenciones quirúrgicas. No alergias medicamentosas conocidas. Fumadora de
20 cigarrillos/ día. No consumo de alcohol ni de drogas. Antecedentes ginecoobstétricos:
3 partos eutócicos; lactancia materna. Tiempo medio de menstruación: 4 / 28 días. No
dismenorrea. Revisiones periódicas anuales. Ultima hace 3 meses con citología normal.
Exploración: genitales externos y vagina normales; cérvix de multípara bien epitelizado;
útero de tamaño y forma normal; anexos no se palpan patológicos. USG: útero en
anteversión, regular, histerometría 75mm; anexos normales. ¿Qué método
anticonceptivo aconsejaría?

1. 1. Anticonceptivos orales combinados.


2. 2. DIU.
3. 3. Diafragma.
4. 4. Minipíldora de gestágenos.
Gráfico de respuestas
Comentario

En una paciente fumadora de más de 35 años, la anticoncepción hormonal sería una


contraindicación absoluta, debido al riesgo cardiovascular, con lo que las opciones 1 y 4 no serían
planteables. En cuando al diafragma, recuerde que debe utilizarse junto con una crema

!
!
!
!
espermicida y su eficacia está por debajo de la del DIU. Por lo tanto, tampoco sería la mejor opción
para esta paciente. En caso de que éste no pueda utilizarse, tal vez sería planteable, pero no como
primera opción.

Con respecto al DIU, sería interesante que recordase las siguientes ideas:

- Mecanismo de acción: reacción inflamatoria endometrial que evita la implantación.

- Está contraindicado ante la sospecha de embarazo, infección pélvica aguda o reciente,


sangrado uterino anormal, tratamiento con anticoagulantes o tumores cervicales o uterinos de
cualquier tipo.

- El momento habitual de colocación es durante la menstruación.

- El DIU previene mejor el embarazo normal que el ectópico, por lo que aumenta la frecuencia
relativa de éste último.

- El DIU es un factor de riesgo para enfermedad inflamatoria pélvica, al poner en contacto una
cavidad séptica (la vagina) con el interior del útero.(R2)

97. Actualmente no existe una vacuna frente al virus respiratorio sincitial (VRS), principal
causante de bronquiolitis aguda en niños pequeños. ¿Cuál de los siguientes anticuerpos
se utiliza como inmunoterapia pasiva para prevenir la infección y reducir su gravedad?

1. 1. Abciximab.
2. 2. Basiliximab.
3. 3. Rituximab.
4. 4. Palivizumab.
Gráfico de respuestas
Comentario

Los anticuerpos monoclonales son uno de los últimos avances en terapeútica con cada vez más
indicaciones.

El abciximab es un antiagregante que inhibe el receptor plaquetario glicoproteína IIb/IIIa.

El basiliximab es un inmunosupresor al inhibir el CD25, receptor del la IL- 2 básico para la


expansión de los linfocitos T.

El rituximab es un anti-CD20, presente en los linfocitos B, útil en el tratamiento de leucemias,


linfomas y enfermedades autoinmunes.

La respuesta correcta es la 4, el Palivizumab. Concede inmunidad pasiva frente al virus respiratorio


sincitial (VRS). Ha demostrado reducir la hospitalización de los niños de riesgo por bronquiolitis.
Está indicado en sujetos de riesgo: prematuros, displasia broncopulmonar, inmunodeficiencias y
cardiopatías hemodinámicamente significativas. Se administra de forma intramuscular
mensualmente durante la temporada de VRS.(R4)

98. Pre-escolar de 2 años de edad, ingresa al servicio de urgencias con signos de


deshidratación severa, se intenta canalización venosa periférica sin éxito en varias

!
!
!
!
oportunidades, por lo que se decide colocar catéter venoso central con abordaje por la
vena yugular interna, iniciándose por esta vía hidratación inicial con cloruro de sodio en
bolo, mejorando la intensidad de los ruidos cardiacos al segundo bolo. Sin embargo el
paciente se pone polipneico, pálido, con saturación de 50%, politirajes. El diagnóstico
probable es:

1. 1. Embolismo aéreo.
2. 2. Arritmia.
3. 3. Neumotórax.
4. 4. Perforación de vaso sanguíneo.
Gráfico de respuestas
Comentario

Neumotorax: la mayoría se deben a la ruptura de las bullas subpleurales aunque el atrapamiento


aéreo secundario a tapones de moco y la inflamación también juegan su papel. EN este caso fué
secundario a la colocación del acceso venoso central.

El tratamiento depende de la sintomatología y del tamaño del neumotorax, así se emplean los
siguientes:

- Asintomático y con tamaño menor del 20% del hemitórax: oxigenoterapia y reposo.

- Sintomático o tamaño mayor del 20% del hemitórax: drenaje torácico.(R3)

99. Las vasculitis cutáneas comprenden un espectro de síndromes clínicos y hallazgos


histológicos que presentan como hecho común la inflamación de la pared vascular. ¿Cuál
de las siguientes es CORRECTA acerca de las manifestaciones cutáneas de estos
procesos?:

La forma clínica más frecuente es la púrpura palpable, que a medida que evoluciona toma
1. 1.
un tono violáceo hasta que desaparece, dejando a menudo hipopigmentación residual.
2. 2. A veces, las vasculitis provocan la aparición de úlceras y zonas necróticas.
3. 3. Son lesiones asintomáticas siempre.
Pueden aparecer lesiones maculosas, vesiculosas, urticarianas y úlceras, pero estas últimas
4. 4.
siempre por confluencia o complicación de otras iniciales.
Gráfico de respuestas
Comentario

Una pregunta de dificultad media sobre las vasculitis. Analicemos las distintas respuestas:

- Las vasculitis suelen producir hiperpigmentación residual, no hipopigmentación (respuesta 1


falsa).

- Las lesiones secundarias a vasculitis pueden producir dolor como consecuencia de la isquemia
cutánea, por lo que no es obligado que sean asintomáticas (respuesta 3 falsa).

- Las úlceras vasculíticas no siempre aparecen por confluencia ni complicando otras úlceras
previas (respuesta 4 falsa).

!
!
!
!
Lógicamente, la respuesta 2 es correcta, ya que la inflamación de la pared de un vaso puede ser
causa de isquemia tisular; de ahí que puedan aparecer lesiones ulcerosas o necróticas.(R2)

100. La bronquiolitis es una infección viral de la vía respiratoria inferior que produce
ingreso hospitalario hasta en el 3.5% de los recién nacidos. Señale la respuesta FALSA
en relación a su clínica:

1. 1. El cuadro clínico se inicia con un catarro de vías altas que puede durar dos o tres días.
2. 2. Esa fase suele durar de dos a tres días, tras los cuales comienza una mejoría progresiva.
Algunos niños presentan vómitos tras la tos o dificultad para la toma de alimento, por lo
3. 3.
que pueden llegar a ingresar.
Aproximadamente tras una semana del comienzo del cuadro se produce la desaparición de
4. 4.
toda la sintomatología.
Gráfico de respuestas
Comentario

Esta pregunta hace referencia a la historia natural de la bronquilitis, enfermedad que debemos
saber reconocer en una caso clínico.

Es causada por el VSR el cuadro clínico se inicia con un catarro de vías altas que puede durar dos
o tres días. Posteriormente asocia afectación de la vía inferior con tos, taquipnea, fiebre y signos
de dificultad respiratoria.

La dificultad respiratoria alcanza su máxima gravedad a las 24-48 h (cuando se producen la


mayoría de los ingresos) y después mejora gradualmente. En las bronquiolitis moderadas-graves la
tos es intensa y con frecuencia emetizante. El pronostico inmediato de la mayoría de los niños es
excelente y los síntomas se resuelven en una semana. Sin embargo, algunos niños pueden
presentar posteriormente nuevos episodios de sibilancias, niveles de función pulmonar más bajos e
hiperreactividad bronquial residual.

No es típica una tos emetizante en una bronquiolitis por virus sincitial respiratorio. Si nos pusieran
este cuadro clínico, deberíamos sospechar infección por Bordetella pertusis.(R4)

101. A 5-year-old child is brought to the pediatric cardiologist referred by the school
doctor presenting with a murmur. Cardiac auscultation shows the presence of a soft (3/6)
systolic murmur that is heard in supine position but disappears when the child stands or
sits up. What should be done next?

1. 1. Perform an echocardiogram.
2. 2. Perform a chest X ray.
3. 3. Periodic observation.
4. 4. Perform an ECG.
Gráfico de respuestas
Comentario
Periodic observation. An innocent or functional murmur is a heart sound that appears without
structural heart defects. Innocent murmurs are common in childhood. They murmurs are systolic,
have a soft intensity and change in different body positions.(R3)

102. Which of the following is not an immunopathological mechanism of vasculitis?

!
!
!
!
1. 1. Formation of immune complexes in the vessel wall.
2. 2. Deposition of circulating immune complexes in the vessel wall.
3. 3. Association with HLA-DR4.
4. 4. Changes and mechanisms induced by Cytokines (IL-1, TNF-alpha).
Gráfico de respuestas
Comentario

Concepto poco preguntado en el nacional. En general se piensa que las vasculitis están mediadas,
al menos en parte por mecanismos inmunitarios, siendo el mecanismo más comunmente admitido,
el depósito de de complejos inmunes en las paredes de los vasos, aunque no está demostrado
claramente el papel causal de los complejos inmunes. Existen otros mecanismos como la
hipersensiblidad retardada y la lesión medida por células como lo demuestra la formación de
granulomas, aunque los propios complejos inmunes pueden causar respuesta granulomatosa.
Existen mecanismos inducidos por citoquinas, fundamentalmente IL-1 e IFN alfa que producen
daño en la pared vascular como favorecer la adhesión de los leucocitos a la pared vascular. Se ha
demostrado también la formación in situ de inmunocomplejos como posible mecanismo
patogénico.(R3)

103. Which of
the following is the best therapy option for this patient?

1. 1. Liver transplantation.
2. 2. Radiotherapy.
3. 3. Radiofrequency ablation.
4. 4. Arterial embolization.
Gráfico de respuestas
Comentario

Aunque en la imagen que nos aportan no nos indican el tamaño exacto del tumor, parece que se
trata de una lesión única, no muy grande situada en lóbulo hepático derecho. Podríamos incluirlo
dentro del grupo de pacientes pertenecientes a un estadio inicial o A (1 nódulo < 5cm ó 3 nódulos <
3cm), donde se puede plantear una opción terapéutica con intención curativa. Al presentar
hepatopatía de base descartamos la resección quirúrgica de entrada y ofrecemos al paciente, por
tener mayor éxito, la opción del trasplante hepático, que nos permite tratar el hepatocarcinoma y su
hepatopatía de base. La ablación con radiofrecuencia la reservaríamos en este paciente para el
supuesto en el que no fuera posible la cirugía (nos hablarían de un elevado riesgo quirúrgico, ASA
IV ó V) o para un paciente en el que el tiempo de espera del trasplante fuera prolongado, aspectos
que no mencionan en el enunciado. La embolización o la quimioterapia se ofertan como

!
!
!
!
tratamientos paliativos en el hepatocarcinoma irresecable.(R1)

104. A 55 year-old man, smoker and ex-alcoholic. His medical history is remarkable for
upper gastrointestinal bleeding and cirrhosis (HCV+). Child-Pugh score of 6. Image shows
two axial sections corresponding on arterial and late phases of CT scan done to the
patient. Mark the correct answer according to the CT scan.

A focal hypervascular lesion in arterial phase, probably a hepatic artery pseudoaneurysm, is


1. 1.
observed.
An hypervascular node in arterial phase with wash out in the venous phase, probably a
2. 2.
hepatocellular carcinoma, is observed.
3. 3. A focal hypovascular lesion, probably with peripheral cholangiocarcinoma is observed.
4. 4. It is a cystic lesion, probably an hydatid cyst.
Gráfico de respuestas
Comentario

Cuando nos planteamos el diagnóstico diferencial de una lesión hepática focal debemos considerar
el paciente del que partimos, diferenciando entre tres contextos fundamentales: a. Lesión hepática
incidental, sobre paciente sano: considerar primero el hemangioma. b. Lesión hepática sólida,
sobre paciente con antecedente de cualquier cáncer: considerar como primera opción las
metástasis. c. Y como en el caso que nos plantean, lesión hepática focal sobre paciente con
hepatopatía con marcadores virales positivos o alfa-fetoproteina elevada, debemos pensar en el
hepatocarcinoma como primera opción. El estudio hepático se hace con TAC helicoidal con
contraste lo que nos permite obtener imágenes en distintos momentos: fase arterial, fase portal y
de lavado. Característicamente el hepatocarcinoma presenta una imagen hipervascular en fase
arterial que se hace rápidamente iso o hipovascular en fase portal (efecto denominado lavado o
wash out). No obstante, si situamos al paciente en uno de los contextos mencionados previamente
podemos llegar a la respuesta sin necesidad de "conocer exactamente" cual es la imagen
característica del TAC.(R2)

105. El síntoma más frecuente de la otomicosis es:

1. 1. Hipoacusia.
2. 2. Otorrea.
3. 3. Otorragia.
4. 4. Prurito.
Gráfico de respuestas
Comentario

La otomicosis o infección por hongos del conducto auditivo externo se caracteriza principalmente
por el prurito ótico. Aparece generalmente como una sobreinfección tras un tratamiento con
antibióticos tópicos y se debe pensar en ella siempre que una infección no mejore con terapia
antibiótica. Puede presentar un exudado algodonoso (candidas), en ocasiones de color gris
negruzco (Aspergillus). La obstrucción del CAE por este exudado puede producir hipoacusia de
transmisión.(R4)

106. Cuál de los siguientes fármacos NO se utiliza como antiarrítmico:

1. 1. Tolazolina.
2. 2. Ajmalina.
3. 3. Propafenona.

!
!
!
!
4. 4. Procainamida.
Gráfico de respuestas
Comentario

Es una pregunta algo difícil en la que no es necesario perder mucho tiempo. La tolazalina es un
antagonista alfa adrenérgico no selectivo de características similares a la fentolamina (salvo que es
menos potente que esta) que no posee utilidad como fármaco antiarrítmico, si no que se usa con
poca frecuencia y en otro tipo de procesos como la HTA. La procainamida está dentro del I a,
aprindina pertenece al grupo I b y la propafenona se sitúa en el I c.(R1)

107. Mark the incorrect answer concerning intestinal vascular disorders:

Acute mesenteric ischemia (AMI) should be suspected when individuals, especially those at
high risk for acute mesenteric ischemia (for instance: past medical history significant for
1. 1.
cardiac ischemic disease), develop severe and persisting abdominal pain that is
disproportionate to their abdominal findings.
Splenic artery aneurysms are the most common intra-abdominal aneurysms and have a low
2. 2.
rupture risk.
3. 3. The most common cause of acute mesenteric ischemia is arterial embolism.
4. 4. Treatment for AMI includes embolectomy or fibrinolysis.
Gráfico de respuestas
Comentario

1.- Esta opción hace referencia a una patología muy preguntada, la embolia mesentérica. Suelen
presentarla como un anciano con fibrilación auricular e intenso dolor abdominal. Recordemos que
la exploración física suele ser normal al principio. Pasadas unas horas, cuando el intestino se
necrose y perfore, aparecerían signos de irritación peritoneal.

2.- Los aneurismas de la arteria esplénica tienen gran tendencia a la rotura espontánea,
especialmente durante el embarazo.

3.- La opción 3 es correcta: la causa más frecuente de insuficiencia mesentérica aguda es la


embolia. Por eso se relaciona con la fibrilación auricular, que es de donde proceden los émbolos.

4.- El tratamiento de elección de la embolia mesentérica es la embolectomía (no el by-pass ni la


resección con anastomosis término-terminal). También es posible emplear fibrinolíticos, tratando de
lisar el émbolo, pero la embolectomía con sonda de Fogarty es mucho más usada.(R2)

108. A 55-year-old woman comes to her gynecologist for her annual follow-up visit. She
is a heavy smoker since she was 30. On physical examination, a 1.5-cm-diameter lump is
discovered in the upper outer quadrant of her left breast. The lump is tender, as well as
the whole left breast. No axillary or supraclavicular lymph nodes are palpated.
Mammography shows the existence of a nodule with features of malignancy.
Intraoperative biopsy is reported as infiltrating ductal carcinoma and the resected nodule
has tumor-free margins. Which of the following is the most appropriate next step in
management?

1. 1. Radical mastectomy, because the patient has poor prognostic factors, such as her age.
2. 2. Tumorectomy, lymphadenectomy and radiation therapy given to the surgical bed.
3. 3. Subcutaneous mastectomy, lymphadenectomy and letrozole.
4. 4. Chemotherapy, radiation therapy, postoperative hormone therapy (tamoxifen).

!
!
!
!
Gráfico de respuestas
Comentario

Esta pregunta es fácil si recordamos el algoritmo de tratamiento del cáncer de mama. Cuando una
mujer presenta un nódulo palpable en la mama, si además la mastografía tiene signos
sospechosos de malignidad, debemos realizar una biopsia de la tumoración, ya sea en consulta
con una PAAF, o en quirófano mediante una biopsia diferida. Como el diagnóstico es de un
carcinoma infiltrante, es obligado en la cirugía incluir siempre la axila ya sea mediante
linfadenectomía axilar ipsilateral o bien con la biopsia del ganglio centinela. Esta última permite, si
es negativo, conservar el tejido ganglionar. En este caso, además, al ser una tumoración pequeña
y la biopsia incluir toda la pieza tumoral con márgenes quirúrgicos libres, no es necesario realizar
mastectomía, pero sí será obligado añadir radioterapia en el lecho quirúrgico de la mama (ya que
la tumorectomía es una cirugía conservadora), por lo que la respuesta correcta es la 2.(R2)

109. Paciente de 75 años, diagnosticado de enfermedad de Parkinson hace 5 años y


desde entonces en tratamiento con L-Dopa. Se encuentra en un estadio II-III de Hoehn y
Yahr. Comienza a presentar alucinaciones visuales y auditivas. ¿Cuál sería la primera
medida terapéutica a realizar?:

1. 1. Disminuir la dosis de L-dopa.


2. 2. Asociar clozapina.
3. 3. Añadir anticolinérgicos.
4. 4. Comenzar tratamiento con haloperidol.
Gráfico de respuestas
Comentario

Pregunta difícil sobre un tema importante. Se nos presenta un paciente diagnosticado de


enfermedad de Parkinson en tratamiento con L-Dopa que presenta alucinaciones. Lo más probable
es que el paciente esté intoxicado, ya que este efecto es dosis dependiente, por lo que se debe
disminuir la dosis de L-Dopa.

Recuerde que la enfermedad por cuerpos de Lewy se caracteriza por la existencia de


parkinsonismo y alucinaciones y responde bien a fármacos anticolinérgicos.(R1)

110. ¿Cuál de las siguientes características es más útil para establecer la diferencia entre
la rubeóla y el exantema súbito?

1. 1. Edad de presentación.
2. 2. Adenopatías.
3. 3. Tipo de exantema.
4. 4. Biometría hemática.
Gráfico de respuestas
Comentario

Tema muy importante para el nacional. Para el diagnóstico diferencial entre ambas entidades:

Rubeola (tres días): Es rojo, morbiliforme y confluente en la cara. Se resuelve mediante una
mínima descamación.

!
!
!
!
Exantema súbito o 6ª enfermedad (24 horas): de forma brusca, el niño se queda afebril y aparece
un exantema maculopapuloso rosado y poco confluente en el tronco y extremidades superiores. Es
rara la afectación facial y distal de los miembros.(R3)

111. Factor más frecuente de Infertilidad:

1. 1. Factor tubo peritoneal.


2. 2. Factor masculino.
3. 3. Factor ovario.
4. 4. Factor cervical.
Gráfico de respuestas
Comentario

La infertilidad está causada:

-En un tercio de los casos por el hombre

-En otro tercio por la mujer

-En otro tercio la causa es mixta.

De las cuatro opciones, la respuesta correcta es la 2, pues como hemos dicho, el factor masculino
es responsable de un tercio de los casos de infertilidad.

Las demás opciones son causas de infertilidad en la mujer. Recordemos que sumando todas las
causas de infertilidad en la mujer, explicamos un tercio de los casos, así que cada una de estas
opciones es responsable de menos de un tercio de los casos.(R2)

112. La causa más frecuente de obstrucción intestinal en un paciente apendicectomizado


es:

1. 1. Hernia estrangulada.
2. 2. Divertículo de Meckel invaginado.
3. 3. Vólvulo de sigma.
4. 4. Presencia de adherencias y bridas.
Gráfico de respuestas
Comentario

Pregunta que se contesta de forma directa si sabe las dos principales causas de obstrucción
intestinal. Saber este tipo de datos es importante de cara al ENARM porque permite contestar
rápido y ganar tiempo. La causa más frecuente de obstrucción del ID son las adherencias,
seguidas de las hernias (que son la primera causa en pacientes sin cirugía previa). Ya de paso,
recuerde que la causa más frecuente de la obstrucción del intestino grueso es el cáncer colorrectal,
sobre todo a nivel de recto-sigmoides.(R4)

113. El trastorno convulsivo más frecuente en la infancia es:

1. 1. La convulsión febril.
2. 2. La epilepsia mioclónica juvenil.
3. 3. La crisis tónico-clónica generalizada

!
!
!
!
4. 4. El espasmo infantil.
Gráfico de respuestas
Comentario

Pregunta muy sencilla. La respuesta correcta es la 1, las convulsiones febriles.(R1)

114. Señale la respuesta INCORRECTA en relación con los pacientes ancianos con
deterioro cognitivo:

1. 1. La enfermedad de Alzheimer es la causa más frecuente de demencia.


La masa cerebral no es un buen indicador del grado de funcionalismo intelectual, por lo que
2. 2.
una atrofia cerebral generalizada en la TC no siempre se asociará a demencia.
En la enfermedad de Pick, las placas seniles y los ovillos neurofibrilares se distribuyen
3. 3.
preferentemente a nivel fronto-temporal.
4. 4. En un 25% de los casos de enfermedad de Alzheimer, existen antecedentes familiares.
Gráfico de respuestas
Comentario

La demencia de Pick se diferencia del Alzheimer en que aparecen en más jóvenes, no cursa con
amnesia, ni apraxias ni agnosias, pese a ser una demencia cortical; las alteraciones ocnductuales
y del lenguaje son más precoces y no aparecen ovillos, ni placas neuríticas.(R3)

115. Paciente masculino de 55 años presenta un bocio multinodular de larga evolución,


que ha crecido en los últimos 6 meses de forma global. A la exploración presenta un
bocio a expensas del lóbulo derecho con un nódulo predominante de 2 cms, con
extensión hacia región intratorácica. El paciente presenta dificultad para la deglución. El
paciente aporta varias PAAF realizadas hace 2 años que eran benignas. Se decide cirugía
por bocio multinodular compresivo, realizándose hemitiroidectomía derecha completa y
dejando tejido tiroideo en itsmo y lóbulo derecho, que eran de características normales.
El estudio histopatológico a posteriori del nódulo muestra un carcinoma papilar de
tiroides. ¿Cuál sería su actitud terapéutica?:

Nueva intervención, completando la tiroidectomía total y exploración de cadenas


1. 1.
ganglionares.
2. 2. No se requiere ampliar la cirugía, y se realizará seguimiento ecográfico periódico.
3. 3. Completar la tiroidectomía con vaciamiento ganglionar derecho cervical radical.
4. 4. Rastreo corporal para ver la cantidad de tejido tiroideo restante.
Gráfico de respuestas
Comentario

En este caso clínico se presenta a un paciente con antecedentes de bocio multinodular, que
presenta sintomatología compresiva, crecimiento intratorácico y aumento del tamaño tiroideo en los
últimos meses. Además en la exploración se aprecia la existencia de un nódulo predominante en el
lóbulo derecho. Debido a la existencia de síntomas compresivos está indicada la cirugía. En este
paciente, tras el estudio histológico, se aprecia la existencia de un carcinoma papilar tiroideo en el
nólulo del lóbulo derecho. La actitud correcta es ampliar la cirugía con una tiroidectomía total y la
exploración de las cadenas ganglionares. El vaciamiento ganglionar de inicio no está indicado. Si
se hubiera diagnosticado de un carcinoma papilar menor de 1 cm, no hubiera sido necesario
ampliar la cirugía, y el seguimiento se realizaría con ecografía cervical. El tratamiento con
radioyodo y el tratamiento supresor con levotiroxina estarán indicados pero después de ampliar la
cirugía.(R1)

!
!
!
!
116. Respecto a la incompatibilidad maternofetal en el sistema ABO de grupos
sanguíneos, es cierto que:

1. 1. Afecta a primogénitos.
2. 2. Es más grave que la isoinmunización D.
3. 3. La detección prenatal es muy importante porque se asocia a anemia fetal severa.
4. 4. Es una causa infrecuente de enfermedad hemolítica en el recién nacido.
Gráfico de respuestas
Comentario

Tema preguntado con frecuencia en el ENARM, no confunda la incopatibilidad RH con el ABO. La


incompatibilidad Rh requiere que una madre, Rh negativa, sea puesta en contacto previamente con
sangre Rh positiva. En cambio, la incompatibilidad ABO puede afectar al primer hijo sin necesidad
de sensibilización previa (respuesta 1 correcta). Esta circunstancia es más frecuente (respuesta 4
falsa), pero menos grave, que la incompatibilidad Rh, por lo que no se asocia a anemias fetales
severas (respuestas 2 y 3 falsas).

El test de Coombs indirecto es positivo y el directo también suele serlo, salvo en raras
excepciones.(R1)

Isoinmunización del recién nacido (RN)

!
!
!
!
117. Un hombre de 61 años, hipertenso bien controlado desde hace años y con DM tipo
2, acude a su consulta porque en una ecografía realizada para valorar su dispepsia
habitual se le ha detectado una masa renal. Se trata de una masa de 3 cm, bien definida,
con algunas irregularidades en la cara inferior de sus bordes que por lo demás son lisos,
de contenido hipoecoico homogéneo y con refuerzo ecográfico posterior. Su actitud será:

1. 1. Solicitar una TAC abdominal de forma inmediata.


2. 2. Solicitar una PAAF de la masa.
3. 3. Programar una nefrectomía parcial.
4. 4. Derivar al paciente a oncología.
Gráfico de respuestas
Comentario
Se trata de una masa renal compleja, ya que no cumple estrictamente los criterios de quiste simple
por no tener unos bordes completamente lisos, así que hay que estudiarla más en profundidad con
una TAC abdominal para poder descartar la presencia de un adenocarcinoma.(R1)

118. Señale cuál de las siguientes anemias macrocíticas NO muestra rasgos


megaloblásticos en la médula ósea:

1. 1. Anemia de la enfermedad de Biermer.


2. 2. Infestación por Diphylobotrium latum.
3. 3. Anemia del hipotiroidismo.
4. 4. Anemia asociada a la toma de metotrexato o de trimetropim.
Gráfico de respuestas
Comentario

Causas de anemia megaloblásticas.

Recuerde que todas las anemias megaloblásticas son macrocíticas, pero hay otras anemias
macrocíticas que no son megaloblásticas como, por ejemplo, el hipotiroidismo o las anemias
refractarias, las secundarias a tratamiento quimioterápico, etc. El resto de las respuestas hacen
referencia a patologías que causan un déficit de vitamina B12 y, por tanto, generan una anemia
megaloblástica. Un dato curioso es que la anemia perniciosa también se llama enfermedad de
Biermer.(R3)

119. Con relación a sepsis neonatal, marque el enunciado INCORRECTO:

1. 1. Son factores de riesgo: prematuridad, corioamnionitis y ruptura de membranas.


2. 2. Los gérmenes para sepsis neonatal temprana son los organismos del tracto genital materno.
3. 3. La presentación de la sepsis tardía es generalmente fulminante, multisistemica.
Se debe realizar cultivo de liquido cefalorraquídeo en todo paciente que se considere
4. 4.
séptico.
Gráfico de respuestas
Comentario

Factores de riesgo para la sepsis neonatal son:

- prematuridad

- corioamnionitis

!
!
!
!
- ruptura de membranas

Suelen ser organismos del tracto genital materno y la presentación de la sepsis neonatal precoz
suele ser multisistémica, mientras que la tardía suele ser focal (meningitis...)

Siempre deberemos solicitar cultivo del liquido cefalorraquideo en todos los neonatos que
consideremos sépticos.(R3)

120. Niña de 1 año con fiebre de


hasta 39 ºC de 2 días de evolución. No presenta a la exploración foco infeccioso claro
para la fiebre, salvo faringe algo hiperémica. Se toman muestra de orina mediante bolsa
colectora detectándose nitritos y leucocitos mediante tira reactiva, por lo que se procede
a toma de orina estéril mediante punción suprapúbica y la tinción de Gram de dicha orina
muestra más de 1 bacteria por campo de gran aumento. En los estudios de laboratorio
destaca leucocitosis con neutrofilia así como elevación importante de la PCR. Tras el
diagnóstico y el tratamiento adecuado para este caso, la niña es estudiada en la consulta
de nefrología donde se le decide practicar una cistoureterografía miccional seriada
(CUMS) que se muestra en la imagen. Respecto al cuadro clínico citado señale la
respuesta FALSA:

1. 1. La infección urinaria es la causa más frecuente de fiebre sin foco en menores de 1 año.
2. 2. La imagen muestra reflujo vesico-ureteral severo.
3. 3. Para el diagnóstico de infección urinaria es necesario un cultivo positivo de orina.
El tratamiento de elección en estos casos con sospecha de infección urinaria con fiebre debe
4. 4.
ser siempre con antibióticos intravenosos de entrada.
Gráfico de respuestas
Comentario

!
!
!
!
La sola sospecha de infección urinaria no justifica la administración de antibióticos en todos los
casos. Cuando éstos se utilizan, tampoco tienen por qué ser necesariamente intravenosos
(depende de la gravedad y de la edad del paciente). Una respuesta como la 4, aunque sólo fuera
por lo radical de su expresión, debería hacerle desconfiar muy seriamente. La administración de
antibióticos intravenosos queda reservados para los casos más graves, además, lo más adecuado
es confirmar la infección.(R4)

121. La niña de la pregunta anterior fue diagnosticada finalmente de infección urinaria al


presentar un urocultivo con >1000 UFC/ml de E. coli que respondió bien al tratamiento
antibiótico. En relación al manejo adecuado de este tipo de pacientes tras la resolución
del proceso infeccioso, señale la opción FALSA:

Se recomienda realizar ultrasonido renal y vesical tras un episodio de IVU febril para
1. 1.
descartar hidronefrosis y abscesos renales.
La CUMS no sólo permite el diagnóstico del reflujo vesico-ureteral sino que lo clasifica en
2. 2.
grados que pueden determinar la actitud terapéutica más adecuada.
La gammagrafía renal con DMSA es la prueba más sensible para el diagnóstico de
3. 3.
cicatrices renales tras una pielonefritis.
El reflujo vesico-ureteral primario se debe a una anomalía congénita en la unión urétero-
4. 4. vesical, mientras que el secundario, mucho más frecuente que el primario, puede tener
múltiples etiologías.
Gráfico de respuestas
Comentario

Para el diagnóstico de IVU, se precisan más de 100,000 UFC, pero en este caso nos bastaría con
una sola, ya que el método de recolección de orina fué a través de punción suprapúbica. Una vez
resuelto el caso, debemos plantearnos si existe una anomalía morfológica que lo justifique, por lo
que realizaremos un ultrasonido renal y vesical, para empezar el estudio. También se utilizará la
cistoureterografía miccional seriada (CUMS) para ver el grado de reflujo y elegir el tratamiento más
adecuado. La respuesta 4 es falsa por un pequeño detalle: el RVU es más frecuente en su forma
primaria que el secundario.(R4)

122. ¿Cuál de las siguientes pruebas NO estudia la fibrinólisis?:

1. 1. Determinación del contenido antigénico de antitrombina III.


2. 2. Análisis de los dímeros D.
3. 3. Determinación del plasminógeno.
Dosificación del inhibidor del activador del plasminógeno (PAI) mediante sustratos
4. 4.
cromogénicos.
Gráfico de respuestas
Comentario

El estudio de la fibrinolisis puede tener mucho interés en ciertas situaciones, como puede ser la
coagulación intravascular diseminada (CID), por lo que nos debemos familiarizar con
determinaciones como las que aparecen en esta pregunta. El aumento de PDFs y de los D-
dímeros son dos pruebas muy indicativas de activación de la fibrinolisis, ya que refleja la presencia
de productos derivados de esta activación. Por supuesto también descienden el fibrinógeno, el
plasminógeno y todos los factores de coagulación que se están gastando constantemente en
estados de fibrinolisis. El PAI es un factor de control de este proceso de degradación, por lo que
también puede ser útil conocer sus niveles. La antitrombina III también disminuye en estos
procesos en un intento de frenar el exceso de coagulación, pero su contenido antigénico no aporta
ninguna prueba de la existencia de un estado de fibrinólisis.(R1)

!
!
!
!
123. Durante los primeros meses de embarazo la curva de temperatura basal es:

1. 1. Constantemente hipotérmica.
2. 2. Constantemente hipertérmica.
3. 3. Primero normo y después hipertérmica.
4. 4. Primero hipo y después normotérmica.
Gráfico de respuestas
Comentario

Durante los primeros meses del embarazo, el aumento de la progesterona produce un incremento
de la temperatura basal (respuesta 2 correcta).

Esta pregunta, en realidad, la podría haber deducido. Ya sabe que, a partir de la ovulación, existe
una tendencia a la hipertermia, también debida a la acción de la progesterona.(R2)

124. A 21-day-old child is brought to the pediatrician's office presenting with non-bile-
stained projectile vomiting. The child is constantly hungry. Palpation of the abdomen
reveals a mass in the epigastrium and peristaltic waves are seen. Signs of dehydration
are present and potassium levels are 2.5 mEq/L. What should be done next?

1. 1. Intravenous hydration and potassium replacement.


2. 2. Start antibiotic coverage.
3. 3. Perform an abdominal ultrasound.
4. 4. Emergency surgery.
Gráfico de respuestas
Comentario
Intravenous hydration and potassium replacement. The child in this case suffers from pyloric
stenosis. This condition is caused by an excessive hypertrophy of the pyloric muscles. The
symptoms start around the third week of life. Vomiting can be severe with dehydration and
electrolyte imbalances. In such cases corrective therapy must be promptly started.(R1)

125. Paciente de 17 años, diagnosticado hace un año de leucemia linfoide aguda en


tratamiento de mantenimiento con 6-mercaptopurina y metotrexato. Acude a Urgencias
por cefalea, náuseas y parálisis del III par craneal, por lo que usted sospecha una recidiva
de su leucemia. ¿Qué prueba de las siguientes le parece más razonable realizar para
confirmar el diagnóstico en este paciente?

1. 1. TC craneal.
2. 2. Punción lumbar.
3. 3. Ecografía ocular.
4. 4. Frotis de sangre periférica.
Gráfico de respuestas
Comentario

Las leucemias agudas linfoblásticas invaden con frecuencia el sistema nervioso central, y por la
clínica, parece que esto es lo que le ha ocurrido a nuestro paciente. Para diagnosticar esta
afectación, lo más útil es estudiar la celularidad del LCR mediante la realización de una punción
lumbar.(R2)

!
!
!
!
126. Paciente VIH(+), primigesta, que acude al servicio de urgencias en la semana 39 de
gestación por sensación de pérdida de líquido amniótico desde el día anterior. A la
exploración se observa un cérvix sin modificar, cerrado, formado, en posición posterior
y de consistencia media. Durante la gestación ha realizado controles en la consulta de
alto riesgo obstétrico, y actualmente está en tratamiento con triple terapia. Presenta una
última carga viral de 1600 copias. El nivel de CD4 es de 400. Señale cuál de las siguientes
afirmaciones es CORRECTA:

1. 1. Es indicación de cesárea electiva para evitar la transmisión vertical.


2. 2. Puede permitirse el parto vaginal, ya que la paciente no presenta deterioro inmunológico.
El tiempo de evolución de bolsa rota no es un factor indicativo de cesárea frente a parto
3. 3.
vaginal en ninguna de las circunstancias.
Los únicos criterios que determinan la vía de parto en una paciente VIH(+) son la carga
4. 4.
viral y el nivel de linfocitos CD4.
Gráfico de respuestas
Comentario
En una paciente VIH(+) se recomienda cesárea electiva para evitar la transmisión vertical en casos
de carga viral detectable, bolsa rota de > 4 horas de evolución, diagnóstico reciente sin tratamiento
o con tratamiento diferente a la triple terapia, deterioro inmunológico o prematuridad. Por tanto, en
este caso clínico, la paciente presenta una carga viral >1000 copias y rotura prematura de
membranas de larga evolución por lo que es de elección la cesárea, aunque la paciente no
presente deterioro inmunológico o haya realizado tratamiento adecuado durante la gestación.(R1)

127. El tumor maligno más común de células germinales corresponde a:

1. 1. Neoplasia escamosa.
2. 2. Serosa.
3. 3. Serosa papilífera.
4. 4. Disgerminoma.
Gráfico de respuestas
Comentario

Los tumores germinales suelen aparecer en mujeres jóvenes. La gran mayoría corresponden al
teratoma quístico maduro o quiste dermoide, que es benigno.

Otros tumores de este grupo son el disgerminoma (maligno que deriva de la célula germinal), tumor
del seno endodérmico (produce AFP), carcinoma embrionario (produce AFP), coriocarcinoma
(produce HCG).

El resto de opciones hablan de tumores epiteliales y en estos los serosos son los más
frecuentes.(R4)

128. Paciente de 68 años que acude a urgencias disneico y con tinte cianótico. Fumador
de 2 paquetes/día desde los 20 años de edad. La PaO2 es de 59 mmhg y la PaCO2 es de
43 mmhg. En la placa de tórax existe aplanamiento diafragmática e hiperinsuflación, con
silueta cardiaca alargada. El FEV1 es menor del 50% respecto al esperado. ¿Qué
diagnóstico sospecha y qué tratamiento inicial realizaría?:

1. 1. Enfisema; bromuro de ipatropio + salbutamol.


2. 2. Neumonia por S. aureus; cloxacilina.

!
!
!
!
3. 3. Enfisema; oxprenolol + teofilina.
4. 4. Enfisema; glucocorticoides + formeterol.
Gráfico de respuestas
Comentario

Pregunta de dificultad media en la que te piden llegar al diagnóstico de un caso clínico expuesto a
partir de unos datos clínicos y de pruebas complementarias, y dar el tratamiento más adecuado. No
debería tener ningún problema para resolverlo. Los datos clave son que el paciente llega disneico,
en insuficiencia respiratoria, con hiperinsuflación y silueta cardiaca alargada. La mayoría de los
datos apuntan a un EPOC reagudizado, descartando fácilmente la neumonía por la ausencia de
fiebre y la imagen radiológica. Para llegar al tipo de EPOC es clave observar la RX: aplanamiento
diafragmático (por atrapamiento aéreo) y silueta cardiaca alargada, lo que nos lleva a considerar el
enfisema como opción principal. Una vez hecho el diagnóstico, debemos corregir el posible
desencadenante (mayormente infección) e iniciar el tratamiento con oxígeno y dos
broncodilatadores de primera línea: beta agonista de acción corta y anticolinérgico. Si la respuesta
clínica no es buena, se debe plantear añadir la teofilina y corticoides.(R1)

129. Mujer de 30 años que presenta un acufeno en OD sincrónico con el pulso e


hipoacusia derecha con las siguientes pruebas de diapasones: Weber lateralizado a la
derecha, Rinne OD - y Rinne OI +. En la otoscopia se observa una masa rojiza que protruye
en el CAE. ¿Cuál de las siguientes patologías es más probable?:

1. 1. Pólipo sangrante de Killian.


2. 2. Fístula carótido cavernosa.
3. 3. Colesteatoma.
4. 4. Tumor glómico.
Gráfico de respuestas
Comentario

El paciente presenta, según los diapasones, una hipoacusia de transmisión que no es


característica del pólipo sangrante de Killian (situado en las fosas nasales) ni de la fístula carótido-
cavernosa. La maso rojiza en la caja que protruye en el CAE no es típica del colesteatoma y sí del
tumor glómico, sobretodo si presenta acúfeno pulsátil con el pulso. El hemotímpano se presenta
como sangre en la caja del tímpano produciendo hipoacusia pero no tiene la característica de
acúfeno pulsátil del glomus.(R4)

130. Tratamiento de la nefropatía lúpica difusa (Tipo IV de la clasificación de la OMS):

1. 1. Esteroides e inmunosupresores.
2. 2. Cloxacilina.
3. 3. Antiagregantes y fibrinolíticos.
Ocho semanas de prednisona con pauta descendente posterior y suspensión de esteroides a
4. 4.
los tres meses.
Gráfico de respuestas
Comentario

De la nefropatía lúpica debe recordar que la GN proliferativa difusa lúpica es la forma más grave y
la más frecuente en enfermos sintomáticos. Los pacientes presentan sedimento urinario activo,
importante proteinuria, HTA y alteración de la función renal en un 50% de los casos. El tratamiento
se inicia con corticoides a dosis altas asociando inmunosupresores si la situación clínica es muy

!
!
!
!
grave o si se necesitan altas dosis de corticoides o durante mucho tiempo. La plasmaféresis puede
utilizarse en los casos más graves con mucho deterioro funcional.(R1)

131. Indique la asociación INCORRECTA entre patología y signo exploratorio:

1. 1. Pulso bisferiens ... insuficiencia aórtica.


2. 2. Ondas a cañón ... bloqueo AV completo.
3. 3. Soplo de Austin-Flint ... estenosis aórtica.
4. 4. Soplo de Graham-Steele ... insuficiencia pulmonar.
Gráfico de respuestas
Comentario

Pregunta teórica de gran dificultad como es la semiología cardíaca.

La onda "a" se debe a la contracción auricular que tiene lugar al final de la diástole, por lo que no
existe en la fibrilación auricular. De la misma forma una onda "a"grande se debe a un aumento de
la resistencia al llenado del ventrículo derecho, como en la estenosis tricúspide, en la hipertensión
pulmonar, etc...El grado máximo de esta resistencia ocurre cuando la válvula tricuspídea se
encuentra cerrada mientras la AD se contrae, y entonces aparecen ondas a "en cañón".

El pulso bisferiens consisten en dos ondas sistólicas y se produce cuando se eyectan grandes
cantidades de sangre en muy poco tiempo, por ejemplo en la insuficiencia aórtica y en
miocardiopatía hipertrófica.

El soplo de Austin Flint se produce en casos de insuficiencia aórtica grave, y es un soplo


mesodiastólico que se produce por el choque de la valva mitral anterior contra el chorro
regurgitante.

El de Graham-Steell se produce en la insuficiencia pulmonar y aumenta con la inspiración.(R3)

132. La base del apéndice siempre está a unos 3 cm por debajo de la unión ileocecal, en
el punto en que se unen las tres tenias del ciego. El resto del órgano, sin embargo, puede
tener una orientación variable. ¿Cuál de las siguientes es la orientación más frecuente?:

1. 1. Laterocecal derecha.
2. 2. Pélvica.
3. 3. Mesocelíaca.
4. 4. Retrocecal.
Gráfico de respuestas
Comentario

Esta pregunta es de anatomía y de cara al ENARM no tiene mucha importancia. Cabe decir que la
orientación del apendice es muy variable y que lo más frecuente es que se oriente de forma
retrocecal.(R4)

133. Señale en qué caso consideraría patológico el desarrollo psicomotor de un niño:

1. 1. Se sienta a los 6 meses.


2. 2. No presenta sonrisa social a los 3 meses.
3. 3. No dice papá y mamá a los 8 meses.

!
!
!
!
4. 4. No hace una torre con dos cubos a los 12 meses.
Gráfico de respuestas
Comentario

Esta pregunta acerca del desarrollo psicomotor de un niño es muy relevante. Sobre este tema
quédese con los hitos mas importantes: inicio sonrisa social (1.5 meses), inicio sostén cefálico (3
meses), inicio sedestación (6 meses), dice su primera palabra y da los primeros pasos en torno al
año.(R2)

134. Respecto al EPOC y sus variantes señale la FALSA:

Su diagnóstico se basa en la clínica y las pruebas de imagen radiológicas, pero es necesario


1. 1.
la demostración de obstrucción al flujo aéreo no reversible con la espirometría.
En la bronquitis crónica la radiografía de tórax ofrece una imagen típica en «sombras en
2. 2.
vías de tranvía», debido al engrosamiento de las paredes bronquiales.
3. 3. El cor pulmonale es una complicación típica del enfisema
La hipertensión pulmonar es más frecuente y grave en pacientes con bronquitis crónica que
4. 4.
en el caso del enfisema.
Gráfico de respuestas
Comentario

Esta pregunta es muy importante para el ENARM ya que es un tema relevante y debe saber
diferenciar, dentro del EPOC, las características que son más propias del enfisema o de la
bronquitis crónica. El fenotipo del enfisema es el "soplador sonrosado", siendo las características
claves la intensa disnea, la hiperinsuflación radiológica que provoca aplanamiento del diafragma y
la hiperclaridad retroesternal y la disminución de la difusión de CO; sin embargo, el fenotipo del
bronquítico es el "azul abotargado" y en él son típicas la tos y expectoración abundante,
alteraciones gasométricas graves, cianosis, radiología en "sombras de vías de tranvía",
hipertensión pulmonar y cor pulmonale (que raramente se da en el enfisema, y si lo hace es en
estadios terminales).(R3)

135. En una familia los tres hijos presentan desde la juventud una artritis poliarticular
severa con condrocalcinosis radiológica. Su sospecha diagnóstica es:

1. 1. Forma hereditaria de artropatía por cristales de pirofosfato cálcico dehidratado.


2. 2. Hiperparatiroidismo.
3. 3. Hemocromatosis.
4. 4. Poliartritis por oxalato cálcico.
Gráfico de respuestas
Comentario
La mayoria de los pacientes que presentan condrocalcinosis son ancianos sin ninguna enfermedad
asociada que presentan depósito de estos microcristales. Sin embargo, en sujetos de menos de 55
años debemos sospechar la presencia de una enfermedad metabólica que induce el depósito de
estos cristales o bien la aparición de una forma hereditaria. La condrocalcinosis familiar
clásicamente se ha descrito como una artropatía de inicio temprano, entre la tercera y quinta
década de la vida, con afectación poliarticular grave e incapacitante. En la actualidad se han
descrito más de 50 familias distribuidas en distintos paises, entre ellos España. El mecanismo de
trasmisión parece ser autonómico dominante, observándose en algunas familias defectos
genéticos en los cromosomas 5p y 8q.(R1)

!
!
!
!
136. Señale cuál de estas afirmaciones NO es correcta en cuanto al tratamiento
conservador (sin diálisis ni trasplante), de la insuficiencia renal crónica:

1. 1. Se debe instaurar precozmente.


2. 2. Puede controlar la mayoría de los síntomas urémicos.
3. 3. Previene las secuelas a largo plazo.
Su efectividad se controla con la determinación de los niveles séricos de creatinina y
4. 4.
nitrógeno ureico.
Gráfico de respuestas
Comentario

Recuerde el tratamiento conservador de la IRC y distínguelo del sustitutivo (diálisis o trasplante). El


tratamiento conservador consta de: 1. El control de la HTA, principalmente con IECAs, ralentiza la
progresión 2. Dieta, pues la reducción de proteínas en la dieta de forma precoz enlentece la
progresión del fallo renal 3. Disminuir el aporte de sodio, agua, potasio 4. Quelantes de fosfatos,
suplementos de Ca++ y vitamina D para prevenir y tratar la osteodistrofia renal 5. Bicarbonato
sódico v.o. 6. EPO, entre otras medidas. Este tratamiento no sólo controla síntomas sino que hace
que su progresión a la fase terminal sea más lenta, sobre todo si algunas medidas se instauran
precozmente (como la restricción proteica), e incluso algunas manifestaciones se intentan evitar
como la aparición de la osteodistrofia renal. El nitrógeno ureico no se utiliza para medir la
efectividad de este tratamiento, en general la medida más común es el aclaramiento de creatinina,
aunque resulta molesta para el paciente y está sometida a errores, después de establecer el FG
inicial basta con vigilar la evolución de la creatinina en plasma, y tener en cuenta las posibles
causas de modificación.(R4)

137. Paciente de 20 meses presenta cuadro de deshidratación y convulsiones


generalizadas. En los exámenes de laboratorio destaca un Na de 110. Seleccione lo
CORRECTO:

Se debe corregir rápidamente la hiponatremia con bolos de ClNa hasta alcanzar un nivel de
1. 1.
135.
El déficit de Na se calcula con la fórmula: Déficit: (135-Real)* 0.3* Peso y administrarlo
2. 2.
en 24 horas.
Tratamiento con bolos de ClNa para alcanzar niveles séricos de 120 mEq/L y el resto del
3. 3.
déficit en las siguientes 24 horas.
4. 4. Es raro que se produzcan convulsiones con un Na sérico menor de 120 mEq/l.
Gráfico de respuestas
Comentario

Esta pregunta hace referencia al manejo de un paciente con una deshidratación hipotónica grave
(Na < 120), es difícil, pero es importante que sepa su manejo.

El déficit de Na se calcula con la fórmula: déficit (mEq) = (Na deseado- Na real) x 0.6 x pero (kg).
La rehidratación será vía intravenosa.

En casos graves es frecuente que aparezcan convulsiones como consecuencia de la


hiperhidratación neuronal, en cuyo caso se puede administrar salino hipertónico al 3%.

El déficit de Na se debe reponer rápidamente hasta alcanzar niveles de Na de > 120, lo que suele
detener las convulsiones , y posteriormente el resto del déficit en las siguientes 24h.

!
!
!
!
La corrección rápida hasta niveles superior de 130 puede asociarse a mielinolisis pontina central.
Por regla general no hay que aumentar ni disminuir el Na a más de 15 mEq /24h. (R3)

138. A 22-year-old female comes to the emergency room complaining of recent onset
fever, chills, dysuria and abdominal pain. She has no past medical history. Her
temperature is 38.7ºC. Her blood pressure is 120/80, her pulse is 110/min and her
respirations 25/min. Examination shows tenderness at the right costovertebral angle.
Laboratory results show 20,000 leukocytes (90% neutrophil count). Urinalysis shows
bacteriuria and pyuria. She is prescribed ciprofloxacin and sent home. After 5 days she
returns for follow-up. Her temperature is 38.5ºC. Her symptoms haven't subsided. What
is the best next step?

1. 1. Start intravenous gentamicin.


2. 2. Start intravenous ampicillin.
3. 3. Perform renal ultrasound.
4. 4. Continue ciprofloxacin for 10 days.
Gráfico de respuestas
Comentario
Perform renal ultrasound. We are presented with a case of acute pyelonephritis. First step in
management is starting empirical antibiotics. If there is no response in 72 hours an imaging study
should be performed to rule out complications or underlying urological abnormalities.(R3)

139. A 30-year-old woman with a history of irregular menses, hirsutism, obesity and acne
presents to your consultation reporting a 2-year history of infertility. Further inquiries
reveal that she took contraceptives but she stopped taking them three years ago. Physical
examination shows normal sexual characters. What would be the most appropriate
treatment?

1. 1. Subcutaneous progesterone (1 injection every month).


2. 2. Clomiphene citrate.
3. 3. Indomethacin.
4. 4. Cabergoline.
Gráfico de respuestas
Comentario

Pregunta sencilla sobre el síndrome de ovarios poliquísticos. Los datos de relevancia que nos
proporciona el enunciado son: irregularidad menstrual, hirsutismo, obesidad, acné e infertilidad. El
tratamiento se realizará con citrato de clomifeno. Respuesta correcta 2.

140. Which of the following is the most common causative agent of acute laryngitis during
childhood?

1. 1. Parainfluenzae virus.
2. 2. Haemophilus influenzae type B.
3. 3. Pneumococcus.
4. 4. Respiratory syncytial virus infection.
Gráfico de respuestas
Comentario

!
!
!
!
De los principales gérmenes implicados en la patología respiratoria del lactante es importante que
recuerde que el principal patógeno implicado en la laringotraqueitis aguda es el virus
parainfluenzae, en la epiglotitis aguda es el Haemofilus influenzae, y en la bronquiolitis el VRS. El
parainfluenza tipo 1 es el responsable del 75% de los casos de LAV. También pueden producirla
los tipos 2 y 3, influenza, VRS, adenovirus, rinovirus, enterovirus y Mycoplasma pneumoniae.(R1)

141. La causa más frecuente de hemoperitoneo asociada a tumoración anexial


complicada es:

1. 1. Teratoma.
2. 2. Endometrioma.
3. 3. Cuerpo lúteo.
4. 4. Quiste folicular.
Gráfico de respuestas
Comentario

Pregunta de dificultad alta, no se preocue si la ha fallado. El cuerpo lúteo normal suele contener
un poco de sangre en su centro, pues se produce un pequeño sangrado tras la ovulación. Si el
sangrado es excesivo, el cuerpo lúteo puede romperse y producir un hemoperitoneo. Es la causa
más frecuente de hemoperitoneo de origen ovárico, por tanto la respuesta correcta es la 3.

142. Le llaman a usted a la sala de partos para atender el parto del hijo de una primípara
de 23 años, bebé pequeño para la edad del embarazo (1,980 gramos, 38 semanas) que
tiene microcefalia, defectos cardíacos, ictericia y cataratas. ¿Qué orden daría para el
niño?

1. 1. Cuidados ordinarios de enfermería.


2. 2. Aislamiento precautorio e investigación de infecciones congénitas.
3. 3. Cuidados ordinarios y estudios cromosómicos y consejo genético para los padres.
4. 4. Cuidados ordinarios e investigación de diabetes mellitus en la madre.
Gráfico de respuestas
Comentario

Después de las cromosomopatías, la causa más frecuente de crecimiento intrauterino retardado


son las infecciones connatales. Piense en infecciones TORCH si el niño presenta, además de
antecedentes de CIR, si desarrolla ictericia precozmente, tiene hepatoesplenomegalia,
adenopatías o alteraciones de la BH.

A su vez, la presencia de alteraciones cardíacas y oculares le indican la posibilidad de rubéola


congénita (recuerde que son parte de la tríada de Gregg: sordera neurosensorial, cataratas y
malformaciones cardíacas). Ante semejante sospecha, está indicado el aislamiento del niño y
realizar las pruebas oportunas para confirmar la sospecha diagnóstica.(R2)

143. Ante un cuadro clínico de amenorrea galactorrea y pérdida de campo visual, el


primer diagnóstico a considerar es:

1. 1. Adenoma hipofisario no funcionante.


2. 2. Prolactinoma.
3. 3. Meningioma del tubérculo solar.
4. 4. Intoxicación por benzodiacepinas.
Gráfico de respuestas

!
!
!
!
Comentario

Una pregunta muy sencilla. Ha de saber que los prolactinomas son los únicos adenomas
hipofisarios cuyo tratamiento de elección es el farmacológico, sean macro o microprolactinomas.
Las indicaciones de tratamiento del microprolactinoma son:

•! Varones: disminución de la libido o potencia sexual.


•! Mujeres: deseo de embarazo, hipogonadismo severo con alto riesgo de osteoporosis,
aparición de síntomas molestos (galactorrea, disminución de la libido).

Por otra parte, debe recordar que la hipófisis anterior, que es donde asientan los prolactinomas, se
relaciona anatómicamente con el quiasma óptico. Por esto, los tumores hipofisarios pueden
conllevar alteraciones en el campo visual por compresión de esta estructura, cuando alcanzan
cierto volumen.(R2)

144. De las siguientes neumoconiosis señale cuál se asemeja más a la sarcoidosis:

1. 1. Pulmón de granjero.
2. 2. Bagazosis.
3. 3. Siderosis.
4. 4. Beriliosis.
Gráfico de respuestas
Comentario
La beriliosis es una enfermedad producida por la exposicón al berilio. Este agente se encuentra en
las fábricas de aleaciones, cerámica, electrónica de alta tecnología y lámparas fluorescentes. El
berilio puede producir una neumonitis aguda o, lo que es mucho más frecuente, una neumonitis
intersticial crónica. Desde el punto de vista patológico produce granulomas similares a los de la
sarcoidosis, por lo que es fundamental la determinación de berilio en el tejido para su
diferenciación.(R4)

145. Niños de 18 meses de edad, eutrófico, bruscamente presenta crisis de llanto, palidez,
vómitos alimentarios en un inicio y posteriormente biliosos, evacuaciones
sanguinolentas “jalea de grosella”. El diagnóstico clínico más probable es:

1. 1. Apendicitis aguda.
2. 2. Gastroenteritis disentérica.
3. 3. Invaginación intestinal.
4. 4. Amebiosis.
Gráfico de respuestas
Comentario

Se trata de una pregunta relativamente sencilla sobre invaginación intestinal. La edad del niño es
típica ya que la invaginación intestinal es la causa más frecuente de obstrucción intestinal entre los
3 meses y los seis años, mostrando mayor incidencia entre los 4 y los 12 meses.

La etiología es en la mayoría de los casos desconocida aunque en un pequeño porcentaje de


casos es secundario a infecciones, divertículo de Meckel, pólipos etc. El diagnóstico se basa en la
anamnesis y la exploración física, la Rx simple de abdomen en las primeras horas puede ser
normal.

!
!
!
!
Ante la sospecha fundada como ocurre en este caso debe realizarse ultrasonido abdominal que
mostrará la imagen de “dona” es decir la imagen de un segmento intestinal que se introduce en
otro segmento intestinal inmediatamente distal a él.(R3)

146. Paciente de 50 años con


cirrosis grado B de Child, presenta en la TAC lo que muestra la imagen nº ##. ¿Cuál entre
los siguientes sería el tratamiento de elección?

1. 1. Quimioembolización.
2. 2. Sorafenib.
3. 3. Alcoholización.
4. 4. Trasplante hepático.
Gráfico de respuestas
Comentario

Se observa una única masa hepática que refuerza con contraste en fase arterial, además de tener
un paciente en Child B, sin más comorbilidades.

Para diagnosticar el cáncer hepático no es necesario una biopsai si las pruebas de imagen son
altamente sugestivas de carcinoma hepatocelular (TAC en fase arterial y venosa).(R4)

!
!
!
!

147. Mujer de 83 años con antecedentes


personales de diabetes mellitus, hernia del hiato, úlcera duodenal y herniorrafia inguinal
bilateral. Sigue tratamiento habitual con metformina, omeprazol y hierro por anemia
desde hace meses. Acude a Urgencias por dolor abdominal cólico con náuseas, vómitos
y estreñimiento de 48 h de evolución. Exploración física: abdomen distendido y doloroso
difusamente, sin signos de irritación peritoneal. Ruidos hidroaéreos disminuidos. No
masas palpables. Laboratorio: Hb: 8.5 g/dl, VCM 80 fl, plaquetas 240,000/Ul, leucocitos
10,200/Ul (81% granulocitos). pH: 7.31, HCO3: 17 mmol/l, Na + 134 mmol/l, K 3.1 mmol/l.
Amilasa 150 U/l (28-100). LDH 252 Ul (135-225). Creatinina 1.1 mg/dl. Resto normal.
Radiología de abdomen: ver imagen 1. Su sospecha diagnóstica sería:

1. 1. Estreñimiento secundario a tratamiento con hierro.


2. 2. Pancreatitis aguda.
3. 3. Íleo paralítico secundario a alteraciones iónicas.
4. 4. Obstrucción intestinal mecánica a nivel de íleon distal o ciego.
Gráfico de respuestas
Comentario

Las manifestaciones clínicas que nos presentan: náuseas, vómitos, estreñimiento, distensión
abdominal, etc., junto con los hallazgos radiológicos (radiografía anteroposterior de abdomen
donde se observa dilatación de las asas del intestino delgado con ausencia de aire a nivel del
intestino grueso) son compatibles con obstrucción intestinal.

Entre los antecedentes personales de la paciente, cabe destacar la herniorrafia inguinal bilateral,
ya que en pacientes con cirugía previa la causa más frecuente de obstrucción de intestino delgado
son las adherencias. Todos estos hallazgos hacen que la primera sospecha diagnóstica sea la de
obstrucción intestinal mecánica a nivel del íleo distal o ciego.(R4)

148. El manejo clínico inicial más adecuado para el paciente sería:

Dieta absoluta, sonda con aspiración nasogástrica y sueroterapia con cloruro de potasio.
1. 1.
Solicitar TAC abdominal urgente.
2. 2. Enemas de limpieza hasta resolución del cuadro. Repetir Rx de control.

!
!
!
!
3. 3. Sueroterapia con bicarbonato 1/6 M, cloruro de sodio y de potasio intravenosos.
Dieta absoluta 8 h, probar tolerancia y alta a domicilio con lactulosa y domperidona vía
4. 4.
oral.
Gráfico de respuestas
Comentario

Ante la sospecha diagnóstica de obstrucción intestinal mecánica a nivel de intestino delgado, el


tratamiento será dieta absoluta, sonda nasogástrica con aspiración y reposición hidroelectrolítica.
Para confirmar la sospecha diagnóstica y la etiología de la misma se solicitará TC abdominal. El
90% de las obstrucciones de intestino delgado se resolverán con tratamiento médico. El
tratamiento quirúrgico está indicado si existe sospecha de estrangulación, el dolor y la fiebre
aumentan y si no existe resolución en un plazo de 3-5 días.(R1)

149. RN de 42 semanas de edad gestacional presenta en las primeras horas de vida


taquipnea, quejido respiratorio y cianosis. En la Rx de tórax se aprecian infiltrados
parcheados, aumento del diámetro anteroposterior y aplanamiento de los diafragmas con
imagen de neumomediastino. En relación a este cuadro, señale la afirmación FALSA:

1. 1. Es típico de los postérmino.


2. 2. El riesgo de aparición de este cuadro no se ve influido por la hipoxia fetal.
3. 3. La expresión radiológica del cuadro suele estar dominada por la hiperinsuflación.
4. 4. Las secuelas pulmonares son raras en estos niños.
Gráfico de respuestas
Comentario

Lo que nos están describiendo es un síndrome de aspiración meconial. Para reconocerlo, es típico
que se trate de un RN postérmino con sufrimiento fetal, y en la Rx suele haber datos de
hiperinsuflación. Por supuesto, la respuesta 2 es incorrecta, ya que la hipoxia fetal es uno de los
factores que puede producir una expulsión precoz del meconio, con la consiguiente aspiración del
mismo cuando el niño rompe a llorar.(R2)

150. En la UCIN nos avisan porque un RNPT de 28 semanas, con 15 días de vida, actual
portador de una vía central, y con antecedentes de enfermedad de membrana hialina
grave, está desde hace dos días menos reactivo, tiene mal aspecto y controla mal la
temperatura. Tiene distrés respiratorio moderado y una leve distensión abdominal, no
especialmente llamativa. En la BH presenta: leucocitos: 4,700 (900 neutrófilos, múltiples
bandas), Hb: 8 g/dl. Plaquetas: 100,000/mm3. La Rx simple de abdomen es normal. ¿Cuál
es el diagnóstico más probable?:

1. 1. Sepsis nosocomial.
2. 2. Enterocolitis necrotizante.
3. 3. Ileo meconial.
4. 4. Metabolopatía.
Gráfico de respuestas
Comentario

Un neonato con vía central es un candidato al desarrollo de sepsis. Tenga en cuenta que los
criterios que definen sepsis en el adulto no son válidos ni aplicables al período neonatal, donde
esta situación se define a través de alteraciones de la BH. La BH indicativa de sepsis es aquél que
presenta leucopenia menor de 5000, neutropenia menor de 1500 y desviación izquierda (cociente
IT>0.16). El niño del caso clínico cumple rigurosamente lo dicho sobre estas líneas.(R1)

!
!
!
!
151. Un paciente consumidor de heroína intravenosa consulta por presentar artritis aguda
de rodilla y fiebre. En el líquido obtenido de la artrocentesis se aprecian cocos GRAM
positivos. El tratamiento antibiótico adecuado sería:

1. 1. Ceftazidima i.v.
2. 2. Eritomicina i.v.
3. 3. Ciprofloxacina oral.
4. 4. Cloxacilina i.v y Gentamicina i.v.
Gráfico de respuestas
Comentario
Esta pregunta es rentable en el capítulo de artritis sépticas, y enlaza con las infecciones en los
ADVP. En un paciente ADVP con artritis, lo más probable es que el agente causal sea S.aureus,
por lo que su tratamiento empírico es cloxacilina i.v. (asociamos gentamicina para cubrir otros
microorganismos a la espera del resultado del cultivo). La artrocentesis es obligada ante una
monoartritis aguda para filiar su origen probable, y en este caso confirma la presencia de cocos
gram +. Este paciente tiene además riesgo de endocarditis? Si la artritis fuera de localización
condrocostal, debemos pensar más bien en Candida.(R4)

152. Indique con qué tipo de fármaco puede tratarse a demanda la eyaculación precoz:

1. 1. Beta-bloqueante.
2. 2. ISRS.
3. 3. Sildenafilo.
4. 4. Apomorfina.
Gráfico de respuestas
Comentario
La dapoxetina, inhibidor selectivo de la recaptación de la serotonina de vida media corta es el
tratamiento actual a demanda para la eyaculación precoz.(R2)

153. Varón de 10 años que presenta un exoftalmos derecho muy moderado y defecto
pupilar aferente parcial. En la radiografía simple de cráneo se aprecia un ensanchamiento
del agujero óptico. ¿Cuál será el diagnóstico más probable?:

1. 1. Rabdomiosarcoma.
2. 2. Meningioma del nervio óptico.
3. 3. Glioma del nervio óptico.
4. 4. Hemangioma orbitario.
Gráfico de respuestas
Comentario
La existencia de exoftalmos traduce la presencia de una masa orbitaria. La presencia de un
defecto pupilar aferente relativo, pone de manifiesto la existencia de daño en el nervio óptico. Todo
ello nos debe hacer sospechar la presencia de un tumor derivado del nervio óptico. Nuestra
sospecha se confirma radiológicamente, pues el canal óptico se aprecia ensanchado. Este
ensanchamiento se debe a la erosión que el tumor produce en su lento crecimiento. Debes
recordar que el tumor derivado del nervio óptico más frecuente en el niño, es el glioma, y que se
trata de un tumor benigno de crecimiento muy progresivo. El meningioma es el tumor derivado del
nervio óptico más frecuente en el adulto, y también es benigno. El resto de las opciones quedan
descartadas, pues al no tratarse de tumores derivados del nervio óptico, en caso de producir un
defecto pupilar aferente, lo harían en fases muy avanzadas del tumor.(R3)

!
!
!
!
154. Usted ve en consulta por primera vez a un paciente de 45 años diagnosticado de
enfermedad pulmonar obstructiva crónica (EPOC). Desde los 25 años había fumado 5
cigarrillos al día (5 paquetes/año). El síntoma fundamental es una disnea de mínimos
esfuerzos. La auscultación pulmonar es normal, aunque en la exploración física destaca
la presencia de aumento de presión venosa y edemas en ambos miembros inferiores.
¿Cuál de las siguientes actuaciones considera errónea en este paciente?

Realizar una espirometría con prueba broncodilatadora para demostrar la existencia de


1. 1.
obstrucción crónica al flujo aéreo.
Iniciar inmediatamente tratamiento con altas dosis de broncodilatadores, corticoides
2. 2.
inhalados y teofilina para mejorar su situación clínica.
Cuestionar el diagnóstico de EPOC y valorar otras patologías que cursen con insuficiencia
3. 3.
cardíaca derecha.
4. 4. Ampliar el estudio del paciente con radiografía de tórax y ecocardiograma.
Gráfico de respuestas
Comentario

Es poco probable que este paciente padezca en realidad una EPOC. A pesar de que es fumador,
sólo fuma cinco cigarrillos diarios (cinco paquetes/año). Para padecer esta enfermedad, se
estima que el promedio necesario es el equivalente a 20 paquetes/año (es decir, una cajetilla
diaria durante 20 años, o dos cajetillas diarias durante diez años).

Por otra parte, la normalidad de la auscultación pulmonar también iría en contra de este
diagnóstico. La presencia de disnea y edemas podría ser explicada por otras patologías
(cardiopatías, hipertensión pulmonar…), de manera que convendría reenfocar el estudio de
este paciente y no tratarlo dando por hecho que es una EPOC (respuesta 2 falsa).(R2)

155. Desde el punto de vista hormonal, ¿qué es lo patognomónico de la menopausia?

1. 1. Prolactina elevada.
2. 2. FSH normal y estradiol alto.
3. 3. FSH elevado y estradiol bajo.
4. 4. Delta 4 androstendiona alta.
Gráfico de respuestas
Comentario

Aunque en esta pregunta se habla de “patognomónico”, se comprende perfectamente y puede ser


respondida.

El agotamiento de los folículos determina un descenso de los niveles de estradiol. Como


consecuencia de ello, se activa el feedback hipofisario, elevándose las gonadotropinas (FSH y LH).
Por lo tanto, la respuesta correcta es la 3. Como decíamos, este perfil hormonal no es
patognomónico de la menopausia. Podemos encontrarlo en muchas otras situaciones donde existe
un descenso del estradiol, como en el síndrome de Turner, en el fallo ovárico precoz, etcétera.(R3)

156. Femenino de 45 años, con antecedentes de obesidad y DM tipo 2, presenta dolor


cólico en hipocondrio derecho, de algunas horas de evolución, asociado a fiebre, mareos,
nauseas, vómitos. ¿Cuál es el manejo y el pronóstico?

!
!
!
!
1. 1. Si se realiza un cultivo biliar se encontraría positivo en más del 90%.
2. 2. La colecistectomía laparoscópica está contraindicada.
3. 3. Una adecuada cobertura antibiótica incluye anaerobios y gram positivos.
La mejor conducta es diferir el tratamiento quirúrgico una vez resulto el cuadro infeccioso
4. 4.
para una colecistectomía laparoscópica.
Gráfico de respuestas
Comentario

La colecistitis aguda puede ser de carácter leve e involucionar en forma espontánea o progresar
hacia el empiema de la vesícula biliar, lo que se conoce como “piocolecisto”, y hasta la necrosis y
gangrena con perforación del órgano y peritonitis biliar, lo cual se asocia con una elevada tasa de
mortalidad (20%). La respueta correcta es la 4. Los microorganismos más frecuentes en la bilis de
la colecistitis aguda son entéricos, entre ellos E coli, Klebsiella spp, Streptococcus fecalis,
Clostridium welchii, Proteus spp, y Bacteroides.

Entre 15% y 30% de los pacientes con colelitiasis presenta bilis infectada, pero la infección de la
bilis es casi universal en los casos de colecistitis aguda, coledocolitiasis, estenosis u obstrucción
biliar no tumoral.(R4)

157. de 68 años que acude a


Urgencias disneico y con tinte cianótico. Fumador de 2 paquetes/día desde los 20 años
de edad. La PaO2 es de 59 mmHg y la PaCO2 es de 43 mmHg. En la placa de tórax existe
lo que muestra la imagen nº ##. El FEV1 es menor del 50% respecto al esperado. ¿Qué
diagnóstico sospecha y qué tratamiento inicial realizaría?

1. 1. Enfisema; bromuro de ipatropio + salbutamol.


2. 2. Bronquitis crónica; salmeterol + bromuro de ipatropio.
3. 3. Enfisema; oxprenolol + teofilina.
4. 4. Enfisema; glucocorticoides + formeterol.
Gráfico de respuestas
Comentario

!
!
!
!
Pregunta de dificultad media en la que te piden llegar al diagnóstico de un caso clínico expuesto a
partir de unos datos clínicos y de pruebas complementarias, y dar el tratamiento más adecuado. No
deberías tener ningún problema para resolverlo. Los datos clave son que el paciente llega disneico,
en insuficiencia respiratoria, con hiperinsuflación y silueta cardíaca alargada como se muestra en la
radiografía de tórax. La mayoría de los datos apuntan a una EPOC reagudizada, descartando
fácilmente la neumonía por la ausencia de fiebre y la imagen radiológica. Para llegar al tipo de
EPOC es clave observar la Rx: aplanamiento diafragmático (por atrapamiento aéreo) y silueta
cardíaca alargada, lo que nos lleva a considerar el enfisema como opción principal. Una vez hecho
el diagnóstico, debemos corregir el posible desencadenante (principalmente infección) e iniciar el
tratamiento con oxígeno y dos broncodilatadores de primera línea: betaagonista y anticolinérgico.
Si la respuesta clínica no es buena se debe plantear añadir la teofilina y corticoides.(R1)

158. El himen imperforado es un defecto de:

1. 1. Conductos de Wolf.
2. 2. La cresta genital.
3. 3. Tubérculo del seno urogenital.
4. 4. Cloaca ectodérmica.
Gráfico de respuestas
Comentario

-La respuesta 1 es incorrecta: los conductos de Wolf degeneran en la mujer, y en el hombre dan
lugar a epidídimo, conducto deferente y canal eyaculador.

-La respuesta 2 es falsa: la cresta genital es una prominencia del mesodermo intermedio que da
lugar a las gónadas.

-La respuesta 3 es verdadera. Recordemos cómo se forma la vagina. Los conductos de Müller dan
lugar al útero y las trompas. El útero resulta de la fusión de la parte distal de los dos conductos,
para dar un conducto único. La parte distal de este conucto único se fusiona con la pared del seno
urogenital; entre los dos, dan lugar a un tubérculo, llamado tubérculo de Müller o del seno
urogenital, que finalmente se canaliza, para dar lugar a la vagina. Si no se canaliza del todo, el
himen permanece imperforado.

-La respuesta 4 es falsa: Si bien es cierto que una parte de la cloaca, el seno urogenital, contribuye
a formar la vagina, la cloaca no procede del ectodermo, sino del endodermo.(R3)

159. Niño de 6 años de edad, acude por náuseas y vómitos de dos días de evolución.
Presenta fiebre y rinorrea por lo cual recibió antipiréticos, descongestionante nasal y
vitamina C. Posteriormente cursa con ictericia, bilirrubinas y transaminasas muy
elevadas. Usted podría sospechar intoxicación por:

1. 1. Salicilatos.
2. 2. Loratadina.
3. 3. Clorferamina.
4. 4. Acetaminofén.
Gráfico de respuestas
Comentario

Cuadro típico de intoxicación por acetaminofén... Le relatan un cuadro de infección de vías aereas
superiores probablemente viral, con la administración de antipiréticos, con posterior ictericia y

!
!
!
!
alteración de PFH y le preguntan por INTOXICACIÓN, por lo que debe elegir la opción 4. Podría
dudar con salicilatos, ya que la administración de acido acetilsalilico en pacientes con influenza B o
varicela pueden desencadenar un síndrome de Reye con datos de hepatitis aguda y encefalopatía,
pero por eso insisto, debe leer cuidadosamente ya que le solicitan que elija la intoxicación, no el
efecto adverso.(R4)

160. En el renograma isotópico de una niña de 14 años con hidronefrosis congénita


unilateral por síndrome de la unión pieloureteral, es característico que:

1. 1. No existe captación del radiofármaco en el riñón alterado.


2. 2. El tiempo arterial está prolongado en el riñón alterado.
El tiempo de excreción prolongado no se modifica significativamente al suministrar un
3. 3.
diurético.
El tiempo de tránsito parenquimatoso y el tiempo de excreción, ambos prolongados, se
4. 4.
modifican significativamente al suministrar un diurético.
Gráfico de respuestas
Comentario

Aparentemente difícil, pero esta pregunta puede resolverse por sentido común. De entrada, habría
que dudar entre las opciones 3 y 4, porque son opuestas entre sí. ¿Se modifica el tiempo de
excreción con un diurético, o no? Dado que nos hablan de una alteración estructural (síndrome de
la unión pieloureteral), lógicamente no se modificará por dar diuréticos (intentar solucionar un
problema estructural con una pastilla sería bastante optimista). Por ello, la opción correcta es la
3.(R3)

161. ¿Cuál de las siguientes afirmaciones es correcta respecto a la talla baja asociada a
déficit de hormona de crecimiento (GH)?

1. 1. La deficiencia de GH es la causa más frecuente de hipocrecimiento armónico patológico.


2. 2. En las formas congénitas de deficiencia de GH, el crecimiento prenatal suele ser normal.
La maduración ósea en la deficiencia de GH, al contrario de lo que ocurre en otras
3. 3.
endocrinopatías, se encuentra muy elevada.
4. 4. Los valores séricos de IGF-I se encuentran claramente elevados en la deficiencia de GH.
Gráfico de respuestas
Comentario

Con respecto al déficit de hormona de crecimiento en la infancia, los pacientes que aquejan dicho
déficit muestran una velocidad de crecimiento inferior a la normal y su curva de crecimiento se
desvía progresivamente del canal normal. Pero si el déficit es congénito, se manifiesta a partir de
los 6-12 meses de edad, siendo la talla y el peso al nacer normales. Eso es debido a que en esta
etapa de la vida el crecimiento no depende de GH, sino de la insulina. La mayoría responden al
tratamiento con GH sintética con una aceleración de la velocidad de crecimiento hasta un límite
normal o, incluso, por encima de lo normal.(R2)

162. En los casos de deshidratación grave con natremia normal o baja, los signos
predominantes son:

1. 1. Diarrea y mucosas pastosas.


2. 2. Irritabilidad y fontanela hundida.
3. 3. Convulsiones.
4. 4. Pliegue cutáneo y signos de hipoperfusión periférica.

!
!
!
!
Gráfico de respuestas
Comentario

Esta pregunta resulta útil para entender el capítulo de las deshidrataciones. La clínica de las
deshidrataciones depende del compartimiento afectado. En la hipertónica el compartimiento
afectado es el intracelular, y predomina la afectación neurológica, fiebre y sed intensa. En los
casos mas graves es posible hemorragia intracraneal consecuencia de la elongación y ruptura de
los vasos intracraneales al deshidratarse la masa encefálica. En los casos de deshidratación iso e
hiponatrémica el compartimiento más afectado es el extracelular y los signos predominantes son:
pliegue cutáneo, mucosas secas, fontanela deprimida y signos de hipoperfusión periférica. En los
casos graves predomina la afectación del espacio vascular siendo frecuente el choque. En algunos
casos graves de deshidratación hiponatrémica pueden aparecer convulsiones por hiperhidratación
neuronal.(R4)

163. Paciente varón de 60 años


intervenido de prótesis total de cadera derecha hace 10 días. Es traído a Urgencias por
febrícula de 37,6º C que se acompaña de taquipnea y taquicardia de 6 horas de evolución.
La gasometría muestra una saturación basal de oxígeno del 87%. La imagen nº 11 muestra
la radiografía simple de tórax. ¿Cuál debe ser nuestra primera sospecha diagnóstica?

1. 1. Distrés respiratorio del adulto.


2. 2. Tromboembolismo pulmonar.
3. 3. Sepsis nosocomial.
4. 4. Infección protésica.
Gráfico de respuestas
Comentario

Se trata de un paciente que ha sido hospitalizado recientemente e inmovilizado con intervención


sobre el miembro inferior, por lo que existen factores de riesgo para la infección nosocomial y para
la enfermedad tromboembólica. La radiografía de tórax muestra elevación del hemidiafragma
izquierdo sin condensaciones parenquimatosas evidentes. A priori, la neumonía atípica no se ve
refl ejada en los hallazgos radiográficos, por lo que se puede descartar razonablemente. También
puedes descartar el distrés respiratorio del adulto pues no se evidencian infiltrados alveolares
bilaterales típicos.

!
!
!
!
La infección protésica y la sepsis se deben tener presentes pero no serán la primera opción
diagnóstica, y esperaríamos encontrar fiebre elevada y que nos aportasen datos analíticos de
leucocitosis con neutrofilia intensa y elevación de reactantes de fase aguda. El diagnóstico que
debemos plantear en primer lugar es el del tromboembolismo pulmonar: paciente con factores de
riesgo para TVP y que súbitamente inicia disnea con taquicardia y febrícula, y radiografía
compatible con elevación hemidiafragmática.(R2)

164. En relación con el síndrome de muerte súbita infantil o del lactante (SMSL), señale
cuál de los siguientes NO se asocia a un aumento de riesgo:

1. 1. Sobrecalentamiento del entorno.


2. 2. Colchón plano y firme.
3. 3. Prematuridad.
4. 4. La postura en prono para dormir.
Gráfico de respuestas
Comentario

La respuesta 2 es falsa; de hecho es justo al contrario, un colchón blando y poco firme puede ser
causa de apneas obstructivas en el lactante, sobre todo si se asocia este factor a una postura del
sueño en decúbito prono o lateral.

Hay que recordar los dos factores de riesgo más importantes implicados en el SMSL: posición para
dormir en decúbito prono y lateral, y en segundo lugar, madre fumadora en el embarazo.(R2)

165. A 23-year-old man is brought to the emergency


department after he drove his motorcycle into a guardrail at a high speed, receiving a
strong impact on his right leg. Physical examination reveals right knee swelling, rotation
of the leg and the patient refers great pain while the exam is made. Common peroneal
nerve dysfunction is found. Which of the following is the most appropriate next step in
management?

1. 1. Surgical repair of common peroneal nerve.


2. 2. Immobilization and further imaging studies.

!
!
!
!
3. 3. Closed reduction and immobilization of the limb.
4. 4. Arthroscopic knee surgery, repairing ligament and meniscal injuries.
Gráfico de respuestas
Comentario
La radiología nos muestra una luxación de rodilla con una incongruencia articular femorotibial. Ante
una luxación de rodilla, debemos realizar una reducción cerrada con extrema suavidad (bajo
anestesia general, para evitar lesiones mayores de las partes blandas) e inmovilizar la pierna con
una férula posterior. La reparación de las lesiones ligamentosas y meniscales debe hacerse de
manera diferida (con la rodilla “enfriada”), para evitar una respuesta inflamatoria mayor que nos
abocaría a una rigidez y a un mal resultado funcional. El nervio peroneo común posiblemente sufra
una neuroapraxia por la contusión y no precise de reparación quirúrgica. No nos indican la
presencia de herida que nos obligue a realizar cobertura antibiótica y antitetánica de entrada.(R3)

166. Cuando el VHS I ó II producen afectación del SNC, suelen afectar predominantemente
a:

1. 1. Area frontal.
2. 2. Area temporal.
3. 3. Meninges.
4. 4. Area occipital.
Gráfico de respuestas
Comentario
Pregunta sobre conceptos básicos de la encefalitis por VHS que no se debe fallar. Repasemos los
aspectos fundamentales. Más del 95% de los casos de meningoencefalitis por VHS se deben al
subtipo VHS- I. Es el agente causal más frecuentemente identificado en la meningitis linfocitaria
recurrente (meningitis de Mollaret), que ha sido preguntado en el MIR. La encefalitis por VHS se
manifiesta en forma de fiebre y alteraciones conductuales. Su diagnóstico es inicialmente clínico.
En el TC craneal se pueden encontrar hipodensidades en lóbulos temporales y en el EEG
alteraciones en dicho lóbulo, uni o bilaterales. La confirmación del diagnóstico se realiza mediante
la demostración del ADN viral por PCR en LCR y su tratamiento de elección es el aciclovir iv.(R2)

167. La presencia de proteínas en la orina, así como su cantidad, puede ser un marcador
importante de diferentes enfermedades renales. Señale la respuesta INCORRECTA:

La excreción urinaria de proteínas por encima de 3 g/24 horas hace muy probable la
1. 1.
existencia de enfermedad glomerular.
2. 2. La presencia de proteinuria debería hacernos dudar del diagnóstico de síndrome nefrítico.
La proteinuria, cuando sobrepasa cierta cuantía, es un criterio de gravedad en la
3. 3.
preeclampsia.
La presencia de proteinuria selectiva es sugestiva de nefropatía de cambios mínimos y
4. 4.
produce una pérdida fundamentalmente de albúmina.
Gráfico de respuestas
Comentario

La proteinuria menor de 3 g/día es una característica clínica más del síndrome nefrítico. Recuerde
que los otros componentes son hematuria y reducción aguda del filtrado glomerular con oliguria,
insuficiencia renal y retención de agua y sal. Respuesta 2 correcta.

!
!
!
!
168. Acerca de los requerimientos nutricionales, marque lo INCORRECTO:

1. 1. El RN a término el requerimiento hídrico del primer día es de 60–70 mL/Kg/d.


2. 2. Los requerimientos de glucosa en el RN son de 4 a 8 g/Kg/d.
3. 3. Los requerimientos calóricos en el primer trimestre es de 100 a 120 Kcal/Kg/d.
4. 4. Los requerimientos proteicos es de 2 a 3 g/Kg/d.
Gráfico de respuestas
Comentario

Requerimientos hidratos de carbono: RNT 10-17 g/kg/dia, Prematuro 13-15 g/kg/dia.

Lípidos: RNT 4-6 g/kg/dia y prematuro hasta 9 g/kg/dia.

Hídricos: 1-2 días: 6080 ml/kg/dia, 3-7 día 100-150 ml/kg/dia, 8-30 día 120-180 ml/kg/dia

Proteinas: RNT 2.25-2.5 g/kgdia, prematuro 3-4.5 g/kg/dia.

Por lo que la respuesta que debió elegir es la 2.

169. A 66-year-old male comes to your office due to anorexia, fatigue, and weight loss for
the past six months. His previous medical history is significant for type 2 diabetes
mellitus and hypertension. He denies alcohol consumption and does not use illicit drugs.
He has smoked 20 cigarettes per day for the past 50 years. His current medications are
enalapril and metformin. On arrival temperature is 37ºC, BP is 135/76 mmHg, pulse is
80/min, and respirations are 12/min. Physical examination reveals mild jaundice. The rest
of examination is unremarkable. Blodd tests are normal except for high bilirubin and
alcaline phospatase levels. An abdominal ultrasound shows a dilated choledochal duct
with no presence of gallstones. What is the next best step in the management of this
patient´s condition?

1. 1. ERCP.
2. 2. Percutaneous transhepatic cholangiography).
3. 3. Abdominal CT.
4. 4. Serum amilase and lipase.
Gráfico de respuestas
Comentario
Abdominal CT. In a smoker who presents with weight loss, anorexia and jaundice, an abdominal
ultrasound should be performed. In this case, the patient has non-lithiasic biliary dilation so an
abdominal CT scan is necessary to rule out pancreatic cancer.(R3)

170. ¿Cuál de los siguientes NO es una hernia de la pared abdominal?:

1. 1. Hernia de Spiegel.
2. 2. Hernia incisional.
3. 3. Hernia epigástrica.
4. 4. Hernia de Hesselbach.
Gráfico de respuestas
Comentario

!
!
!
!
Es una pregunta difícil.

La hernia de Spiegel se produce en el punto de unión del borde lateral del recto abdominal con la
línea semilunar de Douglas.

La hernia incisional se produce después de una intervención en la que se haya realizado una
incisión en la pared abdominal.

La hernia epigástrica aparece en la línea alba superior al ombligo.

La hernia de Hesselbach es una profusión de divertículos a través de la vaina femoral, usualmente


asociada a una hernia inguinal oblicua.(R4)

171. Considera FALSO, respecto a la clínica de la estenosis aórtica:

La angina de pecho es el síntoma más precoz y más frecuente de la estenosis aórtica


1. 1.
sintomática.
2. 2. El síncope de reposo es un dato de mal pronóstico.
3. 3. Existe riesgo de muerte súbita.
4. 4. La disnea suele ser el síntoma más precoz.
Gráfico de respuestas
Comentario

Pregunta importante sobre la clínica de la Estenosis Aórtica. La EAo puede permanecer


asintomática durante muchos años. Cuando aparecen los síntomas, son principalmente tres: la
angina, el síncope y la disnea (de mejor a peor pronóstico). La angina de esfuerzo es el síntoma
más frecuente y más precoz, y se debe sobre todo a la hipertrofia del VI. El síncope suele ser de
esfuerzo y ortostático, por la incacpacidad de aumentar el gasto cardíaco con el ejercicio. Si
aparece en reposo es un dato de mal pronóstico. Los pacientes con EAo tienen más riesgo de
muerte súbita, pero ésta suele darse en pacientes que ya son sintomáticos. La disnea por IC
izquierda es el síntoma de peor pronóstico y suele ser el más tardío (opción 4 falsa). La fibrilación
auricular puede disminuir el gasto cardíaco y empeorar rápidamente el estado del paciente,
desencadenando una IC aguda.(R4)

172. Eczema es un término que designa un tipo de reacción inflamatoria cutánea a


diferentes estímulos, y que puede clasificarse en distintos grupos. Señale la afirmación
que le parece correcta en cuanto a este tema:

Los diferentes tipos de eczemas comparten básicamente las características clínicas,


1. 1.
dependiendo los hallazgos histológicos de la etiopatogenia de cada una.
El eczema alérgico de contacto es una reacción de hipersensibilidad tipo IV, por el contacto
2. 2. cutáneo o de las mucosas con una sustancia a la que el paciente está previamente
sensibilizado.
3. 3. El níquel es el agente implicado con más frecuencia en los eczemas irritativos de contacto.
4. 4. Es raro que el eczema atópico se manifieste antes de los dos años de edad.
Gráfico de respuestas
Comentario

Tal como se explica en la respuesta 3, el eccema de contacto es una reacción de hipersensibilidad


retardada. El resto de las opciones son falsas, por los motivos siguientes:

!
!
!
!
R1. La histología no depende del origen del eccema, sino de su evolución (agudo, subagudo,
crónico).

R2. Los antihistamínicos son poco eficaces para el prurito del eccema. El tratamiento de elección
son los esteroides tópicos.

R4. El níquel está implicado en eccemas alérgicos, no irritativos.

R5. El eccema atópico es muy frecuente en la infancia. De hecho, existe la llamada forma del
lactante, bastante anterior a los dos años de edad.(R2)

173. Niña RN de 3 días y medio de vida presenta ictericia mucocutánea de pocas horas
de evolución. Ha sido alimentada al pecho sin incidencias hasta el momento. La
exploración física, a parte de la ictericia, es normal para su edad. Las heces y la orina son
normales. Se realizan estudios de laboraotrio donde se reporta bilirrubina total 18.7
mg/dL. La hemoglobina, hematocrito y los reticulocitos son normales. El grupo
sanguíneo materno es A- y el de la niña es O+. Antecedentes familiares: padre de 33 años
sano, madre de 34 años (G1-A0-P1-C0), diagnosticada de enfermedad de Crohn desde los
14 años. El diagnóstico más probable y el tratamiento indicado en este caso serían:

Ictericia fisiológica. Alta a domicilio y control de la ictericia por su pediatra de forma


1. 1.
ambulatoria.
2. 2. Ictericia por isoinmunización anti-D. Iniciar exanguinotransfusión de forma inmediata.
3. 3. Ictericia fisiológica. Ingreso e iniciar fototerapia simple continua.
Ictericia fisiológica probablemente agravada por lactancia materna. Iniciar fototerapia
4. 4.
simple continua y retirar lactancia materna.
Gráfico de respuestas
Comentario

En los casos clínicos de ictericia neonatal es importante distinguir entre fisiológica y patológica:

- De las patológicas (sobretodo precoz) es muy frecuente las isoinmunizaciones por


incompatibilidad de grupo sanguíneo madre-hijo. Identifique el grupo sanguíneo materno (ABO y
Rh). Posible isoinmunización anti A/B si madre O y recién nacido A, B o AB.

- Fisiológica: Ictericia monosintomática, con predominio de bilirrubina indirecta, fugaz (aparición 2º


y desaparición hacia el 7º día de vida y con bilirrubina por debajo de 12 mg/dl (14 mg/dL en el
pre- término).

El tratamiento de la hiperbilirrubinemia dependerá de las cifras de bilirrubina, de la edad


gestacional, de la edad de vida en horas del recién nacido y de la patología de base.

Existen unas gráficas con las cifras de riesgo según estos factores y los niveles que indican
exanguintransfusión, fototerapia u observación.

En el recién nacido a término considere indicada la fototerapia si bilirrubina total >12 (24h), 15
(48h), 17 (72h) y 19 (>4 días) y la exanguintransfusión por encima de 17 (24h), 22 (48h) y 25 en
adelante.

La ictericia no contraindica la lactancia materna.(R3)

!
!
!
!
174. En la foliculitis vulvar, los principales agentes bacterianos involucrados en su
etiología son:

1. 1. Ureaplasma urealyticum y Mycoplasma hominis.


2. 2. Streptococcus y Staphylococcus.
3. 3. Gardnerella vaginalis y Hemophilus ducreyi.
4. 4. Neisseria gonorroheae y Chlamydia trachomatis.
Gráfico de respuestas
Comentario

La foliculitis se puede desarrollar en cualquier superficie corporal que contenga pelo. Puede
presentarse con pústulas foliculocentricas y se puede observar el pelo en el centro de cada lesión.
Los microorganismos más frecuentes son los cocos gram positivos como Staphylococcus aureus y
Streptococcus.

Como bien sabes Neisseria y Chlamydia son causas de uretritis, cervicitis y EIP. Gardnerella
produce vaginosis bacteriana y H. ducreyi produce el chancroide. Mientras que U. urealyticum y M.
hominis producen infección en las vías urinarias, cervicitis y EIP.(R2)

175. Los padres de una niña de 4 años le


consultan por haber sido remitidos por presencia de cojera en su hija. Refieren que la
paciente lleva así unas 8 semanas y que ha sido valorada por un especialista en
traumatología quien ha realizado una resonancia magnética encontrando en la
exploración un aumento de líquido sinovial, motivo por el cual se ha realizado una
artroscopia con extracción de un líquido cuyos resultados aporta, con 35.400 leuc/mm3
y glucosa de 60 y proteínas de 35 sin encontrar gérmenes en la tinción de gram. En la
exploración física se detecta un aumento de volumen de la rodilla izquierda, con
limitación en los últimos grados de extensión y un aumento de volumen con rubor, de la
tercera interfalángica proximal del cuarto dedo del pie derecho. En la exploración ocular
se detecta la anomalía que aparece en la imagen. Responda la respuesta correcta:

Se trata como primera opción de una artritis idiopática juvenil de inicio sistémico, debemos
1. 1.
iniciar tratamiento con corticoides y, una vez descartadas otras causas, con ciclosporina.
Se trata, como primera posibilidad, por la presencia de una oligoartritis asimétrica de
2. 2.
miembros inferiores, de una artritis relacionada con entesitis por lo que el hallazgo analítico

!
!
!
!
mas probable será la presencia de HLA B27. En este caso, la paciente tien riesgo de
dearrollar uveítis posteriores agudas.
Lo más probable es que la paciente tenga anticuerpos antinucleares positivos y presenta una
elevada posibilidad de que el cuadro ocular sea una uveítis crónica por lo que debemos
3. 3.
solicitar una exploración e indicar tratamiento con ibuprofeno, corticoides y metotrexato, y
si no hay respuesta incluir terapia biológica.
Se trata de una artritis idiopática juvenil pero, en este caso, por la edad de aparición lo más
4. 4. probable es que se trate de una artritis poliarticular seropositiva, por lo que requerirá
tratamiento con Metotrexato y Etanercept en la mayoría de los casos.
Gráfico de respuestas
Comentario

La dificultad de esta pregunta está en reconocer el tipo de artritis crónica juvenil que padece esta
paciente, que sería una forma oligoarticular precoz. Como sabes, ésta suele complicarse con
uveítis crónica, de ahí que podamos encontrarnos alteraciones en el iris como las que se aprecian
en esta imagen, sin que exista excesiva inflamación en el polo anterior del ojo, dado que se trata
de un cuadro crónico. En esta forma, tal como se explica en la respuesta 4, es muy frecuente la
presencia de anticuerpos antinucleares (ANAs). El tratamiento, al menos de inicio, consiste en
AINEs, corticoides y/o metotrexato. Cuando no existe una respuesta adecuada, durante los últimos
años se ha comenzado a utilizar el infliximab u otros anticuerpos anti- TNF, conocidos como
fármacos “biológicos” (respuesta 4 correcta).(R3)

176. Un mioma producirá con mayor frecuencia:

1. 1. Sangrado abundante en el momento esperado de la regla (hipermenorrea).


2. 2. Hemorragia acíclica (metrorragia).
3. 3. Reglas muy frecuentes (polimenorrea).
4. 4. Flujo vaginal serosanguinolento.
Gráfico de respuestas
Comentario

El cuadro clínico característica del mioma submucoso es la hipermenorrea, o sangrado abundante


en el momento de la regla, respuesta correcta 1.

177. Señale la FALSA en relación con la displasia broncopulmonar:

1. 1. Son antecedentes la prematuridad y la enfermedad de membrana hialina.


2. 2. Los diuréticos se han demostrado poco útiles en el manejo a largo plazo.
3. 3. La dexametasona i.v. aumenta el riesgo de hipertensión, infecciones y parálisis cerebral.
4. 4. En la adolescencia algunos pacientes pueden seguir presentando hiperreactividad bronquial.
Gráfico de respuestas
Comentario

La displasia broncopulmonar es aquella situación en la cual un RN sigue necesitando oxígeno a los


28 días de vida. Tiene un correlato radiológico que se conoce como pulmón en esponja. Es
especialmente frecuente en prematuros que sufrieron enfermedad de membrana hialina.
Predispone de forma inespecífica al desarrollo futuro de hiperreactividad bronquial. En su
tratamiento se emplean, además de oxígeno, fisioterapia respiratoria, restricción de líquidos,
diuréticos (opción 2, falsa) y broncodilatadores. El uso de corticoides se ha asociado con
complicaciones graves.(R2)

!
!
!
!
178. Tras una discusión con su novio, una chica de veinte años presenta un episodio de
pérdida de conciencia; en urgencias no responde a órdenes ni presenta respuesta ante
estímulos dolorosos, oponiendo resistencia a la apertura de los párpados. ¿Cuál de los
siguientes datos NO sería congruente con el probable diagnóstico?:

1. 1. Indiferencia afectiva al recuperarse del coma.


2. 2. Antecedentes de episodios similares en su madre.
3. 3. Respuesta a la sugestión.
4. 4. Ausencia de reflejos oculocefálicos.
Gráfico de respuestas
Comentario

Pregunta de fácil deducción, si intuimos que es un cuadro conversivo exclusivamente por un dato,
que no es la discusión previa, sino la resistencia a la apertura parpebral, dado que es un signo
incongruente con la exploración, lo lógico sería que no ofreciera resistencia. Dado que si nos
fijamos en la discusión, podríamos también sugerir que tuvo un aumento importante de la TA y por
ello una rotura de una malformación arterial cerebral que produjera el cuadro... Centrándonos en
las respuestas, las cuatro primeras sugieren características disociativa, excepto la ausencia de
reflejos oculocefálicos que es un signo neurológico claro, y que debe hacernos pensar en cuadros
no psiquiátricos.(R4)

179. Niño de 3 años hijo de madre con VIH es llevado a su consultorio, su actitud con
respecto al niño es:

1. 1. Iniciarle TARGA.
2. 2. Iniciarle profilaxis con cotrimoxazol a dosis pediátricas.
3. 3. Tomarle un ELISA para VIH.
4. 4. Darle Nevirapina 1 dosis y Zidovudina por 6 semanas.
Gráfico de respuestas
Comentario

En hijos de madres diabéticas se debe iniciar la evaluación con la realización de ELISA para
detectar VIH y en caso de ser positivo, identificar los criterios para inicio de antirretrovirales.
Detecte que la pregunta dice 3 años de edad, por lo que a esta edad ya desaparecieron los
anticuerpos de la madre.(R3)

180. Con respecto a las parasomnias, señale la respuesta INCORRECTA:

1. 1. El sonambulismo aparece en la fase delta del sueño no REM.


2. 2. Los terrores nocturnos aparecen durante el sueño REM.
3. 3. El bruxismo aparece en la fase 2 del sueño.
4. 4. El sonambulismo tiende a asociarse a otras parasomnias.
Gráfico de respuestas
Comentario

Los terrores nocturnos aparecen durante el sueño profundo, es decir, sueño no- REM (respuesta 2
correcta). Son las pesadillas las que aparecen durante la fase REM. En cuanto al sonambulismo,
que también se menciona entre las opciones, tiene cierta asociación con los terrores nocturnos, por
lo que también aparece en fase no- REM.(R2)

!
!
!
!
181. Todas las siguientes circunstancias, EXCEPTO una, se han asociado a la trombosis
venosa renal:

1. 1. Deshidratación en la lactancia.
2. 2. Síndrome nefrótico.
3. 3. Fibrilación auricular.
4. 4. Embarazo y anticonceptivos orales.
Gráfico de respuestas
Comentario
Esta pregunta es muy sencilla y NO DEBES FALLARLA. La fibrilación auricular se relaciona con
embolismos mientras que el resto de las opciones tienen que ver con situaciones de
hipercoagulabilidad (embarazo y anticonceptivos orales), hiperviscosidad sanguínea
(deshidratación, Síndrome nefrótico) o situaciones que afecten directamente la vena renal (es
típico en caso del TRAUMATISMO DEL CINTURON DE SEGURIDAD).(R3)

182. Indique cuál de las siguientes características respecto a la invaginación intestinal es


FALSA:

1. 1. La mayor parte de las invaginaciones aparecen en niños menores de 2 años.


2. 2. La imagen en "dona", en el ultrasonido abdominal, es característica de este cuadro.
3. 3. La presencia de peritonitis como complicación es precoz.
4. 4. La reducción hidrostática con enemas no está indicada ante invaginaciones recidivantes.
Gráfico de respuestas
Comentario

La peritonitis en el contexto de una invaginación, aparece como complicación tardía (respuesta 3


falsa).

La localización más frecuente de la invaginación intestinal es ileocólica. En estos casos, su causa


más frecuente es de origen desconocido. Sin embargo, la colocólica, que es una rareza, sí que se
asocia a tumores malignos.

Por último, merece la pena recordar la clínica típica de la invaginación intestinal, que es un dolor
abdominal cólico, acompañado de crisis de llanto, palidez cutánea y encogimiento de miembros
inferiores.(R3)

183. Agente etiológico más frecuentemente causante de bronquiolitis y neumonía en


niños menores de 1 año.

1. 1. Virus parainfluenzae.
2. 2. Haemophilus influenzae.
3. 3. Virus Sincitial Respiratorio.
4. 4. Virus influenzae.
Gráfico de respuestas
Comentario

El agente más frecuentemente implicado en la bronquiolitis aguda es el virus respiratorio sincitial.


Respuesta 3 correcta.

!
!
!
!
184. Un paciente de 34 años de edad ha sido recientemente diagnosticado de
hipertensión arterial. En la exploración física se encuentra debilidad en los pulsos de
extremidades inferiores con TA de 100/70 mmHg en los mismos, y una TA en los
miembros superiores de 165/90 mmHg. Además, se ausculta un soplo continuo en la
parte anterior del tórax y en la región interescapular. Este paciente presenta con más
probabilidad una de las siguientes patologías:

1. 1. Mixoma auricular.
2. 2. Síndrome de Romano-Ward.
3. 3. Aneurisma de aorta abdominal.
4. 4. Coartación de aorta.
Gráfico de respuestas
Comentario

Las cardiopatías congénitas constituyen un tema difícil, ya que hay muchas y son difíciles de
recordar. Sin embargo, existen unas pocas muy preguntadas, y por ello más rentables, como
sucede con la coartación aórtica. Lo primero que debes recordar es la asimetría que existe entre
los pulsos de los MMSS (normales o aumentados, porque los vasos subclavios salen antes de la
coartación) y los de los MMII (disminuidos, porque obviamente las arterias ilíacas y femorales son
distales a la coartación). Debido a la disminución de flujo sanguíneo a las zonas distales a la
obstrucción, se desarrolla circulación colateral para suplir el déficit y de ahí surgen el resto de
signos clínicos y RX:

- HTA por encima del nivel de la coartación.

- Soplo a nivel de la coartación.

- Muescas costales en la RX tórax, por gran desarrollo de las arterias subcostales (signo de
Roesler).

- Signo del “3” en la Rx tórax. Se debe a la dilatación pre y postestenótica que ocurre a nivel de la
propia coartación.

- Signo de la “E” en el esofagograma con bario. Su explicación es semejante al signo del 3.(R4)

185. Recién nacido a término que experimenta cianosis en la sala de partos. Después de
la intubación y medidas para su estabilización se observa abdomen escafoideo y
disminución del murmullo vesicular en hemitorax izquierdo. El diagnóstico más probable
es:

1. 1. Neuroblastoma.
2. 2. Atelectasia.
3. 3. Hernia diafragmática.
4. 4. Neumotórax.
Gráfico de respuestas
Comentario

La respuesta correcta es la 3, hernia diafragmática, ya que te dan todos los datos: cianosis,
abdomen escafoideo y los ruidos respiratorios no se escuchan en el hemitórax izquierdo. La hernia
en la que debes pensar es la de BochdaLek recuerda Back and Left.(R3)

!
!
!
!
186. Which of the following statements regarding the central venous pressure is FALSE?

The internal jugular vein is the most appropriate vein for estimation of central venous
1. 1.
pressure.
High central venous pressure should be examined with the patient lying in supine position
2. 2.
in order to observe the venous pulse wave.
Central venous pressure can be considered elevated when the filling level of the jugular
3. 3.
vein is more than 3 cm vertical height above the sternal angle.
The most common cause of elevated central venous pressure is increased right ventricular
4. 4.
diastolic pressure.
Gráfico de respuestas
Comentario

Pregunta muy fácil sobre la presión venosa yugular. Esta pregunta es muy intuitiva, aunque no
conociera todas las opciones, ya que si ha visto explorar alguna vez la PVY no podría dudar.

La presión que hay en las venas yugulares (PVY) equivale a la presión auricular derecha (presión
venosa central, PVC). Su equivalente en el lado izquierdo sería la presión de enclavamiento
pulmonar (PCP, equivalente a la presión en aurícula izquierda, PAI) que se mide con un catéter de
Swan-Ganz. A su vez, la presión de las aurículas durante la diástole, en ausencia de obstrucción
en las valvas auriculoventriculares, es equivalente a la presión del ventrículo derecho (o del
izquierdo si nos referimos a la PCP).

La PVY se mide viendo el latido yugular con el paciente en decúbito y levantado el tórax unos 45º
(opción 2 falsa: correcta). Esto se explica por la teoría de los vasos comunicantes. Se mide la
altura con respecto al ángulo de Louis (manubrio esternal, que está a unos 5 cm de AD), y rara vez
se eleva más de 3 cm por encima de éste (lo que equivaldría a unos 8-9 cm de agua) (opción 3
cierta). La causa más frecuente de elevación de la presión venosa es un aumento de la presión
diastólica ventricular derecha (opción 4 cierta). Si tenemos un catéter central, medimos
directamente la PVC en la vena yugular interna (opción 1 cierta).(R2)

Medición de la presión venosa central

!
!
!
!

187. Paciente femenino de 48 años con


antecedentes patológicos de asma y apendicectomía en la adolescencia, que presenta
dolor abdominal y vómitos, con hallazgos exploratorios de aumento del peristaltismo. Se
decide realizar una Rx simple de abdomen en bipedestación que revela lo que se muestra
en la imagen. ¿Cuál de las siguientes premisas NO es cierta?:

En los casos secundarios a herniación la reducción manual es inadmisible si existe sospecha


1. 1.
de estrangulación.
El tratamiento es fundamentalmente quirúrgico, pudiendo ser necesaria, en ocasiones, la
2. 2.
resección del segmento no viable.
3. 3. La mortalidad se estima alrededor del 10%.
4. 4. El examen del paciente debe completarse con la exploración de los puntos herniarios.
Gráfico de respuestas
Comentario

La presencia de niveles hidroaéreos nos debe hacer sospechar que nos plantean un caso de
obstrucción intestinal. Recordemos que la principal causa de obstrucción mecánica de intestino
delgado son las adherencias postquirúrgicas. En pacientes que nunca han sido sometidos a cirugía
abdominal, serían las hernias; pero en nuestro caso existen antecedentes de apendicectomía y,
por tanto, la exploración del paciente debe incluir los puntos herniarios (opción 4 correcta). Su
mortalidad se sitúa en el 10% (opción 3 correcta).

Dado que las hernias son una de las principales causas de obstrucción de intestino delgado, debe
conocer su tratamiento. Así, ante una hernia sin sospecha de estrangulación, se puede intentar la
reducción manual bajo sedación suave. Sin embargo, ante la sospecha de hernia estrangulada
siempre se debe abrir el saco herniario para valorar la viabilidad del segmento intestinal ya que, en
este caso, también existiría un compromiso de la vascularización. En obstrucciones por otras
causas, debes conocer que, en la mayor parte de los casos, se resuelven con tratamiento
conservador y no se precisa intervención quirúrgica.(R2)

!
!
!
!
188. An 18-year-old patient comes to the emergency department with abdominal pain for
the last 4-6 hours. The pain was initially periumbilical but it is now located in the right
lower quadrant (RLQ). His temperature is 38 ° C and the arterial pulse is 100 l/min. Physical
examination reveals tenderness in the RLQ. Lab tests show: WBC count 17,500 / mm3
with a left shift. Which of the following is the most appropriate next step in management?

1. 1. Abdominal X-ray
2. 2. Abdominal CT scan
3. 3. Repeat physical examination and lab tests in 12-24 hours
4. 4. Laparotomy
Gráfico de respuestas
Comentario
Laparotomy. The patient presents with symptoms and signs that suggest accute appendicitis.
Appendicitis is defined as an inflammation of the inner lining of the vermiform appendix that spreads
to its other parts. Most patients present with anorexia and periumbilical pain followed by nausea,
right lower quadrant (RLQ) pain, and vomiting. Once the diagnosis is clear, patients should undergo
an appendicectomy, which can be open (laparotomy) or with laparoscopy.(R4)

189. Respecto a la exploración funcional del paciente con EPOC señale la FALSA:

1. 1. El porcentaje del FEV1 en relación con el teórico se utiliza para clasificar la gravedad.
2. 2. Existe un aumento del VR, del CRF y del VR/CPT.
3. 3. Existe una clara relación, en pacientes no graves, entre el FEV1 y la gasometría.
4. 4. La DLCO disminuye cuando predomina el enfisema.
Gráfico de respuestas
Comentario
Pregunta sencilla si conoces bien la fisiopatología del paciente EPOC y si tienes alguna duda
repásala bien e intenta comprenderla, ya que casi todos los años cae alguna pregunta sobre ella.
Debes recordar que el acontecimiento fisiopatológico clave del EPOC es la obstrucción a la salida
del flujo aéreo, por lo que se traduce en un aumento del VR por secuestro, lo que lleva
consecuentemente al aumento de la CRF y del VR/CPT. Pese a esto el FEV1 por sí solo no sirve
para diagnosticar la enfermedad, pero sí si se pone en relación con la CVF, en lo que se denomina
índice de Tiffeneau. Sin embargo la utilidad del FEV1 es para fijar la gravedad y el pronóstico del
paciente, una vez diagnosticado éste, ya que no existe relación clara entre el FEV1 y la gasometría
(por tanto la repercusión clínica y complicaciones). Otro dato típico que debes conocer por su
importancia MIR es que el enfisema se caracteriza por la disminución de la DLCO (este dato ha
sido muy preguntado).(R3)

190. Varón de 6 años de edad que comenzó hace 1 año con episodios de epistaxis y
petequias recurrentes y cada vez más severos. Desde entonces ha presentado 2 cuadros
de neumonía por Pseudomonas aeruginosa. Acude a urgencias por presentar un cuadro
de astenia y palpitaciones, así como nuevo episodio de epistaxis. A la exploración
destaca palidez cutaneomucosa, manchas café con leche y microcefalia. Se objetiva
cierto grado de retraso mental. En el análisis sanguíneo destaca: hemoglobina 9 g/dl;
reticulocitos corregidos 0,5%; neutrófilos 450/mm3; plaquetas 20.000/mm3. Teniendo en
cuenta el diagnóstico más probable de este paciente, ¿cuál de las siguientes
aseveraciones es FALSA?:

La disminución del numero de células hematicas, se correlaciona con la menor frecuencia


1. 1.
de leucemias en estos pacientes.

!
!
!
!
La mayor parte de anomalías cromosómicas se deben a un defecto de la reparación del
2. 2.
ADN.
3. 3. En la radiografía de antebrazo puede verse ausencia del radio.
4. 4. La estatura suele ser baja.
Gráfico de respuestas
Comentario
Pregunta tipo caso clínico, de dificultad elevada, donde lo importante es identificar el diagnostico
más probable que afecta a este paciente. Nos presentan a un niño con hemorragias, infecciones y
palidez, y luego con el hemograma se confirma el diagnostico de pancitopenia, con reticulocitos
disminuidos. Causas de pancitopenia hay muchas, pero que afecten a un niño de 6 años, asociado
a manchas café con leche y microcefalia, nos debe hacer de sospechar en una aplasia medular.
Las aplasias medulares congénitas o constitucionales, son enfermedades con base genética que
se detectan en los primeros años de vida; su causa más frecuente es la anemia de Fanconi que se
asocia a malformaciones óseas debido a un trastorno en la reparación del ADN (opción 3 correcta),
de herencia autosómica recesiva (opción 1 correcta), con un aumento de la fragilidad
cromosómica; esto explica el numero elevado de neoplasias malignas, no solo de origen
hematológico de estos pacientes (opción 2 falsa). Entre las malformaciones se encuentra la baja
estatura (opción 5 correcta), malformaciones del pulgar y del radio (opción 4 correcta), microcefalia,
anomalías vasculares , manchas cutáneas y una fascie característica, aunque el retraso mental no
es un hallazgo habitual se puede encontrar como en este paciente.(R1)

191. Femenino de 19 años, con 40 días de amenorrea, a la que se administra una dosis
de 200 mg de progesterona IM, produciéndose sangrado vaginal 2 días después. Este
resultado permite afirmar que:

1. 1. Hay un adecuado nivel de estrógenos.


2. 2. No ha habido ovulación.
3. 3. Hay un endometrio intacto hormonalmente receptivo.
4. 4. Todas las anteriores.
Gráfico de respuestas
Comentario

Si la amenorrea se debe a anovulación, típicamente responde a la administración de progesterona,


produciéndose la menstruación después. En estas amenorreas, hay folículos en los ovarios
liberando estrógenos en los niveles adecuados, pero al no haber ovulación no se forma el cuerpo
lúteo, y por tanto el ovario no libera progesterona, necesaria para que el endometrio alcance la
fase secretora y finalmente se desprenda en la menstruación.

Por último, si la paciente ha respondido a la administración de progesterona, significa que su


endometrio es normal y perfectamente receptivo a las hormonas.(R4)

192. RN con síndrome de Down presenta vómitos biliosos a las 36 horas de vida. A la
exploración destaca abdomen excavado. El diagnóstico más probable es:

1. 1. Estenosis hipertrófica de píloro.


2. 2. Enfermedad de Hirschprung.
3. 3. Atresia duodenal.
4. 4. Invaginación intestinal.
Gráfico de respuestas
Comentario

!
!
!
!
Le hablan de vómitos biliosos, por lo que puede descartar estenosis hipertrófica de píloro. La
opción 2 le da una clínica totalmente distinta, y la invaginación intestinal de igual forma. Si recuerda
un dato típico de niños con síndrome de Down es la atresia duodenal, "signo de la doble burbuja"
respuesta correcta 3.

193. Primigesta de 32 años que acude a urgencias refiriendo sangrado genital y prueba
de embarazo en farmacia positiva. Se realiza exploración ginecológica, ultrasonido y
exámenes de laboratorio que sugieren el diagnóstico de mola vesicular. ¿Cuál sería el
tratamiento de elección?

1. 1. Quimioterapia con metotrexato.


2. 2. Legrado uterino con legra de Recamier y pinza Winter.
3. 3. Legrado por aspiración.
4. 4. Histerectomía simple (conservando ovarios).
Gráfico de respuestas
Comentario

Pregunta que hace referencia a la mola hidatiforme, que consiste en una proliferación excesiva del
trofoblasto junto con edema de vellosidades. El curso clínico supone un embarazo con tamaño
uterino mayor que el que corresponde por amenorrea con metrorragia a partir del 2º mes.

La prueba diagnóstica de elección es el ultrasonido, en el que se aprecia una imagen característica


en “copos de nieve” que corresponde a vesículas hidrópicas.

El diagnóstico de certeza nos lo da la anatomía patológica.

El tratamiento consta de dos partes: evacuación de la mola y seguimiento posterior de la


enfermedad.

El tratamiento evacuador de elección es el legrado por aspiración. Debe evitarse la histerectomía,


dado que se trata de una mujer joven nulípara.(R3)

194. ¿Cuál de los siguientes recursos es el más adecuado para tratar la


hiperbilirrubinemia (11.2 mg/dl) en un lactante de 3 semanas alimentado al pecho y con
crecimiento y desarrollo normales?:

1. 1. La fototerapia.
2. 2. Exanguinotransfusión.
3. 3. Retirar definitivamente la lactancia.
4. 4. Continuar con lactancia materna y tranquilizar a la madre.
Gráfico de respuestas
Comentario

El síndrome de Arias es la modalidad de ictericia evocada en esta pregunta.

Este síndrome, presente en 1/200 alimentados al pecho, genera en la mayor parte de los casos
una hiperbilirrubinemia de predominio indirecto discreta (como la del enunciado).

En su patogenia se ha implicado a los pregnanos de la leche de madre, que inhiben la


glucoroniltransferasa.

!
!
!
!
El manejo del síndrome de Arias es conservador: hay que explicarle a la madre que la ictericia
cederá a las tres o cuatro semanas de vida del niño, e invitarle a que continúe ofreciéndole el
pecho.(R4)

195. En la pleuritis tuberculosa:

1. 1. En general se encuentra el bacilo de Koch en el examen del líquido pleural.


Hay una alta probabilidad de aparición de lesiones tuberculosas tardías si no se trata
2. 2.
adecuadamente.
3. 3. El líquido tiene las características de un trasudado.
4. 4. El germen causal suele ser resistente a hidracida.
Gráfico de respuestas
Comentario
Dentro de las enfermedades de la pleura, el tema del derrame pleural es una de las partes más
importantes y que se debe dominar, sobre todo en relación con las características del líquido
pleural, su etiología y tratamiento. En este caso nos hablan de un tipo en concreto y nos piden sus
características, pero también lo pueden preguntar al revés. En el caso de la TBC, lo que hay que
conocer es que puede ser la causa más frecuente de exudado (opción 4 incorrecta) en países en
vías de desarrollo (aunque recuerda que en general, la causa más frecuente es el derrame
paraneumónico). Suele ser unilateral y presentar un predominio linfocitario (pero ten en cuenta que
en fases iniciales puede haber un predominio de polimorfonucleares), disminución de la glucosa
(<60mg/dl aunque esto también puede aparecer en el paraneumónico, en tumores, en la AR siendo
en esta última donde vamos a encontrar los niveles más bajos), presencia de ADA (aunque es muy
orientativa hay que saber que también aparece en la AR, linfoma y empiema), IFN gamma(que es
mucho más específico). Sin embargo nos debe hacer dudar en TBC si encontramos eosinófilos
(que aparecen en relación con aire, sangre, fármacos, parásitos, vasculitis), o >5% de células
mesoteliales. Para el diagnóstico se requiere en general una biopsia pleural cerrada, ya que el
mantoux es negativo en 1/3 de los casos (opción 1 incorrecta) y el cultivo sólo es positivo en el 15-
20% de los casos (opción 2 incorrecta). El tratamiento es exactamente igual que el de la TBC
pulmonar, con los tres fármacos durante 6 meses (opción 5 incorrecta)(R2)

196. Niño de 8 años de edad, que es traído porque la mamá lo nota “muy gordito” y lo
considera obeso. ¿cuál de los parámetros en percentil usaría usted para definir si el niño
tiene obesidad? (IMC= índice de masa corporal; P/T=peso para talla; P/E= peso para la
edad):

1. 1. IMC mayor de 95.


2. 2. P/T mayor de 50.
3. 3. P/E mayor de 90.
4. 4. IMC de 85 a 90.
Gráfico de respuestas
Comentario

Se considera obesidad infantil, cuando el IMC es superior a 95, mientras que de 85 a 95 se


considera sobrepeso. Respuesta 1 correcta.

197. Un paciente de 23 años, adicto a drogas por vía parenteral, presenta desde hace 24
horas inflamación y dolor en rodilla izquierda. No se ha termometrado, pero tiene
escalofríos frecuentes. Se realiza artrocentesis sin obtener líquido sinovial. Ante la
sospecha de artritis séptica iniciaría el tratamiento antibiótico con:

!
!
!
!
1. 1. Cloxacilina oral y revisión en 48 horas.
2. 2. Cloxacilina más gentamicina i.v.
3. 3. Antifúngicos i.v.
4. 4. Amoxicilina vía oral.
Gráfico de respuestas
Comentario

El tema de las artritis sépticas sigue teniendo importancia en el ENARM. El paciente es un adulto
joven ADVP con una monoartritis y probable fiebre o estado séptico. Se ha realizado la obligada
artrocentesis, que lamentablemente no nos ayuda a confirmarlo. De todas formas la sospecha es
firme: artritis por S.aureus, con el peligro de endocarditis, etc. El tratamiento de elección del
estafilococo es Cloxacilina (confiamos que es un germen no resistente dado que proviene de la
comunidad), y se asocia a gentamicina los primeros días para que, si hay otros gérmenes
implicados, no se escapen a la espera del cultivo. La vía es i.v., es un cuadro severo, y desde
luego nunca antibióticos intraarticulares. Debe drenarse el derrame por artrocentesis todos los
días, y si persiste o evolucional mal, se practica artrotomía o más modernamente artroscopia.(R2)

198. La otitis externa maligna o necrosante:

1. 1. Está generalmente producida por gérmenes gramnegativos (P. aeruginosa).


2. 2. Afecta fundamentalmente a niños sanos.
3. 3. No suele sobrepasar el conducto auditivo externo.
4. 4. Rara vez es necesario realizar una limpìeza quirúrgica.
Gráfico de respuestas
Comentario

La otitis externa maligna se desarrolla a partir de una otitis externa difusa, por lo tanto el germen
más frecuente es el mismo en ambas, esto es, la Pseudomonas aeruginosa. Suele afectar a
pacientes inmunodeprimidos, diabéticos o ancianos. Se debe a una afectación más allá de las
partes blandas del CAE afectando al hueso subyacente desde donde puede extenderse al hueso
de la base del cráneo provocando una osteomielitis. Su avance provoca lesiones de pares
craneales (primero el VII, luego el V, IX, X y XI) y la muerte del sujeto. El tratamiento debe ser
enérgico empleándose antiboticoterapia endovenosa durante largo tiempo y, en ocasiones, cirugía
para desbridar tejido necrótico.(R1)

199. El diagnóstico de artritis reumatoidea juvenil es fundamental por:

1. 1. Presencia de factor reumatoideo.


2. 2. Por descarte clínico.
3. 3. Por transaminasas altas.
4. 4. Por ninguno de los anteriores.
Gráfico de respuestas
Comentario

El diagnóstico de ARJ se realiza por descarte clínico, respuesta 2 correcta. Repase la siguiente
tabla de las características principales de esta enfermedad.

!
!
!
!

200. Mujer de 22 años que acude a consulta de ginecología remitida por su médico de
cabecera porque en una exploración abdominal se le palpa una masa de probable
localización ovárica. La paciente está asintomática. La exploración física y el USG
transvaginal confirman la existencia de una masa en ovario izquierdo de aspecto mixto
sólido-quístico. Se decide intervenir con fines diagnóstico terapéuticos. La anatomía
patológica nos confirma la existencia de un tumor ovárico de tipo disgerminoma limitado
al ovario, y que no invade su superficie del mismo. Dado que la mujer desea tener
descendencia en un futuro y el tumor está en estadio Ia, se plantea tratamiento quirúrgico
conservador, para el que hay que considerar los siguientes datos EXCEPTO:

1. 1. Estadio Ia, grados de diferenciación G1 o G2.


2. 2. Extirpación del ovario contralateral cuando la mujer haya completado sus deseos genésicos.
3. 3. Ovario contralateral normal.
4. 4. Edad menor de 25 años.
Gráfico de respuestas
Comentario

El conocimiento de los requisitos para el tratamiento quirúrgico conservador del cáncer de ovario
requiere un estudio en profundidad de esta materia y pueden no ser muy rentables.

Simplemente recuerde que en mujeres con deseos reproductivos, con tumor bien diferenciado y
limitado al ovario con cápsula intacta, sin ascitis tumoral ni lavado peritoneal positivo (IAG l) es
posible cirugía conservadora con anexectomía unilateral y completar la extirpación después de
cumplir con sus deseos reproductivos.

La falsa es claramente la respuesta 4 ya que la edad de la paciente no desaconseja esta actitud


quirúrgica, pero tampoco sería necesario quitar el ovario contralateral.

!
!
!
!
Recuerde que en el caso de tumor bien diferenciado limitado al ovario no es necesario la
quimioterapia adyuvante.(R4)

201. RN que al minuto de vida, presenta FC de 110 latidos por minuto, acrocianosis con
esfuerzo respiratorio ausente, así como hipotonía y leve mueca al introducir la zona de
aspiración. La puntuación de Apgar será:

1. 1. 0.
2. 2. 2.
3. 3. 5.
4. 4. 6.
Gráfico de respuestas
Comentario

A: APARIENCIA, 0 azul, 1 sonrosado, 2 totalmente sonrosado

P: PULSO (FRECUENCIA CARDIACA) 0 no, 1 menos de 100, 2 más de 100

G: GESTO O MUECA 0 ausente, 1 mueca, 2 tos o estornudo.

A: ACTIVIDAD O TONO MUSCULAR 0 debil, 1 ligera flexión, 2 activo

R: RESPIRACIÓN (PRESENCIA Y NO FRECUENCIA) 0 ausencia, 1 débil, 2 bueno llanto.(R4)

202. Un paciente de 28 años de edad ingresa por dificultad respiratoria, sin antecedentes
de enfermedad obstructiva pulmonar previa. En la exploración física se deteta
ginecomastia. En su radiografía de tórax y TC torácico se muestran múltiples imágenes
y nódulos pulmonares sugestivos de suelta de globos. En los exámens de laboratorio
destaca una elevación de beta-HCG (12.000 U/l). ¿De qué tumor primario es más probable
que estemos hablando?

1. 1. Carcinoma embrionario de testículo.


2. 2. Seminoma.
3. 3. Tumor seno endodérmico.
4. 4. Coriocarcinoma testicular.
Gráfico de respuestas
Comentario

Un caso clínico de moderada dificultad.

Dados los diagnósticos que aparecen entre las opciones, junto con la elevación de la beta-hCG, ya
podríamos imaginarnos que se trata de algún tipo de cáncer testicular. De todos ellos,
coriocarcinoma sería la opción más razonable, por las razones siguientes:

•! Es el que con más frecuencia eleva la beta-hCG, y en mayor cantidad.


•! Tiende a producir metástasis hematógenas precoces, incluso antes de que el paciente se
haya percatado de tener una masa en el testículo.

El seminoma, aunque es el cáncer testicular más frecuente, no suele producir este patrón
metastásico sin producir primero una importante masa en testículo.(R4)

!
!
!
!
203. En la diarrea acuosa de tipo osmótico, lo siguiente es probable, EXCEPTO:

1. 1. Un agente frecuente es el rotavirus.


2. 2. El pH de las heces es generalmente alcalino.
3. 3. La cantidad de sodio en las heces es menor que en las diarreas secretoras.
4. 4. Presencia de cuerpos reductores en heces.
Gráfico de respuestas
Comentario

El pH de las heces es generalmente en las diarreas osmóticas es ÁCIDO, por lo que la respuesta
incorrecta es la 2, recuerda que por eso se irrita la zona perianal.(R2)

204. ¿Con cuál de las siguientes técnicas puede diagnosticar un gastrinoma con más
seguridad?:

1. 1. Prueba de la D-xilosa.
2. 2. Arteriografía selectiva.
3. 3. Test del aliento.
4. 4. Prueba de la estimulación con secretina.
Gráfico de respuestas
Comentario
En aquellos pacientes en los que se sospecha un gastrinoma, se debe determinar en primer lugar
la gastrina basal. Pero si su valor es menor de 1.000 pg/mL, está indicada la realización de una
prueba de estimulación con secretina. En los gastrinomas, tras la infusión de secretina, las cifras
de gastrina se elevarían enormemente. En cambio, el resto de las pruebas no serían adecuadas
para este diagnóstico.(R4)

205. A 32-year-old patient comes to your office due to depressed mood over the past 8
months. He has been feeling very tired, without any appetite, and he no longer wants to
go out with his friends. He can't get enought sleep during the night, and as a result, he is
unable to concentrate at work. You decide to start fluoxetine. Which side effect should
you warn the patient about?

1. 1. Hypothyroidism
2. 2. Hypertension
3. 3. Impotence
4. 4. Elevated liver enzymes
(R3)

206. Femenino de 43 años que presenta tumoración de 5 cm en ovario izquierdo, con


índices de resistencia disminuidos y zona sólidas-mixtas en su interior en la ecografía.
Hace 8 años fue intervenida por una masa anexial en el mismo lado, con diagnóstico
anatomopatológico de endometrioma. Presenta elevación de marcadores tumorales,
especialmente del CA 125. ¿Cuál es la estirpe tumoral que más probablemente presente
la paciente?:

1. 1. Teratoma.
2. 2. Adenoma mucinoso.
3. 3. Tumor de células claras.

!
!
!
!
4. 4. Tumor del seno endodérmico.
Gráfico de respuestas
Comentario

En las pacientes con antecedentes de endometriosis los tumores ováricos que se asocian con
mayor frecuencia son los carcinomas endometriodes y los carcinomas de células claras, siendo
estos últimos los mas frecuentes relacionados con la endometriosis. Recuerda que en ambos
existe contraindicación para la terapia hormonal sustitutiva, ya que dicho tratamiento puede
favorecer su proliferación por responder al estímulo estrogénico.(R3)

207. Los tumores ováricos más frecuentes son:

1. 1. Epiteliales.
2. 2. Del estroma.
3. 3. Quiste dermoide.
4. 4. Ninguna.
Gráfico de respuestas
Comentario

Los tumores epiteliales son la estirpe más frecuente de cáncer de ovario y suelen aparecer en
mujeres de edad avnazada. De ellos, la variedad más habitual son los serosos, seguidos de los
mucinosos.

Los tumores germinales suelen aparecer en mujeres jóvenes. La gran mayoría corresponden al
teratoma quístico maduro o quiste dermoide, que es benigno Otros tumores de este grupo son el
disgerminoma, tumor del seno endodérmico (produce AFP), carcinoma embrionario (produce AFP),
coriocarcinoma (produce HCG ).

Los tumores de los cordones sexuales-estroma son la estirpe menos frecuente y se caracterizan
por la síntesis de hormonas esteroideas que permiten que se puedan diagnosticar más
precozmente que otros tipos. Los más habituales son los tumores de la granulosa (síntesis
estrogénica), seguidos de los tecafibromas (síntesis de andrógenos).(R1)

208. Una primigesta de 37 semanas de edad gestacional ingresa por rotura prematura de
membranas de 24 horas de evolución. No hay trabajo de parto ni signos de infección
amniótica. El test de Bishop (de maduración cervical) es de 6 puntos. El feto está en
cefálica y no hay signos de sufrimiento fetal. ¿Cuál es la conducta obstétrica indicada?

1. 1. Inducción del parto con oxitocina.


2. 2. Administar antibióticos y corticoides e inducir el parto 48 horas después.
3. 3. Administrar antibióticos y esperar el comienzo espontáneo del parto.
Esperar el comienzo espontáneo sin tratamiento y hacer cesárea si aparecen signos de
4. 4.
infección amniótica.
Gráfico de respuestas
Comentario

Caso clínico sencillo, ya que te dan todos los datos necesarios para saber cuál es la respuesta
correcta.

Para enfocar este caso clínico tiene que darse cuenta de que es una gestante a término, de modo
que la actitud correcta es terminar la gestación. Las opciones 2, 3 y 4 incluyen esperar, por lo que

!
!
!
!
hay que descartarlas, puesto que sólo implican aumento de la morbilidad materna y fetal (aparición
de corioamnionitis), sin aportar beneficios.

Para decidir la mejor forma de inducir el parto, tenemos que basarnos en el valor del test de
Bishop. Si es mayor o igual a 5, el procedimiento es la inducción con oxitocina.(R1)

209. Todos los siguientes procesos se manifiestan frecuentemente como colestasis


neonatal, EXCEPTO:

1. 1. Hepatitis B.
2. 2. Hipotiroidismo.
3. 3. Galactosemia.
4. 4. Tirosinemia.
Gráfico de respuestas
Comentario

La hepatitis B, en el período neonatal cursa de modo rigurosamente asintomático. A pesar de esta


inocencia sintomática, hay una clara predisposición a desarrollar un estado de portador crónico. De
ahí la peculiaridad de la infección transmitida de forma vertical. El resto de las patologías
enumeradas, al igual que otras entidades (hepatitis neonatal idiopática, atresia biliar extrahepática,
etc), cursan con colestasis.

Por cierto debes manejar a a perfección la profilaxis ante el nacimiento de una madre portadora de
hepatitis B es muy importante para el nacional.(R1)

210. A 5-year-old child is brought to the pediatrician's office presenting with irritability,
headache, malaise and food refusal. The girl was previously healthy but yesterday she
began with fever and diarrhea. Physical examination shows an erythematous pharynx and
diffuse abdominal pain. She vomits during the visit. You notice nuchal rigidity. Lumbar
puncture is performed and biochemistry results of cerebrospinal fluid are: Glucose 55
mg/dL, Proteins 79 mg/dL, RBC 11/mm3, WBC 100/mm3 (lymphocytes 80%). Which of the
following is the most likely causative agent?

1. 1. M. tuberculosis.

!
!
!
!
2. 2. Listeria monocytogenes.
3. 3. S. aureus.
4. 4. Echovirus.
Gráfico de respuestas
Comentario
Echovirus. The most common causes of acute meningitis during childhood are viruses, especially
Echovirus. The cerebrospinal fluid analysis will show an increase in lymphocyte count and proteins
with normal glucose levels. Abdominal pain and erythematous pharynx support the suspicion of a
viral cause.(R4)

211. Acude a su consulta una paciente de 22 años consultando por amenorrea desde
hace un año. Menarquia a los 14 años con reglas cada 29 días de 5 días de duración hasta
hace un año. Las determinaciones hormonales son las siguientes: prolactina 18 ng/ml,
FSH 3 mUI/ml, LH 1 mUI/ml, estradiol 13 pg/ml, progesterona 0.3 ng/ml. Tras la
administración de acetato de noretisterona 5 mg durante 10 días no consigue
menstruación. Con un preparado anticonceptivo con 30 µg de etinilestradiol y 3 mg de
drospirenona se consigue menstruación al suspenderlo. Respecto a la amenorrea
secundaria que tiene la paciente, es CIERTO que:

1. 1. La causa de la amenorrea se debe a etiología ovárica.


2. 2. En la ecografía, el endometrio está atrófico.
3. 3. Se debe pedir RMN de silla turca para descartar la presencia de un prolactinoma.
4. 4. El IMC de la paciente suele estar por encima de 18.
Gráfico de respuestas
Comentario

Caso clínico en que se nos cuenta una amenorrea secundaria en la que sólo con los datos de
laboratorio podemos establecer el diagnóstico. Tenemos una paciente con un hipogonadismo
(estradiol <20 pg/ml) hipogonadotropo (FSH y LH <5 mUI/ml), por lo que la causa es central y no
ovárica (hipergonadotropo), por tanto 1 es falsa. Al no tener estradiol, el endometrio no se estimula
y ecográficamente se ve atrófico. En las amenorreas centrales se puede realizar una RM de silla
turca para descartar adenomas de hipófisis, pero en este caso no buscamos un prolactinoma (>100
ng/ml) ya que el valor de la prolactina es normal (<20 ng/ml). Estas amenorreas centrales suelen
deberse a bajo peso de la paciente, entrando en amenorrea secundaria cuando su IMC es <18 o la
masa grasa <23%.(R2)

212. A 3-year-old boy is brought to the physician because of a 10-day-history of fever,


weight loss, skin rash and nonexudative conjunctivitis bilaterally. Physical examination
shows mild splenomegaly, a painful cervical lymphadenopathy and peeling of skin at
fingertips. Which of the following is the most likely diagnosis?

1. 1. Schönlein-Henoch syndrome.
2. 2. Rubella.
3. 3. Kawasaki's disease.
4. 4. Lupus erythematosus in children.
Gráfico de respuestas
Comentario
La enfermedad de Kawasaki se define por una serie de criterios clínicos. Como puedes ver, se
adaptan bastante bien a lo que nos cuentan en esta pregunta (respuesta 3 correcta).

!
!
!
!
213. Un hombre de 30 años de edad, que hace 2 meses comenzó a trabajar en una
empresa que se dedica a la manufacturación de cloruro de polivinilo, acude refiriendo
que "está convencido de que está desarrollando un cáncer de hígado como consecuencia
de la exposición a la sustancia con la que trabaja". Refiere que desde que les impartieron
la charla de seguridad, con las normas de protección y los posibles efectos dañinos de
la sustancia en cuestión, no ha podido dejar de pensar en ello, que se nota cada día que
pasa "más amarillo". Tras la exploración física y analítica, que se encuentran dentro de
los parámetros de la normalidad, además de tranquilizar al paciente, le recordaremos el
contacto prolongado con cloruro de polivinilo, que no es su caso, se ha relacionado con
la aparición en el hígado de:

1. 1. Angiosarcoma.
2. 2. Colangiocarcinoma.
3. 3. Hepatoblastoma.
4. 4. Hepatocarcinoma fibrolamelar.
Gráfico de respuestas
Comentario

Una pregunta que difícilmente se presta al razonamiento lógico, con lo que te quedan pocas armas
si no se conoce la asociación. Efectivamente, el cloruro de polivinilo se ha relacionado con el
angiosarcoma hepático.(R1)

214. A 34-year-old-woman comes to the Emergency Department with a suspected


diagnosis of bacterial meningitis. She reports acute onset of headache, muscle pain, and
nausea with stiff neck for one day. She works in a nursery. Her temperature is 39ºC, blood
pressure is 70/43 mmHg, pulse is 115/min. Physical examination shows a left pupil with
nonreactive mydriasis. Which of the following is the best next step in management?

1. 1. Perform a lumbar puncture


2. 2. CT of the brain
3. 3. First dose of antibiotic treatment
4. 4. Get blood cultures
Gráfico de respuestas
(R3)

215. En cuanto a la ictericia neonatal, marque lo CORRECTO:

1. 1. El momento de inicio no ayuda en el diagnóstico.


2. 2. El riesgo de Kernicterus se presenta con valores de bilirrubina > 30 mg/dl.
3. 3. La reabsorción de hematomas puede causar ictericia.
4. 4. La ictericia fisiológica aparece en las primeras 24 horas.
Gráfico de respuestas
Comentario

Las características de la ictericia no fisiológica son las siguientes:

!
!
!
!
• Inicio en las primeras 24 horas de vida.

• Duración superior a 10-15 días.

• Bilirrubina total mayor de 12 mg/dl en RNT o mayor de 1 5 mg/dl en

RNPT.

Incremento de la bilirrubina superior a 5 mg/dl/24 horas.

• Bilirrubina directa mayor a 1 mg/dl o superior al 20 % de la bilirrubina

total.

La ictericia fisiológica nunca comienza el primer día de vida, ni se prolonga más allá de los días 10-
15. Cursa con buen estado general. La billrrubina no supera los 12 mg/dl en los RNT ni los 14
mg/dl en los RNPT.

En los RNPT, la ictericia fisiológica suele ser de inicio algo más tardío que en el RNT, y también
habitualmente es algo más prolongada, así como también puede normalmente alcanzar niveles
más altos.(R3)

216. Neonato de 17 horas de vida, trasladado desde una clínica de primer nivel de
atención, sin cuidados intensivos. Embarazo con diabetes gestacional. Parto
espontáneo, cefálico, amniorrexis 11 horas antes, con líquido claro. La edad gestacional
es de 36 semanas. En la exploración física se observa mal estado general, color terroso
de piel, cianosis acra, taquipnea, aleteo nasal, retracciones intercostales, quejido. FC:
150 lpm. FR: 80 rpm. Buena ventilación bilateral. Se palpa hígado a 1 cm, llanto agudo,
hipotonía generalizada. TA: 47/31 mm Hg. ¿Cuál sería la sospecha clínica?

1. 1. Hemorragia cerebral.
2. 2. Sepsis.
3. 3. Enfermedad de membranas hialinas.
4. 4. Hipoglucemia en hijo de madre diabética.
Gráfico de respuestas
Comentario

Un caso clínico complicado, donde resulta bastante difícil decidirse entre las opciones 2 y 4. Las
otras son relativamente fáciles de descartar, por las razones siguientes:

•! R1: no existen signos de hemorragia intraventricular, que normalmente cursaría con


bradicardia, hipoventilación, convulsiones, hipotonía y fontanelas a tensión (sólo coincide la
hipotonía, que es un dato inespecífico).
•! La verdadera dificultad estaría en distinguir la hipoglucemia de la sepsis neonatal. Al ser
hijo de madre diabética tiene un claro factor de riesgo para padecer una hipoglucemia. Sin
embargo, al ser un recién nacido pretérmino, también lo tiene para sepsis. Por tanto,
debemos guiarnos por las manifestaciones clínicas.

La presencia de acrocianosis y malestar general, así como el color terroso, podrían aparecer en
ambos casos. Sin embargo, cuando se trata de una hipoglucemia, lo habitual sería la aparición de

!
!
!
!
temblores o algún otro tipo de clínica neurológica (irritabilidad, convulsiones) que en este caso
faltan. Como en esta ocasión predominan las manifestaciones inespecíficas, sin foco infeccioso
claramente identificable, resulta más probable la respuesta 2 que la 4.(R2)

217. Respecto al struma de ovario, es FALSO:

1. 1. Es un tumor de células germinales.


2. 2. Se engloba dentro de los teratomas.
3. 3. Imita al tejido tiroideo: produce TSH.
4. 4. Es propio de mujeres jóvenes.
Gráfico de respuestas
Comentario

Esta pregunta hace referencia a una rara entidad. El struma ovárico es tejido tirodeo ectópico que
se localiza en ovario y produce hipertiroidismo por secreción de hormonas tiroideas T3 y T4, pero
no TSH. Es un tumor germinal que se engloba dentro de los teratomas como teratoma
monodérmico especializado. Recuerde que la captación gammagráfica del tiroides será nula.(R3)

218. A 19-month-old infant is brought to the pediatrician's office for a routine visit. She
has normal physical development. She drinks six glasses of milk daily. She can say pairs
of rhyming words and short sentences. Blood laboratory tests results are: Hemoglobin 9
g/dL, RDW 13%. MCV 70 fl, Platelets 300000, Leukocyte 6000/mm3. Physical examination
shows mild pallor. What is the most likely laboratory finding that would be expected in
this case?

1. 1. Low reticulocyte count


2. 2. Hematuria
3. 3. High serum creatinine level
4. 4. High serum copper level
Gráfico de respuestas
Comentario
Low reticulocyte count. Microcytic anemia in children of this age should make you suspect iron
deficiency anemia, which is the most common type of anemia at this age. Also, it is specified that
the child drinks large amounts of milk. Probably this child's diet is deficient in ferrous iron. The
physical and social milestones achieved are normal for this age.(R1)

219. Sobre la displasia congénita de cadera señale la afirmación FALSA:

1. 1. El test de Ortolani reduce la cadera luxada y el test de Barlow luxa la cadera luxable.
2. 2. El test de Galeazzi es muy específico, sobre todo en las luxaciones bilaterales.
3. 3. La cadera subluxada presenta artrosis secundaria con más frecuencia que las luxadas.
4. 4. El empleo de pañales en esta patología puede ser un tratamiento adecuado.
Gráfico de respuestas
Comentario
Esta pregunta hace referencia a uno de los temas más importantes de la ortopedia infantil como es
la displasia de la cadera en desarrollo, y es uno de los temas que necesitas dominar, junto con el
de fracturas. En concreto esta pregunta abarca el tema de la semiología que, salvo cosas
concretas, no es muy difícil de dominar. . En la exploración física el miembro afecto está acortado
(la luxación es posterior), se explora en el paritorio con las maniobras de Ortolani y Barlow (como
nos comenta este enunciado), y es más frecuente en la mujer. Cuidado con la respuesta 3, que es

!
!
!
!
verdad, pues la cadera subluxada tiene más probabilidad de dañar el cartílago articular. La 5 no es
el tratamiento idóneo, pero la respuesta 2 es más falsa: el test de Galeazzi consiste en flexionar
caderas y rodillas a 90º u observar la altura de las rodillas: en luxaciones bilaterales este test no es
muy específico. (respuesta 2 falsa).(R2)

220. Un fumador crónico presenta expectoración hemoptoica desde ayer y en la


radiografía torácica se aprecia un aumento de tamaño del hilio derecho, con pérdida de
volumen del pulmón ipsilateral. En la exploración física se objetivan adenopatías
supraclaviculares. En su hemograma presenta anemia. ¿Cuál es la explicación más
probable?:

1. 1. Ferropenia.
2. 2. Anemia de trastornos crónicos.
3. 3. Mieloptisis.
4. 4. Hemólisis inmune.
Gráfico de respuestas
Comentario
Este es el típico caso clínico de hematología, donde en el enunciado encontramos una patología
subaguda o crónica y luego nos cuentan que tiene anemia. Las opciones 4 y 5 quedan
descartadas, no nos hablan ni de ictericia, ni esplenomegalia, ni aumento de la LDH, ni de la
haptoglobina (hallazgos típicos de las hemólisis). La mieloptisis es un fenómeno de ocupación de
la medula ósea (por micrometástasis), que cursa con una reacción leucoeritroblastica y la
presencia de dacrocitos (descartamos también la opción 3). La duda puede surgir entre las
opciones 1 y 2; la anemia ferropénica requiere al menos haber terminado con los depósitos de
hierro para hacerse manifiesta, y es poco probable que por una expectoración hemoptoica de 24
hs se produzca (opción 1 falsa). Por otro lado la clínica de este paciente se ajusta más al
diagnostico de anemia de enfermedad crónica.(R2)

221. Señala la respuesta FALSA:

1. 1. La aparición de ondas Q es indicativa de necrosis miocárdica.


2. 2. El segmento ST descendido indica lesión subendocárdica.
3. 3. El segmento ST elevado es patognomónico de lesión subepicárdica isquémica.
4. 4. El infarto puede presentarse en ausencia de ondas Q.
Gráfico de respuestas
Comentario
Pregunta de baja dificultad de un tema que hay que controlar a fondo como es el ECG en la
cardiopatía isquémica. Las ondas T positivas indican isquemia subendocárdica, mientras que las T
negativas indican isquemia subepicárdica. El segmento ST descendido indican lesión miocárdica
en un IAM o angina de pecho mientras que el ST elevado indican lesión subepicárdica en el IAM o
angina vasoespástica. La aparición de ondas Q es índice de necrosis miocárdica. Hay infartos con
ondas Q, que generalmente son transmurales e infartos sin ondas Q que son subendocárdicos o
no transmurales.(R3)

222. En una exploración rutinaria se descubre un soplo en un preescolar. Es un soplo


sistólico eyectivo que se ausculta en la zona media-baja del borde esternal izquierdo. El
soplo aumenta en decúbito y disminuye en sedestación. También aumenta con el
ejercicio físico y la fiebre. Lo más probable es que se trate de:

!
!
!
!
1. 1. Soplo de Still.
2. 2. Soplo de trasposición de grandes arterias.
3. 3. Soplo de Austin-Flint.
4. 4. Soplo de Carey-Coombs.
Gráfico de respuestas
Comentario

Lo que nos están describiendo es el llamado “soplo inocente” o soplo de Still, bastante habitual en
la infancia, habitualmente diagnosticado en el contexto de una revisión rutinaria, como es el caso
de esta pregunta. Las características exploratorias descritas son las típicas, así como el aumento
de intensidad con el ejercicio físico, la fiebre u otras circunstancias que produzcan una circulación
hiperdinámica.(R1)

223. Se debe sospechar taponamiento pericárdico en un paciente con:

1. 1. Dolor torácico con ondas de gran voltaje en el electrocardiograma.


2. 2. Isquemia subendocárdica lateral con hemibloqueo anterior izquierdo.
3. 3. Fiebre, taquicardia y dolor torácico transfixiante.
4. 4. Choque y presión venosa alta.
Gráfico de respuestas
Comentario

Pregunta teórica de baja dificultad sobre un tema muy preguntado en el ENARM como es el
taponamiento cardiaco. En el electrocardiograma existe una disminución de la amplitud del QRS,
así como alternancia eléctrica de P, QRS y T, por lo que la opción primera es falsa. En el
cateterismo aparece un aumento de la presión en la AD, con aumento de la onda "x" y disminución
de la "y". Se igualan las presiones telediastólicas de AD, VD, arteria pulmonar y presión de
enclavamiento pulmonar. La clínica se deriva de la disminución del gasto cardíaco y la congestión
venosa: hipotensión arterial, aumento de la presión venosa central, disnea con ortopnea,
congestión hepática, etc...pero la fiebre y el dolor no son característicos del taponamiento, sino de
la pericarditis aguda.(R4)

224. Which of the following is CORRECT regarding dyplastic nevus?

1. 1. It is an acquired junctional melanocytic nevus that usually appears after puberty.


2. 2. The risk of melanoma in sporadic cases is as high as in familial cases.
It is not important to avoid sunlight or UV light exposure, since there are atypical changes
3. 3.
that are inevitable.
4. 4. Patient education regarding the identification of early signs of melanoma is very important.
Gráfico de respuestas
Comentario

Los nevus displásicos implican una mayor probabilidad de desarrollar un melanoma, por lo que es
importante la educación del paciente para que consulte precozmente en caso de encontrarse una
lesión pigmentaria con las características siguientes:

- A:Asimetría.

- B:Bordes irregulares.

- C:Coloración no uniforme.

!
!
!
!
- D:Diámetro mayor de 5 mm.

Por supuesto, el diagnóstico de melanoma es histológico y habría que confirmarlo con biopsia,
pero estos cuatro datos son suficientes como screening.(R4)

225. Félix es un niño de 2 años que acude con un cuadro de 2 días de evolución,
inicialmente presentó tos con rinorrea. Actualmente presenta tos perruna y ruidosa. Al
examen presenta temperatura de 38.3 °C, estridor inspiratorio y tiraje moderado. El
tratamiento que indicaría usted es:

1. 1. Nebulización con salbutamol.


2. 2. Ceftriaxona.
3. 3. Epinefrina en aerosol.
4. 4. Prednisona.
Gráfico de respuestas
Comentario

La clínica de laringitis, con un catarro previo de origen probablemente viral que se convierte en una
larigitis con la tríada clásica de tos perruna, estridor inspiratorio y dificultad respiratoria.

El tratamiento de la laringitis, si es leve o moderada se puede tratar con budesonida y corticoides,


en los casos más graves se puede utilizar adrenalina o derivado en aerosol. Incluso se puede
llegar a la intubación y a la ventilación mecánica.(R3)

226. Un lactante de 10 meses que presenta vómitos, diarreas, taquicardia, presión arterial
normal, mucosas secas, llenado capilar menor de 2 segundos e irritabilidad ¿Qué
porcentaje de peso ha perdido el paciente?

1. 1. 13-15%
2. 2. 3-5%
3. 3. 1-2%
4. 4. 6-9%
Gráfico de respuestas
Comentario

La respuesta correcta es la 4. Repase la siguiente tabla.

!
!
!
!

227. En el cuadro clínico del lupus NO encontraremos regularmente:

1. 1. Leucocitosis.
2. 2. Anemia.
3. 3. Mialgias.
4. 4. Artritis.
Gráfico de respuestas
Comentario
El lupus eritematoso sistémico es una enfermedad del tejido conectivo caracterizada por la
presencia de múltiples autoanticuerpos y el daño muy variado a distintos órganos o sistemas. Las
alteraciones hematológicas ocurren en el 85% de los casos y suelen ser asintomáticas. La anemia
suele ser de trastornos crónicos, aunque también puede aparecer en algunos casos en forma de
hemólisis. En el 60% de los casos se produce leucopenia, que no es intensa, ni favorece las
infecciones, y por lo tanto no requiere tratamiento. No es característica la leucocitosis, que será la
respuesta que elijamos. Es frecuente la linfopenia. La presencia de trombopenia no suele tener
importancia clínica, ya que solo en menos del 5% de los casos es intensa y se acompaña de
fenómenos hemorrágicos. Otras manifestaciones muy habituales son las musculoesqueléticas
como la artritis y las mialgias. Desde el punto de vista cutáneo el 70% presentan
fotosensibilidad.(R1)

228. En la diabetes mellitus pueden tomarse varias medidas para frenar el avance de la
nefropatía diabética. No obstante, algunas de ellas son más importantes que otras.
Señale la de mayor peso:

1. 1. Control metabólico adecuado de la diabetes mellitus.


2. 2. Control de la hiperpotasemia.
3. 3. Control de la presión arterial, preferentemente con IECA o ARA-II.
4. 4. Inicio precoz de la diálisis.
Gráfico de respuestas

!
!
!
!
Comentario

Es importante que recuerdes la respuesta de esta pregunta ya que ha sido preguntada


reiteradamente en el examen MIR (la última vez en la convocatoria 03- 04): el control de la tensión
arterial con ARA- II o IECAS es la medida más efectiva para reducir la progresión de la neuropatía
diabética ya que reducen la lesión vascular y la hipertensión intraglomerular. De hecho, los IECAs
o ARA- II están indicados en todo paciente con nefropatía diabética, sea hipertenso o no, para
disminuir la presión de filtración intraglomerular.(R3)

229. Paciente femenino de 35 años G3P2+ embarazo de 41 semanas de amenorrea que


acude al hospital por presentar dinámica de parto. El embarazo actual ha cursado con
normalidad. En el registro cardiotocográfico la frecuencia cardíaca fetal es de 120 lpm, la
variabilidad de 5-10 y existen deceleraciones variables. ¿Hacia qué tipo de patología nos
orienta el registro?:

1. 1. Compresión de la cabeza fetal en el canal del parto.


2. 2. Patología del cordón.
3. 3. Rotura de vasa previa.
4. 4. Desprendimiento prematuro de placenta normalmente inserta.
Gráfico de respuestas
Comentario

Caso clínico muy fácil en el que se nos presenta a una tercigesta a términoque está de parto. En el
registro cardiotocográfico se muestran dos alteraciones como son la variabilidad baja (5-10
latidos/minuto) y deceleraciones variables. Recuerde que cada tipo de deceleración está
relacionada con un tipo de patología, así, las deceleraciones variables (o también llamadas
umbilicales) son características de la patología del cordón (opción 2). En la compresión de la
cabeza fetal (estimulación del parasimpático fetal) solemos encontrar deceleraciones precoces o
DIPS de tipo I; en la situación premórtem (por ejemplo en la rotura de vasos previo) encontramos
una sinusoidal. Los DIPS tipo II o deceleraciones tardías los encontramos en patología placentaria
como puede ser el desprendimiento prematuro de placenta normalmente inserta (opción 4).(R2)

230. Mujer de 31 años con antecedente de alcoholismo, que consultó por fiebre, astenia,
dolor en hipocondrio derecho e ictericia de 6 días de evolución. En la exploración física
destacaba ictericia y hepatomegalia de 6 cm, dolorosa a la palpación. En los exámenes:
19.000 leucocitos con desviación izquierda, Hb 12,4 g/dl, 175.000 plaquetas/mm3, GOT
485 UI/l, GPT 407 UI/l, GGT 305 UI/l, F. Alcalina 246 UI/l, Bb 4,9 g/l. El USG demostró
imágenes compatibles con esteatosis hepática. La IgM VHA, el Ag S y anti VHC fueron
negativos. La conducta más adecuada para el correcto manejo de esta paciente
consistiría en:

1. 1. Tratamiento con esteroides.


2. 2. Biopsia hepática.
3. 3. Profilaxis del síndrome de abstinencia.
4. 4. Solicitar Ac antimitocondriales.
Gráfico de respuestas
Comentario

El antecedente de alcoholismo, la clínica típica, la leucocitosis con desviación a la izquierda, y la


elevación moderada de las transaminasas con predominio de la GOT sobre la GPT orientan a un

!
!
!
!
cuadro de hepatitis aguda alcohólica. Los datos ecográficos y los estudios serológicos de hepatitis
virales negativos apoyan este diagnóstico.

En las formas leves y moderadas, el tratamiento es de soporte: nutrición adecuada, abstinencia


alcohólica y evitar los síntomas de abstinencia. Los esteroides sólo están indicados en las formas
graves. La hepatitis de este paciente no se podría clasificar como tal, ya que no tiene signos de
disfunción hepática (encefalopatía, ascitis, alargamiento del tiempo de protrombina,
hiperbilirrubinemia intensa). La lactulosa estaría indicada en caso de encefalopatía, para prevenir
el estreñimiento, pero en este caso no existe alteración neurológica.(R3)

231. En la exploración de un recién nacido a término, de dos días de vida, encontramos


palidez muco-cutánea y taquicardia. En la BH hallamos Hb 12.5 g/dl; al día siguiente
encontramos Hb 10 g/dl. La prueba de Kleihauer-Betke es positiva, mientras que el test
de Coombs directo es negativo. ¿Cuál es su diagnóstico de sospecha?:

1. 1. Hemorragia por ligadura anómala del cordón.


2. 2. Enfermedad hemorrágica del recién nacido.
3. 3. Enfermedad hemolítica del recién nacido.
4. 4. Transfusión feto-materna.
Gráfico de respuestas
Comentario

La transfusión feto-materna es una entidad patogénica responsable de la aparición de cuadros de


anemia en el recién nacido (generalemente no demasiado graves) que se caracteriza porque
hematíes del feto pasan al compartimento de la madre. La prueba para detectarla es el test de
Kleihauer- Betke, que detecta en sangre materna la presenia de hematíes ricos en Hb fetal.(R4)

232. En relación a la fibrosis quística (FQ), todas las afirmaciones siguientes son ciertas,
EXCEPTO una. ¿Cuál?

El diagnóstico por despistaje neonatal se realiza mediante la determinación del


1. 1.
tripsinógeno o tripsina inmunorreactivos séricos.
2. 2. Las formas mucoides de Pseudomonas aeruginosa son diagnósticas de FQ en niños.
3. 3. El hallazgo de concentraciones bajas de cloro en sudor permiten confirmar el diagnóstico.
4. 4. Un 15-20% de los recién nacidos con FQ presenta íleo meconial.
Gráfico de respuestas
Comentario

Con respecto a la fibrosis quística, el diagnóstico se basa en las manifestaciones clínicas típicas
(respiratorias, digestivas y genitourinarias) o antecedentes de fibrosis quística en un hermano o
prueba de cribado neonatal positivo, más datos de laboratorio de disfunción de CFTR (test del
sudor positivo en dos resultados positivos en días distintos) o dos mutaciones de FQ en estudio
genético o alteraciones en la diferencia de potencial nasal.

Diagnóstico de fibrosis quística

!
!
!
!

!
!
!
!

233. Acude a consulta de urgencias un niño con


babeo, dificultad respiratoria y estridor inspiratorio. Cuando intenta acostarlo para
explorarlo, empeora su disnea, encontrándose mejor sentado, apoyado sobre sus manos.
Usted sospecha una epiglotitis. ¿Cuál es la actitud menos indicada?

1. 1. Valoración en unidad de críticos o UCI pediátrica.


Examen laríngeo en quirófano, para visualizar epiglotis con aspecto característico "rojo
2. 2.
cereza".
3. 3. Radiografía lateral de faringe. Hemocultivo y laboratorios.
Exploración en consulta con abatelenguas para descartar otras causas que puedan simular
4. 4.
la clínica, como faringitis o amigdalitis.
Gráfico de respuestas
Comentario

Ante una sospecha de epiglotitis no debe manipularse la cavidad oral. La epiglotis se encuentra
adyacente a la base de la lengua, y en los niños la epiglotis es muy alta, dado que tienen una
faringe corta. La manipulación de la epiglotis, puede desencadenar un aumento de la inflamación,
resultando en una situación crítica.

Ante una sospecha leve puede iniciar el estudio con radiografía lateral, buscando tumefacción.
Hemocultivo y exámenes de laboratorio (reactantes de fase aguda).

!
!
!
!
En caso de alta sospecha, mantendrá al niño en una unidad en la que pueda intubarse en caso de
necesidad. Y se realizará una exploración en quirófano. Si se comprueba la epiglotitis, se
procederá a la intubación.(R4)

234. En el 90% de los carcinomas pulmonares de células pequeñas se puede encontrar


una deleción del brazo corto de uno de los siguientes cromosomas. Señale cuál es:

1. 1. Cromosoma 13.
2. 2. Cromosoma 3.
3. 3. Cromosoma 5.
4. 4. Cromosoma 7.
Gráfico de respuestas
Comentario

Pregunta de extrema dificultad; no te preocupes si la has fallado. La respuesta correcta es la 3


(cromosoma 3). Parece haber varios oncogenes recesivos candidatos localizados en el cromosoma
3p, e implicados en varios cánceres pulmonares. A título anecdótico, recuerda que, en el
hipernefroma, también se ha encontrado implicación del cromosoma 3 en su génesis.(R2)

235. Señale la respuesta INCORRECTA sobre el colangiocarcinoma:

Se relaciona con enfermedades que producen colestasis crónica, como la enfermedad de


1. 1.
Caroli, los quistes de colédoco o la colangitis esclerosante primaria.
2. 2. Se relaciona con el virus de la hepatitis B.
3. 3. El tratamiento de elección es la resección hepática, aunque pocas veces es posible.
4. 4. Es más frecuente en pacientes ancianos que a edades jóvenes.
Gráfico de respuestas
Comentario

El colangiocarcinoma no presenta relación con el virus de la hepatitis B ni con la cirrosis. Ten


cuidado con esta pregunta porque es un engaño muy común. Lo que sí predispone a este tumor es
la infección por Clonorchis sinensis, las infecciones biliares de repetición y las malformaciones de
las vías biliares que determinen cierta lentitud del flujo biliar, como la enfermedad de Caroli.(R2)

236. Un recién nacido a término de peso adecuado, presenta ictericia el primer día de vida
con bilirrubina de 15 mg/dl y hemoglobina de 10 g/dl. El test de Coombs directo es
positivo. La madre tiene grupo sanguíneo O y Rh positivo, el padre es grupo A y Rh
negativo y su hermano mayor es A positivo. No hay antecedentes familiares de anemia.
¿Qué diagnóstico establecería?:

1. 1. Anemia hemolítica por incompatibilidad de grupo A.


2. 2. Anemia hemolítica por incompatibilidad de grupo B.
3. 3. Ictericia fisiológica.
4. 4. Esferocitosis hereditaria.
Gráfico de respuestas
Comentario
Queda descartada de entrada la ictericia fisiológica, pues nunca ésta aparece el primer día de vida
(y en un término jamás supera los 12 mg/dl). Con el test de Coombs positivo, la ictericia debe
obedecer a trastornos isoinmunes. Queda descartada la incompatibilidad Rh porque la madre es
Rh+; por tanto, nos queda la incompatibilidad ABO, especialmente frecuente en los hijos de madre
grupo O e hijos A.(R1)

!
!
!
!

237. Una paciente de 36 años fue diagnosticada de esclerosis múltiple recidivo-remitente


a los 28 años por un episodio de neuritis óptica izquierda y hemihipoestesia izquierda,
con RM cerebral compatible y presencia de bandas oligoclonales en el LCR. En los dos
últimos años ha tenido tres brotes (uno cerebeloso y dos sensitivos) con leve
inestabilidad en la marcha residual. Sobre el tratamiento médico que podemos ofrecerle,
NO es cierto:

Los bolos de metilprednisolona de 1 g/día durante 3 días disminuyen la progresión y el


1. 1.
número de brotes de la enfermedad.
Está indicado en nuestra paciente el uso de interferón beta 1a, 6 millones de unidades en
2. 2.
una inyección intramuscular a la semana.
El tratamiento con esteroides se basa en la actuación sobre el edema y la permeabilidad de
3. 3.
la barrera hematoencefálica.
Los efectos adversos más frecuentes del interferón beta incluyen un síndrome gripal,
4. 4.
mialgias, fiebre y reacciones locales en el sitio de la inyección.
Gráfico de respuestas
Comentario

Nos presentan una paciente con una forma remitente- recurrente de esclerosis múltiple. El
tratamiento de los brotes consiste en corticoides a dosis altas (1 gr/día) por vía intravenosa. Este
tratamiento pretende reducir las secuelas de ese brote, pero no actúan disminuyendo el número de
brotes posteriores (respuesta 1 falsa). El tratamiento de fondo se realiza con interferón o con el
acetato de glatirámero, que sí disminuyen el número de brotes, aunque sólo en las formas
recurrente- remitentes. Para las formas progresivas, el uso de interferón o glatirámero no aporta
beneficios, pudiendo emplearse inmunosupresores.(R1)

238. Una mujer de 76 años, con hipertensión arterial bien controlada, antecedentes de
depresión y artrosis en tratamiento con paracetamol, acude al Servicio de Urgencias por
presentar desde hace unas horas un dolor en fosa ilíaca izquierda, con aumento del
número de deposiciones (4-5 al día) de carácter líquido, pero sin productos patológicos.
En la exploración, existe dolor a la palpación superficial y profunda en la misma zona.
Ruidos hidroaéreos abdominales presentes. No hay signos de irritación peritoneal.
Laboratorios de uregencia normales, salvo leucocitosis con desviación izquierda. Un
TAC abdominal urgente muestra imágenes de diverticulosis, con signos de diverticulitis
aguda, pero sin datos de perforación. ¿Qué actitud terapéutica considera más correcta?:

Laparotomía exploradora, dado que la clínica no es concordante con los hallazgos


1. 1.
radiológicos.
2. 2. Drenaje percutáneo urgente guiado por ultrasonido del área inflamada.
3. 3. Alta domiciliaria con analgesia y recomendaciones dietéticas (dieta rica en fibra).
4. 4. Reposo intestinal, reposición hidroelectrolítica y antibioterapia.
Gráfico de respuestas
Comentario

Se trata de un paciente con una diverticulitis aguda no complicada, que inicialmente debe tratarse
con reposo intestinal, reposición hidroelectrolítica y antibióticos. En caso de que se objetivara un
absceso habría que hacer punción y drenaje guiado por TAC.(R4)

!
!
!
!
239. Nos encontramos ante un hombre de 60 años diagnosticado de cirrosis hepática con
serología positiva para e virus de hepatitis C y ascitis que no responde al tratamiento con
400 mg de espironolactona con restricción de sal (menos de 800 mg de sodio) y reposo.
¿Cuál debe ser nuestra actitud si queremos controlar la ascitis?

1. 1. Estableceremos un cortocircuito portocava laterolateral.


Dispondremos un shunt peritoneovenoso, conscientes de las complicaciones infecciosas y
2. 2.
de coagulación que puede tener el enfermo.
Duplicaremos la dosis de espironolactona con especial atención al nivel de potasio en
3. 3.
sangre.
4. 4. Añadiremos al tratamiento furosemida de 20 a 80 mg al día.
Gráfico de respuestas
Comentario

En el tratamiento escalonado de la ascitis, se debe iniciar con restricción de sodio y reposo. A


continuación, se deben añadir diuréticos, aumentando progresivamente la dosis hasta llegar al
máximo permitido. Aunque la espironolactona es el diurético de elección, su inicio de acción es
lento, y tiene potencia diurética baja; por ello, se debe asociar furosemida, aumentando la dosis
progresivamente hasta llegar al máximo tolerado o hasta 160 mg al día.(R4)

240. Señale la asociación FALSA:

1. 1. Enfermedad poliquística del adulto - poliglobulia.


2. 2. Enfermedad quística medular - imagen en cepillo.
3. 3. Síndrome de Alport - esferofaquia.
4. 4. Acidosis tubular distal - nefrocalcinosis.
Gráfico de respuestas
Comentario
Es una pregunta fácil, más si tenemos en cuenta que sabiéndote los pocos dato típicos de la
enfermedad, se responden la mayoría de preguntas. La imagen "en cepillo" corresponde a la
imagen característica que podemos observar al realizar una radiografía simple de abdomen a un
paciente con riñón esponjoso medular. Éste se define por la presencia de dilataciones en los
conductos de Bellini, que al depositarse calcio en su interior muestra la imagen descrita.
Bibliografía: Tema 11.3 de Nefrología, Manual 4ª Ed, pág. 31.(R2)

241. Un paciente de 64 años fumador y diabético, asintomático desde el punto de vista


cardiológico se realiza un electrocardiograma de rutina como parte de un estudio
preoperatorio y un Holter que se indica por palpitaciones ocasionales. En el Holter se
detectan descensos del segmento ST que cursan de forma asintomática. ¿Cuál cree que
es el diagnóstico más probable del cuadro de este paciente?:

1. 1. Isquemia silente.
2. 2. Angina estable.
3. 3. Angina inestable.
4. 4. Falsa positividad del Holter.
Gráfico de respuestas
Comentario
La isquemia silente es la demostración en el Holter de cambios electrocardiográficos sugerentes de
isquemia miocárdica, pero que no se acompañan de dolor ni otros síntomas . Estos episodios de
isquemia miocárdica asintomática son frecuentes en los enfermos que tienen angina de pecho,
pero también pueden

!
!
!
!
darse en individuos asintomáticos que nunca han tenido angina. La presencia de episodios
frecuentes de isquemia silente es un factor de mal pronóstico en los individuos con angina de
pecho. No está claro que el disminuir los episodios de isquemia silente mediante tratamiento
farmacológico mejore el pronóstico. No obstante, para estos enfermos, una buena elección serían
los betabloqueantes.(R1)

242. Un niño de 6 años, trasplantado cardíaco, que recibe ciclosporina, azatioprina y


corticoides en bajas dosis, ha presentado en el último año varios episodios de rechazo
que han requerido ciclos de dosis altas y prolongadas de corticoides. Para valorar, si
como efecto secundario producen enlentecimiento del crecimiento, ¿cuál es la velocidad
de crecimiento mínima, de entre las siguientes, que consideramos normal en este niño?

1. 1. 2 cm/año.
2. 2. 4 cm/año.
3. 3. 7 cm/año.
4. 4. 8 cm/año.
Gráfico de respuestas
Comentario

Se trata de una pregunta que podemos considerar como específica del tema, y que no debe darle
demasiada importancia si la ha fallado. No obstante, como muchas veces ocurre en las preguntas
sobre cifras o porcentajes, la respuesta correcta está, una vez más, en el valor intermedio....

Se habla de talla baja cuando se encuentra por debajo del percentil 3 para su edad, o por debajo
de - 2 DS. A la hora de evaluar la talla de un niño, hay que tener en cuenta la correlación con la
talla de los padres, y la velocidad de crecimiento, de forma que está indicado un estudio de talla
baja en los niños con velocidad de crecimiento por debajo del percentil 25 o menor de 4 cm / año,
de forma mantenida. Repuesta 2 correcta.(R2)

243. Cuál de los siguientes NO es útil en el diagnóstico prenatal de las cromosomopatías:

1. 1. Funiculocentesis.
2. 2. Baja cifra de alfafetoproteina en sangre materna.
3. 3. Biopsia corial.
4. 4. Test de COOMBS INDIRECTO.
Gráfico de respuestas
Comentario

Pregunta fácil sobre los métodos de diagnóstico prenatal. La biopsia corial es el método que
podemos realizar de forma más precoz (a las 8 semanas de gestación), siendo útil para el
diagnóstico de enfermedades metabólicas en el feto y cromosomopatías. La amniocentesis
genética es la siguiente, realizándose a la 14 semana; es la más utilizada y la más sencilla con
diferencia. La funiculocentesis o cordocentesis es la última prueba invasiva, realizándose a la 18
semana de gestación, obteniendo el diagnóstico de forma bastante rápida. Hemos citado métodos
invasivos, pero también disponemos de marcadores ecográficos y bioquímicos entre los que
destaca la alfafetoproteína, que en el síndrome de Down está muy descendida, y en los defectos
del tubo neural está muy elevada. El test de Coombs indirecto no es un método de diagnóstico
prenatal, por lo que la opción 4 es la que debemos escoger; es utilizado para el diagnóstico de Ac
anti- D maternos, para así poder identificar gestaciones con riesgo de tener una isoinmunización
Rh.(R4)

!
!
!
!
244. La causa de muerte en los pacientes afectos de metástasis del SNC es:

1. 1. Hidrocefalia.
2. 2. Hipertensión intracraneal.
3. 3. Recidiva del tumor sistémico.
4. 4. Herniación transtentorial.
Gráfico de respuestas
Comentario
Esta pregunta es importante para el examen, pues es subtema de tumores encefalicos ha salido
año tras año en el MIR. Recuerda que las metastasis son los tumores cerebrales más frecuentes
en adultos y que se localizan generalmente a nivel de la unión corticosubcortical de los hemisferios
cerebrales. La gran mayoria son de origen pulmonar, siendo mas habituales en el carcinoma de
células pequeñas (oat cell). Sin embargo, el tumor que tiene más tendencia a metastatizar es el
melanoma. El tratamiento de elección es la radioterapia holocraneal, aunque las lesiones únicas y
accesibles, cuando el tumor primario está controlado, deben tratarse con cirugía más radioterapia
postoperatoria. Una vez que un cáncer sistémico ha metastatizado en el cerebro es, con raras
excepciones, incurable, siendo la supervivencia media una vez tratado de unos 6 meses.(R3)

245. ¿Cuál de las siguientes complicaciones es la más frecuente en un RN hijo de una


madre con varicela clínica 2 días antes del parto?:

1. 1. Meningoencefalitis.
2. 2. Coriorretinitis.
3. 3. Persistencia del conducto arterioso.
4. 4. Neumonitis.
Gráfico de respuestas
Comentario
La presencia de varicela materna en el intervalo temporal que va desde 5 días antes del parto
hasta 2 días después de producido éste supone un factor de riesgo para desarrollar varicela
neonatal, que es un cuadro muy grave, y que provoca en el RN afectación sistémica consistente en
vesículas, neumonitis, hepatitis, encefalitis y potencialmente la muerte del niño. Debe ser
prevenida con gammaglobulina específica antes de que se declare en el RN.(R4)

246. A 40-year-old woman comes after a gas explosion at her home. She has extensive
third-degree burns. IV fluids and antibiotics are started. After five days, she complains of
right upper cuadrant abdominal pain and fever. Murphy sign is positive. WBC is
16,000/mm3. Ultrasonogram shows no gallstones, although the gallbladder is dilated. The
most likely diagnosis is:

1. 1. Septicemia
2. 2. Shock
3. 3. Curling syndrome
4. 4. Acalculous cholecystitis
Gráfico de respuestas
Comentario
Acalculous cholecystitis. Acute acalculous cholecystitis is an inflammation of the gallbladder without
evidence os stones. It is closely associated with a gangrenous bladder, empyema and high
mortality rates. Conditions associated include burns, severe trauma, total parenteral nutrition or
sepsis. Typical signs are an elevated WBC count, fever and increased liver function tests.(R4)

!
!
!
!
247. What is the most common cause of secondary parkinsonism?

1. 1. Postencefalitis.
2. 2. Antidopaminergic drugs.
3. 3. Hypoparathyroidism.
4. 4. Brain tumors.
Gráfico de respuestas
Comentario

Todas las situaciones de las diversas opciones son causa de parkinsonismo secundario, pero entre
ellas la más frecuente es la toma de fármacos antidopaminérgicos (opción 2 correcta), tanto
depletores presinápticos (reserpina) o postsinápticos (neurolépticos), estos últimos más
frecuentemente. El parkinsonismo es el efecto 2º más frecuente del tratamiento con neurolépticos,
aparece de forma subaguda, y responde al tratamiento con anticolinérgicos.(R2)

248. A 5-year-old Senegalese child is brought to the emergency department presenting


with fever and malaise. She has a medical history of sickle cell anemia and recurrent
hospital admissions for osteomyelitis and splenic infarctions. Which of the following is
the most likely causative agent of her present condition?

1. 1. Staphylococcus aureus
2. 2. Streptococcus pneumoniae
3. 3. Influenza virus
4. 4. Salmonella
Gráfico de respuestas
Comentario
Streptococcus pneumoniae. Splenic involvement confers a very high risk of infections in patients
with sickle cell anemia, especially by encapsulated bacteria such as Streptococcus pneumoniae.
Invasive pneumococcal disease is typical of this disease (with a risk up to x300). Pneumococcal
meningitis and septicemia are important causes of death in SCD patients, with case / fatality rates
of 15 to 35%(R2)

249. Un paciente alcohólico presenta en su domicilio, durante la convalecencia de una


neumonía, un cuadro de confusión y ataxia severa. La exploración muestra un nistagmus
horizontal en ojo derecho, en la mirada hacia la derecha. La familia asegura que no ha
bebido alcohol. ¿Cuál es su opinión?

1. 1. El cuadro es sugerente de una enfermedad de Marchiafava-Bignami.


2. 2. Padece un infarto cerebeloso.
3. 3. Debe recibir tiamina con urgencia.
4. 4. El primer diagnóstico a descartar es el de hematoma subdural.
Gráfico de respuestas
Comentario

La aparición de un cuadro de confusión, ataxia y nistagmo horizontal en un paciente alcohólico,


debe hacernos pensar en una encefalopatía de Wernicke, por lo que administraremos como
tratamiento tiamina o vitamina B1.(R3)

!
!
!
!

250. Acude a su consulta


un paciente varón de 79 años que refiere otalgia del oído derecho desde hace unos diez
días, asociado a leve otorrea de dicho oído. Como antecedentes de interés refiere haber
sido de un adenocarcinoma gástrico hace 5 meses, y hace 2 semanas finalizó el último
ciclo de quimioterapia. Acudió a su médico de familia, quien le pautó gotas óticas con
corticoides, a pesar de lo cual no ha mejorado. En la exploración objetivamos a la
otoscopia las paredes del conducto auditivo externo edematosas, ocluyendo de forma
completa la luz del conducto. Se objetiva edema también de la piel de la región
preauricular, justo delante del trago. La exploración neurológica del paciente es normal.
Se decide realizar una prueba de imagen, que se muestra (ver imagen). Señale la
respuesta verdadera:

1. 1. Se trata con toda seguridad de una otomicosis, por la descripción de la otoscopia.


Con la imagen se confirma la presencia de una otitis externa maligna, ya que la porción
2. 2. ósea del conducto auditivo externo, tanto en su parte superior como en la inferior, se ve
claramente erosionada y adelgazada por una masa de partes blandas.
La imagen de tomografía que se muestra es normal, tanto a nivel de la caja timpánica
3. 3.
como a nivel del conducto audititivo externo.
Por la imagen otoscópica descrita y la imagen de tomografía que se muestra, estaría
4. 4.
indicado pedir una resonancia magnética para estudiar el ángulo pontocerebeloso.
Gráfico de respuestas
Comentario

Es posible que los antecedentes del paciente nos hagan pensar, en principio, en un diagnóstico
potencialmente grave. Sin embargo, la presencia de edema en conducto auditivo externo junto a la
imagen que nos ofrecen, donde se distinguen claramente las estructuras indemnes de oído medio
y oído interno, son sugestivas de una otitis externa de origen bacteriano.

Recuerda que la causa más frecuente es la otitis por Pseudomonas, que es lo que probablemente
padecerá este paciente. Por eso no ha funcionado el tratamiento pautado por su médico de
atención primaria, ya que incluía corticoide para combatir la inflamación, pero no antibiótico.(R3)

251. ¿Cuál sería la actitud que seguiría con este paciente?

!
!
!
!
1. 1. Tratamiento antibiótico de amplio espectro intravenoso durante seis semanas.
Añadiría gotas tópicas con corticoide y antibiótico que cubra pseudomonas
2. 2.
(ciprofloxacino), y además antibiótico vía oral.
3. 3. Aspiraría la otorrea del conducto auditivo externo y continuaría con el mismo tratamiento.
4. 4. Le colocaría un drenaje transtimpánico.
Gráfico de respuestas
Comentario

Considerando lo explicado en la pregunta anterior, habría que incluir un antibiótico potencialmente


eficaz contra el género Pseudomonas, como el ciprofloxacino (respuesta 2 correcta). También
sería aconsejable añadir tratamiento oral, que debería ser el mismo antibiótico, ya que el
ciprofloxacino es la mejor alternativa por vía oral para luchar contra este microorganismo.
Recuerda que, para combatir el género Pseudomonas, también podría recurrirse a la ceftazidima,
cefepima, aminoglucósidos, piperacilina- tazobactam… Pero estas alternativas se administran por
vía intravenosa, que en este caso no parece necesaria.(R2)

252. Una recién nacida (26 semanas de edad gestacional, 650 gramos de peso al
nacimiento, cesárea urgente, Apgar 5/6) presenta a los pocos minutos de nacer aleteo
nasal, tiraje intercostal moderado, retracción subxifoidea y quejido espiratorio continuo
audible sin fonendo. FR 70 respiraciones por minuto. Ingresa en la Unidad de Cuidados
intensivos neonatales intubada y se administra surfactante endotraqueal. Posteriormente
requiere ventilación mecánica para mantener buenas saturaciones de oxígeno. Presenta
una buena evolución desde el punto de vista clínico y gasométrico, por lo que se va
reduciendo progresivamente la concentración de oxígeno hasta alcanzar una FiO2
mínima de 0,25. A las 72 horas de vida aparece de nuevo tiraje intercostal y taquicardia
requiriendo un aumento de la concentración de oxígeno. A la exploración presenta un
latido hiperdinámico y se ausculta un soplo continuo. La diferencia entre la presión
arterial sistólica y diastólica es de 25 mmHg. Con respecto a la complicación que ha
desarrollado la paciente es FALSO que:

1. 1. En este caso no estaría indicado el uso de una perfusión de prostaglandinas intravenosa.


No se han encontrado diferencias significativas en cuanto a la eficacia de la indometacina
2. 2. y el ibuprofeno, pero el ibuprofeno tiene menos efectos adversos renales que la
indometacina.
El aumento de las presiones pulmonares que se produce como consecuencia de una mejor
3. 3. oxigenación produce un cortocircuito de derecha a izquierda causante de los síntomas
estando indicada la intervención ya sea médica o quirúrgica.
La presencia de una trombopenia, enterocolitis necrotizante o hemorragia intraventricular
4. 4.
contraindicarían el tratamiento conservador.
Gráfico de respuestas
Comentario

El caso clásico de un conducto arterioso persistente en el prematuro es el de un recién nacido con


un cuadro inicial de dificultad respiratoria (enfermedad de membrana hialina) que requiere oxígeno
y ventilación mecánica con lo que va mejorando su situación respiratoria. Posteriormente hablarán
de un nuevo empeoramiento (o que se detiene la mejoría) y la aparición de un soplo continuo,
pulsos saltones y aumento del diferencial entre TAS y TAD.

La incidencia de DAP es mayor cuanto menor sea la edad gestacional y el peso del recién nacido.
El cierre del DAP se produce cuando aumenta la oxigenación de la sangre.

!
!
!
!
En los RN con EMH esto no ocurre debido a su enfermedad pulmonar. Los capilares pulmonares
permanecen contraídos como respuesta a la hipoxia y las presiones pulmonares elevadas. Al tratar
la enfermedad respiratoria con surfactante y mejorar la oxigenación los capilares pulmonares se
dilatan, cayendo las resistencias pulmonares por debajo de las sistémicas. Esto provoca que el
flujo de la aorta tenga más facilidad para ir hacia los pulmones vía ductus (cortocircuito izquierda
derecha) provocando un hiperaflujo pulmonar con riesgo de edema agudo de pulmón, causa del
empeoramiento respiratorio típico.

El tratamiento en los casos sintomáticos hemodinámicamente significativos hay que hacerlo con
fármacos (ibuprofeno/indometacina). Misma eficacia, menos efectos renales- oliguria -(con el
primero), se dan 2-3 ciclos (cada uno con 3 dosis) antes de pasar a la cirugía.

Las contraindicaciones del tratamiento médico son: plaquetopenia, enterocolitis necrotizante,


insuficiencia renal y hemorragia cerebral. Las opciones quirúrgicas son: Ligadura, sección o cierre
con clip.(R3)

253. Ante un paciente drogadicto que ha sido tratado de tuberculosis pulmonar activa
con isoniacida 300 mg/día, rifampicina 600 mg/día, piracinamida 2g/día y etambutol 15
mg/kg/día dos meses seguido de isoniacida y rifampicina a las mismas dosis durante
cuatro meses y que sufre una recaída. ¿Qué explicación sería la más probable para el
fracaso del tratamiento?:

1. 1. Negligencia del enfermo.


2. 2. Pauta terapéutica insuficiente.
3. 3. Inmunodepresión del paciente.
4. 4. Error diagnóstico.
Gráfico de respuestas
Comentario

Esta pregunta no la olvide. La causa más frecuente de recaída en un paciente con tuberculosis es
la incorrecta realización del tratamiento (es además más típico de ciertos pacientes como el del
enunciado). En efecto, a pesar de que M. tuberculosis tiene una probabilidad de manifestar
resistencias a alguno de los fármacos, si se realiza bien la terapia con tres o más fármacos ésta
suele ser muy eficaz.(R1)

254. En la evaluación por infertilidad de una pareja se encuentra: factor masculino


anormal (oligospermia severa), trompas obstruidas y reserva ovárica con FSH mayor de
30. La edad de la mujer es de 39 años y del varón 60. ¿Qué procedimiento de fertilización
asistida sería el más indicado?

1. 1. Inducción de ovulación + inseminación intrauterina


2. 2. Fertilización in vitro (FIV)
3. 3. Microinyección espermática (ICSI)
4. 4. Ovodonación + Microinyección espermática (ICSI)
Gráfico de respuestas
Comentario

Se trata de una pregunta muy complicada, que exige conocer detalles muy concretos.

El ICSI (inyección intracitoplasmática de espermatozoides), consiste en la inyección del


espermatozoide directamente en el citoplasma del óvulo. Está indicado en casos de esterilidad

!
!
!
!
masculina grave, como es este caso. En estos casos, la fecundación in vitro tendría una eficacia
demasiado baja, por eso se prefiere el ICSI.

Además, la mujer del caso presenta una reserva ovárica prácticamente agotada, como refleja una
FSH en valores prácticamente postmenopáusicos (>30). De ahí que se decida recurrir a la
donación de óvulos, en lugar de a una estimulación de la ovulación que, casi seguro, no tendría
resultados.

Por todo ello, la respuesta correcta es la 4.

255. La disgenesia gonadal ovárica que se caracteriza por tener un cariotipo 46 XY se


llama:

1. 1. Síndrome de Swyer.
2. 2. Síndrome de Rösle.
3. 3. Síndrome de Asherman.
4. 4. Síndrome de Morris.
Gráfico de respuestas
Comentario

Lo más importante de las amenorreas es conocer su diagnóstico diferencial, además debe conocer
unos datos básicos de las causas más frecuentes o más características. El síndrome de Swyer es
una disgenesia gonadal pura, sin malformaciones ni enanismo. El cariotipo es 46 XY. El
cromosoma Y no se expresa, por lo que funciona como un 45 X0. Son frecuentes los cánceres de
ovario: el más frecuente el gonadoblastoma.

No es un tema preguntado en el ENARM.(R1)

256. Las perlas de Epstein situadas en el paladar duro del RN deberían tratarse con:

1. 1. Corticoterapia tópica.
2. 2. Escisión y drenaje.
3. 3. Observación.
4. 4. Peroxido de hidrogeno.
Gráfico de respuestas
Comentario

Son quistes blanquecinos y amarillentos que se forman en las encías y en el paladar de RN. No es
necesario ninguna tratamiento. Respuesta correcta 3.

257. A 5-year-old Senegalese girl is brought to the emergency room presenting with fever
and pain in her right leg. She has a medical history of sickle cell anemia. Physical
examination shows a swollen leg, especially on the anterior tibial area, which is tender to
palpation. Blood tests and cultures are pending. Whichof the following is the most likely
causative agent of this clinical manifestation?

1. 1. Salmonella species
2. 2. Clostridium species
3. 3. Pseudomonas species
4. 4. Cryptococcus species

!
!
!
!
Gráfico de respuestas
Comentario
Salmonella species. The most common causative agent of osteomyelitis is Staphylococcus aureus.
Nevertheless, in children suffering from sickle-cell anemia, Salmonella species are the most likely
pathogens.(R1)

258. Un paciente acude a Urgencias presentando dolor y tumefacción medial tras sufrir
eversión forzada de tobillo derecho. No presenta dolor a la palpación bimaleolar ni en la
base del V metatarsiano. Movilidad articular conservada con cierta inestabilidad articular.
¿Qué estructura anatómica esperaría encontrar lesionada?:

1. 1. Ligamento deltoideo.
2. 2. Ligamento peroneoastragalino anterior.
3. 3. Ligamento peroneocalcáneo.
4. 4. Ligamento tibioperoneo posterior.
Gráfico de respuestas
Comentario
En una torsión del tobillo en eversión forzada el ligamento que se tensa y se lesiona es el deltoideo
o ligamento lateral interno. El resto de los ligamentos citados se localizan en la región externa
formando el ligamento lateral externo (peroneoastragalino anterior y posterior, y peroneocalcáneo)
y parte de la sindesmosis (tibioperoneo posterior).(R1)

259. Un chico de 13 años viene con su madre al final de nuestra consulta de “viernes
tarde”, debido a que al salir del colegio tenía fiebre (38,5 ºC) y ha vomitado algo de
comida. “Seguro que es de la garganta, su hermano estuvo igual hace 5 días”, comenta
la madre. En la historia del Centro de Salud no se observa ningún antecedente de interés.
El paciente se queja de dolor de cabeza, al tragar saliva y de dolor abdominal, no tiene
tos, ni rinorrea, ni ronquera. La exploración es normal excepto la presencia de varias
adenopatías cervicales anteriores bilaterales de más de 1 cm de diámetro dolorosas,
exudado blanco-grisáceo en pared posterior de la faringe y amígdalas grandes y
eritematosas. ¿Cuál sería la actitud más adecuada?

Instaurar tratamiento con una penicilina a dosis y tiempo adecuado por una posible
1. 1.
faringitis estreptocócica.
Hacer una toma de exudado para cultivo, prescribir analgésicos y antipiréticos, en espera
2. 2.
del resultado previsto a los 6-7 días.
Instaurar tratamiento con un macrólido a dosis y tiempo adecuado, evitando las penicilinas
3. 3.
por el riesgo de reacción ante una posible mononucleosis.
4. 4. Administrar moxifloxacino para asegurarnos de la eficacia antibiótica.
Gráfico de respuestas
Comentario

Impugnable.

El manejo correcto ante un cuadro que sugiere faringitis estreptocócica (como parece este caso)
sería realizar un test de diagnóstico rápido y comenzar tratamiento con penicilina si fuera positivo;
en caso de que el test de diagnóstico rápido fuera negativo, habría que realizar cultivo faríngeo
cuyo resultado se obtendría en 24-48 horas (en ningún caso 6-7 días). El hecho de que la consulta

!
!
!
!
se realice el viernes a última hora no justifica que no se sigan las instrucciones de las guías de
práctica clínica.(R1)

260. ¿Cuál de los siguientes tratamientos tópicos del acné polimorfo juvenil NO está
indicado?:

1. 1. Peróxido de benzoilo.
2. 2. Acido retinoico.
3. 3. Dipropionato de betametasona.
4. 4. Acido salicílico.
Gráfico de respuestas
Comentario

En el tratamiento del acné, se emplean antibióticos tópicos (eritromicina, clindamicina) e incluso


sistémicos (tetraciclinas). También pueden ser beneficiosas ciertas sustancias queratolíticas, como
el ácido salicílico o el peróxido de benzoilo (este último, además, es comedolítico y bacteriostático).
Los retinoides tópicos también tienen un papel, por su carácter comedolítico y exfoliante. Sin
embargo, lo que no tiene sentido es tratar el acné con corticoides, precisamente porque ellos
mismos pueden producir acné (acné esteroideo).(R3)

261. En relación al tratamiento del hipertiroidismo durante la gestación, ¿cuál considera


INCORRECTA?:

1. 1. Los antitiroideos deben evitarse, en la medida de lo posible, durante la gestación.


2. 2. El I-131 está contraindicado de manera absoluta durante la gestación.
3. 3. La cirugía puede ser una alternativa en determinadas pacientes.
La pauta bloqueo-sustitución (dosis alta de antitiroideo y L-tiroxina para evitar un
4. 4.
hipotiroidismo yatrógeno) es la pauta de elección.
Gráfico de respuestas
Comentario

Pregunta en relación al tratamiento del hipertiroidismo durante la gestación. Para fines del ENARM
lo único que debe recordar es que el fármaco aconsejado durante la gestación es el propiltiouracilo
por tener el menor paso placentario y, por tanto, tener menor riesgo de provocar un hipotiroidismo
en el feto. No obstante, no todas las guías están de acuerdo en este punto. El mayor riesgo de
hepatotoxicidad con propiltiouracilo hace que se aconseje valorar cambiar el tratamiento a
metimazol tras el primer trimestre. Aún así, cualquier antitiroideo siempre debe utilizarse en el
embarazo, sólo si es imprescindible y a la menor dosis posible. Durante la gestación, el tratamiento
con I-131 está contraindicado y, por lo tanto, sólo quedaría como alternativa la cirugía en caso de
mal control o efecto secundarios con los antitiroideos. La pauta bloqueo-sustitución puede utilizarse
fuera de la gestación pero es inasumible durante la gestación ya que conlleva el uso de dosis más
elevadas de antitiroideos y mayor riesgo de hipotiroidismo fetal.(R4)

262. Los gérmenes que con mayor frecuencia infectan las heridas quirúrgicas limpias
son:

1. 1. Género Staphylococcus.
2. 2. Enterobacteriáceas.
3. 3. Pseudomona aeruginosa.
4. 4. Streptococcus sp.
Gráfico de respuestas

!
!
!
!
Comentario

La infección de la herida quirúrgica es la segunda causa de infección en los servicios quirúrgicos,


después de las infecciones del tracto urinario, que son las más frecuentes en el ámbito nosocomial.
Los gérmenes más usualmente implicados son:

•! Heridas que no afecten al periné y no estén implicados el tracto biliar ni gastrointestinal:


Staphylococcus aureus (con mayor frecuencia) o estreptococos (respuesta 1 correcta).
•! Heridas que sí afectan periné, tracto gastrointestinal o biliar: gramnegativos y anaerobios.

Lo habitual es que se genere a partir de la flora del propio paciente, introducida durante la cirugía.
También merece la pena recordar los agentes más probables en función de la cronología de la
infección:

•! Precoz (24-48 horas): Streptococcus pyogenes (fascitis necrotizante), Clostridium


(gangrena gaseosa).
•! A los 4-6 días postoperatorios: lo más frecuente (Staphylococcus).
•! Más de siete días postoperatorios: bacilos gramnegativos y otros anaerobios.(R1)

263. Señale la respuesta CORRECTA:

1. 1. La dehidroepiandrosterona se produce fundamentalmente en el ovario.


2. 2. En la mujer la testosterona se origina fundamentalmente por conversión periférica.
3. 3. La testosterona en la mujer se produce fundamentalmente en el ovario.
4. 4. La mayor parte de la testosterona circula en estado libre.
Gráfico de respuestas
Comentario

Concepto básico. Dentro de los andrógenos naturales, el más importante es la testosterona,


aunque su derivado, la dihidrotestosterona, es más potente desde el punto de vista biológico.
Como andrógeno natural de origen suprarrenal está la dehidroepiandrosterona y con origen mixto
gonadal y suprarrenal la androstendiona. La testosterona libre sólo representa entre 1.2- 2.2% de
la testosterona total.(R3)

264. ¿Qué cantidad de leche debe tomar un lactante de 4 meses que pese 5 Kg?:

1. 1. 750 cc.
2. 2. 900 cc.
3. 3. 400 cc.
4. 4. 300 cc.
Gráfico de respuestas
Comentario

Esta pregunta sobre lo requerimientos nutritivos del lactante, tema my importante, no es tiene
especial importancia. La cantidad de leche que debe tomar se calcularía sabiendo que las
necesidades calóricas para un lactante en el segundo trimestre son de: 100-110 Kcal/Kg/día. Si
este niño necesita 105x5= 525 Kcal/ día y la leche tiene 670 kcal/l, utilizando una regla de tres, la
cantidad de leche que debe tomar es: 525/670= 0.700 l =700cc.(R1)

!
!
!
!

265. A 47-year-old female is concerned about her risk of ovarian cancer because her aunt
died of it and her mother has just been diagnosed. She is asymptomatic and her physical
examination is completely normal. She smokes one pack of cigarettes per day and she is
diabetic. Which of the following should be consider for ovarian cancer screening in this
patient?

1. 1. Biannual pelvic examination


2. 2. Yearly transvaginal ultrasound
3. 3. CA125 serum test if tested positive for BRCA1
4. 4. Yearly transabdominal ultrasound if tested positive for BRCA2
Gráfico de respuestas
Comentario
CA125 serum test if tested positive for BRCA1. Screening has only been proved valid for high
genetic risk patients(R3)

266. La toma de anovulatorios disminuye el riesgo de todos los siguientes EXCEPTO uno.
Señálelo:

1. 1. Embarazo ectópico.
2. 2. Cáncer de endometrio.
3. 3. Cáncer epitelial de ovario.
4. 4. Carcinoma ductal de mama.
Gráfico de respuestas
Comentario

Los anticonceptivos orales por su efecto anovulatorio disminuyen el riesgo de embarazo ectópico y
de carcinoma de ovario de estirpe epitelial (recuerde la teoría de la ovulación incesante como
hipótesis etiológica y que todas aquellas situaciones en las que disminuya el número de
ovulaciones como gestaciones, toma de ACOs, son factores protectores frente al cáncer de ovario).
Al conseguirse una regularidad en el ciclo endometrial que se descama todos los meses, no damos
lugar al desarrollo de hiperplasias endometriales que podrían degenerar en un adenocarcinoma de
endometrio. Con la toma de ACOs se produce un espesamiento y mayor viscosidad del moco
cervical inducidos por el gestágeno, de manera que se evita el ascenso de bacterias y disminuye la
incidencia de EPI. Existe un riesgo incrementado muy pequeño de carcinoma de mama en
usuarias recientes o actuales de ACO que desaparece a los 10 años de abandonarlo. Además los
cánceres de mama diagnosticados en usuarias de ACO están más localizados y tienen menor
tendencia a metastatizar lo que supondría un aumento de la supervivencia de estas pacientes.(R4)

267. Una de las siguientes sustancias NO se encuentra entre los numerosos mediadores
preformados en los gránulos citoplasmáticos de los mastocitos humanos:

1. 1. Serotonina.
2. 2. Histamina.
3. 3. Hidrolasas ácidas.
4. 4. Proteasas neutras (quinasa y triptasa).
Gráfico de respuestas
Comentario

!
!
!
!
Pregunta compleja sobre un tema poco preguntado en el MIR, por lo que no le des muchas vueltas
y recuerda tener claro los conceptos claves. Los gránulos citoplasmáticos de basófilos y mastocitos
contienen numerosos mediadores preformados que son secretados tras la unión de la IgE o IgG4 a
los receptores de su membrana. Dentro de las sustancias secretadas se encuentran
fundamentalmente la histamina, la heparina, la serotonina (NO en humanos), hidrolasas, proteasas
neutras (triptasa y quimasa) y factores quimiotácticos para los eosinófilos y neutrófilos.
Posteriormente se sintetizan nuevas sustancias que serán secretadas, derivadas del ácido
araquidónico, como el ácido hialurónico, y el PAF. Ten cuidado porque aunque los macrófagos
producen serotonina, no lo hacen en humanos encontrándose aquí la dificultad de esta
pregunta.(R1)

268. Indique el número de anastomosis que hay que realizar en una técnica de Whipple
clásica:

1. 1. 1.
2. 2. 3.
3. 3. 4.
4. 4. 5.
Gráfico de respuestas
Comentario

La técnica de Whipple implica la resección de cabeza del páncreas junto con la segunda porción
del duodeno y reconstrucción del tránsito con un asa de yeyuno. Para ello son precisas tres
anastomosis:

1. Pancreáticoyeyunal: entre el ducto pancreático seccionado y el yeyuno (para la secrección


pancreática)

2. Hepáticoyeyunal: entre el conducto hepático común y el yeyuno (para la bilis)

3. Gastroyeyunal: para restablecer el tránsito alimentario.(R2)

269. En pacientes con dilatación de la vía biliar, sin aparente causa ¿Que estudios
diagnóstico podrían realizarse?:

1. 1. Ultrasonido y endoscopia.
2. 2. Ultrasonido y tomografía.
3. 3. Colangiografía y resonancia magnética.
4. 4. Ultrasonido y marcadores tumorales.
Gráfico de respuestas
Comentario

Después de la realización del ultrasonido, los siguientes estudios que podrían ser de utilidad es la
colangiografía y RMN. Respuesta 3 correcta.(R3)

270. Una mujer de 18 años consulta por amenorrea primaria. En la exploración se observa
un fenotipo femenino normal pero con ausencia de desarrollo de los caracteres sexuales
secundarios. Los genitales externos son femeninos de aspecto infantil y los genitales
internos femeninos e hipoplásicos. La talla es normal. El cariotipo es 46,XX. Los niveles
plasmáticos de

!
!
!
!
gonadotropinas (FSH y LH) están muy elevados. ¿Cuál de los siguientes diagnósticos
corresponde a este cuadro?

1. 1. Síndrome de insensibilidad a los andrógenos (feminización testicular).


2. 2. Disgenesia gonadal tipo síndrome de Turner.
3. 3. Disgenesia gonadal pura.
4. 4. Síndrome adrenogenital.
Gráfico de respuestas
Comentario

La disgenesia gonadal pura, que es el cuadro que nos describen en el enunciado, presenta las
características siguientes:

•! El cariotipo puede ser normal (46,XX) o 46,XY.


•! Los genitales externos y el aspecto general corresponden a una mujer inmadura. Lo mismo
ocurre con el útero y las trompas.
•! Las gónadas no están desarrolladas, y son simplemente cintillas bilaterales.
•! A diferencia del síndrome de Turner, la talla es normal o elevada.

El tratamiento de la disgenesia gonadal pura es la extirpación gonadal, especialmente cuando el


cariotipo es 46,XY, por la elevada incidencia de tumores. Además de la administración de
estrógenos.(R3)

271. En una laparotomía por un tumor ovárico encontramos afectación de ambos ovarios.
La biopsia intraoperatoria informa de la existencia de numerosas células en anillo de
sello, con lo que se confirma su naturaleza metastásica. ¿Dónde buscaría usted el tumor
primario?:

1. 1. En el riñón.
2. 2. En el pulmón.
3. 3. En el tubo digestivo.
4. 4. En las glándulas salivares.
Gráfico de respuestas
Comentario

Pregunta directa que puede correlacionar. Lo que nos describe este caso clínico es un tumor de
Krukenberg. Acerca del mismo, se deben recordar las siguientes características:

- Origen más frecuente: estómago, aunque puede proceder de otras regiones del tubo digestivo.

- Histología: células en anillo de sello.

- Llegada al ovario por vía hematógena, es decir, se trata de una metástasis.(R3)

272. En relación a la infección por H. pylori, señalar la respuesta FALSA:

Se recomienda la erradicación de esta bacteria en personas con historia previa de


1. 1.
enfermedad ulcerosa péptica, aunque se encuentren asintomáticas.

!
!
!
!
Es recomendable la realización de un amplio despistaje y tratamiento de esta infección en
2. 2. la población general para disminuir la incidencia de cáncer gástrico y de enfermedad
ulcerosa péptica.
Para comprobar la eliminación de la infección por H. pylori con la prueba del aliento con
3. 3. urea marcada tras el tratamiento se debe suprimir la toma de inhibidores de la bomba de
protones al menos 2 semanas antes.
Se recomienda la erradicación de esta bacteria en familiares de primer grado de pacientes
4. 4.
con cáncer gástrico.
Gráfico de respuestas
Comentario

No está indicada la erradicación sistemática en la población general de H. pylori (opción 2 falsa).


Se consideran indicaciones de erradicación de H. Pylori:

•! -Gastritis crónica con metaplasia (gastritis atrófica)


•! - Úlcera gástrica y duodenal
•! - LNH de bajo grado de tipo MALT
•! - Familiares de primer grado de paciente afectos de adenocarcinoma gástrico
•! - Gastrectomizados
•! - Dispepsia no ulcerosa
•! - Anemia ferropénica no explicada
•! - Púrpura trombocitopénica idiopática
•! - Exigencia por parte del paciente
•! - Antes de iniciar AINEs en un paciente AINE naive

Interfieren en las pruebas diagnósticas de H pylori el consumo de antisecretores y antibióticos.


Motivo por el que antes de hacer un test diagnóstico es necesario suspender la toma de inhibidores
de la bomba de protones 15 días, la de antibióticos 15 días y de anti-H2 4 días. La confirmación de
la erradicación en la úlcera duodenal se recomienda con test del aliento. En cambio en la úlcera
gástrica (dado que en el 5% de los casos se deben a adenocarcimas gástricos) o úlcera
complicada, es necesario la confirmación con endoscopia de la cicatrización motivo por el que se
recomienda como test confirmatorio de erradicación el test de la ureasa o la histología.(R2)

273. Niño de 12 años con una talla para la edad en el percentil 10 y cuya velocidad de
crecimiento está por encima del percentil 25. Existen antecedentes de baja talla familiar
en la rama materna y de pubertad retrasada en la paterna. La exploración física es normal,
siendo el volumen testicular de 3 cc. Señale la respuesta CORRECTA:

1. 1. La opción terapéutica de elección es el empleo de hormona del crecimiento.


El indicador clínico que mejor refleja la normalidad del proceso es la velocidad de
2. 2.
crecimiento.
3. 3. Es aconsejable estudiar el eje de la hormona del crecimiento.
4. 4. Se debe realizar control periódico de la talla cada 2 meses.
Gráfico de respuestas
Comentario

Una pregunta de dificultad media que se responde de forma directa a partir del Manual CTO.

Tal como explicamos en el manual de la Sección de Pediatría, el indicador más sensible para
detectar precozmente este tipo de alteraciones es la velocidad de crecimiento (respuesta 2

!
!
!
!
correcta). Ésta es máxima durante los cuatro primeros años de la vida, produciéndose un segundo
incremento (aunque menor) niños.

Durante la pubertad, siendo más precoz en niñas que en Los datos que hay que tener en cuenta
para evaluar una talla baja son la velocidad de crecimiento y la correlación con la talla de los
padres. En este caso clínico se trata de un niño con una velocidad de crecimiento normal. Por otra
parte, la talla se puede considerar baja pero dentro de los límites normales y, sobre todo, acorde
con la talla materna.

Además, nos explican que en la rama paterna, existen antecedentes de pubertad retrasada.

Por consiguiente, no está indicado utilizar hormona del crecimiento ni realizar estudios del eje
hipotálamo-hipofisario. En otras palabras, no estamos ante un caso en el que haya que pensar en
patología de ningún tipo, al menos con estos datos, por lo que no precisa que tomemos medidas
extraordinarias.(R2)

274. Niño de 3 meses de edad que acude a urgencias porque en los últimos días ha
presentado clínica respiratoria. La madre comenta que presenta episodios frecuentes de
tos, en accesos, que a veces finalizan en cierta cianosis labial y se siguen de un vómito.
La madre lleva varias semanas con tos. La radiografía de tórax no muestra alteraciones
significativas. En los exámenes sanguíneos se observa leucocitosis a expensas de
linfocitosis. Ante la entidad clínica que sospecha, ¿cuál sería el tratamiento más
apropiado?

1. 1. Azitromicina.
2. 2. Aciclovir.
3. 3. Amoxicilina-clavulánico.
4. 4. Rifampicina.
Gráfico de respuestas
Comentario

Si se fija bien la pregunta describe el cuadro de tosferina clásico.

Recuerde que se produce por Bordetella pertussis, generalmente en niños menores de 1 año
donde la vacuna no ofrece aún buena inmunidad, aunque en los últimos años existe un repunte
entre adolescentes y adultos jóvenes, por atenuación del efecto vacunal.

Se precede de una clínica catarral inespecífica para dar lugar a la fase paroxística en forma de
accesos de tos emetizante, en ocasiones finalizados en apneas o gallo.

En la biometría hemática existe unalinfocitosis característica.

El tratamiento de elección son losmacrólidos, que se administrarán al paciente y a todos los


contactos como quimioprofilaxis (opción 1 correcta).

275. A una mujer de 43 años se le ha realizado una tumorectomía de mama derecha con
biopsia de ganglio centinela en la que la anatomía patológica muestra “carcinoma ductal
infiltrante de 0.6 cm de diámetro mayor, con bordes quirúrgicos libres. Dos ganglios
centinelas sin células tumorales”, y sin signos de metástasis a distancia. Presenta un
cáncer de mama en el siguiente estadio:

!
!
!
!
1. 1. pT3N1M0.
2. 2. pT1aN0M0.
3. 3. pT1bN0M0.
4. 4. pT2N1M0.
Gráfico de respuestas
Comentario

Una pregunta muy difícil, puesto que exige conocer al mínimo detalle el TNM del cáncer de mama.
Hablamos de T1 cuando se trata de tumores de 2 cm o menos.Dado que en este caso mide 0.6
cm, debemos precisar todavía más, dentro de que se trata de un T1. Entre 0.5 y 1 cm de diámetro,
el estadio exacto sería un T1b.Dada la ausencia de metástasis ganglionares y a distancia,
añadiríamos N0 y M0.(R2)

!
!
!
!
276. Todos los datos de laboratorio pertenecen a las anomalías producidas por infección
VIH en el sistema inmune, EXCEPTO:

1. 1. Deplección de linfocitos T4.


2. 2. Producción de inmunoglobulinas incrementada.
3. 3. Aumento de la producción de interferón gamma en respuesta a los antígenos.
4. 4. Aumento de beta-2-microglobulina.
Gráfico de respuestas
Comentario

La infección por VIH produce una inmunodepresión celular, por lo que no cabría esperar que
aumente la producción de IFN gamma ante antígeno alguno (respuesta 3 falsa). El resto de las
opciones sí ocurren como consecuencia de esta infección. Según ésta avanza, el número de
linfocitos CD4+ sufre un descenso progresivo. Como consecuencia de ello, dejan de ejercer su
función reguladora sobre el resto de las células de la inmunidad, por lo que se produce una
activación de las células B, que aumentan su producción de anticuerpos, lo que se expresa
mediante una hipergammaglobulinemia.

En cuanto a la beta- 2- microglobulina, es una proteína que forma parte de la membrana de las
células, y que se libera normalmente como resultado de la muerte natural de las células
sanguíneas en general, especialmente los glóbulos blancos. De ahí su elevación en enfermedades
donde se produce un intenso recambio de estas células, como los linfomas o el mieloma múltiple.
El nivel normal es de unos 3 microgramos por litro. En pacientes infectados por el VIH, esta cifra
suele estar aumentada, como consecuencia de la paulatina destrucción que van sufriendo sus
linfocitos a medida que progresa la enfermedad.(R3)

277. Señale la respuesta INCORRECTA en relación con los trastornos de la personalidad:

Se caracterizan por un patrón de comportamiento rígido y constante, estable a lo largo del


1. 1.
tiempo, que acaban por producir perjuicio al sujeto.
2. 2. El sujeto es capaz de reconocer sus peculiaridades, pero no le resultan desagradables.
Los pacientes con trastorno obsesivo de la personalidad suelen ser extrovertidos y con
3. 3.
facilidad para expresar sus emociones.
4. 4. En el trastorno paranoide de la personalidad es característica la desconfianza.
Gráfico de respuestas
Comentario

Lo característico de los trastornos obsesivos de la personalidad es la introversión, por la dificultad


para demostrar sus sentimientos y emociones. Por tanto, es radicalmente erróneo que sean
extrovertidos y que se expresen tal y como dice la opción 3, que es falsa. Este tipo de pacientes
suelen ser muy trabajadores, se adaptan con facilidad a una rutina (lógico, porque les da una
mayor sensación de control) y suelen tener actitudes rígidas.(R3)

278. ¿Qué tipo de carcinoma mamario es especialmente difícil de detectar por


mastografía?

1. 1. Tumor filodes.
2. 2. Tumor medular.
3. 3. Comedocarcinoma.
4. 4. Carcinoma inflamatorio.
Gráfico de respuestas

!
!
!
!
Comentario

El carcinoma medular de mama es una variedad histológica poco común. Se caracteriza por una
corona de linfocitos que rodea al tumor. El pronóstico es más favorable que en el resto de los
carcinomas mamarios, y tiene la peculiaridad de ser radiológicamente poco expresivo en las
mastografías.(R2)

279. El quiste hidatídico se localiza en diferentes partes del cuerpo. Indique cuál es el
órgano más afectado:

1. 1. Bazo.
2. 2. Hígado.
3. 3. Pulmón.
4. 4. Riñón.
Gráfico de respuestas
Comentario

La localización más frecuente donde se localiza el quiste hidatídico es la respuesta 2 hígado,


aunque también podría presentarse en las otras localizaciones como el pulmón.(R2)

280. Si el recién nacido está deprivado de oxigeno y entra en un estado de apnea


secundaria, su frecuencia cardiaca…………………… y su presión
sanguínea…………………

1. 1. Aumentará / aumentará.
2. 2. Disminuirá / disminuirá.
3. 3. Aumentará / disminuirá.
4. 4. Aumentará / no variará.
Gráfico de respuestas
Comentario

La única respuesta posible es la número 2. Traten de no olvidar el algoritmo de reanimación


neonatal.(R2)

281. A 17-day-old full-term newborn is brought to the pediatrician's office presenting with
yellowish skin and white stools. His mother is worried about the adverse effects of
breastfeeding. Gestational tests (including serological screening) were all normal.
Vaginal delivery was uncomplicated. Abdominal palpation shows an increased liver
silhouette. Laboratory blood test results are HB 19 g/dL, Total bilirubin 6 mg/dL, Direct
bilirubin 5 mg/dL, Blood type A + (mother B-), coombs' test negative. What is the most
likely diagnosis?

1. 1. Biliary atresia
2. 2. Congenital toxoplasmosis
3. 3. Newborn hemolytic disease
4. 4. Physiological jaundice
Gráfico de respuestas
Comentario
Biliary atresia. Jaundice in the newborn has several possible causes. The most common is
physiological jaundice, which appears during the first or second day of life. Breastfeeding-related

!
!
!
!
jaundice is also common. They are both secondary to an increase in indirect bilirubin and they tend
to resolve spontaneously when the liver increases its metabolic function. However, in this case, high
direct bilirubin levels and white stools suggest biliary disease. The most likely cause is biliary
atresia.(R1)

282. Señale cuál es el signo electrocardiográfico más característico de la hipercalcemia:

1. 1. Elevación de la onda T.
2. 2. Fusión del segmento ST.
3. 3. Alargamiento del intervalo PR.
4. 4. Acortamiento del intervalo QT.
Gráfico de respuestas
Comentario
Los trastornos del metabolismo del calcio constituyen un tema bastante preguntado en el MIR.
Recuerda que las cifras normales de Ca en sangre son de 9 a 10,5 y que la hipercalcemia suele
ser asintomática hasta cifras de 11,5- 12,5. Los síntomas que aparecen son: astenia, depresión,
anorexia, vómitos, estreñimiento, confusión y diabetes insípida nefrogénica. También acorta el QT,
pudiendo llegar a producir arritmias mortales. La hipocalcemia crónica, en cambio, no suele ser
asintomática sino que da síntomas neuromusculares y neurológicos (espasmos carpopedales,
convulsiones, irritabilidad, alteración de la memoria, depresión). En el ECG produce prolongación
del QT e inversión de la T, y disminuye la eficacia de los digitálicos.(R4)

283. Durante la etapa fetal, el (los) órganos (s) que recibe (n) proporcionalmente más
oxígeno es (son):

1. 1. Pulmones.
2. 2. Bazo.
3. 3. Cerebro.
4. 4. Intestinos.
Gráfico de respuestas
Comentario

El cerebro es el órgano que durante la etapa fetal recibe más oxígeno. Es el órgano más
importante ya que si se detiene el aporte de oxigeno enseguida se van a manifestar las
consecuencias, como por ejemplo la parálisis cerebral en los partos en los que el recine nacido ha
pasado momentos de anoxia.(R3)

284. Femenino de 18 años con amenorrea primaria y cariotipo 45,XO, NO esperaría


encontrar:

1. 1. Cúbito valgo.
2. 2. Linfedema en manos y pies.
3. 3. Escleróticas azules.
4. 4. Tórax en escudo.
Gráfico de respuestas
Comentario

El sindrome de Turner tiene cariotipos 45,XO, 46,XX y mosaicos. Presentan talla baja y frecuentes
malformaciones extragenitales: pliegue cervical (pterigion colli), cubitus valgus, tórax en escudo,
alteraciones renales, cardíacas (coartación aórtica en los cariotipos 45,XO)...Son causa de abortos

!
!
!
!
y a veces se detectan en vida embrionaria por presentar higromas quísticos, que son tumoraciones
linfáticas visibles en el ultrasonido desde el primer trimestre. Son frecuentes los gonadoblastomas.

La respuesta incorrecta es la 3, ya que ésta se presenta en pacientes con osteogénesis


imperfecta.(R3)

285. ¿Cuáles de las siguientes formas clínicas de neumopatía supurada es el más grave,
y con mayor tasa de mortalidad?:

1. 1. Neumotórax compresivo hipertóxico.


2. 2. Enfisema subpleural masivo.
3. 3. Parenquimal masiva.
4. 4. Ninguna de las anteriores
Gráfico de respuestas
Comentario

La neumoppatía supurada parenquimal masiva debemos recordarla como una forma clínica de
extrema gravedad y por lo tanto es la opción que debemos marcar en este caso. Respuesta 3
correcta.(R3)

286. En relación a la Neoplasia Trofoblástica Gestacional, NO es cierto que:

1. 1. El síntoma más frecuente de la mola total es la hemorragia genital.


2. 2. En la etiología de la mola completa, el origen es paterno.
3. 3. En la mola parcial, el cariotipo embrionario es diploide en el 90% de los casos.
4. 4. En la mola total es frecuente la aparición de hiperemesis gravídica muy precoz y severa.
Gráfico de respuestas
Comentario

Pregunta de un tema importante en el ENARM, que puedes contestar por descarte. En un


embarazo normal, el trofoblasto invade el endometrio hasta que encuentra un área rica en oxígeno.
En ausencia de mesodermo fetal que produce vasos sanguíneos, no hay buena transmisión de
oxígeno y el trofoblasto prolifera permanentemente dando origen a la enfermedad trofoblástica. El
trofoblasto, que se desarrollaría como placenta, produce mucha más cantidad de beta-HCG (útil en
diagnóstico y seguimiento) que un embarazo normal, lo que a su vez producirá la hiperemesis
gravídica y preeclamsia precoz. Además de la hiperemesis, será característico de la clínica la
metrorragia +/- dolor en hipogastrio +/- expulsión de vesículas (no frecuente pero patognomónico).
El ultrasonido es la prueba diagnóstica de elección, en la cual veremos la típica imagen en "copos
de nieve". En cuanto a la diferencia entre la mola completa y mola parcial, propongo los siguientes
puntos: MOLA COMPLETA: más frecuente, ausencia de tejido embrionario y amnios, fecundación
de un óvulo vacío (genes del padre), fecundación por un solo espermatozoide (23X que se divide
en 46XX) o fecundación por dos espermatozoides (23X+23Y --> 46XY), riesgo de enfermedad
trofoblástica persistente 15-20%; MOLA PARCIAL: menos frecuente, hay tejido embrionario y
amnios, hay genes maternos, fecundación por dos espermatozoides 47XXY (así pues la mola
parcial no sería diploide), riesgo de ETP 4%.(R3)

287. Son criterios de Ranson para el pronóstico de la pancreatitis aguda todos los
siguientes excepto:

1. 1. Edad mayor de 55 años.


2. 2. Amilasa > 250 UI.

!
!
!
!
3. 3. Calcio menor a 8 mg.
4. 4. Caída del hematocrito en más de 10 puntos.
Gráfico de respuestas
Comentario

Los criterios de Ranson se utilizan para determinar la severidad de la pancreatitis aguda. Se basan
en parámetros clínicos y laboratorio que se evalúan a las 0 y 48 horas. El encontrar un puntaje a
las 48 horas >3 habla de una pancreatitis aguda severa.

288. La forma más frecuente de trasmisión del citomegalovirus es:

1. 1. Lactancia materna.
2. 2. Intrahospitalario.
3. 3. Transplacentario.
4. 4. Transfusional.
Gráfico de respuestas
Comentario

Pregunta un poco complicada que puede resolver sin complicaciones.

El riesgo de contraer citomegalovirus (CMV) a través del contacto ocasional es muy bajo. El virus
generalmente se transmite de las personas infectadas a otras personas a través del contacto
directo con líquidos corporales, tales como la orina, la saliva y la leche materna. El CMV se
transmite a través del contacto sexual y también se puede propagar a través de trasplantes de
órganos y transfusiones de sangre.

!
!
!
!
Aunque el CMV puede diseminarse en la leche materna, las infecciones que se producen debido a
la lactancia por lo general no producen síntomas ni enfermedad en el recién nacido, y no se
recomienda interrumpir la lactancia materna.

Transmisión del CMV durante el embarazo. Aproximadamente entre el 30 y el 50% de las mujeres
embarazadas nunca han tenido una infección por CMV. Alrededor de 1 a 4 de cada 100 mujeres
que nunca antes han tenido una infección por este virus, durante el embarazo contraen la primo
infección. Cerca de un tercio de las mujeres (que se infectan por el CMV por primera vez durante el
embarazo) transmiten la infección al producto.

289. Respecto a la fisiología del embarazo, señale cuál de las siguientes afirmaciones es
INCORRECTA:

1. 1. Existe un aumento del flujo plasmático renal y del filtrado glomerular.


2. 2. Existe una ligera hiperventilación y alcalosis respiratoria.
El mecanismo fundamental de la anemia en el embarazo es el aumento en las necesidades
3. 3.
de hierro.
4. 4. El eje del corazón está ligeramente desviado hacia la izquierda.
Gráfico de respuestas
Comentario

Pregunta fácil sobre las adaptaciones materna al embarazo. Existe un aumento del flujo plasmático
renal gracias al HPL y también del filtrado glomerular en un 40%, concepto repasado previamente.

Existe un aumento de la ventilación pulmonar sobretodo a expensas del aumento del volumen
circulante, lo cual conduce a una alcalosis respiratoria.

El aumento de la capacidad abdominal hace que se eleve el diafragma, y el eje del corazón se
desvía a la izquierda.

El aumento del volumen plasmático es responsable del descenso de las proteínas plasmáticas
(aunque su síntesis pueda estar aumentada). Durante el embarazo aumenta la eritropoyesis y las
necesidades del hierro, pero es el aumento del volumen plasmático el mecanismo fundamental de
la anemia en el embarazo, ya que el plasma aumenta en mayor proporción que la masa
eritrocitaria.

290. Femenino de 28 años de edad, G2C1+embarazo de 38 semanas en periodo de


dilatación (4 cm, primer plano); disfruta de anestesia epidural. Hace un año se le realizó
cesárea por situación transversa. Estando previamente bien y de forma súbita presenta
un sangrado que coincide en el registro cardiotocográfico fetal con 4 desaceleraciones
tipo DIP 1 seguidos de una bradicardia fetal a 70 latidos por minuto. La presentación fetal
se palpa sobre el estrecho superior de la pelvis y la dinámica uterina prácticamente ha
cesado. Su actitud es:

Sospecha de placenta de inserción baja; esperará a la normalización de la frecuencia


1. 1.
cardíaca fetal y permitirá seguir el curso del parto.
2. 2. Sospecha de rotura uterina; realiza cesárea.
3. 3. Sospecha de rotura de vasa previa; realiza cesárea.
Sospecha de placenta de inserción baja; utilizará oxitocina intravenosa para mejorar la
4. 4.
dinámica uterina.
Gráfico de respuestas

!
!
!
!
Comentario

Una pregunta sencilla, de vital importancia para el ENARM, que se responde fácilmente con el
cuadro de diagnóstico diferencial que aparece en el Manual, sobre las hemorragias del tercer
trimestre.

•! Los datos de malestar fetal irían en contra de una placenta previa, donde no suelen
afectarse el estado materno ni el fetal (respuestas 1 y 4 falsas).
•! En la rotura de vasa previa, el dato típico sería un sangrado coincidente con la rotura de la
bolsa, que aquí no mencionan. Por otra parte, no encaja el dato del cese de la dinámica
uterina (respuesta 3 falsa).

En cambio, existen varias razones a favor de la respuesta 2, la rotura uterina, que sería la solución
correcta:

•! El antecedente de cesárea aumenta mucho el riesgo. De hecho una cesárea previa se


considera indicación de nuevas cesáreas en partos posteriores, para evitar esta
complicación.
•! La exploración uterina también sugiere una rotura, ya que el órgano está adinámico, aparte
de que el feto se palpa por encima del estrecho superior.

En este caso, no presenta dolor, que es uno de los datos más típicos de la rotura uterina. No
obstante, dado que nos insisten en que estamos usando anestesia epidural, no es raro que falte
este dato.

291. Señale, entre los siguientes elementos, cuál sería de utilidad para establecer el
estadiaje TNM del carcinoma invasor de cuello uterino:

1. 1. Metrorragia.
2. 2. Dolor pélvico.
3. 3. Pérdida de peso.
4. 4. Afectación de los parametrios.
Gráfico de respuestas
Comentario

Recuerde la importancia de la afectación de los parametrios en los cánceres de cervix.

El estadio II A tiene afectados los 2/3 superiores de la vagina sin afectar a los parametrios. Esto
hace que sea susceptible de ser tratado mediante la intervención de Werthein-Meigs. Sin embargo,
la afectación de los parametrios, nos pone directamente en un estadio II B que es susceptible de
tratamiento con radioterapia.(R2)

!
!
!
!

292. El anillo inguinal superficial esta formado por:

1. 1. Aponeurosis del oblicuo mayor.


2. 2. Aponeurosis del oblicuo mayor ipsilateral y del homólogo.
3. 3. Aponeurosis del oblicuo mayor contralateral.
4. 4. Aponeurosis del oblicuo mayor + arcada del transverso.
Gráfico de respuestas
Comentario

Pregunta clásica de anatomía. El anillo inguinal superficial lo forma la aponeurosis del oblicuo
mayor.(R1)

293. El anillo inguinal superficial esta formado por:

1. 1. Aponeurosis del oblicuo mayor.


2. 2. Aponeurosis del oblicuo mayor ipsilateral y del homólogo.
3. 3. Aponeurosis del oblicuo mayor contralateral.

!
!
!
!
4. 4. Aponeurosis del oblicuo mayor + arcada del transverso.
Gráfico de respuestas
Comentario

Pregunta clásica de anatomía. El anillo inguinal superficial lo forma la aponeurosis del oblicuo
mayor.(R1)

294. Hablando de antígenos de histocompatibilidad, en el síndrome de Sjögren primario


son frecuentes los antígenos de histocompatibilidad:

1. 1. CW3 y DR8.
2. 2. B7 y DR1.
3. 3. B27 y DR1.
4. 4. B8 y DR3.
Gráfico de respuestas
Comentario
Es una pregunta francamente difícil, los HLA son bastante preguntados en el MIR, pero no el del
síndrome de Sjögren. De recordar algo al respecto, que sea el HLA DR3, y está mencionado en la
respuesta 5. El HLA B27 se asocia a la espondilitis anquilopoyética; el HLA DR4 y DR1 a la artritis
reumatoide; el HLA DR2 y DR3 al LES, mientras que DR4 y DR7 al lupus farmacológico; el HLA
DQ2 y DB8 se asocian a enfermedad celíaca; el Graves- Basedow al HLA DR3 y DB8; y la
asociación más fuerte es la de la narcolepsia al HLA DR2 (=DoRmiDOS).(R4)

295. Un niño de 15 kg, ¿qué necesidades basales de líquidos precisa para 24 horas?:

1. 1. 2000 cc.
2. 2. 1500 cc.
3. 3. 1250 cc.
4. 4. 1000 cc.
Gráfico de respuestas
Comentario

Esta pregunta no tiene mucha importancia de cara al ENARM (si desde el punto de vista práctico).
Nos piden que calculemos las necesidades basales de agua. En una estado de normohidratación
nos podemos guiar por la siguiente estimación: 100 cc/kg por los primeros 10 Kg de peso, 50cc /kg
de 10-20 kg y 20 cc/kg a partir de los 20 de peso. Máximo 2-2.5 litros. Así, en este caso: (10 x 100)
+ (50 x5) = 1000+250 =1250 cc.(R3)

296. La causa más frecuente de fiebre persistente o recurrente durante el tratamiento de


una endocarditis infecciosa es:

1. 1. Aparición de resistencia durante el tratamiento.


2. 2. Embolias sépticas.
3. 3. Absceso miocárdico.
4. 4. Fiebre medicamentosa.
Gráfico de respuestas
Comentario

Pregunta bastante difícil sobre la endocarditis infecciosa, no te preocupes si la has fallado.

!
!
!
!
La fiebre existe en casi todos los enfermos de endocarditis, salvo ocasionalmente en ancianos, ICC
o intensa debilidad. Excepto cuando la enfermedad es aguda, la fiebre suele ser moderada, menos
de 39,4º C.

La fiebre suele desaparecer de 3 a 7 días después del tratamiento antimicrobiano. Durante el


tratamiento deben tomarse hemocultivos periódicos, que suelen volverse negativos varios días
después del inicio del tratamiento.

Si persisten la fiebre y la bacteriemia debe sospecharse la formación de abscesos miocárdicos o


metastáticos.

La causa más frecuente de fiebre persistente o recidivante durante el tratamiento es una reacción
medicamentosa (RC 5) y, en menor grado, las embolias.(R4)

297. Las lesiones malignas de la mama están localizadas más frecuentemente en:

1. 1. Cuadrante súpero interno.


2. 2. Cuadrante ínfero externo.
3. 3. Cuadrante súpero externo.
4. 4. Cuadrante ínfero interno.
Gráfico de respuestas
Comentario

La localización más frecuente del cáncer de mama es el cuadrante superoexterno: es algo que hay
que aprender de memoria simplemente.(R3)

298. Which of the following is regarded as the tumor marker of choice for diagnosis and
monitoring of colorectal carcinoma?

1. 1. Alpha-fetoprotein (AFP).
2. 2. Chromogranin A.
3. 3. Carcinoembryonic antigen (CEA).
4. 4. Cancer antigen 15.3 (Ca 15.3).
Gráfico de respuestas
Comentario

Una pregunta de dificultad media-baja sobre los marcadores tumorales. Veamos, a continuación,
cuáles son las neoplasias relacionadas con cada una de las opciones.

La alfafetoproteína se produce en el hígado y en el saco vitelino del feto. Su nivel es máximo


durante el primer trimestre del embarazo, descendiendo paulatinamente hasta el nacimiento. Poco
a poco, la albúmina va en aumento y reemplaza a la alfafetoproteína. Sus aplicaciones clínicas, por
lo tanto, estarán en relación con esto:

•! Durante el embarazo puede utilizarse para diagnosticar determinadas anomalías fetales.


Como ya sabemos, forma parte del triple test (beta-hCG, AFP y estriol).
•! En el adulto se eleva en determinados tumores, como el hepatocarcinoma y algunos
cánceres de origen testicular.

!
!
!
!
El CA-125 es un marcador tumoral que puede elevarse en diferentes tipos de cánceres. Aunque no
es patognomónico de ninguno de ellos, debemos recordarlo por su relación con el cáncer de
ovario. Recordemos que también puede aumentar en otras enfermedades ginecológicas, como la
endometriosis.

La cromogranina A es una proteína asociada a los gránulos neurosecretores. Éstos se encuentran


en células de estirpe neuroendocrina. Sabiendo esto, puedes imaginar a qué tumores puede
asociarse: adenomas hipofisarios, tumores del páncreas endocrino, feocromocitomas, carcinoma
medular de tiroides, cáncer de pulmón microcítico, tumores carcinoides, etcétera.

El antígeno carcinoembrionario puede encontrarse elevado en enfermedades no tumorales, como


traumatismos digestivos, infartos intestinales, enfermedades hepáticas, etcétera. No obstante, es
más conocido por su asociación a varios tumores digestivos, especialmente el cáncer de colon
(respuesta 3 correcta).

El CA 15.3 debemos recordarlo en relación con el cáncer de mama. Existe otro marcador, el CA
27.29, que aumenta la sensibilidad del CA 15.3 si se utilizan juntos. Recordemos que el CA 15.3 no
es útil como prueba de screening, ya que solamente se encuentra elevado en el 20-25% de las
pacientes con cáncer de mama en estadio I, II o III. En cánceres muy avanzados (estadio IV), su
elevación es bastante más frecuente.(R3)

299. Las células clave en el frotis de secreción vaginal sirve para el diagnostico de:

1. 1. Candidiasis vaginal.
2. 2. Vaginosis bacteriana.
3. 3. Vaginosis mixta.
4. 4. Vaginitis por Gram (-).
Gráfico de respuestas
Comentario

Las células claves son células del epitelio vaginal con borde mal definido de aspecto granuloso por
el gran número de bacilos de Gardnerella Vaginales unidos a su superficie. Se detectan células
claves cuando se mezcla el líquido vaginal con solución salina fisiológica y se observa al
microscopio.(R2)

300. Gestante de 24 semanas que acude a la consulta porque a su hijo de 4 años le


diagnosticaron hace 5 días varicela. La paciente no recuerda si padeció la enfermedad en
la infancia, pero sí sabe que no fue vacunada y está muy preocupada por la posible
afectación fetal. ¿Qué actuación sería la CORRECTA?

Solicitar cuantificación de Ig G, y si fuera negativo, administrar la gammaglobulina


1. 1.
específica.
2. 2. Tranquilizar a la paciente informándole de la ausencia de riesgos fetales.
3. 3. Administrar aciclovir oral a dosis de 800 mg, 5 veces al día, durante 5-7 días.
Administrar gammaglobulina específica y tranquilizar a la madre informándole de la
4. 4.
ausencia de riesgos fetales.
Gráfico de respuestas
Comentario

Pregunta clave para el nacional. El período de máximo riesgo de transmisión de la varicela son las
primeras 20 semanas de embarazo, sobre todo el primer trimestre (primeras 12). No obstante, ni

!
!
!
!
siquiera en estos casos supera el 1-2%. Más allá de las 20 semanas, el riesgo es mucho menor,
prácticamente nulo, por lo que la mejor respuesta es la 2. De acuerdo que lo de AUSENCIA puede
sonar muy extremista, pero está bastante cerca de la verdad, mientras que el resto son claramente
falsas.

La administración de gammaglobulina dentro de los cuatro días que siguen a un posible contagio
es de dudosa eficacia. La única indicación actual de administración es si el cuadro aparece las tres
semanas previas al parto para evitar la varicela neonatal que presenta una elevada mortalidad.(R2)

301. ¿Cuál de las siguientes afirmaciones en relación a la leucemia mieloide crónica es


CORRECTA?

El cromosoma Ph se produce a partir de una translocación recíproca entre los cromosomas


1. 1.
15 y 17.
2. 2. El cromosoma Ph aparece exlusivamente en precursores granulocíticos.
El oncogén BCR-ABL codifica una proteína (p210) con actividad tirosinquinasa
3. 3.
aumentada.
El mesilato de imatinib es un fármaco citotóxico alquilante utilizado en la fase crónica de
4. 4.
la enfermedad.
Gráfico de respuestas
Comentario

Pregunta de dificultad media acerca de la LMC.

La leucemia mieloide crónica (LMC) es un síndrome mieloproliferativo crónico (SMC), en el que


predomina la proliferación de la serie mieloide. Como otros síndromes mieloproliferativos crónicos,
aparece en personas de edad media y sin causa aparentemente conocida.

La LMC está claramente relacionada con un marcador citogenético, el cromosoma Philadelphia,


que aparece hasta en un 95% de los casos y se encuentra no solamente en las células
precursoras de la serie blanca, sino también en precursores eritroides y megacariocitos, incluso en
algunos casos hasta en los linfocitos (opción 2 incorrecta). Dicho cromosoma consiste en una
traslocación del material genético entre los cromosomas 9 y 22 (opción 1 incorrecta). Esta
traslocación da lugar a la unión del oncogén abl del cromosoma 9 con el oncogén bcr del
cromosoma 22, originando un híbrido anormal bcr/abl que codifica una proteína con actividad
tirosina cinasa, causante de la enfermedad. A su vez, esta proteína es la diana de un nuevo
tratamiento llamado mesilato de imatinib, un anticuerpo monoclonal que inhibe específicamente la
tirosina cinasa.(R3)

302. El serotipo de Haemophilus influenzae que causa con más frecuencia patología
infecciosa humana es:

1. 1. a.
2. 2. d.
3. 3. c.
4. 4. b.
Gráfico de respuestas
Comentario
Haemophilus influenzae es un patógeno exclusivamente humano, coloniza la nasofaringe desde
donde puede causar enfermedad. Algunas cepas de Haemophilus influenzae poseen una cápsula
que permite realizar serotipo de ellas. Las cepas con cápsula se clasifican en 6 serotipos (del a al

!
!
!
!
f); las cepas sin cápsula se denominan no tipificables. Los H. influenzae más frecuentes son el
serotipo b y los no tipificables. El serotipo b causa habitualmente enfermedad invasiva en niños
menores de 6 años [meningitis, epiglotitis, neumonía, osteomielitis, pericarditis, bacteriemia, etc.],
mientras que las cepas no capsuladas causan con más frecuencia infecciones mucosas que se
extienden por vecindad [otitis, bronquitis, sinusitis, etc.].(R4)

303. A 33-year-old woman comes to his physician due to recurrent epistaxis, petechiae
and ecchymosis. Lab tests show thrombocytopenia with a platelet count of 4000 platelets
/ microliter. The presumed diagnosis is immune trombocytopenia (ITP). Which of the
following statements is FALSE regarding ITP?

The presence of lymphadenopathy or splenomegaly on physical examination suggests a


1. 1.
different diagnosis.
Bone marrow analysis shows a decreased number of megakaryocytes, but no other
2. 2.
abnormalities.
The CBC shows isolated thrombocytopenia, often with large platelets, without anemia,
3. 3. unless there is significant bleeding or associated autoimmune hemolysis (Evans
syndrome).
The diagnosis of ITP is established by exclusion of other processes that cause
4. 4.
thrombocytopenia.
Gráfico de respuestas
Comentario

En la PTI se produce un aumento de la destrucción de las plaquetas por un mecanismo


inmunológico. Al tratarse de una trombopenia periférica existe un aumento de los magacariocitos
en la médula ósea.(R2)

304. De los mencionados a continuación, señale cuál es el principal estímulo para la


secreción de vasopresina (ADH):

1. 1. Disminución del volumen.


2. 2. Disminución de la presión.
3. 3. Aumento de la osmolalidad plasmática.
4. 4. Aumento del volumen.
Gráfico de respuestas
Comentario
La acción fundamental de la vasopresina es facilitar la absorción de agua libre en el túbulo colector
renal, a través de acciones mediadas por los receptores V2. El estímulo más importante para su
secreción es el aumento de la osmolalidad plasmática, aunque también puede responder
incrementando su secreción ante cualquier situación que curse con disminución de volumen o de la
presión arterial. Recordad que presenta acciones vasoconstrictoras mediadas por receptores
V1a.(R3)

305. Una mujer de 19 años, fue diagnosticada en su país de origen, hace 2 años, de
epilepsia y tratada con carbamacepina y difenilhidantoína, sin conseguir controlar del
todo los episodios comiciales. Acude a urgencias por crisis tónico-clónicas
generalizadas; en los exámenes de laboratorio presenta: calcio 5.8 mg/dl, fósforo 7 mg/dl
y magnesio 2 mg/dl. Existe alargamiento del QT en el electrocardiograma y calcificación
de los ganglios basales en la TC craneal. Puede tratarse de:

!
!
!
!
1. 1. Meningioma calcificado.
2. 2. Hipoparatiroidismo.
3. 3. Hipocalcemia secundaria a tratamiento con antiepilépticos.
4. 4. Intoxicación por vitamina D.
Gráfico de respuestas
Comentario

Aunque tiene apariencia de caso clínico difícil, en realidad la pregunta es sencilla:

El meningioma se descarta, por la sencilla razón de que la prueba de imagen no muestra nada de
esto. Sólo nos hablan de calcificación de los ganglios basales, que probablemente tendrá que ver
con algún trastorno del metabolismo fosfocálcico.

La intoxicación por vitamina D hubiera producido hipercalcemia, no hipocalcemia.

Una hipocalcemia por antiepilépticos sería debida al efecto inductor que tienen estos fármacos
sobre el metabolismo de la vitamina D, que se termina agotando, disminuyendo tanto el calcio
como el fosfato.

La respuesta correcta es la 2. Recuerde que la acción de la PTH aumenta el calcio y disminuye el


fosfato. Cuando existe un déficit de esta hormona, se produce la situación contraria (calcio bajo y
fosfato elevado, como en esta paciente). Parece que el diagnóstico de epilepsia no era correcto, de
ahí la falta de control con anticomiciales, y las crisis convulsivas serían debidas a la
hipocalcemia.(R2)

306. Mujer de 75 años que consulta por insomnio. Refiere que desde hace años se
despierta por la noche con sensación de hormigueo, quemazón en las piernas y a veces
en los brazos; estos síntomas se presentan también por la tarde. Nota mejoría al mover
las piernas, pero los síntomas recurren durante el reposo, por lo que no puede volver a
conciliar el sueño. Este cuadro nos podría orientar al diagnóstico de un síndrome de
piernas inquietas. ¿Cuál de las siguientes es FALSA?:

1. 1. El diagnóstico de esta afectación se basa en criterios clínicos.


Habría que realizar una analítica básica que incluyera perfil férrico, hormonas tiroideas y
2. 2.
B12 y ácido fólico.
3. 3. El diagnóstico se confirma con biopsia muscular.
4. 4. El pramipexol y el ropirinol se utilizan en el tratamiento.
Gráfico de respuestas
Comentario
El diagnóstico de síndrome de piernas inquietas se basa en criterios clínicos, siendo en la mayoría
de las ocasiones un cuadro de origen desconocido. En cualquier caso se debe realizar un estudio
de hierro (están descritos en anemias ferropénicas), estudio de hormonas tiroideas (descritas en
hipotiroidismo), y estudio de vitamina B12 y ácido fólico. El tratamiento es sólo sintomático, y debe
realizarse en aquellos casos en los que se altera de forma significativa la calidad del sueño. El
tratamiento se basa en el empleo de agonistas dopaminérgicos (pramipexol o ropinirol). La biopsia
muscular no es útil para el diagnóstico de este cuadro clínico.(R3)

307. ¿Cuál de los siguientes marcadores tumorales puede ser útil para el diagnóstico y
seguimiento del cáncer de páncreas?

!
!
!
!
1. 1. CA 15.3.
2. 2. CA 125.
3. 3. CA 19.9.
4. 4. Gonadotropina.
Gráfico de respuestas
Comentario

Los marcadores tumorales, en general, son poco sensibles e inespecíficos, pero tienen valor
pronóstico y utilidad en el seguimiento. En el cáncer de páncreas se eleva característicamente el
CA 19.9.

El CA 125 se eleva en la endometriosis y el cáncer de ovario.

El CA 15.3 se eleva en los tumores de mama.

La alfafetoproteína se eleva en muchos tumores como, por ejemplo, el de hígado o en testiculares


no seminomatosos.(R3)

308. Hombre de 72 años de edad. Antecedentes: ex fumador importante. Hipertenso.


Diabetes mellitus tipo 2 en tratamiento con antidiabéticos orales. Infección dentaria que
precisó extracción de pieza dental 1 año antes del episodio actual. Ingresa en Infectología
por presentar fiebre y tumoración en ángulo de la mandíbula; refería haber presentado
tumefacción en dicha localización en varias ocasiones en el último año, por lo que había
acudido al dentista y había recibido varias pautas de tratamiento antibiótico con
amoxicilina, con disminución de la tumefacción mientras tomaba el tratamiento
antibiótico y reaparición posterior. ¿Qué etiología le sugiere el cuadro clínico del
paciente?:

1. 1. Tuberculosis ósea.
2. 2. Actinomicosis.
3. 3. Celulitis facial.
4. 4. Carcinoma epidermoide.
Gráfico de respuestas
Comentario

El caso descrito en un paciente con manipulación de la mucosa oral y que presenta una
tumoración inflamatoria de evolución subaguda es muy sugestivo de actinomicosis. Esta bacteria
es un microorganismo Gram positivo cuyo tratamiento de elección es la penicilina, y en el caso
comenta una respuesta parcial a la amoxicilina. En algunos casos la lesión inflamatoria progresa y
fistuliza al exterior.

El caso clínico de esta pregunta sería típico de esta patología. La historia de infección dental con
extracción es un factor de riesgo importante y a la vez el antecedente clave que nos debe hacer
pensar en este diagnóstico. Las lesiones en la mucosa oral suelen ser la puerta de entrada del
microorganismo, flora normal oral en humanos. La DM es otro factor predisponente. La mandíbula
suele ser el órgano más afectado. La cronicidad del cuadro y evolución insidiosa también son
típicos. El retraso en el diagnóstico es frecuente por su baja incidencia. La osteomielitis por
cándida, la TB ósea y el carcinoma no mejorarían con amoxicilina. La celulitis facial suele tener una
evolución más aguda.(R2)

309. ¿Cuál de estas características NO se corresponde con Shigella?:

!
!
!
!
1. 1. Es un bacilo gramnegativo oxidasa negativo.
2. 2. Es lactosa positiva.
3. 3. No produce sulfhídrico.
4. 4. Produce diarreas en homosexuales.
Gráfico de respuestas
Comentario

Las infecciones del tracto digestivo son un tema bastante preguntado en el examen, sobre todo en
lo que se refiere a los distintos mecanismos fisiopatológicos de producción de diarreas. Mucho
menos preguntadas son las características microbiológicas de los distintos agentes que en ellas
intervienen. Las bacterias de la familia enterobacteriacea (entre las que se encuentra Shigella) son
huéspedes habituales del tracto digestivo. Salmonella, Shigella, Citrobacter, Yersinia y algunas
especies de E. coli entre otros son patógenos primarios; el resto son oportunistas. Son bacilos
gramnegativos oxidasa negativos (opción 1 correcta) no esporulados, aerobios o anaerobios
facultativos, móviles, que fermentan la glucosa con producción de ácido y que reducen los nitratos
a nitritos. Shigella no produce sulfhídrico (es Salmonella typhi la que característicamente produce
sulfhídrico en medio de "triple sugar iron" (opción 3 correcta) y es lactosa negativo (opción 2 falsa).
Al igual que otras especies, puede producir diarrea en homosexuales (opción 4 correcta).(R2)

310. Qué fármaco le parece más adecuado para el bloqueo de las crisis de angustia de
repetición:

1. 1. Benzodiacepinas.
2. 2. Imipramina.
3. 3. Fluoxetina.
4. 4. Buspirona.
Gráfico de respuestas
Comentario
Es bien sabido que las crisis de ansiedad son las más preguntadas dentro del núcleo de preguntas
de las neurosis en el MIR. Suelen preguntar por su clínica y diagnóstico en caso clínico, menos
sobre el tratamiento, pero lo debes conocer. Existen dos tipos de tratamiento: el de la propia crisis,
con BZD principalmente, y el de mantenimiento o profilaxis, con antidepresivos. Existen diversos
estudios al respecto comparando entre ISRS, IMAOs y ADT: en general los ISRS son mejor
tolerados, mientras que los más eficaces son los ADT (el más estudiado la imipramina).
Bibliografía: Manual Psiquiatría CTO- Medicina 4ªEd, pág 4(R2)

311. Which of the following conditions is a contraindication of breastfeeding in developed


countries?

1. 1. HBV infection in the mother


2. 2. HIV maternal infection
3. 3. Beast milk jaundice
4. 4. Prematurity
Gráfico de respuestas
Comentario
HIV maternal infection. When there is a chronic HBV infection, vaccination and immunoglobulin
administration protect the child against transmission.(R2)

312. Femenino de 65 años de edad, obesa, que acude a la consulta de ginecología


remitida por su médico de cabecera por pequeñas hemorragias uterinas indoloras como

!
!
!
!
única sintomatología. En la exploración ginecológica encontramos atrofia de los
genitales externos con útero de tamaño normal. ¿Cuál sería la siguiente exploración que
indicaría para descartar su principal sospecha diagnóstica?:

1. 1. Legrado fraccionado.
2. 2. Colposcopía.
3. 3. Laparoscopía.
4. 4. Biopsia dirigida.
Gráfico de respuestas
Comentario

Es importante que conozca las nociones básicas sobre las metrorragias postmenopaúsicas; ya que
ha sido un tema preguntado en el ENARM, tanto el diagnóstico como el tratamiento presentándolo
en forma de caso clínico. Debemos estar alerta ante toda hemorragia genital, ya que es el principal
síntoma y habitualmente el más precoz de la mayoría de los tumores genitales. Actualmente la
visualización de la cavidad endometrial por histeroscopia permite obtener muestras de biopsias
dirigidas que aumentan la sensibilidad del legrado - biopsia fraccionaoo, con lo cual el diagnóstico
de elección ha pasado a ser la biopsia dirigida por histeroscopia. La histerectomía está indicada en
las pacientes perimenopáusicas en las que el examen anatomopatológico demuestre una
hiperplasia endometrial atípica.(R4)

313. Niño estudiado por cianosis ligera y soplo en borde esternal izquierdo. Tras la
punción para una extracción sanguínea para conocer el hematocrito comienza a llorar
intensamente, se agita, aparece taquipnea, taquicardia y la cianosis se hace intensa. El
soplo desaparece y entra en una fase de somnolencia. El hematocrito era del 33%. ¿Qué
diagnóstico clínico establecería?:

1. 1. Crisis convulsiva atónica.


2. 2. Crisis de ansiedad.
3. 3. Síncope vasovagal.
4. 4. Crisis hipoxémica.
Gráfico de respuestas
Comentario
El tema de las cardiopatías congénitas es un tema muy amplio por lo que al estudiarlo debes
quedarte sobre todo con las características principales de las cardiopatías más preguntadas que te
permitan identificar un caso clínico como éste. Se trata de un niño con una tetralogía de Fallot que
sufre una crisis hipoxémica, que suele desencadenarse por ejercicio, llanto o dolor. Durante esta
crisis, aumenta la cianosis y disminuye el soplo de la estenosis pulmonar. La policitemia se justifica
por la hipoxia crónica que existe en estos pacientes. Aprovecha para repasar el tratamiento médico
de las crisis hipoxémicas, muy susceptible de ser preguntado en el MIR.(R4)

314. Una mujer postmenopáusica fue tratada de un carcinoma mamario infiltrante de 1


cm mediante tumorectomía y linfadenectomía. Esta última fue negativa. ¿Qué tratamiento
añadiría?:

1. 1. Ovariectomía bilateral.
2. 2. Poliquimioterapia.
3. 3. Radioterapia.
4. 4. Radioterapia y tamoxifeno.
Gráfico de respuestas

!
!
!
!
Comentario

Pregunta básica, no puede dudar la respuesta por nada. Sobre esta pregunta, debemos hacer una
serie de observaciones. Durante los últimos años, han cambiado algunos conceptos sobre el
cáncer de mama, por lo que podría ser anulada.

Ante un tumor tan pequeño, puede plantearse una cirugía conservadora en vez de una
mastectomía radical, con iguales resultados en términos de supervivencia. Pero debe quedar clara
una cosa: para que los resultados sean los mismos, es preciso complementar la tumorectomía con
radioterapia.

No está indicada la quimioterapia, puesto que los ganglios axilares son negativos y no existe
ningún tipo de factor de mal pronóstico. Recuerde que el factor pronóstico más importante en el
cáncer de mama es el número de ganglios afectados, y por ello la indicación más típica de
quimioterapia.

La hormonoterapia, actualmente, se reserva para las pacientes con receptores estrogénicos


positivos. Como en esta paciente no nos dicen nada al respecto, la pregunta no está bien
construida. Es necesaria esta información para decidir si indicar hormonoterapia.

Hasta hace poco, se trataba con hormonoterapia a todas las pacientes postmenopáusicas,
independientemente de la positividad de los receptores estrogénicos. Actualmente, esto ya no es
así, por lo que la 4 no sería correcta dado que no nos dicen nada sobre el estatus de los
receptores hormonales estrogénicos y la más adecuada sería la 3.(R3)

315. ¿Qué porcentaje de la luz del tronco de la arteria coronaria izquierda debe estar
obstruido para indicar tratamiento quirúrgico?:

1. 1. Más del 20%.


2. 2. Más del 50%.
3. 3. Más del 75%.
4. 4. Más del 90%.
Gráfico de respuestas
Comentario
Es una pregunta que no tiene mucha explicación. La única manera de contestarla es sabiéndosela.
Si nos la sabemos es muy fácil y si no prácticamente imposible de contestar. En el tronco
coronario, se considera una lesión significativa si obstruye el 50% de la luz. Para el examen, toda
lesión de esta características es indicación de cirugía de revascularización. En el resto de arterias,
se considera el 70% de la luz como límite de la severidad.(R2)

316. Neonato pretérmino que presenta al nacer un episodio de intensa dificultad


respiratoria. Tiene tiraje subcostal e intercostal, disociación toraco-abdominal y cianosis
generalizada que mejora cuando el niño llora. Además tiene rinorrea mucosa bilateral,
que se aspira, pero la sonda no pasa más allá de la cavidad nasal. ¿Qué tratamiento sería
el más adecuado para este paciente?

1. 1. Quirúrgico, diferido a que el niño pese más de 5 kg.


2. 2. Surfactante pulmonar.
3. 3. Tetina de McGovern.
4. 4. Cirugía urgente.

!
!
!
!
Gráfico de respuestas
Comentario

Esperamos que, el día del ENARM, no tenga ningún problema si aparece una atresia de coanas.
Lógicamente, al ser la atresia de coanas un problema estructural, el tratamiento de la misma es
quirúrgico. Respuesta correcta 4.

317. Respecto a la acromegalia inducida por GHRH, es cierto que:

1. 1. Característicamente no se acompaña de hipertensión arterial.


2. 2. Histológicamente se caracteriza por la presencia de hiperplasia de células somatotróficas.
3. 3. Se asocia con la hipersecreción de TSH.
El adenocarcinoma gástrico es el tumor que produce GHRH ectópicamente con más
4. 4.
frecuencia.
Gráfico de respuestas
Comentario
La causa más frecuente de acromegalia es la presencia de un adenoma hipofisario secretor de
GH. Existen raros casos descritos de acromegalia por producción ectópica de la hormona
liberadora de GH (GHRH), cuya etiología más frecuente son los tumores carcinoides torácicos o
abdominales. Estos pacientes presentan las características clínicas típicas de la acromegalia,
elevación de los niveles de GH, además de aumento del tamaño hipofisario en las pruebas de
imagen por hiperplasia de las células hipofisarias productoras de hormona del crecimiento. Algunos
tumores hipotalámicos también pueden producir cantidades elevadas de GH.(R2)

318. Una mujer de 55 años acude por tos. La radiografía de tórax muestra una masa hiliar
derecha. La tomografía computarizada del tórax revela un tumor de 5 cm cerca del
bronquio principal derecho a unos 4 cm de la carina. La mediastinoscopia y la
broncoscopia revelan que la masa es un carcinoma pulmonar de células pequeñas y que
también están afectados los ganglios linfáticos de las regiones subcarinales del hilio
derecho. El tratamiento más apropiado en este momento sería:

1. 1. Quimioterapia combinada.
2. 2. Quimioterapia combinada más radioterapia.
3. 3. Cirugía.
4. 4. Radioterapia.
Gráfico de respuestas
Comentario

El cáncer de pulmón es un tema muy importante para el MIR. Sobre esta pregunta, debes saber
que, para el carcinoma microcítico de pulmón, no se utiliza la clasificación TNM. En este caso
concreto, se tiende a simplificar en dos estadios tumorales, dependiendo de si existe o no
afectación extratorácica:

- Enfermedad localizada. Indica que la enfermedad está confinado a un hemitórax y a sus ganglios
regionales (campo de radioterapia torácica). Comprende la enfermedad confinada al tórax
(incluidos ganglios linfáticos mediastínicos, hiliares contralaterales y supraclaviculares
ipsilaterales), afectación del nervio recurrente y obstrucción de la cava superior. Tratamiento:
Quimioterapia + radioterapia.

!
!
!
!
- Enfermedad avanzada. No abarcable por la radioterapia, es la forma más frecuente de
presentación. Tratamiento: Quimioterapia.(R2)

319. Una de las siguientes afirmaciones referentes al sarampión NO es cierta:

1. 1. Puede producir púrpura trombocitopénica.


La ausencia de manchas de Koplik en el período catarral de la enfermedad impide el
2. 2.
diagnóstico.
3. 3. La morbilidad tuberculosa aumenta tras el sarampión.
4. 4. Es una enfermedad que causa morbilidad tanto inmediata como tardía muy importantes.
Gráfico de respuestas
Comentario

Pregunta de dificultad media acerca del sarampión. Esta enfermedad es una de las pocas que
tiene un dato patognomónico, las manchas de Koplik. Éstas son manchas blanquecinas sobre un
halo eritematoso que aparecen en la mucosa subyugal y que son fugaces, pudiendo desparecer en
12-18 horas. Por lo tanto, el no evidenciar estas manchas no excluye el diagnóstico de esta
enfermedad.(R2)

320. Embarazadade 29 semanas de


gestación que refiere dolor abdominal tipo cólico. Embarazo controlado de curso normal.
Como antecedente de interés, refiere una cesárea hace 2 años por parto pretérmino en
presentación podálica a las 27 semanas de amenorrea. Se realiza una monitorización fetal
no estresante con el resultado que se muestra en la imagen. La exploración muestra un
cérvix con una dilatación de 1 cm. La cervicometría medida por USG transvaginal es de
24 mm. ¿Cuál sería la conducta obstétrica más adecuada?:

1. 1. Observación hospitalaria.
2. 2. Ingreso para administración de corticoides y atosibán.
Ingreso para administración de corticoides, atosibán y antibioterapia al ser un gran
3. 3.
prematuro.
4. 4. Cesárea urgente por sufrimiento fetal.
Gráfico de respuestas
Comentario

La imagen muestra un registro cardiotocográfico fetal no estresante, en el cual observamos un feto


reactivo con ascensos, sin hallazgos patológicos. La dinámica uterina materna es regular y de

!
!
!
!
intensidad alta. Este dato debe de activar la alarma ante una posible amenaza de parto prematuro.
Para completar el diagnóstico es correcto realizar una exploración cervical junto con una
cervicometría, que es el mejor parámetro para predecir el riesgo de parto prematuro. En el USG
vaginal la cervicometría al ser < de 30 mm, también predice la amenaza franca de parto. Además
no debemos olvidar que el mayor riesgo de presentar un parto prematuro es tener antecedente de
uno previo. Por todos estos datos la conducta mas correcta es maduración pulmonar con
corticoides y tocolíticos, de elección el atosibán. Los antibióticos no se deben administrar salvo que
exista una sospecha o certeza de rotura prematura de membranas.(R2)

321. A las 6 de horas de instaurado tratamiento, la paciente avisa por la pérdida de líquido
amniótico. A la exploración se confirma la rotura prematura de membranas con expulsión
de líquido amniótico claro y dilatación de 4 cm y presentación cefálica. ¿Cuál será su
conducta en este momento?:

1. 1. Mantendría el mismo tratamiento.


Mantendría el mismo tratamiento asociando profilaxis antibiótica por rotura prematura de
2. 2.
membranas.
3. 3. Dejar evolucionar el parto por vía vaginal sin otros tratamientos asociados.
Dejar evolucionar el parto por vía vaginal con profilaxis antibiótica por rotura prematura
4. 4.
de membranas.
Gráfico de respuestas
Comentario
La exploración de la paciente ha cambiado y se encuentra en periodo de dilatación (4 cm) por lo
que está contraindicado el tratamiento tocolítico por lo que se suspenderá y se dejará evolucionar
el parto por vía vaginal. Debemos de administrar antibioterapia al ser una rotura prematura de
membranas en una gestación pretérmino.(R4)

322. El tratamiento de los ciclos anovulatorios incluye:

1. 1. Administración de clomifeno.
2. 2. Administración cíclica de progesterona.
3. 3. Administración cíclica de estrógenos y progesterona.
4. 4. Todas las anteriores.
Gráfico de respuestas
Comentario

Como las causas de la anovulación son múltiples y muy variadas: hiperprolactinemia,


hiperandrogenismo, trastornos tiroideos, resistencia a la insulina, incremento de corticoides,
posibilidad de patología tumoral, malos hábitos, etc., lo recomendable es detectar patología
susceptible de corrección pues el tratamiento etiológico no solo mejora el pronóstico sino que ataca
de fondo a un problema. Todas las opciones anteriores son posibilidades terapéuticas.(R4)

323. Todos los siguientes son indicación de realizar laparotomía exploradora tras herida
por arma blanca en abdomen, EXCEPTO:

1. 1. Ausencia de ruidos hidroaéreos.


2. 2. Evisceración de epiplón o vísceras.
3. 3. Lavado peritoneal diagnóstico negativo.
4. 4. Shock inexplicado.
Gráfico de respuestas

!
!
!
!
Comentario

Es una pregunta algo intuitiva ya que los demás datos de las opciones revisten gravedad. Si se fija
para realizar una laparotomía exploradora hay que tener evidencia de penetración en el peritoneo,
silencio abdominal persistente, evisceración, shock inexplicado, evidencia de sangre en estómago,
vejiga o recto y evidencia radiológica de lesión visceral: neumoperitoneo, desplazamiento de
vísceras. El lavado peritoneal diagnóstico negativo sería un dato de no gravedad y por ello la
opción a marcar.(R3)

324. El cuadro de hemorraga uterina disfuncional se asocia frecuentemente con:

1. 1. Hiperplasia endometrial.
2. 2. Hiperplasia secretora.
3. 3. Endometrio atrófico.
4. 4. Endometrio normal.
Gráfico de respuestas
Comentario

La respuesta correcta es la 1 hiperplasia endometrial, ya que se la HUD se puede presentar con


dicho dato.(R1)

325. Varón fumador de 57 años, que acude a su consulta por dolores, calambres en las
extremidades inferiores, que aparecen con el ejercicio y desaparecen con el reposo. Al
explorarlo encuentra que no palpa los pulsos distales en la extremidad, y cierto grado de
atrofia muscular. Señale la FALSA:

1. 1. La obstrucción arterial estará localizada probablemente en la femoral o la poplítea.


2. 2. La angiografía debe emplearse como diagnóstico de rutina.
3. 3. La mayor parte de la mortalidad en estos enfermos se debe a infarto de miocardio.
4. 4. La prueba de esfuerzo se emplea para objetivar las limitaciones funcionales.
Gráfico de respuestas
Comentario
Por la clínica que presenta este paciente, puedes deducir fácilmente que se trata de una isquemia
arterial crónica manifestada con claudicación intermitente. Recuerda que generalmente el dolor se
localiza en la pantorrilla porque casi siempre las oclusiones son femorales o femoropoplíteas.
Como se señala en la opción 3, la supervivencia de estos pacientes está limitada por la presencia
de enfermedad ateroesclerosa a otros niveles como las coronarias. El diagnóstico de la
enfermedad oclusiva arterial casi siempre se hace mediante la historia clínica y la exploración
física, pero pueden ser muy útiles técnicas no invasivas para determinar la gravedad de la
enfermedad, como la ecografía, el doppler o las pruebas de esfuerzo. Sin embargo, la arteriografía
es una prueba invasiva que no debe emplearse como prueba de rutina, pero sí tiene utilidad para
planificar la cirugía.(R2)

326. Un niño de 7 años presenta fiebre, adenopatías retroauriculares y cervicales


posteriores, exantema maculopapuloso y en la biometría hemática: leucopenia con
células plasmocitarias (células de Turk) y linfoides reactivas. ¿Cuál es el diagnóstico más
probable?

1. 1. Sarampión.
2. 2. Rubéola.

!
!
!
!
3. 3. Mononucleosis infecciosa.
4. 4. Varicela.
Gráfico de respuestas
Comentario

Tema que no falla su presencia cada año en el nacional. La rubéola es una enfermedad infecciosa,
producida por un Togavirus, cuyo período de incubación es de 2-3 semanas. El pródromos
consiste en un cuadro catarral leve, con fiebre baja o moderada, conjuntivitis sin fotofobia y un
enantema no patognomónico (manchas de Forcheimer). El signo más característico de esta fase
son las adenopatías retroauriculares, cervicales posteriores y postoccipitales. Poco después,
aparece un exantema, que es morbiliforme y confluyente en la cara. Se resuelve a través de una
mínima descamación. Es posible que aparezcan ciertas complicaciones, como la artritis (la más
frecuente), la encefalitis o la trombopenia.(R2)

327. ¿De qué tejidos están constituidos básicamente los leiomiomas uterinos?

1. 1. Tejido vascular.
2. 2. Tejido nervioso.
3. 3. Tejido endometrial.
4. 4. Tejido muscular liso.
Gráfico de respuestas
Comentario

Pregunta muy fácil y memorística. Tal y como indica la palabra los leiomiomas estan formados por
tejido muscular liso.(R4)

328. Señale entre las siguientes la afirmación QUE MEJOR defina a las artritis sépticas:

1. 1. El líquido articular suele ser de baja viscosidad y pobre en glucosa


2. 2. El germen causal más frecuente en todos los grupos etarios es el neumococo
3. 3. La articulación más típicamente afecta es la metacarpofalángica
En neonatos la manifestación más típica es la inflamación local y dolor localizado en la
4. 4.
articulación afecta.
Gráfico de respuestas
Comentario
Las artritis sépticas se manifiestan típicamente como monoartritis de comienzo brusco, y la
articulación más frecuentemente afecta es la rodilla. En neonatos la forma de manifestación
habitual es con una clínica general, no localizada (fiebre, síntomas sépticos…). El S.aureus es el
microorganismo más frecuentemente implicado en las artritis sépticas en adultos, salvo en los
estudios de EEUU donde en el rango de edad entre los 14 y 40 años (adultos jóvenes sexualemtne
activos) lo es el gonococo. Para otros grupos etarios el S. aureus puede ser el germen
predominante, pero depende ya de otras factores de riesgo (ej. Vacunación frente a H. influenzae
en niños). El neumococo prácticamente es una etiología casual de artritis infecciosa.(R1)

329. Todas las afirmaciones siguientes SALVO una son ciertas respecto a la asociación
de Helicobacter (anteriormente Campylobacter) pylori con gastritis:

1. 1. Se supone que el desarrollo de H.pylori en el estómago ocasiona gastritis crónica.


H. pylori es un bacilo gramnegativo que invade la mucosa gástrica, produciendo así una
2. 2.
inflamación de la mucosa.

!
!
!
!
3. 3. H. pylori puede identificarse por su capacidad para descomponer la urea.
4. 4. La eliminación de la bacteria logra una mejoría histológica de la mucosa gástrica.
Gráfico de respuestas
Comentario
La infección por H.pylori provoca inicialmente una gastritis aguda y, posteriormente, una gastritis
crónica. Esta inicialmente es antral y, posteriormente, se extiende por todo el territorio gástrico
causando atrofia. Es un bacilo gram negativo con un flagelo cuyo hábitat está en relación con las
capas más superficiales de la barrera gástrica, en concreto se sitúa entre el moco y el epitelio.
Puede cursar con gastritis, pero asintomática. Y se ha asociado a cáncer gástrico tanto difuso
como intestinal y linfomas. El tratamiento erradicador asocia ampicilina y claritromicina, pudiendo
utilizarse metronidazol en caso de alergia.(R2)

330. Mujer de 35 años que acude a su consulta por amenorrea secundaria de 5 meses de
evolución. Entre sus antecedentes ginecológico-obstétricos destacan un parto y tres
abortos posteriores. Le realiza un test de embarazo que es negativo. El estudio hormonal
muestra los siguientes resultados: PRL 10ng/ml, LH 5 mUI/ml, FSH 7 mUI/ml, estradiol 22
pg/ml. Le administra progesterona natural micronizada durante 10 días y no menstrúa. Le
administra entonces un anticonceptivo oral y tampoco menstrúa. ¿Cuál de los siguientes
será la causa de la amenorrea?:

1. 1. Prolactinoma.
2. 2. Síndrome de Asherman.
3. 3. Fallo ovárico precoz.
4. 4. Síndrome de Kallman.
Gráfico de respuestas
Comentario

Los antecedentes obstétricos de abortos de repetición ya nos orientan acerca de un síndrome de


Asherman ya que la causa son las sinequias uterinas tras legrados repetidos. Además tiene unas
determinaciones hormonales normales (se descarta el prolactinoma – prolactina sería alta- y el fallo
ovárico precoz- gonadotropinas estarían elevadas), no menstrúa tras la administración de un
gestágeno (el SOP sí lo haría), y es una amenorrea secundaria (el síndrome de Kallman causa la
típica asociación de anosmia y amenorrea primaria).(R2)

331. Dentro de las exploraciones de una pareja estéril, se debe realizar una
histerosalpingografía:

1. 1. El día 6-8 del ciclo.


2. 2. En el momento de la ovulación.
3. 3. El día 16-18 del ciclo.
4. 4. Inmediatamente antes de la menstruación.
Gráfico de respuestas
Comentario

Esta pregunta es normal fallarla porque no tiene porque saber algo que es de nivel especialista,
como cuando se debe realizar una hiterosalpingografía. Lo que debe saber es que esta prueba
permite valorar la obstrucción tubárica o uterina. Ocasionalmente puede resultar terapéutica y
repermeabilizar una obstrucción tras su realización.(R1)

332. Son técnicas sin tensión para reparación de hernias inguinales. EXCEPTO.

!
!
!
!
1. 1. Shouldice.
2. 2. Rives.
3. 3. Nyhus.
4. 4. Lichtenstein.
Gráfico de respuestas
Comentario

Pregunta díficil ya que exige conocer las técnicas de reparación de hernias. De entre las opciones
dadas, todas son técnicas de reparación sin tensión excepto la opción 1, técnica de Shouldice.(R1)

333. Femenino de 28 años que acude al ginecólogo por prurito vulvovaginal y leucorrea.
En la exploración existe abundante flujo amarillo-grisáceo espumoso y la vagina y el
exocérvix muestran un enrojecimiento difuso y un punteado más intenso. ¿Cuál sería el
tratamiento más adecuado en este caso?:

1. 1. Amoxicilina y clavulánico.
2. 2. Estrógenos tópicos.
3. 3. Doxiciclina.
4. 4. Metronidazol.
Gráfico de respuestas
Comentario

Tema fundamental en el ENARM, cada año se pregunta, te intentarán confundir con el aspecto del
flujo, pero no se deje engañar.

Recordemos las características de la tricomoniasis vaginal:

- Leucorrea espumosa, amarillo-grisácea, con pH básico (5-6).

- Prurito vulvar y ardor vaginal.

- Petequias (cuello en fresa).

- Diagnóstico mediante examen microscópico en fresco.

- Tratamiento con metronidazol.(R4)

334. Los característicos "granos de azufre" de la actinomicosis están formados


principalmente por:

1. 1. Microorganismos.
2. 2. Neutrófilos y monocitos.
3. 3. Monocitos y linfocitos.
4. 4. Eosinófilos.
Gráfico de respuestas
Comentario
Existen algunos datos microbiológicos típicos que conviene recordar, este es uno de ellos. Los
"granos de azufre" corresponden al cúmulo de estructuras filamentosas que forman los
actinomyces, y que podemos ver si observamos al microscopio una muestra del pus que drena de
los abscesos que produce este germen (respuesta correcta por tanto es la 1).

!
!
!
!

335. Hombre de 14 años de edad que


acude a urgencias por dolor escrotal izquierdo de aparición brusca de 1 hora de
evolución, que comenzó durante un partido de baloncesto. El paciente no refiere
antecedente traumático, ni fiebre. A la exploración física, el paciente se encuentra con
fascies dolorosas intensas, con náusea y diaforético. La auscultación cardiopulmonar es
normal y el abdomen es blando y depresible, sin signos de irritación peritoneal ni dolor
a la palpación profunda. En la inspección del área genital se observa lo que se observa
en la imagen. Ante la sospecha clínica, ¿qué otros hallazgos NO esperaría encontrar en
la exploración de los siguientes?

1. 1. Dolor a la palpación del testículo.


2. 2. Empeoramiento del dolor con la elevación del testículo afectado.
3. 3. Reflejo cremastérico abolido.
4. 4. Signo de la lágrima o mancha azul en el polo superior del testículo afectado.
Gráfico de respuestas
Comentario

El paciente de esta pregunta presenta un escroto agudo, es decir, un cuadro de tumefacción


dolorosa del escroto.

Dentro del diagnóstico diferencial del escroto agudo se distinguen fundamentalmente 3 cuadros:
torsión testicular, torsión del apéndice testicular y orquiepididimitis.

Es fácil distinguirlos según la edad de presentación, la clínica y la exploración.

En este caso estamos ante una torsión testicular, que supone la causa más frecuente de escroto
agudo a partir de los 12 años.

Estos niños presentan un intenso dolor a la palpación del testículo, que empeora con la elevación
del mismo, lo cual se conoce como signo de Prehn negativo. Otro dato muy característico es la
ausencia de reflejo cremastérico, ya que en la torsión testicular se tuerce el cordón espermático, lo
que condiciona una compresión del paquete vásculo-nervioso.

El signo de la mancha azul se presenta en la torsión del apéndice testicular, un cuadro típico de los
niños menores de 12 años.(R4)

!
!
!
!
336. Al paciente de la pregunta anterior se le realizó un ultrasonido Doppler. ¿Cuáles cree
que fueron los hallazgos ultrasonográficos y la actitud terapéutica más CORRECTA entre
las siguientes?

1. 1. Aumento del flujo testicular. Exploración quirúrgica urgente con detorsión testicular.
2. 2. Aumento del flujo testicular. Detorsión quirúrgica urgente con orquidopexia bilateral.
3. 3. Disminución del flujo testicular. Detorsión quirúrgica urgente con orquidopexia bilateral.
4. 4. Aumento del flujo testicular. Detorsión quirúrgica urgente con orquidopexia unilateral.
Gráfico de respuestas
Comentario

El ultrasonido Doppler en la torsión testicular muestra una reducción del flujo sanguíneo del
testículo afectado, ya que supone una torsión del cordón espermático y, por tanto, una compresión
del paquete vásculo-nervioso.

Si la sospecha clínica es de torsión testicular no es necesario perder el tiempo realizando un


ultrasonido ya que se trata de una urgencia quirúrgica.

El tratamiento consiste en la detorsión del testículo afectado y la fijación de ambos al escroto


(orquidopexia bilateral, ya que la causa es una túnica vaginal redundante que permite una excesiva
movilidad al testículo dentro del escroto, y este defecto suele ser bilateral).

La orquidectomía (extirpación del teste) se reserva sólo para los casos en que el testículo no es
viable, lo cual es poco probable en nuestro caso que lleva sólo una hora de evolución.(R3)

337. En el caso de que a un lactante le tengamos que mantener a dieta, por ejemplo, por
una intervención quirúrgica, si pesa 5 kg, ¿qué cantidad de líquido intravenoso
deberemos administrarle al día?:

1. 1. 500 cc.
2. 2. 600 cc.
3. 3. 400 cc.
4. 4. 750 cc.
Gráfico de respuestas
Comentario

Esta pregunta no tiene mucha importancia de cara al ENARM, aunque es importante desde el
punto de vista práctico. Nos piden que calculemos las necesidades basales de agua. En una
estado de normohidratación nos podemos guiar por la siguiente estimación: 100cc/kg por los
primeros 10 Kg de peso, 50 cc /kg de 10-20 kg y 2 0cc/kg a partir de los 20 de peso. Máximo 2-2.5
litros. Así, en este caso: 100 x 5 = 500 cc.(R1)

338. La subclase de IgG que NO fija complemento por la vía clásica es:

1. 1. IgG1.
2. 2. IgG2.
3. 3. IgG4.
4. 4. IgG5.
Gráfico de respuestas
Comentario

!
!
!
!
Pregunta de dificultad media pero que debes dominar a la perfección, ya que los aspectos
relacionados con las Ig son muy importantes en el examen MIR. Estudia bien las características
propias de cada Ig y haz hincapié en las peculiaridades que más han aparecido en el examen.
Recuerda que la frecuencia de las distintas Igs responde a la regla GAMDE. La IgG es la Ig más
abundante en las secreciones internas, tanto del RN como del adulto, y posee 4 subclases. Dentro
de sus características está atravesar la placenta y fijar complemento (no la IgG4). La IgA es la
segunda en abundancia en las secreciones internas y la más abundante en las externas. La forma
sérica es monomérica y la secretada es dimérica unida por el componente J (producido por las
linfocitos B) y el componente secretor (producido por las células epiteliales). Su principal acción es
antivírica. La IgM es la mayor de las Igs, de estructura pentamérica, lo que le confiere excelente
capacidad para fijar complemento pero le imposibilita para atravesar la placenta. Es la principal Ig
de la respuesta timo independiente y de la fase primaria de la timo dependiente. La IgD actúa
sobre todo como receptor de superficie de los linfocitos B. La IgE es la menos abundante y tiene
implicaciones sobre todo en las reacciones alérgicas (al unirse a los mastocitos provoca la
degranulación de éstos) y en la protección frente a helmintos.(R3)

339. El tumor de ovario que se presenta más frecuentemente en mujeres jóvenes:

1. 1. Cistoadenoma seroso.
2. 2. Fibroma de ovario.
3. 3. Quiste dermoide.
4. 4. Tecoma.
Gráfico de respuestas
Comentario

El tumor de ovario más frecuente en mujeres jóvenes es el teratoma maduro, también llamado
quiste dermoide. Es un tumor benigno y fácilmente reconocible en la ecografía transvaginal.
Pertenece al grupo de los tumores germinales.(R3)

340. Es frecuente el siguiente cuadro clínico asociado a infección por M. kansansii:

1. 1. Adenitis.
2. 2. Osteoarticular.
3. 3. Enfermedad diseminada.
4. 4. Pulmonar.
Gráfico de respuestas
Comentario

Los dos síntomas frecuentes de la infección con M. kansasii son dificultad para respirar y fiebre.
Otros síntomas incluyen: sudores nocturnos, escalofríos, pérdida de peso, pérdida de masa
muscular, dolores abdominales, fatiga (por lo general, como resultado de la anemia) y diarrea. El
M. kansasii también puede causar agrandamiento del hígado, del bazo y de los ganglios
linfáticos.(R4)

341. ¿A cuál de estas circunstacias patológicas NO se asocia la gastritis atrófica?:

1. 1. Anticuerpos anticélula parietal positivos.


2. 2. Déficit de factor intrínseco.
3. 3. Anemia perniciosa.
4. 4. Hipersecreción ácida.

!
!
!
!
Gráfico de respuestas
Comentario
En general, la atrofia gástrica va a condicionar una hipofunción gástrica y, por tanto, hiposecreción
ácida, con elevación del pH luminal y descenso de los niveles séricos de pepsinógeno I. La atrofia
puede ser debida tanto a la infección por H.pylori como a un origen autoinmune, en este caso se
asocia a anticuerpos anticélula parietal positivo y antifactor intrínseco. Estos anticuerpos
neutralizan o bloquean al factor intrínseco, el cual a su vez puede estar disminuido al estar
reducida la masa celular productora del mismo, y, por todo ello, reducirse la absorción de vitamina
B12 y condicionar la anemia perniciosa secundaria.(R4)

342. A woman diagnosed with primary adrenal insufficiency complains of high fever,
chest pain and cough with sputum for the past 2 days. She started with intractable
vomiting and abdominal pain 4 hours ago. Which of the following is the most appropriate
next step in management?

1. 1. Double the dosage of oral corticosteroid drugs and increase salt intake.
2. 2. Give antiemetics and increase the dosage of oral steroids.
Start antibiotic treatment for community-acquired pneumonia and double the usual dose of
3. 3.
steroids.
4. 4. Hydration and high-dose intravenous corticosteroid therapy.
Gráfico de respuestas
Comentario

Nos presentan el caso clínico de una paciente diagnosticada de una insuficiencia suprarrenal
primaria que tras un cuadro de infección respiratoria presenta vómitos y dolor abdominal. Esta
sintomatología nos debe orientar al diagnóstico de crisis suprarrenal. El tratamiento de elección de
este cuadro es la reposición de volumen con suero salino y administración de hidrocortisona por
vía i.v. en dosis elevadas. Recuerde que en las crisis suprarrenales no es preciso administrar
mineralocorticoides porque la hidrocortisona en dosis elevadas ejerce estos efectos. En caso de
que la paciente no hubiera presentado el cuadro de vómitos, el tratamiento correcto hubiera sido
duplicar o triplicar la dosis de glucocorticoide habitualmente administrada. Por último recuerden
que la causa más frecuente de crisis suprarrenal no son las infecciones, sino el abandono brusco
de un tratamiento corticoideo prolongado.(R4)

343. En la menopausia, asociada a los estrógenos, se aconseja el empleo de los


gestágenos en cada ciclo de tratamiento, durante un período de varios días cuando
usamos una pauta cíclica. Estos días son:

1. 1. 7 días.
2. 2. 8 días.
3. 3. 12 días.
4. 4. 15 días.
Gráfico de respuestas
Comentario

Pregunta de las difíciles en cuanto a la THS en la menopausia. Cuando optamos por una pauta
cíclica de THS, la duración de la administración de gestágenos se establecerá únicamente desde
la óptica de obtener un mínimo de hiperplasias de endometrio con un mínimo de efectos
secundarios. Así, las dosis utilizadas serán un mínimo de 10-12 días al mes del gestágeno (opción

!
!
!
!
correcta la 3). Recuerde que otra de las opciones en la THS es la pauta contínua en la que se
administran gestágenos todos los días.(R3)

344. Una niña de 14 años que sufre un traumatismo directo sobre la rodilla izquierda en
semiflexión presenta bloqueo de dicha rodilla, gran deformidad lateral y dolor ante
cualquier intento de movilización. Señale el enunciado que considere FALSO si usted
sospecha una luxación de rótula:

1. 1. La artrocentesis probablemente demuestre hemartros.


2. 2. Es obligado el estudio radiográfico previo a cualquier maniobra de reducción.
El manejo inicial de esta lesión, tras la reducción, debe ser ortopédico, siempre y cuando
3. 3.
no haya otras lesiones asociadas.
4. 4. Las probabilidades de que este cuadro se repita en el futuro son mínimas.
Gráfico de respuestas
Comentario
La luxación de rótula en niñas de 14 años típicamente se asocia a hemartros por rotura de vasos
de los alerones rotulianos. Antes de pensar en maniobras de reducción debemos realizar una
radiografía simple, pues en muchos casos la rótula se reduce espontáneamente poco después de
la luxación. El test de la aprensión (o aprehensión) consiste en desplazar lateralmente la rótula, a
lo que la paciente (por miedo a una luxación provocada por esta maniobra) suele responder con la
contracción del cuádriceps y dolor brusco. El tratamiento inicial de estas lesiones es ortopédico y
rehabilitador. Muy rara vez se precisa la cirugía. Las probabilidades de que el cuadro se repita son
elevadas, como su nombre habitual indica (luxación recidivante de rótula).(R4)

345. ¿Cuál es el método más accesible para detectar precozmente una hidrocefalia en un
lactante?:

1. 1. Medición del perímetro cefálico.


2. 2. US cerebral.
3. 3. TAC craneal.
4. 4. RM craneal.
Gráfico de respuestas
Comentario

El término hidrocefalia implica la presencia de un sistema ventricular dilatado secundariamente a


un aumento de presión del líquido cefalorraquídeo. La dilatación del sistema ventricular en un
pequeño que no tiene cerradas las fontanelas genera el efecto de un aumento del perímetro
craneal. Si este PC crece a ritmo inusual, ha de sospechar la presencia de hidrocefalia.(R1)

346. Uno de los siguientes NO forma parte del tratamiento de ningún tipo de urticaria, sea
ésta aguda o crónica. Señale cuál:

1. 1. Cloroquina.
2. 2. Cimetidina.
3. 3. Hidroxicina.
4. 4. Ondansetron.
Gráfico de respuestas
Comentario
Los antihistamínicos H1 son el tratamiento de primera línea de la urticaria. En general son seguros
y tienen pocos efectos adversos. Se dividen en clásicos (producen sedación), como la hidroxicina,

!
!
!
!
y de segunda y tercera generación, estos dos últimos grupos no producen sedación o si la
producen, ésta es mínima. Se pueden utilizar solos o combinar un clásico con uno de 2ª ó 3ª
generación. Cuando no existe una respuesta adecuada a estos fármacos pueden asociarse a
antihistamínicos H2 como la cimetidina. La ciproheptadina está especialmente indicada en el
tratamiento de la urticaria por frio. La cloroquina se utiliza en el tratamiento de la urticaria solar,
también se ha descrito su utilización en algunos casos aislados de urticaria vasculitis. Las formas
severas se manejan con corticoides sistémicos y en los casos graves con anafilaxia se utiliza la
adrenalina.(R4)

347. Which of the following options should be initiated as soon as possible in a patient
with neonatal sepsis at an early stage?

1. 1. Prostaglandine infusion.
2. 2. Mechanical respiratory support and surfactant administration.
3. 3. Nil per os (NPO) and platelet transfusion.
4. 4. Aminoglycosides + ampicillin or a third-generation cephalosporin
(R4)

348. Paciente de 34 semanas de gestación, que acude a urgencias por presentar


contracciones uterinas con la sospecha de amenaza de parto pretérmino. Se realiza
registro cardiotocográfico fetal, comprobándose la existencia de 2 contracciones en 20
minutos y una longitud cervical medida por USG de 3 cm. ¿Cuál sería la actitud
CORRECTA?

Ingresar a la paciente y dejar evolucionar, pues el feto ya está maduro bajo el punto de
1. 1.
vista pulmonar.
Dejar que se desplace nuevamente a su domicilio advirtiéndole que realice reposo y que
2. 2.
acuda de nuevo a Urgencias si la dinámica se hace más frecuente.
3. 3. Ingresar a la paciente y administrarle tocolíticos para frenar la dinámica.
Ingreso en la sala de expectantes repitiendo, si sigue la dinámica, la ecografía vaginal para
4. 4.
ver si existe modificación del cérvix.
Gráfico de respuestas
Comentario

Pregunta difícil acerca del parto pretérmino.

Factores promotores del parto son la oxitocina (el reflejo de Ferguson está mediado por la
oxitocina), las hormonas suprarrenales y el tono simpático. Otro factor a tener en cuenta es la
distensión uterina. Factores uteroinhibidores son la correcta hidratación, el reposo y la
progesterona.

Actualmente, el mejor indicador del riesgo de parto pretérmino es la longitud cervical medida por
USG, de tal modo que una longitud inferior a 3 cm supone claro riesgo. La detección de
fibronectina en el cérvix o la vagina más allá de la semana 20 también indica aumento del riesgo.

Actualmente, el fármaco de elección es el atosibán, que es un antagonista de los receptores de la


oxitocina, acompañado de una correcta hidratación, sedación y reposo absoluto. Si no se puede
usar, utilizaremos el ritodrine (b2-agonista).

!
!
!
!
Fíjese bien en que la paciente de la pregunta no está en trabajo de parto, porque tiene dos
contracciones / 20 min (tiene que ser al menos dos contracciones / 10 min) y no está dilatada,
puesto que el dato que te dan es la longitud cervical. Por consiguiente, la conducta a seguir es
enviar a la paciente a su domicilio recomendando reposo y que vuelva si la dinámica se hace más
frecuente (repuesta 2 correcta).

349. Una niña de 8 años presenta desde hace unos meses alteración de la atención y del
rendimiento escolar. Frecuentemente presenta episodios de mirada fija y parpadeo. En el
EEG aparecen descargas generalizadas con punta-onda a 3 ciclos/segundo. ¿Qué tipo de
epilepsia presenta?

1. 1. Epilepsia mioclónica de la infancia.


2. 2. Crisis de ausencia.
3. 3. Síndrome de Lennox-Gastaut.
4. 4. Crisis parciales complejas.
Gráfico de respuestas
Comentario

El “pequeño mal”, o crisis de ausencia típicas, son crisis generalizadas que aparecen en la infancia
y que se caracterizan por desconexiones del nivel de conciencia, que ocurren de modo brusco,
muchas veces al día y que son de corta duración. Su frecuencia aumenta con la hiperventilación.
En el electroencefalograma, se observan con frecuencia complejos punta- onda a 3 Hz, como nos
muestran en esta pregunta.

Recuerda que, en este cuadro, podemos considerar dos tratamientos como de primera elección: la
etosuximida y el ácido valproico.(R2)

350. Which of the following treatment improves the quality of the cervical mucus?

1. 1. Estrogens.
2. 2. Danazol.
3. 3. Progestogens
4. 4. Gonadotropins.
Gráfico de respuestas
Comentario

No hay que precuparse en exceso por haber fallado esta pregunta, es de una gran dificultad. Lo
que sí que debesmos conocer es que el citrato de clomifeno es un inductor de la ovulación a través
del aumento de secreción de hormonas hipofisarias, y que se utiliza, por ejemplo, en mujeres con
SOP que desean descendencia. Los estrógenos tiene un efecto sobre el moco cervical, haciendo
que mejore su calidad, en el momento del ciclo en que deben pasar los espermatozoides para una
posible fecundación.(R1)

351. ¿Cuál de estas situaciones constituye una verdadera contraindicación para la


vacunación?:

1. 1. Reacción leve a dosis previa, como dolor, malestar general o febrícula.


2. 2. Lactancia materna.
3. 3. Intolerancia a la lactosa.
4. 4. Triple vírica en inmunodeprimido grave.

!
!
!
!
Gráfico de respuestas
Comentario

Existen falsas contraindicaciones para la vacunación infantil. Recuerden que el niño


inmunodeprimido grave no debe ser vacunado con virus vivos atenuados (triple vírica).(R4)

352. ¿Cuál de estas respuestas sobre el asma en la infancia es FALSA?

1. 1. La eosinofilia es típica del asma, tanto en el extrínseco como en el intrínseco.


Como broncodilatadores, los fármacos beta agonistas son muy superiores a los
2. 2.
anticolinérgicos.
Los corticoides inhalados son menos eficaces que en el adulto, por lo que apenas se
3. 3.
utilizan por esta vía.
Los antagonistas de los receptores de los leucotrienos mejoran los síntomas del asma y
4. 4.
reducen las necesidades de los beta-agonistas complementarios.
Gráfico de respuestas
Comentario

No es cierto que los corticoides inhalados sean menos eficaces en el asma infantil que en el adulto,
ni que apenas se usen en la infancia. El asma persistente moderado y grave se maneja con
corticoides inhalados en los niños como en los adultos. En el asma persistente leve se puede
intentar sustituir el corticoide inhalado a dosis bajas por el cromoglicato o el nedocromil.(R3)

353. En un enfermo con LES, ¿cuál es la prueba más sensible para medir la activación
del complemento?:

1. 1. CH 50
2. 2. C 1
3. 3. C 3
4. 4. C 4
Gráfico de respuestas
Comentario
Los niveles séricos elevados de ANA y anti- DNAds con concentraciones bajas de complemento
suelen reflejar cierto grado de actividad de la enfermedad, especialmente en los pacientes con
nefritis. Los niveles totales de complemento hemolítico funcional (CH50) constituyen la medición de
mayor sensibilidad para conocer la activación del complemento. Los niveles cuantitativos de C3 y
C4 se obtienen fácilmente. Las concentraciones muy bajas de CH50 con niveles normales de C3
sugieren un déficit hereditario de un componente del complemento que se asocia muchas veces a
LES con ANA negativos. Ten en cuenta este último detalle que fue preguntado en una ocasión en
una pregunta MIR. En el LES encontraras habitualmente descenso de CH50, de C3 y de C4.(R1)

354. ¿Cuál de las siguientes respuestas NO es cierta en la hepatopatía alcohólica?

La α-glutamiltranspeptidasa sérica es un marcador biológico, no exclusivo de consumo de


1. 1.
etanol.
2. 2. La relación GOT:GPT es generalmente =1.
3. 3. En la fase de cirrosis, existe un aumento en la incidencia de hepatocarcinoma.
4. 4. En las formas graves de hepatitis alcohólica, es adecuado el tratamiento con corticoides.
Gráfico de respuestas
Comentario

!
!
!
!
Ésta es una pregunta clásica, que ya se ha preguntado de forma similar en ENARM previos.

En la hepatopatía alcohólica es típica la elevación moderada de las transaminasas con una GOT (o
AST) 2 veces superior a la GPT; por tanto, la opción 2 es falsa. Regla mnemotécnica: GOT/AST de
alcohol.

Todas las demás respuestas son ciertas en la hepatopatía alcohólica.(R2)

355. ¿Qué significa el término acantosis?:

1. 1. Reacción inflamatoria de la epidermis.


2. 2. Aumento del espesor de la epidermis.
3. 3. Disminución del espesor de la epidermis.
4. 4. Aumento del espesor de la capa córnea.
Gráfico de respuestas
Comentario
Pregunta de dificultad intermedia sobre la anatomía patológica de las lesiones cutáneas. La
acantosis es el aumento del estrato espinoso ( opción 2 correcta). Hay otros sustrtos
anaomopatológicos básicos en dermatología que vale la pena recordar, la queratinización con
retención de núcleos en el estrato córneo es una paraqueratosis, y es típica del psoriasis. La
hiperplasia del estrato córneo es una hiperqueratosis y se da en muchos procesos, generalmente
crónicos, eccemas, psoriasis, ictiosis#La pérdida de conexiones entre los queratinocitos es una
acantolísis, es típica del péfigo vulgar y da lugar a ampollas intraepidérmicas.El edema intercelular
de la epidermis se denomina espongiosis y es característico del eccema agudo y subagudo Por
último el edema intracelular o balonización que es típica de los virus herpes.(R2)

356. Acude a su consulta un enfermo de 28 años con un cuadro de 9 meses de evolución


de dolor lumbar bajo asociado a rigidez matutina de 60 minutos, que mejora con el
ejercicio o la actividad. Hace unas semanas comenzó con dolor, tumefacción y limitación
funcional en rodilla izquierda, tobillo derecho y “dedo en salchicha” en el terder dedo de
la mano derecha. Tras la toma de antiinflamatorios no esteroideos, ha mejorado sólo
discretamente. En este momento, indique cuál de las siguientes afirmaciones es cierta:

1. 1. Los corticoides orales serán una parte importante del tratamiento en este paciente.
2. 2. Una radiografía anteroposterior de pelvis y de columna lumbar aportará poco en este caso.
3. 3. La sulfasalazina será una parte importante del tratamiento en este paciente.
4. 4. La uveítis posterior es relativamente frecuente en el curso evolutivo de estos pacientes.
Gráfico de respuestas
Comentario

Pregunta típica del MIR sobre la espondilitis anquilosante, en la que has de recordar los datos
típicos. El HLA-B27 es positivo en el 90% de los casos.

Es imprescindible la realización de una radiografía de sacroilíacas. No existe tratamiento específico


que detenga el curso de la enfermedad, y se trata sintomáticamente con AINE.

La afectación periférica puede responder al uso de sulfasalacina o metotrexato que, sin embargo,
son ineficaces en el control de la enfermedad axial. La uveítis típica es la anterior, y es la
manifestación extraarticular más frecuente.

!
!
!
!
En el siguiente esquema te resumimos las características más destacadas de la espondilitis
anquilosante.(R3)
Espondilitis anquilosante. Características

!
!
!
!

357. ¿Cuál de las siguientes situaciones NO produce disminución de la variabilidad en la


monitorización fetal intraparto?

1. 1. Administración de oxitócicos.
2. 2. Meperidina.
3. 3. Anestesia epidural.
4. 4. Tratamiento materno con betabloqueantes.
Gráfico de respuestas
Comentario

Una pregunta de dificultad media sobre el registro cardiotocográfico.

La meperidina, al ser un opiáceo, es un depresor del sistema nervioso central, disminuyendo


entonces la variabilidad. Lo mismo ocurre con los betabloqueantes, que reducen la variabilidad al
oponerse a la acción de las catecolaminas sobre los receptores beta. La anestesia epidural
también puede disminuirla, sobre todo cuando se administra en bolo a través del catéter epidural (a
veces, incluso puede provocar alguna deceleración). Otra de las causas de variabilidad baja es la
hipoxia fetal. En cambio, lo que hacen los oxitócicos es exactamente lo contrario. Al aumentar las
contracciones uterinas, somete al feto a un mayor estrés, con lo que su reacción es un aumento de
la frecuencia cardíaca y de la variabilidad.(R1)

358. En relación a la ictericia del recién nacido, señale la respuesta INCORRECTA:

Su aparición en las primeras 24 horas de vida obliga a investigar una causa patológica de
1. 1.
la ictericia, siendo la isoinmunización anti-A la más frecuente.
En el contexto de una isoinmunización anti-Rh, la coexistencia de incompatibilidad ABO
2. 2.
puede tener un efecto protector.
La presencia de coluria y heces acólicas se encuentra en un cierto porcentaje de las
3. 3.
ictericias fisiológicas.
En algunas ocasiones, se hallan en la leche materna sustancias que inhiben la actividad de
4. 4.
la enzima glucuronil-transferasa, causando ictericias tardías y prolongadas.
Gráfico de respuestas
Comentario

La hiperbilirrubinemia fisiológica es la causa más frecuente de ictericia (2/3 RN sanos), de inicio al


2º-3er día y de duración aproximada de una semana. Los niveles máximos oscilan entre los 12
mg/dL y los 15 mg/dL en pretérminos. La lactancia materna exclusiva puede ser causa de ictericia
prolongada. La ictericia fisiológica es a expensas de bilirrubina indirecta, por lo que los signos
clínicos de colestasis (ictericia verdínica, prurito, acolia y coluria) están ausentes. La ictericia
precoz (primeras 24 horas) es patológica por definición, siendo la causa más frecuente la hemólisis
por isoinmunización anti-A. La isoinmunización anti-Rh, potencialmente muy grave, puede ser más
leve en casos de incompatibilidad ABO concomitante. En estos casos, los anticuerpos contra los
antígenos del sistema ABO, que están preformados, neutralizarían y destruirían rápidamente los
hematíes fetales que pasan a la circulación materna, dificultando así su sensibilización contra
antígenos del sistema Rh.(R3)

359. Acude a urgencias una mujer de 29 años con disnea de inicio brusco y una
hipotensión de 60/40. Días antes, en la pierna izquierda, presentó dolor, tumefacción y
sensación parestésica. Es madre de dos hijos y actualmente toma anticonceptivos orales.

!
!
!
!
La radiografía de tórax es normal. La gasometría muestra un pH de 7,42, con una PaO2
de 48 mmhg y una PaCO2 de 30 mmHg. La gammagrafía no es resolutiva y se opta por
hacer una arteriografía pulmonar, que muestra un defecto de repleción central, por
posible émbolo "en silla de montar". ¿Qué tratamiento realizaría?

1. 1. Heparina cálcica.
2. 2. Embolectomía.
3. 3. Filtro en vena cava.
4. 4. Estreptocinasa i.v.
Gráfico de respuestas
Comentario

El cuadro que presenta este enfermo debe sugerirte un tromboembolismo pulmonar (TEP). Debes
conocer muy bien el tratamiento de esta enfermedad, muy preguntado en el examen MIR.

Este paciente tiene un signo de extrema gravedad, que es la hipotensión y la imagen compatible
con trombo “en silla de montar”. En resumen, un TEP masivo. Si se tratara simplemente de un TEP
“a secas”, sin signos tan críticos, bastaría instaurar tratamiento con heparina. No obstante, ante un
TEP masivo se precisa mayor agresividad. Los trombolíticos son el tratamiento de elección del
tromboembolismo pulmonar masivo (opción 5 correcta). Su uso ha demostrado disminuir la
mortalidad. También se utilizan ante una trombosis venosa profunda iliofemoral de gran entidad,
siempre que haya bajo riesgo de sangrado. Algunos autores recomiendan extender su uso a
pacientes con TEP y datos ecocardiográficos de disfunción ventricular derecha. La trombólisis
debe instaurarse precozmente, pero ten en cuenta que, en esta enfermedad (a diferencia del
infarto agudo de miocardio) puede haber respuesta hasta dos semanas después del TEP. El
objetivo de la trombólisis es acabar cuanto antes con el trombo, acortando la fase de alto
riesgo.(R4)

360. Un paciente portador de una enfermedad pulmonar obstructiva crónica de intensidad


leve sufre un deterioro brusco de su situación clínica con disnea severa tras una
intervención quirúrgica. ¿Cuál sería el hallazgo radiológico más probable?:

1. 1. Patrón alveolar con aumento del índice cardiotorácico.


2. 2. Derrame pleural bilateral.
3. 3. Patrón miliar.
4. 4. Radiología de tórax normal.
Gráfico de respuestas
Comentario

Antiguamente, se consideraba que la radiografía más frecuente en el TEP era una placa de tórax
normal. Sin embargo, hoy día se sabe que, en la mayor parte de los pacientes, existen mínimas
alteraciones radiológicas (elevación de un hemidiafragma, pequeñas atelectasias laminares,
pequeños derrames…). En cualquier caso, alteraciones mínimas o inespecíficas. De las opciones
que nos presentan, la mejor solución sería la 4, pero no olvides lo que hemos dicho, por si alguna
vez lo preguntaran específicamente.(R4)

!
!
!
!
361. ¿Qué puntuación en el test de Silverman tendría un recién nacido que presenta una
maladaptación pulmonar con tiraje intercostal leve, aleteo nasal y boca abierta
constantemente, quejido espiratorio en la auscultación, disociación toracoabdominal y
depresión esternal marcada?:

1. 1. 1-2.
2. 2. 3-4.
3. 3. 5-6.
4. 4. 7-8.
Gráfico de respuestas
Comentario

El test de Silverman cuantifica el distrés respiratorio de los neonatos y los lactantes más pequeños.

Valora cinco puntos, cada uno de los cuales se puntúa entre 0 y 2 puntos (0 puntos indica la
ausencia de clínica; 1 punto, síntomas leves; 2 puntos, severidad).

Los puntos son: aleteo nasal, quejido espiratorio, tiraje subcostal, retracción xifoidea y disociación
toracoabdominal.

Haga las cuentas, y obtendrá la respuesta 4.

!
!
!
!
362. La causa más frecuente de odinofagia en un paciente con infección VIH es:

1. 1. Candidiasis.
2. 2. Infección por CMV.
3. 3. Infección por herpes.
4. 4. Sarcoma de Kaposi.
Gráfico de respuestas
Comentario

En el paciente VIH, la causa más frecuente de odinofagia es la candidiasis esofágica, cuyo


tratamiento es el fluconazol (respuesta correcta 1). En la imagen endoscópica, encontraríamos una
imagen como “en requesón”, con abundantes grumos blanquecinos sobre una mucosa
inflamada.(R1)

363. La vacuna contra el Virus Papiloma Humano debe indicarse en mujeres hasta la edad
de:

1. 1. 8 años.
2. 2. 11 años.
3. 3. 18 años.
4. 4. 35 años
Gráfico de respuestas
Comentario

Segun la cartilla nacional de vacunación México 2013. Se podrán vacunar hasta los 11 años de
edad. Respuesta 2 correcta.

364. Un paciente de 43 años, en tratamiento por leucemia aguda, comienza con fiebre sin
foco aparente. En la exploración: 36 respiraciones/min, 120 latidos/min, TA 90/70,
auscultación cardiopulmonar normal y abdomen normal. Exámenes de laboratorio: Hb 9
g/dl, neutrófilos 120/ml, plaquetas 80.000/ml, creatinina 1 mg/dl. Gasometría arterial
basal: pH: 7,50, pO2: 45, pCO2: 28. Tras recoger cultivos, ¿qué tratamiento iniciaría?

1. 1. Ceftazidima IV, Amikacina IV y Vancomicina IV.


2. 2. Amoxicilina/Clavulánico y ciprofloxacino, por vía oral.
3. 3. Ceftriaxona IV y Vancomicina IV.
4. 4. Amoxicilina/Clavulánico IV.
Gráfico de respuestas
Comentario

Del enunciado, lo único que nos interesa es que nos encontramos ante un paciente con fiebre sin
foco en tratamiento con quimioterapia (leucemia aguda). Después, el hemograma nos confirma que
se trata de un neutropénico. En este tipo de pacientes, hemos de pensar en BGN, S.aureus y
hongos. Así pues, como primera elección el tratamiento debería cubrir estos microorganismos
(respuesta 1 correcta). Si pasados unos días el cuadro no mejora, se debe añadir al tratamiento
anfotericina B, para cubrir la posibilidad del Aspergillus.(R1)

365. Mujer de 36 años, tercigesta, de 35 semanas de amenorrea. En la exploración física


llama la atención una altura uterina menor de la que corresponde. En el USG se ve
estancamiento del crecimiento de los diámetros abdominales respecto a la eco anterior,
con diámetro biparietal

!
!
!
!
y longitud femoral normales. El eco-doppler fetal muestra aumento de resistencia al flujo
en arterias placentarias. El registro cardiotocográfico basal es no reactivo. ¿Cuál sería la
siguiente actuación para valorar el estado fetal?:

1. 1. Amnioscopía.
2. 2. Registro cardiotocográfico estresante.
3. 3. Inducción del parto.
4. 4. Amniocentesis tardía.
Gráfico de respuestas
Comentario

Caso clínico sobre la evaluación de una gestación en el tercer trimestre que nos presenta a una
gestante de 35 semanas con un CIR tipo II o asimétrico ya que los parámetros óseos de medida
(LF y DBP) son normales y los diámetros abdominales son menores de lo que correspondería. El
USG-doppler demuestra una afectación de los flujos y la monitorización fetal no estresante nos
muestra un RAF negativo. Recuerden que ante un RAF negativo debemos realizar una prueba de
Pose o monitorización fetal estresante provocando contracciones bien con la administración de
oxitocina iv. o estimulando el pezón. También que sólo podemos realizar amnioscopías en
gestaciones a término con lo que descartamos la opción 1. La amniocentesis tardía está casi en
desuso gracias al uso de corticoides, y en este caso clínico la gestación es de más de 34 semanas
con lo que tendrá madurez pulmonar fetal. La inducción del parto lo decidiríamos en función de lo
obtenido mediante la prueba de Pose.(R2)

366. Indique el tratamiento de elección para una paciente de 58 años que acude por
síndrome miccional irritativo de varios meses de evolución. Refiere que su MAP le ha
realizado múltiples urocultivos que han resultado ser negativos. Es fumadora. DM tipo 2
en tratamiento dietético e HTA en tratamiento con enalapril. En el sedimento de orina se
objetiva microhematuria. Las citologías de orina indican sospecha de carcinoma urotelial
y las biopsias vesicales informan de ca in situ difuso:

1. 1. Instilaciones con epirrubicina.


2. 2. Inmunoterapia endovesical con BCG.
3. 3. Resección transuretral.
4. 4. Actitud expectante.
Gráfico de respuestas
Comentario
El tratamiento de elección del carcinoma in situ vesical es la realización de instilaciones vesicales
con BCG.(R2)

367. ¿Cuál de las siguientes NO es indicación de extracción fetal mediante cesárea en la


gestante con VIH?:

1. 1. Deterioro inmunológico (CD4<200).


2. 2. Carga viral detectable.
3. 3. No tratamiento antirretroviral previo al embarazo.
4. 4. Prematuridad.
Gráfico de respuestas
Comentario

!
!
!
!
Lo que contraindica parto vaginal es el no tratamiento antirretroviral durante el embarazo, no el
previo al embarazo. El tratamiento estándar consiste en la asociación de al menos tres
antirretrovirales como la zidovudina, lamivudina y nevirapina.(R3)

368. Indique la relación FALSA entre el agente patógeno y su medio de cultivo específico:

1. 1. Borrelia burgdorferi: BSK.


2. 2. Corynebacterium diphtheriae: Medio de Tinsdale.
3. 3. Leishmania donovani: Medio de Novy-McNeal-Nicolle.
4. 4. Neisseria gonorrhoeae: Medio de Fletcher.
Gráfico de respuestas
Comentario

El cultivo de gonococos a partir de secreciones genitales o respiratorias se realiza en medio de


Thayer-Martin; el medio de Fletcher se utiliza para el cultivo de Leptospira. Legionella no crece en
los medios de cultivo habituales, crece en medios con L-cisteína como el BCYE (Buffered-
Charcoal-Yeast-Extract agar). La Borrelia burgdorferi crece en medio BSK (Barbour-Stoenner-
Kelly). El medio selectivo para Corynebacterium diphtheriae es en medio con telurito y cisteína,
como el agar de Tinsdale. El diagnóstico de Leishmaniasis se realiza por visualización del parásito
- tinción de Giemsa- o cultivo en medio específico como el de Novy-MacNeal-Nicolle.(R4)

369. Está atendiendo a un niño de 2 meses que presenta desde hace tres días un catarro
de vías altas, que hoy asocia tos, rechazo parcial del alimento y febrícula. En la
exploración encuentra una faringe hiperémica, con moco claro en cavum, sibilancias y
subcrepitantes escasos de predominio en bases y ausencia de signos de dificultad
respiratoria. El resto es normal. Los padres le preguntan acerca de la evolución del
cuadro y de sus posibles complicaciones; una de las respuestas que da es FALSA,
señálela:

Si el niño comenzase con fiebre mayor de 38ºC, estamos obligados a descartar una
1. 1.
neumonía, ya que la bronquiolitis cursa sin fiebre, o como mucho con febrícula.
En los niños menores de un año, si la bronquiolitis está producida por VRS, existe cierta
2. 2.
tendencia a complicarse con neumonía.
3. 3. Los síntomas de tipo catarral pueden persistir durante 4 semanas.
Es posible que persista un cierto grado de hiperreactividad bronquial durante los dos
4. 4.
primeros años de vida asociado a este episodio de bronquiolitis.
Gráfico de respuestas
Comentario

En esta pregunta nos describen un caso típico de bronquiolitis, que es importante saber reconocer
y nos preguntan sobre su evolución. La dificultad respiratoria tiene una intensidad creciente
alcanzando su máximo en 24- 48h para luego mejorar gradualmente.

La enfermedad tiene una duración aproximada de 5-7 días aunque los síntomas catarrales pueden
persistir mas tiempo. De las complicaciones, lo más importante es recordar que una proporción
significativa de lactantes con bronquiolitis presenta hiperreactividad de las vías aéreas
posteriormente. Habitualmente cursa sin fiebre o con febrícula por lo que, aunque las
complicaciones bacterianas son poco frecuentes, ante una fiebre alta mantenida debemos
descartar neumonía.(R2)

370. El tratamiento inicial de la obstrucción intestinal de cualquier causa es:

!
!
!
!
1. 1. Laparotomía exploradora.
2. 2. Descompresión endoscópica.
3. 3. Reposición hidroelectrolítica y aspiración gástrica.
4. 4. Analgesia.
Gráfico de respuestas
Comentario

Ante una obstrucción intestinal, el primer paso terapéutico siempre va a ser una medida no
invasiva en forma de tratamiento médico. Se procererá a reposición hidroelectrlítica y aspiración
gástrica junto con la realización de un TC urgente para conocer la etiologia del cuadro. El 90% de
las oclusiones se resuelven con tratamiento médico. Solo en aquellos casos en los que haya
signos de alarma como leucocitosis, fiebre o edema en pared se procederá a cirugía.(R3)

371. Las pacientes con síndrome de ovarios poliquísticos presentan un aumento de


riesgo a largo plazo para los siguientes procesos, EXCEPTO:

1. 1. Diabetes mellitus tipo 2.


2. 2. Hipertensión arterial.
3. 3. Osteopenia.
4. 4. Coronariopatía.
Gráfico de respuestas
Comentario

Pregunta de dificultad media-alta acerca del síndrome de ovarios poliquísticos.

El SOP es una afectación muy frecuente, de etiología desconocida, si bien se piensa que el hecho
principal es la secreción de GnRH en pulsos rápidos que produce una elevación de la LH. Esto
ocasiona una hiperplasia de la teca con la correspondiente sobreproducción de andrógenos, y esto
puede ser la causa de la obesidad, el hirsutismo y la anovulación. Por otro lado, en el SOP hay una
resistencia a la acción de la insulina, que en condiciones normales estimularía la producción de
estrógenos por las células de la granulosa a partir de los andrógenos producidos en las células de
la teca, contribuyendo al hiperinsulinismo. Esta resistencia a la insulina, junto al
hiperandrogenismo, producen una propensión a la dislipemia, aterosclerosis, HTA y cardiopatía
isquémica. Por el contrario, el exceso de andrógenos y la obesidad hace que haya una mayor
producción periférica de estrógenos que en un principio protegen de la osteoporosis que acompaña
a otros trastornos anovulatorios.

Las mujeres afectadas suelen quejarse de trastornos menstruales, infertilidad o hirsutismo, que
junto con la obesidad forman el cuadro clínico típico y constituyen la base para el diagnóstico.

En cuanto al tratamiento del SOP, se utilizan los ACO para las alteraciones menstruales y los
antiandrógenos.Para reducir la resistencia a la insulina es primordial la pérdida de peso que puede
ir acompañada del tratamiento con metformina. El citrato de clomifeno es útil para inducir la
ovulación.(R3)

372. Ante una paciente amenorreica con test de gestágenos positivo, ¿cuál de las
siguientes etiologías de la amenorrea es la más factible?:

1. 1. Anovulación crónica.
2. 2. Síndrome de Asherman.
3. 3. Disgenesia gonadal.

!
!
!
!
4. 4. Fallo ovárico precoz.
Gráfico de respuestas
Comentario

Ante una amenorrea secundaria, lo primero que debe solicitarse es un test de embarazo. En caso
de ser positivo, ya tendríamos cuál es la causa de su amenorrea. En caso de ser negativo, se debe
practicar una determinación de TSH y PRL. Si son normales, el siguiente paso consiste en
administrar progesterona.

El test de la progesterona consiste en administrar una pequeña cantidad esta hormona, durante 5
días. Si la paciente ha producido con normalidad la primera fase del ciclo (proliferativa), pero no
llega a ovular y por tanto no llega a producir progesterona, al administrarle esta progesterona que
le faltaba, tendrá la regla y concluimos que la causa era la anovulación (respuesta 1 correcta)

En caso de no conseguirse la regla con la administración de progesterona, el paso siguiente es


combinar estrógenos y progestágenos para continuar el estudio.(R1)

373. Un hombre de 52 años, fumador e hipertenso, acude a urgencias con un infarto


agudo de miocardio de localización inferior. La fase aguda transcurre sin complicaciones.
El siguiente paso más razonable a llevar a cabo sería:

Tratamiento con aspirina y antianginosos, control de los factores de riesgo y hacer la


1. 1.
próxima revisión a los 6-12 meses.
Realizar una ergometría a los 7-14 días del infarto y según su resultado hacer o no una
2. 2.
coronariografía.
Tratar al paciente con antiarrítmicos (por ejemplo, procainamida) durante al menos tres
3. 3.
meses, para prevenir la aparición de arritmias mortales.
Sólo control de los factores de riesgo y, si vuelve a tener síntomas, realizar una
4. 4.
coronariografía.
Gráfico de respuestas
Comentario

Después de un infarto de miocardio, se debe realizar un ecocardiograma para valorar la función


ventricular. Si tiene datos de mal pronóstico, como disfunción ventricular, angor post-IAM o
arritmias ventriculares secundarias, se realiza directamente una coronariografía, para conocer la
anatomía coronaria y evaluar la posibilidad de angioplastia o cirugía de by-pass.

Si, como en este caso, no existen datos de mal pronóstico, se realiza una prueba de detección de
isquemia, como la ergometría, a partir de los 5-7 días del infarto (respuesta 3 correcta). Observe
que nos dicen que “la fase aguda transcurre sin complicaciones”, en este dato está la clave de la
pregunta.

En los casos en los que se utilizó angioplastia primaria con revascularización como técnica de
reperfusión, no suele ser necesaria la realización de la ergometría.(R2)

374. Una de las entidades que se enumeran a continuación NO está presente en el


síndrome de Gardner. Señale cuál es:

1. 1. Fibromas.
2. 2. Osteomas.
3. 3. Tumores desmoides.

!
!
!
!
4. 4. Tumores malignos del sistema nervioso central.
Gráfico de respuestas
Comentario

La forma más rentable de enfocar el estudio de estas enfermedades es conocer el dato típico. La
respuesta falsa es la 4, ya que los tumores cerebrales aparecen en el síndrome de Turcot, no en el
de Gardner.(R4)

375. Gestante de 39 semanas, nulípara primigesta, con inicio de actividad de parto hace
26 horas, rotura espontánea de bolsas, con líquido amniótico claro, que tras la realización
de anestesia epidural ha alcanzado dilatación completa y ha comenzado con el expulsivo.
El feto está en presentación cefálica. Durante todo el proceso, el registro
cardiotocográfico ha mostrado FCF de 140 lpm, variabilidad en torno a 15 lpm, ascensos
en número de 5 cada 20 minutos y deceleraciones tipo DIP I. La mujer comienza a
presentar señales de agotamiento, por lo que se decide utilizar el vacuoextractor. La
acción fundamental del vacuoextractor es:

1. 1. Estimulación de la dinámica uterina.


2. 2. Extractora.
3. 3. Rotación de la cabeza.
4. 4. Deflexión de la cabeza.
Gráfico de respuestas
Comentario

El vacuoextractor o ventosa es uno de los instrumentos que podemos utilizar para abreviar el
expulsivo. El mecanismo de acción consiste en aplicar una campana que suele ser de silicona
sobre la cabeza fetal y hacer un efecto de vacío. De esta forma, se podrá ejercer tracción
siguiendo el eje del canal del parto.

Los requisitos para poder aplicar la ventosa son:

- Dilatación completa

- Bolsa rota

- Gestación de >37 semanas

- Presentación fetal rotada o casi rotada a partir del II plano de Hodge

De las opciones que plantean debemos quedarnos con la respuesta 2, ya que sólo es útil en el
período expulsivo del parto (a partir de la dilatación completa). La diferencia fundamental de la
ventosa con respecto al fórceps es que no es un instrumento rotador (aparte de que el fórceps no
se emplearía en un II plano de Hodge).(R2)

376. El agente etiológico del condiloma acuminado es:

1. 1. Chlamydia tracohmatis.
2. 2. VPH.
3. 3. Hemophilus ducreyi.
4. 4. Herpes virus II.

!
!
!
!
Gráfico de respuestas
Comentario

Pregunta muy fácil sobre las diferentes manifestaciones clínicas del VPH, que no debe fallar. Los
subtipos deVPH que producen el condiloma acuminado son los no cancerígenos, sobretodo el tipo
6 y 11.(R2)

377. A previously asymptomatic 70 year-old woman comes to the emergency department


because of dizziness, vomiting without nausea and bidirectional nystagmus. She is
unable to sit up in bed. Her medical history is significant for hypertension with poor
control. Which of the following are the most likely diagnosis and appropriate treatment in
this case?

1. 1. Cerebellar hemorrhage. Request a CT scan and refer to neurosurgery.


2. 2. Vestibular neuronitis; vestibular sedatives for 10 days.
3. 3. Benign postural paroxysmal vertigo; liberatory maneuvers.
4. 4. Ménière disease; vestibular sedatives. Vestibular neurectomy within the next 72 hours.
Gráfico de respuestas
Comentario
Lo más probable es que la paciente tenga una hemorragia cerebelosa, que es una urgencia
neuroquirúrgica. A veces, empiezan igual que un vértigo periférico que no responde nada al
tratamiento en 24-48h.(R1)

378. Hombre de 62 años de edad, diagnosticado desde hace 2 años de hemorroides,


tratado con pomadas y modificaciones alimentarias. Desde hace 2 meses se le prolapsan
y tiene que reducirlas manualmente. ¿Qué grado de hemorroides presenta ahora?

1. 1. Primer grado.
2. 2. Segundo grado.
3. 3. Tercer grado.
4. 4. Quinto grado.
Gráfico de respuestas
Comentario

Pregunta poco significativa y bastante difícil, puesto que los grados de las hemorroides nunca
habían sido preguntados en el ENARM.Éstos son:

•! Grado I: permanecen en recto, sin sobrepasar el ano.


•! Grado II: prolapso a través del ano cuando el paciente puja, pero se reducen
espontáneamente.
•! Grado III: prolapso por el ano cuando el paciente puja. La reducción requiere ayuda
manual.
•! Grado IV: prolapso persistente.(R3)

!
!
!
!
379. La asociación de un cuadro de púrpura trombocitopenica y anemia hemolítica se
conoce como:

1. 1. Síndrome de Henoch.
2. 2. Síndrome talasemico.
3. 3. Síndrome de Evans.
4. 4. Enfermedad de Werlhaf.
Gráfico de respuestas
Comentario

El síndrome de Evans debe saberlo se describe como la asociación de trombocitoenia y anemia de


origen autoimmune.

La púrpura de SH es una vasculitis que cursa con plaquetas altas. LA talasemia suele cursar solo
con anemia.(R3)

380. ¿Cuál es el tratamiento de elección de la vaginosis bacteriana?:

1. 1. Metronidazol.
2. 2. Clotrimazol.
3. 3. Ketoconazol.
4. 4. Claritromicina.
Gráfico de respuestas
Comentario

La vaginosis bacteriana es más frecuente durante el periodo fértil y aunque su etiología es


polimicrobiana se ha visto que uno de los agentes infecciosos más importante es Gardnerella
vaginalis; el tratamiento se basa principalmente en los fármacos como: metronidazol y
clindamicina.

Con esto, la respuesta correcta sería la 1, metronidazol.

381. El hemorragia disfuncional que persiste a pesar de curetaje y de una terapia


hormonal con progestágenos es mejor tratada con:

1. 1. Histerectomía.
2. 2. Dosis adecuada de estrógenos y progesterona en segunda fase.
3. 3. Cualquiera de las anteriores.
4. 4. Ninguna de las anteriores.
Gráfico de respuestas
Comentario

El tratamiento de la hemorragia disfuncional se inicia con terapia hormonal, en caso de que no


mejore podría realizarse curetaje, posteriormente podrías realizar histeroscopía o ablación
endometrial. Vemos que una de las opciones es dosis adecuada de estrógenos y progesterona que
es prudente intentar, antes de realizar algo tan definitivo como histerectomía.(R2)

382. ¿Cuál de los siguientes fármacos sería de primera elección para tratar los episodios
de agresión provocados por LSD?

!
!
!
!
1. 1. Clordiazepóxido.
2. 2. Carbamacepina.
3. 3. Propranolol.
4. 4. Clorpromacina.
Gráfico de respuestas
Comentario

El LSD (dietilamina del ácido lisérgico) es un alucinógeno semisintético que se obtiene del
cornezuelo del centeno. Existe en forma de polvo, líquido o comprimidos, carentes de olor, color o
sabor. Su uso produce alteraciones en los circuitos serotoninérgicos, con alucinaciones cromáticas,
cronológicas, morfológicas y espaciales. Las percepciones se confunden, hasta el punto de que,
los usuarios de esta droga, a veces dicen oír colores y ver sonidos. En ocasiones, el paciente sufre
inquietud y puede llegar a tener alucinaciones consistentes en visiones terroríficas, en cuyo caso
podrían producirse episodios de violencia y agitación.El tratamiento suele consistir en un
neuroléptico con perfil sedante, como la clorpromacina (respuesta 4).

383. ¿Cuál de las siguientes alteraciones en un paciente con síndrome de Cushing


orientaría más su diagnóstico a un tumor extrasuprarrenal productor de ACTH ectópico?:

1. 1. Existencia de hipotasemia e hipertensión.


Niveles plasmáticos de cortisol altos tras la administración nocturna de una dosis de
2. 2.
dexametasona.
3. 3. Glándulas suprarrenales normales en la TC abdominal.
4. 4. Instauración brusca de un cuadro típico de síndrome de Cushing.
Gráfico de respuestas
Comentario
Pregunta en relación con el Cushing producido por secreción de ACTH ectópica. En la mayoría de
los paciente con Cushing se aprecian alteraciones metabólicas, como la hipopotasemia,
hiperglucemia y alcalosis metabólica y signos como la obesidad y la HTA, luego no son útiles en el
diagnóstico diferencial. La no supresión con dosis elevadas de DXM se produce tanto en los
macroadenomas hipofisarios como en los ectópicos, luego tampoco resulta útil. La existencia de
unas glándulas suprarrenales con morfología normal no nos orienta hacia la existencia de un tumor
ectópico. Sin embargo, el curso clínico sí que puede ser útil, ya que en los tumores ectópicos,
especialmente en los agresivos, la evolución y el desarrollo de las alteraciones metabólicas es más
rápida en relación a los tumores hipofisarios y a la patología suprarrenal.(R4)

384. Señale la FALSA con respecto a la arteritis de Takayasu:

1. 1. Es más frecuente en mujeres.


2. 2. El hallazgo de soplos carotídeos y ausencia de pulsos radiales apoya el diagnóstico.
3. 3. Es necesaria la confirmación histológica para el diagnóstico.
4. 4. La angiografía de la aorta y sus ramas es una prueba muy útil para el diagnóstico.
Gráfico de respuestas
Comentario
La arteritis de Takayasu en una rara enfermedad en nuestro medio, que afecta fundamentalmente
a mujeres jóvenes y cursa con una vasculitis de grandes vasos, presentando predilección por el
cayado aórtico y sus ramas; así, el vaso más frecuentemente afectado son las subclavias. En esta
enfermedad, dado el calibre de los vasos afectados, el diagnóstico se establece por arteriografía
que debe ser extensa, ya que puede existir afectación a varios niveles, y no por biopsia como
ocurre en las vasculitis de pequeño o mediano vaso. El diagnóstico se sospecha ante una paciente
joven que presenta eventos isquémicos o de insuficiencia vascular en alguna localización, y los

!
!
!
!
datos que más apoyan el diagnóstico son la existencia de asimetrías en los pulsos o ausencias, así
como la existencia de soplos. Además, dado que es una vasculitis, las pacientes pueden presentar
síntomas generales con aparición de fiebre, astenia, y elevación de reactantes de fase aguda. El
tratamiento consiste en la reparación quirúrgica de las lesiones establecidas y tratamiento médico
con corticoides para evitar la progresión de la actividad de la enfermedad.(R3)

385. Niño de 3 años, previamente sano, en el cual al examen físico se detecta fiebre,
cefalea y fotofobia, cual sería la etiología más probable:

1. 1. Neuritis óptica.
2. 2. Hemorragia subaracnoidea.
3. 3. Meningitis.
4. 4. Rabia.
Gráfico de respuestas
Comentario

Pregunta muy fácil, que no pude permitirse fallar.

En un niño con fiebre, cefalea y fotofobia, la etiología MAS PROBABLE es meningitis, respuesta 3
correcta. Podría pensar en rabia, pero no es frecuente. La opción 1 y 2 serían más frecuentes en
adultos.(R3)

386. ¿Cuál de los siguientes NO es factor de riesgo del cáncer de mama?:

1. 1. Herencia.
2. 2. Hiperplasia ductal atípica.
3. 3. Mastopatía fibroquística.
4. 4. Obesidad.
Gráfico de respuestas
Comentario

Tema fundamental para el ENARM.

Pregunta fácil sobre uno de los temas importantes del cáncer de mama como son los factores de
riesgo. La edad es el factor de riesgo más importante, aumentando el riesgo rápidamente durante
los años de actividad sexual. La herencia o carácter genético es otro factor de riesgo, pudiendo
aparecer el cáncer familiar y el cáncer hereditario (debido a mutaciones en un gen supresor).
También la obesidad es un factor de riesgo, especialmente durante la menopausia. Las lesiones
benignas no proliferativas (como puede ser la mastopatía fibroquística), no se asocian a un mayor
riesgo de cáncer de mama, con lo que debemos escoger la opción 3. La hiplerplasia ductal atípica
sí está asociada con un mayor riesgo a desarrollar un cáncer de mama.(R3)

387. Las vacunas del papilomavirus frente al carcinoma cervical han demostrado su
eficacia mediante:

1. 1. La demostración de que se producen anticuerpos tras la vacunación.


La demostración de protección frente a la aparición de lesiones preneoplásicas (CIN 2/3)
2. 2.
asociadas a los virus incluidos en la vacuna.
3. 3. La demostración de la remisión del cáncer cervical en las mujeres vacunadas.
4. 4. Estudios clínicos no controlados.

!
!
!
!
Gráfico de respuestas
Comentario

Una pregunta muy importante sobre un tema de bastante actualidad: la vacuna del virus del
papiloma humano. Vamos a exponer algunas ideas que merece la pena conocer:

Se ha demostrado la efectividad de la vacuna para prevenir los cambios citológicos e histológicos


inducidos por los virus contenidos en ella (respuesta 2 correcta).

•! Se trata de una vacuna con un excelente perfil de seguridad.


•! Su uso se recomienda en mujeres.
•! La duración de la protección es desconocida, pero se sabe que a los cinco años sigue
habiendo anticuerpos protectores.
•! Carece de eficacia en personas que ya padecen la infección.

(R2)

388. Which of these increases the rate of malformations?

1. 1. Preeclampsia.
2. 2. Folinic acid.
3. 3. Rubella vaccine.
4. 4. Advanced maternal age.
Gráfico de respuestas
Comentario

Pregunta sobre las malformaciones fetales que no parece nada fácil. De las cuatro opciones,
podemos descartar las dos primeras casi de entrada. La preeclampsia genera mayor riesgo de
CIR, muerte fetal, DPPNI... pero no de malformaciones. El ácido folínico es utilizado en los déficit
de ácido fólico en grandes cantidades (categoría A de la FDA). La duda estaría entre la 3 y la 4. La
vacuna de la rubeola está contraindicada en el embarazo ya que es creada a base de virus
atenuados y será evitada; es categoría C de la FDA (efectos adversos fetales en animales, pero no
demostrado en humanos). Lo que sí aumenta la tasa de malformaciones sería la infección
congénita de rubeola. La respuesta 4 es la que debemos escoger (edad materna), ya que es la
única que sí se ha demostrado tener mayor riesgo de malformaciones y cromosomopatías, de ahí,
que ser mayor de 35 años sea indicación de una prueba de diagnóstico prenatal como puede ser la
amniocentesis.(R4)

389. Which of the following statements about febrile seizures is not correct?

Basic physical examination and full anamnesis must be performed in order to rule out
1. 1.
infectious causes such as bacterial meningitis or sepsis.
It is very important to exclude bacterial meningitis with physical examination or additional
2. 2.
tests and give appropriate treatment for the first episode of febrile seizures.
It is unfrequently seen in patients under 9 months old and/or over 5 years old of age.
3. 3.
Febrile seizures commonly occur between 14 and 18 months of age.
Performing an EEG is not indicated because it would probably show no abnormal findings
4. 4.
and would not change the medical management.
Gráfico de respuestas
Comentario

!
!
!
!
Lo primero que debemos hacer ante una crisis febril es descender la fiebre lentamente y controlar
el cuadro subyacente. Una vez logrado eso y si la crisis es típica, no debemos hacer nada más que
estar atentos a posibles nuevos cuadros de fiebre elevada para aplicar rápidamente un tratamiento
antitérmico. Si las crisis son frecuentes se puede aplicar como profilaxis en caso de cuadro febril
diacepam rectal hasta la resolución del cuadro.(R2)

390. ¿Cuál es el tumor de ovario más frecuente en la mujer joven?

1. 1. Cistoadenocarcinoma.
2. 2. Ginadroblastoma.
3. 3. Quiste dermoide.
4. 4. Tumor del seno endodérmico.
Gráfico de respuestas
Comentario

El tumor de ovario más frecuente en mujeres jóvenes es el teratoma maduro, también llamado
quiste dermoide. Es un tumor benigno y fácilmente reconocible en la ecografía transvaginal.
Pertenece al grupo de los tumores germinales. Y recuerde que en general son los epiteliales los
que se presentan con mayor frecuencia.(R3)

391. Señale la respuesta correcta en relación con la fiebre mediterránea familiar:

Está ligada a una mutación del gen que codifica el receptor tipo 1 de factor de necrosis
1. 1.
tumoral-alfa.
Junto a los episodios de fiebre recurrente y dolor abdominal, la mayoría de los pacientes
2. 2.
presentan intensas mialgias migratorias tanto asociadas a la fiebre como en reposo
Las manifestaciones oculares (edema periorbitario y uveítis) se presentan frecuentemente
3. 3.
en el subgrupo de pacientes menores de 20 años.
Las manifestaciones cutáneas se suelen presentar en forma de eritema erisipeloide
4. 4.
localizado en miembros inferiores.
Gráfico de respuestas
Comentario

Pregunta de dificultad media-alta. La respuesta correcta es la 4.

La fiebre mediterránea familiar (FMF) es una enfermedad hereditaria de etiología desconocida


caracterizada por episodios recurrentes de fiebre, peritonitis y/o pleuritis. También presenta artritis,
lesiones cutáneas y amiloidosis, en algunos pacientes, siendo ésta la complicación más grave.

•! Las lesiones cutáneas suelen consistir en un eritema erisipeloide doloroso, localizado en


extremidades inferiores, sobre todo en el maléolo medial o dorso de los pies (respuesta 4
correcta).
•! Como hemos dicho anteriormente, es una enfermedad genética con un patrón autosómico
recesivo. Está ligada a un defecto en el cromosoma 16 que codifica una proteína de 781
aminoácidos denominado pyrin que se expresa en las células de estirpe mieloide
(respuesta 1 falsa).
•! La clínica se caracteriza por fiebre, inflamación de serosas, junto a lesiones cutáneas y
artritis (no mialgias como en el caso de la respuesta 2). Esta afectación articular
frecuentemente es aguda, si bien ocasionalmente crónica y afecta generalmente a
agrandes articulaciones.

!
!
!
!
•! Los síntomas de la FMF se suelen iniciar entre los cinco y los 15 años de edad, pero no se
han constatado manifestaciones oculares (edema periorbitario y uveítis) como nos relatan
en la respuesta 3 (respuesta 3 falsa).

En el tratamiento de esta enfermedad, se sabe que el empleo de la colchicina a largo plazo no sólo
reduce la aparición de episodios agudos, sino también el desarrollo de amiloidosis.(R4)

392. La corea de Huntington presenta uno de los caracteres que se enumeran a


continuación:

1. 1. Es dominante autosómica.
2. 2. Está ligada al sexo.
3. 3. No está ligada al sexo.
4. 4. No es hereditaria.
Gráfico de respuestas
Comentario
Pregunta de escasa dificultad sobre un tema ampliamente preguntado en el MIR. El corea de
Hungtington es la causa más común de corea hereditario. Tiene una herencia autosómica
dominante (respuesta 1 correcta) localizada en el cromosoma 4. La enfermedad comienza a
manifestarse en la 4ª o 5ª décadas de la vida, con antecedentes familiares en practicamente todos
los casos. Está producida por una expansión de tripletes ( CAG ), lo que hace que sea común el
fenómeno de anticipación ( aparición de la enfermedad a edades más precoces en sucesivas
generaciones) debido al aumento del número de tripletes en el cromosoma 4. El diagnóstico
habitualmente se hace por la historia clínica y por los antecedentes familiares y normalmente no
hay que recurrir al estudio genético, pero la aparición en el propio individuo (no en los familiares)
de más de 40 tripletes en el cromosoma 4 nos da per se el diagnóstico de la enfermedad. NO
existe un tratamiento patogénico eficaz, pero se usan deplectores presinápticos (reserpina) o
postsinápticos (neurolépticos) de dopamina.(R1)

393. Los lactantes con rubéola congénita pueden ser considerados como probables
fuentes de contagio hasta:

1. 1. El mes de edad.
2. 2. Los tres meses de edad.
3. 3. Los seis meses de edad.
4. 4. El año de edad.
Gráfico de respuestas
Comentario

Pregunta totalmente memorística. Además se debe saber de la rubeola que es más grave y más
frecuete cuando la madre se infecta durante el primer trimestre pudiendo causar la tráda de Gregg
(hipoacúsia neurosensorial, cardiopatia, anomalias oculares y alteraciones oseas).(R4)

394. La apendicitis aguda es causada por:

1. 1. Enfermedad celiaca.
2. 2. Hipertrofia de las amígdalas de Roux.
3. 3. Malabsorcion intestinal.
4. 4. Hipotrofia de las placas de Peyer.
Gráfico de respuestas

!
!
!
!
Comentario

Pregunta sencilla sobre apendicitis aguda.

Existen distintas causas. La más frecuente es la hipertrofia de las amígdalas de Roux, en segundo
lugar el fecalito, tercer lugar cuerpos extraños y por último las neoplasias. Pregunta 2 correcta.

395. ¿Qué actitud tomaría ante un varón de 65 años, asintomático y con buen estado
general, que presenta en la placa de tórax residuos fibrosos sugestivos de tuberculosis
antigua (sin presentar signos de tuberculosis activa) y que nunca ha recibido tratamiento
antituberculoso?

1. 1. Abstención terapéutica.
2. 2. Observación y revisión a los 6-12 meses.
3. 3. Quimioprofilaxis con isoniacida 300 mg/día 12 meses.
4. 4. Tratamiento con isoniacida 300 mg/día y rifampicina 600 mg/día durante 9-12 meses.
Gráfico de respuestas
Comentario

La respuesta correcta a esta pregunta es la NÚMERO 3 (Quimioprofilaxis con INH 300 mg al día
durante 12 meses).

Nos presentan un varón de 65 años SIN clínica con una placa sugestiva de infección tuberculosa
("residuos fibrosos sugestivos de tuberculosis antigua"). Por ello en este caso la respuesta correcta
no será tratar al paciente como si estuviese enfermo (es decir, como si padeciese una tuberculosis
antigua; respuesta 4 y 5 incorrectas) sino que habrá que realizar quimioprofilaxis.

Recordad que el tratamiento profiláctico son 6 meses de 300 mg / día de INH excepto en (a) casos
de niños, conversores y convivientes [9 meses] y (b) VIH positivo, inmunodeprimidos y pacientes
con lesiones fibróticas pulmonares [12 meses].(R3)

396. Se debe descartar la existencia de feocromocitoma asociado en todas las siguientes


enfermedades, EXCEPTO en una:

1. 1. Enfermedad de von Hippel-Lindau.


2. 2. Neurofibromatosis tipo 1.
3. 3. Carcinoma papilar de tiroides.
4. 4. Carcinoma medular de tiroides.
Gráfico de respuestas
Comentario

El carcinoma medular produce típicamente calcitonina, cuya elevación puede orientar al


diagnóstico, aparte de que sería de utilidad en el seguimiento postoperatorio. Recuerde que este
tumor puede aparece en el contexto de un MEN 2, que es lo que habría que sospechar en este
caso, dada la asociación a feocromocitoma. La asociación del feocromocitoma es, por lo tanto, con
el carcinoma medular, y no con el papilar.(R3)

397. Paciente de 70 años con diagnóstico histológico de cáncer de endometrio grado 2.


La resonancia magnética nos informa de una afectación de mas del 50% de la pared
miometrial y posible afectación cervical. Además describen ganglios pélvicos

!
!
!
!
aumentados de tamaño con alta probabilidad de invasión carcinomatosa. El resto de la
pelvis no muestra afectación tumoral. ¿En qué estadio tumoral se encuentra la paciente?:

1. 1. IB.
2. 2. IIIB.
3. 3. IIIC1.
4. 4. IIIC2.
Gráfico de respuestas
Comentario

Debe repasar la clasificación del cáncer de endometrio. En este caso nos encontramos ante la
afectación de mas del 50% del miometrio, que sería un estadio IB. Por otra parte nos informan de
la afectación del cervix lo que asciende a un estadio II, al que si sumamos además que existe
afectación ganglionar a nivel de la pelvis obtenemos el estadio final: IIIC1. Con respecto a que
cadena ganglionar está afecta les puede ser utilidad la asociación: C1 son cercanos al tumor
(ganglios pélvicos) y C2 alejados al tumor (ganglios paraaórticos).(R3)

398. En relación con la transmisión vertical del virus de la inmunodeficiencia humana


(VIH) a partir de una madre afecta del síndrome de inmunodeficiencia adquirida (SIDA),
¿cuál es la repuesta FALSA?

1. 1. Puede existir transmisión vertical antes, durante o después del parto.


2. 2. El porcentaje más alto de niños infectados por VIH adquiere el virus durante el parto.
La terapia prenatal, intraparto y posnatal con zidovudina ha disminuido la tasa de
3. 3.
transmisión en el mundo desarrollado hasta menos del 8%.
4. 4. No se ha detectado virus VIH en la leche de madres infectadas.
Gráfico de respuestas
Comentario

Pregunta muy importante, de un tema quese presenta con mayor frecuencia. Con respecto de las
vías de transmisión vertical del VIH neonatal, recuerde que la tasa de transmisión oscila entre 25-
52% y que esta transmisión puede acontecer en tres momentos: vía transplacentaria (30-40% de
los casos), vía de transmisión durante el parto: vía más importante de transmisión maternofetal (60-
70%) y vía postnatal.

Dentro de las variables que facilitan la transmisión destaca la rotura de membranas superior a
cuatro horas; otras son la prematuridad y el bajo peso al nacimiento.

Vía postnatal: existe documentación de transmisión mediante la leche materna (es una de las
pocas contraindicaciones de la lactancia materna, salvo en los países en vías de desarrollo).(R4)

399. ¿Cuál de las siguientes afirmaciones es FALSA?:

La cirugía laparoscópica tiene múltiples ventajas sobre la cirugía convencional en


1. 1.
determinados procedimientos.
2. 2. La exploración laparoscópica, en pacientes inestables es ideal por ser menos traumática.
3. 3. Se ha demostrado su utilidad en la apendicitis aguda.
4. 4. Disminuye el tiempo de recuperación de los pacientes.
Gráfico de respuestas
Comentario

!
!
!
!
Este tipo de preguntas solo son para confundirlo y que aprenda a leer y entender las opciones.

La falsa es la respuesta 2, ya que en pacientes inestables se realizaría una laparatomía


exploradora y no una exploración laparoscópica.(R2)

400. La cardiomegalia acompañada de disnea, taquipnea y fiebre en un lactante de un


mes de edad, debe hacer sospechar una de las siguientes enfermedades:

1. 1. Enfisema lobar.
2. 2. Cardiopatía congénita.
3. 3. Miocarditis.
4. 4. Pericarditis.
Gráfico de respuestas
Comentario
Pregunta difícil y de poca relevancia para el MIR. La presencia de cardiomegalia, acompañada de
disnea y taquipnea en un lactante de un mes de edad puede aparecer tanto en una cardiopatía
congénita, como en una miocarditis. Sin embargo, el dato de la fiebre nos hará sospechar una
miocarditis. Recuerda que en la pericarditis no suele haber disnea y taquipnea. Además, la
miocarditis vírica es más frecuente y grave en los niños.(R3)

401. A 32-year-old female with HIV infection comes to her physician with complaints of
dysphagia and pain on swallowing for the last 4 days. She says that she occasionally
feels substernal burning. No abnormal findings are seen on examination of her oral cavity.
Lungs are clear to auscultation. She is currently taking zidovudine, lamivudine, nelfinavir
and trimethoprim-sulfamethoxazole. Her last CD4 count is 100 cells/microL. What is the
most appropriate next step in management?

1. 1. Oral fluconazole.
2. 2. Oral omeprazole.
3. 3. Oral ganciclovir.
4. 4. Oral acyclovir.
Gráfico de respuestas
Comentario
Oral fluconazole. Patients with HIV disease and a low CD4 count usually present with esophagitis.
The most common causative agent is Candida. Therefore, the most appropriate step would be to
start treatment with fluconazole. Further investigation is recommended if no good response or
improvement is seen after 4-5 days of treatment.(R1)

402. ¿Cuál es la afirmación correcta con relación al asma intrínseco?

1. 1. Asociación con historia personal o familiar de enfermedades alérgicas.


2. 2. Reacción cutánea positiva a la inyección de extractos de antígenos aéreos.
3. 3. IgE sérica aumentada.
4. 4. Las infecciones de vías aéreas inferiores pueden exacerbar una situación clínica basal.
Gráfico de respuestas
Comentario

El asma intrínseco suele ser más grave que el extrínseco, con mayor eosinofilia, sin relación con
alérgenos externos (IgE sérica normal) y, con cierta frecuencia, relacionado con intolerancia a la
aspirina y poliposis nasosinusal (tríada ASA). La respuesta correcta es la 4, pues obviamente

!
!
!
!
cualquier alteración sobreimpuesta a un aparato respiratorio con una enfermedad de base podría
descompensar ésta.(R4)

403. En el ECG de un paciente de 55 años con insuficiencia cardíaca en tratamiento


médico, encontramos los siguientes hallazgos: ondas T picudas, aumento de la amplitud
del QRS y disminución de la amplitud de la onda P. De los siguientes enunciados, señala
lo correcto:

1. 1. El paciente presenta probablemente hipopotasemia.


2. 2. La espironolactona puede ser la causa.
3. 3. Es frecuente en el tratamiento con diuréticos de asa.
4. 4. Es la anomalía característica de la digital.
Gráfico de respuestas
Comentario

Las alteraciones electrocardiográficas que nos describen, especialmente la T picuda, son


sugestivas de hiperpotasemia. Por ello, la respuesta correcta es la 2, dado que la espironolactona
es antagonista de la aldosterona, por lo que podría justificarla (igual que sucede con los
IECAs).(R2)

404. Una mujer de 43 años recibe tratamiento con litio por trastorno bipolar, a pesar de lo
cual presenta recaídas con una periodicidad tal que sugiere que se trata de una cicladora
rápida. Es FALSO que:

Debe suspenderse inmediatamente el litio, pues en estos pacientes su eficacia es nula y, de


1. 1.
hecho, puede inducir cambios de fase.
2. 2. Puede ser más eficaz la carbamacepina que el litio como profiláctico.
3. 3. Puede resultar útil añadir antipsicóticos.
Estos pacientes tienen mayor riesgo de presentar una manía farmacógena tras el
4. 4.
tratamiento con tricíclicos que los bipolares típicos.
Gráfico de respuestas
Comentario

Los cicladores rápidos son pacientes con trastorno bipolar con cuatro o más recaídas anuales.
Esta situación es poco frecuente (10% de los casos) y es más frecuente en mujeres, aunque sin
gran diferencia respecto a los varones. Característicamente, son resistentes al tratamiento con litio
en monoterapia, por lo que suelen precisar asociación con otros fármacos estabilizadores del
estado de ánimo, como la carbamacepina o el ácido valproico. Por ello, la respuesta falsa es la 1,
porque no habría que suspenderlo, sino asociarlo a otros fármacos. En bastantes casos, estos
pacientes padecen un hipotiroidismo subclínico, por lo que también se han utilizado las hormonas
tiroideas en su tratamiento.(R1)

405. Mujer de 60 años de edad, no fumadora y no bebedora. Refiere dolor epigástrico que
no cede con la ingesta y no se irradia; pesadez postprandial, anorexia y astenia. No
cuenta pérdida de peso ostensible. Ha tenido vómitos alimenticios que son cada vez más
frecuentes. Exploración física: discreta palidez mucocutánea, TA 150/85 mm Hg, FC 80
lpm, temperatura 36.8ºC, no adenopatías, resto normal. Laboratorio: Hb 10.1 gr, Hcto 32%,
leucocitos 7,000, actividad de protrombina 90%, urea 30 mgr/100 ml, AST 30 U/L, ALT 35
U/L, fosfatasa alcalina 200 U/L, amilasa 126 U/L. ¿Qué diagnóstico sospecharíamos?:

1. 1. Pancreatitis crónica.

!
!
!
!
2. 2. Colecistitis crónica.
3. 3. Neoplasia gástrica.
4. 4. Neoplasia hepática.
Gráfico de respuestas
Comentario
En este caso clínico, aunque no hay pérdida de peso evidente, sí se observa astenia y anorexia.
Por lo que inicialmente debe sospecharse origen neoplásico. Iría en contra del diagnóstico de ulcus
péptico el que la clínica no cesara con la ingesta. Parece poco probable la pancreatitis crónica, sin
alcoholismo, diarrea ni hiperamilasemia. En la colecistitis crónica, el dolor se localiza en
hipocondrio derecho, más que en epigastrio. También sería poco probable la neoplasia primaria
hepática, sin antecedentes de hepatopatía (AP 90%), y con fosfatasa alcalina normal. Por tanto, el
diagnóstico más probable (epigastralgia, vómitos persistentes con plenitus postpandrial, anemia),
sería el de neoplasia gástrica.(R3)

406. En relación a la embriología del aparato genital femenino, marque el enunciado


CORRECTO:

1. 1. Los conductos mesonéfricos (Wolf) se diferencian en trompa de Falopio y útero.


2. 2. El factor inhibidor de Muller (FIM) se produce en las células de Leydig.
3. 3. La vagina se origina de la cloaca ectodérmica.
4. 4. El himen deriva del tubérculo del seno urogenital.
Gráfico de respuestas
Comentario

Los conductos mesonéfricos o de Wolff desaparecen por falta de testosterona. Desarrollan el


epidídimo, el conducto deferente, la vesícula seminal y el canal eyaculador.

Los conductos paramesonéfricos o Müller forman la mayor parte del aparato genital femenino.
Forman las trompas tubo-ováricas, útero, cérvix y los dos tercios superiores de la vagina, y
obviamente en el hombre desaparecen.

Mientras que el himen deriva del tubérculo del seno urogenital. Respuesta 4 correcta.

407. A 47-year-old female is concerned about her risk of ovarian cancer because her aunt
died of it and her mother has just been diagnosed. She is asymptomatic and her physical
examination is completely normal. She smokes one pack of cigarettes per day and she is
diabetic. Which of the following should be consider for ovarian cancer screening in this
patient?

1. 1. Biannual pelvic examination.


2. 2. Yearly transvaginal ultrasound.
3. 3. CA125 serum test if tested positive for BRCA1.
4. 4. Yearly transabdominal ultrasound if tested positive for BRCA2.
Gráfico de respuestas
Comentario

Las prueas de detección precoz de cáncer, existen exclusivamente para determinadas patologías
como el de mama, cervico-uterino, próstata, colon y pulmón. En el caso de cáncer de ovario en

!
!
!
!
general no existe, solo existiría en el caso que se comprobara alguna mutación en BRCA1, como
explica la respuesta 3.

408. El encontrar una imagen de densidad aérea bajo ambas cúpulas diafragmáticas en la
radiografía de tórax. ¿Qué actitud terapéutica debería ser realizada?:

1. 1. Intervención quirúrgica.
2. 2. Antibióticos.
3. 3. Espasmolíticos.
4. 4. Analgésicos y evolución.
Gráfico de respuestas
Comentario

Simplemente aire libre subdiafragmático. La imagen de densidad aérea bajo el diafragma sugiere
la presencia de un neumoperitoneo que estaría traduciendo la perforación de una víscera hueca y
por tanto haría necesaría la intervención quirúrgica urgente (opción 1).(R1)

409. Con respecto a las convulsiones febriles marque lo INCORRECTO:

1. 1. El antecente de convulsiones febriles no apoya el diagnóstico.


2. 2. Es importante precisar el tipo de crisis para asegurar el diagnóstico.
3. 3. El antecedente de epilepsia en la familia no apoya el diagnóstico.
4. 4. En el post ictal lo menos importante es indicar diazepam.
Gráfico de respuestas
Comentario

Las crisis febriles son un proceso típico de la edad infantil (entre los 5 meses y los 3 años). Se
relacionan con un aumento de la temperatura, lo que da lugar a una crisis el primer dia del proceso
febril, independientemente del origen del mismo. Las crisis febriles simples son generalizadas,
duran menos de 15 minutos, presentan buena recuperación posterior y los hallazgos en el periodo
intercritico son normales o negativos. Con frecuencia existen antecedenes familiares de crsis

!
!
!
!
febriles. Hay que diferenciarlas de las crisis febriles complejas que si que tienen patologia
subyacente.(R1)

410. En el riñón con daño intersticial crónico por ácido úrico (riñón gotoso) es
característico:

1. 1. Eosinofiluria.
2. 2. HTA.
3. 3. Albuminuria.
4. 4. Cristales de oxalato en orina.
Gráfico de respuestas
Comentario

Se trata de una pregunta de escasa importancia para el ENARM pero que le puede servir para que
repase las otras manifestaciones de la gota y sobre todo su tratamiento que son los aspectos sobre
los que más les gusta preguntar en el examen.

La nefropatía gotosa es una afectación tubulointersticial crónica, que se puede dar en la


hiperuricemia duradera. Hay depósitos de cristales de ácido úrico y sales de urato en el
parénquima renal, con cristalización intraluminal en los túbulos distales y colectores, que causan
una obstrucción y una respuesta inflamatoria, con infiltración linfocitaria, reacción de células
gigantes, fibrosis de la región papilar y medular, etc. Clínicamente produce insuficiencia renal de
curso lento y es muy frecuente la HTA. Cada vez es más fuerte la sospecha de que detrás de la
insuficiencia renal asociada a hiperuricemia gotosa exista una intoxicación subrepticia por
plomo.(R2)

411. Ante una paciente perimenopáusica de 42 años, con cáncer de mama, a la que se le
ha realizado tumorectomía y linfadenectomía, con ganglios positivos para infiltración
tumoral y receptores estrogénicos positivos, ¿qué actitud debemos tomar a
continuación?

1. 1. Radioterapia sólo.
2. 2. Radioterapia + tratamiento antiestrogénico.
3. 3. Quimioterapia sólo.
4. 4. Radioterapia + quimioterapia + tratamiento antiestrógenico.
Gráfico de respuestas
Comentario

Caso clínico sencillo sobre el tratamiento del cáncer de mama, que podemos sacar si tienes claro
el esquema terapéutico. Tengamos en cuenta que a esta paciente se le ha practicado una cirugía
conservadora (tumorectomía) acompañada de linfadenectomía. Todas las cirugías conservadoras
deben acompañarse de radioterapia. Dado que los receptores son positivos, añadimos
hormonoterapia. Como la linfadenectomía nos informa de ganglios afectos, es obligado el uso de
quimioterapia adyuvante. Por tanto, la opción correcta es la 4.

412. La causa más frecuente del síndrome de dificultad respiratoria del adulto (SDRA) es:

1. 1. Inhalación de irritantes y toxinas.


2. 2. Sobredosis de narcóticos.
3. 3. Sepsis.
4. 4. TEP.

!
!
!
!
Gráfico de respuestas
Comentario

El SDRA es una forma de edema pulmonar producido por aumento de la permeabilidad de la


membrana alveolocapilar al ser lesionada, bien de modo directo o indirecto. Su causa más
frecuente es la sepsis bacteriana (opción 3 correcta). Patogénicamente, lo que ocurre es que se
estimulan los fagocitos monocíticos y los polimorfonucleares, que se adhieren al endotelio,
produciendo liberación de mediadores de la inflamación, como leucotrienos, tromboxanos y
prostaglandinas. Esto aumenta la permeabilidad vascular, por lo que el líquido intraalveolar se
hace muy rico en proteínas. Tal líquido interfiere con el surfactante, alterándose su síntesis y su
composición, por lesión de los neumocitos II, por lo que puede haber colapso alveolar. Todas estas
alteraciones predominan es las partes más declives del pulmón. Clínicamente, lo esencial es una
hipoxemia que no responde al tratamiento con altas concentraciones de oxígeno.
Radiológicamente, es característica RX tórax con extensos infiltrados alveolointersticiales
difusos.(R3)

413. El test de estadiaje más adecuado en carcinoma de páncreas es:

1. 1. CPRE.
2. 2. Utrasonido endoscópico.
3. 3. TAC de abdomen.
4. 4. Ultrasonido hepático.
Gráfico de respuestas
Comentario

La prueba más completa para realizar el estadiaje del cáncer de páncreas es el TAC abdominal tal
y como señala la opción 3, ya que permite delimitar la afección vascular, determinar la presencia
de alteraciones ganglionares, etc.(R3)

414. ¿En cuál de las siguientes localizaciones de la enfermedad tuberculosa está indicado
el tratamiento coadyuvante con glucocorticoides para mejorar la supervivencia?

1. 1. Pulmonar.
2. 2. Meníngea.
3. 3. Genitourinaria.
4. 4. Osteoarticular.
Gráfico de respuestas
Comentario

Moderadamente fácil.

En la tuberculosis que afecta a serosas, especialmente meningitis y pericarditis, se recomienda


añadir esteroides para disminuir la respuesta inflamatoria, lo que puede influir en las secuelas
neurológicas posteriores.(R2)

415. Mujer de 44 años de edad a la que se le realiza una esplenectomía por


microesferocitosis. La cirugía se realiza sin ninguna complicación. A las 12 horas de
postoperatorio presenta: disnea moderada, taquicardia y taquipnea leve. Temperatura de
38ºC. No síntomas abdominales. Rx de tórax normal. Gasometría arterial: PO2 55 mm de
Hg, PCO2 39 mm Hg. ¿Qué diagnóstico es el más probable?:

!
!
!
!
1. 1. TEP.
2. 2. Neumonía nosocomial.
3. 3. Atelectasia.
4. 4. Embolismo graso.
Gráfico de respuestas
Comentario

Nos encontramos ante un caso de fiebre en el postoperatorio inmediato (12 horas ) de una cirugía
abdominal (en el piso abdominal superior) que en principio es una cirugía limpia según el grado de
contaminación, y que cursa además de con fiebre con síntomas respiratorios (disnea y taquipnea),
que tiene una repercusión en la ventilación del paciente en el que se detecta hipoxemia y aumento
de las cifras de CO2. Por todos los datos expuestos (tiempo transcurrido, clínica respiratoria
asociada y su repercusión analítica) debemos pensar en la existencia de una atelectasia como
causa más frecuente de fiebre.(R3)

416. Lactante de 1 mes de


vida que es llevada a consulta por presentar erupción cutánea localizada en el área del
pañal (imagen nº ##). Nacida a término tras embarazo normal y parto eutócico sin otros
antecedentes de interés. Ante el cuadro que sospecha, indique la respuesta FALSA:

La lesión típica observada se presenta como un eritema papular y confluente con pápulas
1. 1.
rojas satélites en el área del pañal.
2. 2. Se observa a menudo como complicación del tratamiento antibiótico oral.
3. 3. Puede infectar de forma secundaria cualquier otra dermatitis del pañal no infecciosa.
4. 4. El tratamiento de primera elección consiste en glucocorticoides orales.
Gráfico de respuestas
Comentario

Se trata de una sobreinfección por Cándida albicans que se caracteriza clínicamente como un
eritema intenso de color rojo violáceo con formación de pápulas y pápuló-pústulas de extensión
periférica (cuadro clínico encuadrado dentro del espectro clínico de la dermatitis del panal).

En muchas ocasiones se benefician de la aplicación de un antifúngico tópico y, ocasionalmente, de


un corticoide tópico.

Los factores de riesgo generales para la aparición de candidiasis mucocutánea son:


antibioticoterapia, inmunosupresión, diabetes, corticoterapia...(R4)

417. En el paciente de la pregunta anterior, ¿cuál sería la opción terapéutica más


recomendable?:
!
!
!
!
1. 1. Nistatina tópica 10 días y cuidados básicos del área perianal.
Dado que no se observan complicaciones, esperaría la resolución espontánea de las
2. 2.
lesiones.
3. 3. Indicaría antifúngicos por vía sistémica.
4. 4. Ninguna medicación es útil en la resolución completa.
Gráfico de respuestas
Comentario

El abordaje de la dermatitis irritativa del pañal debe orientarse tanto a la prevención como al
tratamiento sintomático de las lesiones establecidas.

Se aconseja realizar una buena higiene de la zona, que debe permanecer siempre seca y limpia.
La limpieza se realizará con agua tibia o con un jabón ácido o neutro.

Asimismo se deben efectuar cambios frecuentes de los pañales, e incluso en niños con episodios
repetidos, puede ser útil mantenerlos unas horas al día sin pañales.

Parece claro que el factor crucial en la prevención de la dermatitis irritativa del pañal es el número
de cambios de pañal al día.

La maceración puede evitarse con polvos de talco u óxido de zinc, aunque estos se encuentran
desaconsejados en erosiones de la piel, ya que podrían generar granulomas por cuerpo extraño.

No se utilizarán de modo profiláctico corticoides, antifúngicos ni antibióticos tópicos, por el riesgo


de sensibilización.(R1)

418. A los diez días de iniciar por primera vez la toma de un anovulatorio de baja dosis,
una paciente de 24 años comienza con metrorragia escasa (spotting). Se produce:

1. 1. En muchos casos; no es preocupante.


2. 2. Por exceso de estrógenos.
3. 3. Por falta de progestágenos.
4. 4. Por fallo en la administración.
Gráfico de respuestas
Comentario

Los anticonceptivos orales más recientes contienen baja dosis de estrógenos para disminuir la
incidencia de efectos adversos. Aunque esta dosis sea suficiente para mantener el efecto
anovulatorio y anticonceptivo, en ocasiones no es suficiente para inducir un adecuado crecimiento
del endometrio el cual se descama parcialmente por disrupción, apareciendo sangrado
intermenstrual escaso (spotting). En otras ocasiones, la dosis de estrógenos no es capaz de
estimular el endometrio quedando éste atrófico y la paciente en amenorrea. Ambos efectos
adversos son relativamente frecuentes y no son preocupantes. Ambos se tratan aumentando la
dosis de estrógenos que contiene el preparado.(R1)

419. Ante un paciente con historia de angina y test de esfuerzo, en el que presenta angina
intensa y desnivel del segmento ST en el primer estadio de la prueba, se hará:

Detener la prueba, iniciar tratamiento médico y revisión con nueva prueba de esfuerzo al
1. 1.
mes.
2. 2. Detener la prueba y solicitar coronariografía.

!
!
!
!
3. 3. Detener la prueba y solicitar un ecocardiograma.
4. 4. Continuar con la prueba hasta su finalización.
Gráfico de respuestas
Comentario
Nos hacen dos preguntas en una. En primer lugar si debemos detener la prueba de esfuerzo y
posteriormente la actitud a tomar con el paciente. La aparición de angina de intensidad moderada o
severa durante la realización de la prueba de esfuerzo es indicación de detener la misma, con lo
cual descartamos la respuesta número 5. Por otro lado, que los síntomas y los cambios en el ECG
comiencen antes de 6 METS [primer y segundo estadío] son datos asociados a mal pronóstico y
son indicación de realizar coronariografía para valorar la posibilidad de revascularización [Resp 2].
Es indudable que este paciente debe controlar sus factores de riesgo e iniciar tratamiento médico
[resp 1 y 3], y serían opciones a valorar si la angina y el descenso del ST aparecieran a cargas
más altas. Lo más importante de esta pregunta es conocer las indicaciones de coronariografía en
la angina estable que son la mala clase funcional pese a tratamiento médico y la aparición de datos
de mal pronóstico en las pruebas diagnósticas [positividad clínica precoz; inapacidad de aumentar
la TA o descenso de la misma durante la prueba; positividad eléctrica precoz o extensa, o que no
se recupera tras tiempo de recuperación; arritmias ventriculares sostenidas, o detección de
isquemia extensa en pruebas de cardiología nuclear o en ecografías de estrés]. Bibliografía:
Manual CTO 4ª edición. Cardiología y Cirugía Vascular. Tema 10.1 Angina de pecho estable. Pag
25.(R2)

420. Ante un paciente hipertenso en tratamiento con tiacidas que presenta cifras de
potasio inferiores a 3 mEq/l en varias determinaciones, ¿cuál sería la actitud más
adecuada?

1. 1. Medir la actividad de la renina plasmática.


2. 2. Realizar sobrecarga intravenosa de sal.
3. 3. Suspender el diurético, dar suplementos de potasio y repetir la medición a las 2 semanas.
4. 4. Dar suplementos de potasio y repetir la determinación de la kalemia en 2 semanas.
Gráfico de respuestas
Comentario
Ante un paciente con HTA que asocie hipopotasemia, siempre se debe considerar la posibilidad de
que presente un hiperaldosteronismo primario, pero la causa más frecuente de hipopotasemia en
esos casos, es que sea debida al tratamiento con diuréticos. La situación presentada en el caso
clínico es frecuente. La actitud a tomar es la administración de potasio, con suspensión del
tratamiento diurético, repitiendo la analítica en las 2 semanas posteriores. En caso de que en la
nueva analítica presentara hipokaliemia, se debería realizar estudio de posible hiperaldosteronismo
primario. La suplementación exclusiva con potasio sin suprimir el diurético no nos descartará, si
persiste la hipopotasemia a las 2 semanas, que sea secundario a la administración del
diurético.(R3)

421. Hombre de 52 años que acude a urgencias por ojo rojo con dolor y visión borrosa
desde hace 2 días. Con respecto a este cuadro señale la opción FALSA:

1. 1. Si la pupila está en midriasis nos orienta hacia un glaucoma agudo de ángulo cerrado
2. 2. Si la pupila está en miosis nos orienta hacia una uveítis o patología corneal
3. 3. Si la hiperemia es ciliar nos orienta hacia un proceso de córnea o cámara anterior
4. 4. Si tiene fenómeno de Tyndall seguro será una uveítis anterior
Gráfico de respuestas
Comentario

!
!
!
!
Para responder a esta pregunta nos ayuda la tabla de diagnóstico diferencial de ojo rojo.

Todas las patologías que producen ojo rojo doloroso pueden además causar visión borrosa,
fotofobia y blefaroespasmo.

El ojo rojo con la pupila en midriasis orienta hacia una glaucoma agudo de ángulo cerrado mientras
que la miosis, que se produce por el dolor, es más inespecífica.

El Tyndall se produce siempre que el iris o el cuerpo ciliar está alterado y aparece en todas las
uveítis anteriores aunque también se puede ver en inflamaciones o infecciones corneales y en
glaucoma agudo.(R4)

422. Una paciente de 76 años consulta por malestar general, dolor abdominal, astenia y
anorexia, que han progresado desde hace 2 meses. La presión arterial es de 171/110
mmHg. Presenta púrpura palpable en miembros inferiores, desde los pies hasta las
rodillas. La auscultación cardiopulmonar revela estertores húmedos en bases
pulmonares, tercer tono y edemas moderados. En la analítica, destaca una creatinina de
4 mg/dl, complemento normal, microhematuria y proteinuria leve en el sedimento, así
como anticuerpos antimembrana basal glomerular negativos, anticuerpos
antimieloperoxidasa positivos. La Rx de tórax muestra un infiltrado bilateral, confluente
en algunas zonas, sin cardiomegalia. En las 24 horas siguientes, el paciente comienza
con hemoptisis, oliguria y elevación de la creatinina a 7.0 mg/dl. El diagnóstico más
probable es:

1. 1. Enfermedad de Goodpasture.
2. 2. Crioglobulinemia mixta esencial.
3. 3. Enfermedad de Wegener.
4. 4. PAN microscópica.
Gráfico de respuestas
Comentario

Nos encontramos ante una paciente que presenta un síndrome renopulmonar, asociado a una
púrpura palpable y a una sintomatología sistémica inespecífica. Con estos datos podemos
descartar el Wegener (asocia alteraciones en las vías aéreas superiores), la PAN clásica (no tiene
afectación pulmonar), y la crioglobulinemia mixta esencial (no dan datos acerca de exposición al
frío).

Para diferenciar entre la PAN microscópica y el Goodpasture hemos de fijarnos en los anticuerpos
antimembrana basal (en este caso, negativos), por lo que la opción 2 es falsa. Por otra parte, la
combinación de HTA y dolor abdominal, aparte de los daños pulmonares, sugieren también una
PAN microscópica.(R4)

423. Varón de 87 años que presenta una rodilla inflamada, caliente y dolorosa. En el
líquido sinovial se identifican cristales alargados con birrefringencia débilmente +, que
se tiñen con alizarina roja S. Nos encontramos ante una artropatía por:

1. 1. Cristales de pirofosfato cálcico deshidratado.


2. 2. Cristales de oxalato cálcico.
3. 3. Cristales de hidroxiapatita cálcica.
4. 4. Cristales de carbonato cálcico.
Gráfico de respuestas

!
!
!
!
Comentario
Se trata de un paciente anciano con monoartritis aguda que afecta a la articulación de la rodilla.
Ante una presentación clínica como esta debes recordar que la monoartritis aguda sugiere como
primera posibilidad la presencia de una artritis por microcristales o de una artritis séptica, siendo
esta segunda posibilidad menos probable en un anciano sin factores de riesgo para una
bacteriemia. El mejor método para poder establecer el diagnóstico de forma definitiva es el estudio
del líquido sinovial utilizando el microscopio de luz polarizada para identificar la presencia de
microcristales. Al realizar el estudio se encuentran cristales de forma rectangular con
birrefringencia débilmente positiva, aspecto característico de los cristales de pirofosfato cálcico. La
tinción con alizarina roja, tiñe tanto los cristales de hidroxiapatita como los cristales de
pirofosfato.(R1)

424. Es FALSO del pseudoquiste pancreático que:

1. 1. Tiene pared propia.


2. 2. Normalmente se localiza en el cuerpo o la cola del páncreas.
3. 3. Se suelen formar en el plazo de 1 a 4 semanas después del inicio de la pancreatitis aguda.
La rotura y la hemorragia son las causas principales de mortalidad en el seudoquiste
4. 4.
pancreático.
Gráfico de respuestas
Comentario
El pseudoquiste pancreático es la complicación más frecuente después de una pancreatitis aguda.
Se trata de una colección líquida que no tiene cápsula y que se localiza con mayor frecuencia en el
cuerpo y cola de páncreas (hasta el 85 % de las veces) apareciendo hasta en el 15 % de los
pacientes con pancreatitis agudas entre la 1 y 4 semana después del inicio de la enfermedad.
Hasta el 25 - 40 % de los pseudoquistes se resuelven espontáneamente. Las principales causas
de mortalidad se deben a rotura del mismo y a hemorragia, siendo mayor cuando ambas
situaciones coinciden.(R1)

!
!
!
!

425. Recién nacido a término CIR tipo


I, presenta microcefalia llamativa, por lo que se realiza ecografía transfontanelar (imagen).
Antecedentes maternos sin interés, la madre tiene otro hijo de 7 años. Periodo neonatal
inmediato sin incidencias salvo que no pasa los potenciales evocados auditivos del oído
derecho. ¿Cúal es su primera impresión impresión?

1. 1. Probablemente sea una cromosomopatía.


2. 2. Enfermedad metabólica.
3. 3. Infección congénita por CMV.
4. 4. Infección congénita por rubeola.
Gráfico de respuestas
Comentario

Una pregunta de dificultad media-alta, porque dependemos bastante de la imagen que nos
aportan. Observe que el niño tiene microcefalia, lo que podría ser compatible con varias
infecciones connatales (salvo con la toxoplasmosis, claro está). Sin embargo, alrededor de los
ventrículos cerebrales vemos áreas hiperdensas, que corresponen a calcificaciones. Y, como ya
sabes, las calficicaciones periventriculares son típicas de la infección congénita por CMV.(R3)

426. ¿Qué otras alteraciones esperaría encontrar?

1. 1. Petequias, ictericia, hepatoesplenomegalia, purpura, convulsiones.


2. 2. Cataratas congénitas.
3. 3. Malformaciones en extremidades.
4. 4. Deterioro progresivo y exitus.
Gráfico de respuestas
Comentario

!
!
!
!
Para completar el cuadro “microcefalia + calcificaciones periventriculares”, tendríamos que confiar
en la opción 1. Recuerde que, en cierto modo, la toxoplasmosis es opuesta al CMV:

- CMV: MICROcefalia y calcificaciones PERIVENTRICULARES.

- Toxoplasma: HIDROcefalia y calcificaciones DIFUSAS.


(R1)

427. Señale la opción FALSA respecto a la patología refractaria del ojo:

1. 1. La miopía dificulta la visión lejana.


2. 2. El astigmatismo puede coexistir bien una hipermetropía, bien con una miopía.
3. 3. La presbicia es una miopía adquirida con la edad.
4. 4. En el seno de una hipermetropía puede aparecer un estrabismo convergente.
Gráfico de respuestas
Comentario

Esta pregunta pertenece al tema de refracción y hace referencia a las cuatro ideas claras que
debes tener. La presbicia es un fenómeno fisiológico asociado al envejecimiento que se caracteriza
por una pérdida de la acomodación del cristalino, que dificulta la capacidad para enfocar objetos
cercanos y no para ver de lejos como en la miopía, por lo que la opción 3 es falsa. En la miopía por
un exceso de convergencia la imagen se forma por delante de la retina causando dificultad en la
visión lejana, mientras que en la hipermetropía ocurre lo contrario, el ojo es poco convergente y la
dificultad está en la visión cercana. Presta atención a la opción 4: en la hipermetropía se produce
un esfuerzo constante de acomodación en un intento de focalizar correctamente los objetos, lo cual
puede producir un estrabismo por el reflejo de sincinesia- acomodación.(R3)

428. Paciente de 36 años,


nulípara, asintomática, que acude al ginecólogo para revisión. Al tacto bimanual se
encuentra un útero ligeramente aumentado de tamaño, no doloroso, móvil, motivo por el
que se le solicita una ecografía transvaginal en la que se observan los hallazgos que se
muestran en la imagen. ¿Cuál será el diagnóstico más probable?:

!
!
!
!
1. 1. Pólipo endometrial.
2. 2. Mioma submucoso.
3. 3. Sarcoma uterino.
4. 4. Mioma intramural.
Gráfico de respuestas
Comentario

Una imagen redondeada en el espesor del miometrio, bien delimitada, sin imágenes quísticas en
su interior, más o menos homogénea, corresponde a la imagen típica del mioma uterino. Los
miomas pueden ser: submucosos si protuyen en la cavidad endometrial, intramurales si se sitúan
en la porción central del miometrio o subserosos si se sitúan bajo el peritoneo visceral. Además
recuerde que la mayoría de los miomas son asintomáticos, sobre todo si son de pequeño tamaño.
Por el contrario, el pólipo endometrial se encuentra en el interior de la cavidad endometrial. El
sarcoma uterino es muy infrecuente y generalmente aparece en la post-menopausia, presentado
un crecimiento muy importante.(R4)

429. ¿Cuál será la actitud más adecuada en esta paciente?:

Miomectomía por laparoscopia puesto que la paciente es nulípara y probablemente querrá


1. 1.
tener hijos.
2. 2. Tratamiento con análogos de la GnRH para evitar que crezca el mioma.
3. 3. Tranquilizar a la paciente y programarle un nuevo control en 6 meses.
4. 4. Miomectomía para evitar la degeneración quística del mioma.
Gráfico de respuestas
Comentario

En los miomas pequeños y asintomáticos la conducta a seguir debe ser el control periódico de los
mismos. En miomas grandes y sintomáticos el tratamiento dependerá de los deseos genésicos y
del tamaño y localización de los miomas: tratamiento quirúrgico conservador o no. Otra alternativa
es la embolización selectiva del mioma. El tratamiento medico con análogos GnRH es un
tratamiento temporal ya que no provoca muerte celular y está indicado antes de la cirugía para
disminuir tanto el tamaño como la vascularización del mismo.(R3)

430. El soplo continuo, en maquinaria o de Gibson, a nivel del 2º espacio intercostal


izquierdo es característico de:

1. 1. Persistencia del conducto arterioso.


2. 2. Estenosis pulmonar congénita.
3. 3. Coartación aórtica.
4. 4. CIV.
Gráfico de respuestas
Comentario

Pregunta básica. Puesto que el tema de las cardiopatías congénitas es muy amplio, debe quedarse
con los datos más característicos de cada una de ellas que permitan reconocerlas en los casos
clínicos. Del conducto arterioso persistente debe recordar que lo típico es la presencia de un soplo
continuo en maquinaria en foco pulmonar que irradia a región infraclavicular izquierda. Recuerde
otros datos clave como que esta cardiopatía es más frecuente en niñas y que se asocia a infección
materna por rubéola en el primer trimestre. Recuerde de la indometacina como tratamiento de
aquéllos prematuros sintomáticos con PCA.(R1)

!
!
!
!
431. Gestante de 37 semanas a la que se le practica monitorización fetal no estresante
con el siguiente resultado: frecuencia cardíaca fetal 135 lpm, ondulatoria normal,
movimientos fetales escasos, sin ascensos en la frecuencia cardíaca y sin
deceleraciones. ¿Qué actitud adoptaría?:

1. 1. Cesárea urgente.
2. 2. Inducción del parto.
3. 3. Amnioscopía.
4. 4. Prueba de Pose.
Gráfico de respuestas
Comentario

Tenemos un caso clínico en el que se nos presenta un caso clínico de una gestante a término que
mientras se le realiza una monitorización fetal no estresante observamos una frecuencia cardiaca
normal (120-160 lat/min), con buena variabilidad (10-25 lat/min), ausencia de desaceleraciones,
pero con movimientos fetales escasos y ausencia de ascensos. En resumen, es un feto con un
RAF negativo, por lo que debemos indicar una prueba de Pose (opción correcta la 4)
omonitorización feta estresante, y en función de su resultado se valoraría terminar la gestación
(opciones 1 y 2) o no.(R4)

432. En relación al examen del cuerpo uterino normal, señale lo incorrecto:

1. 1. La retroversoflexión no es patológico necesariamente


2. 2. El peso oscila entre 70 - 90 gr.
3. 3. El ligamento redondo no es importante para su sostén
4. 4. El sostén más importante es el ligamento ancho
Gráfico de respuestas
Comentario

El útero se sostiene en su posición por varios ligamentos peritoneanos, son varios pero los más
importantes son dos, uno para cada lado del útero:

Nombre Desde Hasta

Ligamento utero-sacro Cervix posterior Hueso sacro

Ligamentos cardinales Cervix lateral Espinas isquiáticas

Ligamento pubocervical

Otros ligamentos cercanos al útero, como el ligamento ancho, el ligamento redondo, el ligamento
suspensorio del ovario y el ligamento infundibulopélvico tienen poca participación en el soporte del
útero.(R4)

433. Un paciente de 57 años con antecedentes de HTA, diabetes y cardiopatía isquémica,


presenta de forma brusca visión borrosa y disminución de la agudeza visual en ojo
izquierdo. No tuvo dolor ocular y tras permanecer 3 días sin mejoría acudió al servicio de
urgencias, donde se realizó un fondo de ojo mostrando un papiledema sectorial no
congestivo y hemorragias en astillas peripapilares. En la exploración neurológica
destacaba un defecto pupilar aferente (pupila de Marcus Gunn). El estudio de laboratorio

!
!
!
!
fue normal incluyendo la velocidad de sedimentación globular. Sobre la enfermedad de
nuestro paciente NO es cierto:

La neuropatía óptica isquémica es la causa más frecuente de pérdida aguda de visión en


1. 1.
individuos de edad media-avanzada.
El tratamiento de elección son dosis altas de esteroides intravenosos, con una respuesta al
2. 2.
tratamiento del 80% si se aplican en las primeras 48 horas.
La sospecha de neuritis óptica isquémica anterior arterítica (arteritis de células gigantes) es
3. 3.
indicación de biopsia de arteria temporal.
4. 4. La embolia es extremadamente rara.
Gráfico de respuestas
Comentario

No se preocupen si ha fallado esta pregunta pues a pesar de que para el ENARM es importante
que conozcan la neuropatía óptica isquémica, no tienen porque saber detalles tales como el que se
refleja en la opción correcta. Para el examen es suficiente que recuerde que la neuropatía óptica
isquémica (NOI) representa un grupo de desórdenes isquémicos del nervio óptico debidos a una
perfusión inadecuada de las arterias ciliares posteriores. Se caracteriza por una pérdida brusca,
indolora y monocular de la visión. Se clasifica en: anterior (NOIA), caracterizada por la tríada de un
defecto pupilar aferente, un defecto del campo visual y un edema del disco óptico. En la neuropatía
óptica isquémica posterior (NOIP), no existe edema del disco óptico. La NOIA a su vez se
subclasifica en dos grandes grupos: arterítica (NOIA- A), asociada a arteritis de la temporal, y no
arterítica (NOIA- NA), siendo esta última la más frecuente (la diabetes y la hipertensión son
factores de riesgo, y todavía no se dispone de tratamiento). Cuando se diagnostica a un paciente
de NOIA se debe instaurar de inmediato el tratamiento con glucocorticoides a dosis altas para
prevenir la ceguera del segundo ojo, pero el pronóstico de recuperación de la visión en el ojo
afecto es infausto. Ante un enfermo con ceguera por posible NOIA arterítica debe hacerse biopsia
de la arteria temporal para confirmar el diagnóstico, sin demorar el inicio del tratamiento
corticoideo.(R2)

434. A 67-year-old man has a 10-year-history of shortness of breath with exertion and dry
cough. He was diagnosed with chronic bronchitis. He presents today with hemoptisis and
malaise. On further questioning, it is known that he used to work in a quarry. Chest X-ray
reveals multiple and small pulmonary nodules in upper lobes with a larger cavitating
nodule. Which of the following is the most likely diagnosis?

1. 1. Tuberculosis in a patient with silicosis.


2. 2. Epidermopid lung carcinoma.
3. 3. Sarcoidosis.
4. 4. Amyloidosis.
Gráfico de respuestas
Comentario

La historia de exposición a la sílice (trabajador de cantera) debe hacernos pensar en el diagnóstico


de silicosis. Una de las imágenes radiológicas característica de la silicosis son los grandes nódulos
de predominio en lóbulos superiores. Estos pacientes tienen mayor riesgo de sufrir tuberculosis,
por lo que el nódulo cavitado debe considerarse en principio de una caverna tuberculosa, porque
además la clínica es bastante sugestiva. No es imposible que se trate, por ejemplo, de un cáncer
de pulmón… Pero nos preguntan la más probable (respuesta 1 correcta).

435. Acude a urgencias una paciente de 31 años con amenorrea de 7 semanas que ha
empezado con dolor y metrorragia. La paciente tiene constantes normales. El test de

!
!
!
!
embarazo en orina es positivo. Le realiza un USG transvaginal, en la cual el útero está
vacío, endometrio decidualizado y en anejo derecho se ve una imagen paraovárica de 28
mm, ecogénica, y líquido libre en fondo de saco de Douglas en cantidad escasa. Ante la
sospecha de que se trate de una gestación ectópica, le solicita beta-HCG en sangre con
un resultado de 3498 mUI/ml. Usted le explica a la paciente las diversas opciones
terapéuticas, decantándose por el metotrexate, para lo cual hay que tener en cuenta todo
EXCEPTO:

1. 1. No debe haber signos de rotura tubárica.


2. 2. Si hubiera latido cardiaco en la gestación ectópica, estaría contraindicado el metotrexate.
3. 3. Evitar nueva gestación en los próximos 3-4 meses, por posibles efectos teratogénicos.
4. 4. Es frecuente la elevación de la beta-HCG hasta el 4º día tras la inyección de metotrexate.
Gráfico de respuestas
Comentario

Nos encontramos ante un caso clínico tipo de gestación ectópica tubárica. Para la administración
de metotrexate debemos recordar que debe ser una gestación menor de 4 cm con beta- HCG
<5000 mUI/ml, que esté hemodinámicamente estable, sin signos de rotura o sangrado activo. La
presencia de líquido libre en Douglas o latido cardiaco en la gestación ectópica no contraindican
por sí solos el tratamiento con metotrexate, pero lo hacen menos aconsejable. Se debe evitar
gestación en los 3- 4 meses posteriores al tratamiento por los efectos teratogénicos.Tras la
inyección se produce un cuadro de dolor por resolución de la gestación ectópica en gran parte de
las pacientes, el cual suele ir acompañado de una elevación de la beta- HCG en gran parte de los
casos.(R2)

436. Paciente femenino de 27 años que consulta por amenorrea de 3 meses. Como
antecedentes obstétricos G1P1 normales hace 5 años. Ex usuaria de drogas por vía
inhalatoria, negando el uso de la vía parenteral. Su pareja sexual es ADVP activo y VIH
positivo en estadio C3. En la serología del primer trimestre se diagnostican anticuerpos
anti-VIH positivos en la gestante. Debemos informarle de todo lo siguiente, EXCEPTO:

La transmisión vertical o materno-fetal de la infección por VIH-1 constituye el principal


1. 1.
modo de adquisición de la infección en los niños.
La introducción de la zidovudina en monoterapia para disminuir la transmisión vertical del
2. 2.
VIH supuso una disminución del 20-30% al 8%.
Deberá renunciar a la lactancia materna, puesto que esta duplica el riesgo de transmisión
3. 3.
madre-hijo.
4. 4. Está aceptada la cesárea electiva de forma sistemática como única vía del parto.
Gráfico de respuestas
Comentario

La tasa de transmisión vertical previa al uso del AZT se situaba en torno al 20%. La mayor parte de
los casos ocurren durante las últimas semanas de gestación y parto. La gestante debe recibir una
pauta de AZT o zidovudina endovenosa y 3TC o lamivudina intraparto, y también el neonato, a
menos que esté contraindicado por toxicidad previa demostrada. Se ha demostrado que la cesárea
electiva (1.8%) disminuye de forma significativa el riesgo de transmisión vertical respecto al parto
vaginal (10.5%). La cesárea debe aconsejarse sin discusión cuando exista deterioro inmunológico
(CD4<200), carga viral detectable o si no ha recibido tratamiento médico durante la gestación. Pero
ten en cuenta que no se realiza en todos los casos, ya que los antirretrovirales disminuyen mucho
la transmisión.

!
!
!
!
Durante el parto, se debe retardar en lo posible la rotura de membranas. Si éstas se rompen y es
previsible un parto prolongado, sería aconsejable realizar una cesárea. Ante una madre VIH, debe
evitarse en lo posible la realización de microtomas, el uso de electrodos y los partos
instrumentados.

En cuanto a la lactancia materna, es cierto que aumenta el riesgo de transmisión, y por ello se
contraindica como recomendación general. No obstante, en los países subdesarrollados sí se
acepta dar lactancia, porque la alternativa sería la malnutrición, con lo que el riesgo-beneficio
aconseja darla.(R4)

437. Mujer de 78 años que acude a urgencias por ictericia cutáneo-mucosa e intenso
prurito que nota desde hace 2 semanas, acompañada de coluria y acolia. Refiere también
pérdida de peso de 15 kilos en los últimos 8 meses, y su hija, que la acompaña, refiere
un importante deterioro físico. Se decide la realización de una TC abdominal donde se
observa una tumoración sólida que ocupa la cabeza del páncreas, y una importante
dilatación de la vía biliar. No se observan metástasis hepáticas. ¿Qué actitud terapéutica
plantearía a la paciente?

1. 1. Derivación biliodigestiva.
2. 2. Colocación de prótesis biliar metálica.
3. 3. Colocación de prótesis biliar plástica.
4. 4. Quimioterapia.
Gráfico de respuestas
Comentario

La paciente presenta un cuadro típico de ictericia secundaria a un tumor de la cabeza de páncreas.


El pronóstico de esta neoplasia es muy pobre: la supervivencia actual a tres años es menor del
5%. A pesar de los avances en técnicas diagnósticas, tan sólo entre el 7% y el 20% de los
enfermos en el momento del diagnóstico tienen un tumor que sea resecable; y aún en este caso
(tumor resecable, limitado a la glándula pancreática y menor de 2 cm de tamaño), la supervivencia
a los cinco años es entre el 20% y el 40%.

No cabe duda que esta paciente no es candidata a cirugía, ni por la edad, ni por el estado de la
misma (pérdida de peso, deterioro…) que sugieren el estado avanzado de la enfermedad y el mal
pronóstico a corto plazo de la misma. El tratamiento por tanto, es paliativo tratando de ayudarla a
mejorar el síntoma por el que consulta: la ictericia. Las opciones de tratamiento estándar para la
misma son:
- Anastomosis o derivación quirúrgica biliodigestiva (que generalmente realizaremos si se
descubre la irresecabilidad en el acto quirúrgico, que no es el caso que nos ocupa) o
- Colocación percutánea radiológica o endoscópica (preferiblemente, por sus menores
complicaciones) de prótesis biliares, que es la opción que elegiríamos para esta paciente.
Las endoprótesis utilizadas para el drenaje endoscópico de la vía biliar son de dos tipos: las
plásticas, y las metálicas.
- Las prótesis plásticas son baratas, pero se ocluyen con material bacteriano y barro biliar
haciendo necesario su recambio. La duración de una prótesis sin obstruirse está relacionada con
su diámetro. La duración media hasta la obstrucción de las prótesis plásticas es de tres a cuatro
meses. En esta paciente, en la que esperamos un desenlace fatal a corto plazo, sería la opción
más correcta. La obstrucción de una endoprótesis se manifiesta por la reaparición de la ictericia o
colangitis, lo que requiere un recambio en forma urgente aunque la administración temprana de
antibióticos permite, en general, hacer el procedimiento en forma electiva.
- Las prótesis metálicas son más caras pero tienen una mayor duración que las plásticas, con una
duración media hasta la obstrucción de ocho meses, la obstrucción por barro biliar es mucho

!
!
!
!
menos frecuente, pero el tumor tiende a crecer a través de la malla metálica y las ocluye por lo que
se están desarrollando nuevas prótesis metálicas recubiertas para evitar el crecimiento tumoral a
través de la malla metálica. Las prótesis metálicas son ventajosas en aquellos pacientes con
supervivencia mayor de seis meses, los cuales necesitarían uno o varios recambios de sus prótesis
plásticas.
Por ello recuerde que debe realizar manejo paliativo con prótesis plásticas, en pacientes con
obstrucción distal por neoplasia, en especial en aquellos con supervivencia esperada menor de
seis meses (diámetro tumoral mayor de 3 cm. Las prótesis metálicas autoexpandibles tienen su
mayor utilidad en pacientes con supervivencia esperada mayor de seis meses (diámetro tumoral
menor de 3 cm). En este grupo también se debe considerar la cirugía paliativa del tipo de doble
derivación.(R3)

438. La insuficiencia suprarrenal por atrofia idiopática:

En nuestro país representa la segunda causa más frecuente de enfermedad de Addison tras
1. 1.
la tuberculosis.
2. 2. Aparece cuando existe una destrucción mayor del 50% de la glándula.
3. 3. Pueden existir anticuerpos contra la enzima 21-hidroxilasa.
4. 4. En el 40% de los casos se asocia a hepatitis crónica autoimnune tipo IIa.
Gráfico de respuestas
Comentario

Una pregunta aparentemente difícil, pero que se podría acertar por sentido común. Aunque se trata
de una pregunta sobre la atrofia idiopática de las glándulas suprarrenales, resulta muy difícil negar
la veracidad de la respuesta 3.

Si esta opción nos dijera que la atrofia es debida a autoanticuerpos, evidentemente sería falsa,
puesto que en ese caso no se podría decir que es idiopática.

Sin embargo, lo que dice no es exactamente esto, sino algo completamente distinto: que pueden
existir anticuerpos contra la enzima 21-hidroxilasa.

Recuerda que existen personas con ANA positivos, sin que padezcan lupus, lo mismo que otros
tienen factor reumatoide en sangre, sin que necesariamente se trate de una artritis reumatoide.

En otras palabras, la presencia de autoanticuerpos no implica necesariamente el padecimiento de


una enfermedad autoinmune. Ante una respuesta tan abierta como la 3, si existen dudas,
deberíamos considerarla correcta.(R3)

439. Pepe es un paciente de 70 años que acude a urgencias por dolor en la rodilla desde
hace 2 meses aproximadamente, por lo que le siguen en reumatología. En una analítica
rutinaria se observa anemia normocítica, creatinina 1.1 mg/dL, urea 50 mg/dL, sodio 140
mEq/L, K 4.2 mEq/L, bicarbonato 16 mEq/L, VSG elevada. En la orina la tira reactiva
muestra glucosuria de 2 cruces, Na 100, K 60, Cl 100. ¿Qué afectación renal sospecha?

1. 1. NTA
2. 2. Prerrenal
3. 3. Riñón de mieloma
4. 4. Síndrome de Fanconi
Gráfico de respuestas
Comentario

!
!
!
!
El paciente impresiona de tener un mieloma pero aun no sabiendo el diagnóstico presenta
glucosuria con bicarbonaturia (la de esta pregunta estriba en saber que un disbalance entre
aniones y cationes en la orina refleja aumento de un anión, normalmente bicarbonato), con
bicarbonato bajo en sangre, todo ello compatible con un Síndrome de Fanconi. Nos faltaría la
aminoaciduria y la fosfaturia para completar el cuadro.(R4)

440. Se consideran manifestaciones de la persistencia del conducto arterioso, EXCEPTO:

Apnea persistente sin causa explicable en un paciente con enfermedad de membrana


1. 1.
hialina.
2. 2. Pulsos periféricos saltones.
3. 3. Soplo continuo.
4. 4. Hipocapnia.
Gráfico de respuestas
Comentario

La presencia de un conducto arterioso abierto implica desde el punto de vista clínico la aparición
de una circulación hiperdinámica, con pulsos saltones, soplo continuo (en maquinaria o de Gibson)
y tendencia a la retención de CO2. Su radiología: habrá dilatación de la silueta cardíaca y,
secundariamente al shunt izquierda-derecha, y congestión pulmonar.(R4)

441. Varón de 32 años que acude a su consulta de especialista remitido desde atención
primaria por leve ictericia desde hace 2 días, fiebre, artromialgias y tos productiva con
expectoración ligeramente verdosa. En la exploración física destacaban únicamente
sibilancias en ambos campos pulmonares. La Rx tórax era normal. La analítica demostró
los siguientes parámetros: Hb 15.2, VCM 86.5 fl, HCM 30 pg, ferritina 94, leucocitos 11500
con 72% neutrófilos, plaquetas 195.000, GOT 32, GPT 23, GGT 32, bilirrubina total 7.8
(directa 0.8), LDH 168, sodio 142, potasio 3.8. ¿Cuál, de entre los siguientes, es el
diagnóstico más probable?:

1. 1. Coledocolitiasis.
2. 2. Fiebre Q.
3. 3. Síndrome de Gilbert.
4. 4. Síndrome de Crigler-Najjar.
Gráfico de respuestas
Comentario
El paciente, en relación con un cuadro gripal que presenta, tiene elevación de bilirrubina con
predominio de indirecta con transaminasas normales. Ello nos orienta hacia un trastorno de la
conjugación de bilirrubina, que, al tener valores de bilirrubina mayores de 5 mg/dl corresponderían
más concretamente a un Síndrome de Crigler- Najjar tipo 2.(R4)

442. El gold standard para el diagnóstico de Enfermedad Inflamatoria Pélvica es:

1. 1. Evaluación clínica.
2. 2. Respuesta terapéutica.
3. 3. Laparoscopía.
4. 4. Resonancia Magnética Nuclear.
Gráfico de respuestas
Comentario

!
!
!
!
El estándar de oro para el diagnóstico de EPI es la laparoscopía, ya que además de visualizar
directamente los órganos pélvicos, permite la toma de muestras para estudios bacteriológicos.(R3)

443. A 7-year-old boy is brought to her pediatrician by her parents, complaining of a 3-


week history of intermittent pruritic rash on his arms. His past medical history includes a
bronchiolitis a year ago treated with albuterol. The rash is itchy, located on his ante-
cubital fossae and erythematous. Which of the following is the most adequate treatment?

1. 1. Biospy first and then medical therapy accordingly.


2. 2. Topical corticosteroid therapy.
3. 3. Oral corticosteroid therapy.
4. 4. Topical antibiotic therapy.
Gráfico de respuestas
Comentario
Topical corticosteroid therapy. This question describes a case of atopic dermatitis. The most
appropriate option is the topical corticosteroid therapy. Remember that topical calcineurin inhibitors
(tacrolimus, pimecrolimus) are effective alternatives to the chronic use of topical corticosteroids,
especially for more delicate areas (face, genitals...) because they do not carry the risks of atrophy,
telangectasias, and striae associated with the chronic use of steroids.(R2)

444. Con respecto a la isoinmunización ABO, señale verdadero ( V ) o falso ( F ) y marque.


La respuesta correcta: ( ) Isoinmunización ABO puede aparecer en el primer hijo, ( ) La
prueba de Coombs directa puede ser negativa. ( ) El antígeno B tiene mayor capacidad
antigénica. ( ) La incompatibilidad ABO protege de la incompatibilidad Rh.

1. 1. FVFV.
2. 2. VVFF.
3. 3. VFFV.
4. 4. VFFF.
Gráfico de respuestas
Comentario

Isoinmunización ABO: es más frecuente en comparación Rh, pero menos severa, puede aparecer
en el primer hijo, las pruebas de Coombs directa pueden ser negativas, es falso que el antígeno B
tiene mayor capacidad antigénica (es el anti-D) y la incompatibilidad ABO NO protege contra la
incompatibilidad Rh. Respuesta 2 correcta.

445. Después de un expulsivo normal y tras 60 minutos de período de alumbramiento, no


se aprecian signos de desprendimiento placentario, a pesar de haberse aplicado masaje
uterino y de haber incrementado moderadamente la dosis de oxitocina. Se indica una
extracción manual de placenta, que resulta imposible por no existir plano de separación
entre la placenta y la pared uterina. ¿Cuál es el diagnóstico más probable?

1. 1. Engatillamiento placentario.
2. 2. Placenta succenturiata con cotiledón aberrante.
3. 3. Placenta adherente por acretismo placentario.
4. 4. Placenta circunvalata.
Gráfico de respuestas
Comentario

!
!
!
!
El caso corresponde a una placenta accreta. En esta situación, la placenta no se inserta sobre el
endometrio, sino sobre el miometrio, por insuficiente decidualización. La incidencia es muy baja:
uno de cada 1000 partos. Predisponen para esta alteración: multiparidad, placenta previa,
legrados, cirugía, miomas, etc. Clínicamente, hay una falta de desprendimiento placentario y
hemorragia durante el alumbramiento. El tratamiento consiste en realizar masaje uterino, intentar
extracción manual, practicar legrado y, si no se consigue el desprendimiento, histerectomía.(R3)

446. La deficiencia congénita de los últimos componentes de la cadena del complemento


(C9- C11) facilita la aparición de meningitis recurrente por:

1. 1. M. tuberculosis.
2. 2. Enterovirus.
3. 3. N. meningitidis.
4. 4. H. influenzae.
Gráfico de respuestas
Comentario
Pregunta fácil que mezcla conocimientos sobre inmunodeficiencias. La deficiencia congénita de los
últimos componentes de la cadena del complemento (C9- C11) facilita la aparición de meningitis
recurrente por Neisserias (meningococo y gonococo). Por otro lado, recuerda que en las
inmunodeficiencias humorales por alteración de las inmunoglobulinas o en pacientes
esplenectomizados predisponen también a las infecciones por gérmenes encapsulados
(neumococo, meningococo y Haemophilus). Si la has fallado repasa la tabla de enfermedades
asociadas a trastornos inmunitarios que aparece en el Manual.(R3)

447. Respecto a la endometriosis, ¿cuál de las siguientes afirmaciones es CIERTA?

1. 1. Todas las lesiones endometriósicas tienen el mismo origen histogénico.


Las recidivas de la endometriosis tras el tratamiento quirúrgico conservador de los ovarios
2. 2.
son raras.
La hormonoterapia posterior a la cirugía aumenta mucho las posibilidades de conseguir
3. 3.
después un embarazo.
4. 4. El embarazo influye favorablemente sobre la endometriosis.
Gráfico de respuestas
Comentario

La endometriosis consiste en la presencia y proliferación de tejido endometrial ectópico, es decir,


fuera de la cavidad uterina. Puede afectar a cualquier órgano. Afecta a un 10% de las mujeres y se
piensa que los ciclos cortos con sangrado abundante podrían ser un factor de riesgo, puesto que
favorecerían el reflujo de sangre a cavidad peritoneal, facilitando los implantes extrauterinos. La
localización más frecuente de la endometriosis es el ovario, donde es habitual la formación de
quistes que se rellenan de sangre, que con el tiempo adquiere un color marrón oscuro (“quistes de
chocolate”). Sus manifestaciones clínicas más frecuentes son:

•! Dolor: es el síntoma más frecuente. Suele ser pélvico y se manifiesta como una
dismenorrea progresiva, que no cede con la toma de anticonceptivos orales. También
puede consistir en dispareunia.
•! Alteraciones menstruales: ciclos cortos y con sangrado abundante.
•! Infertilidad: sin embargo, aunque resulta más difícil conseguir el embarazo, éste tiene una
influencia positiva sobre la evolución de la endometriosis.

!
!
!
!
El diagnóstico de certeza se alcanza por laparoscopia, ya que permite visualizar directamente las
lesiones, que típicamente se describen como “quemaduras de pólvora”.(R4)

448. Mujer de 55 años de edad que consulta por pesadez, distensión abdominal y tenesmo
rectal. La exploración y el ultrasonido muestran la presencia de una tumoración de
densidad irregular en el ovario derecho. Los exámenes de laboratorio muestran un
aumento del marcador CA 125. Se realiza una laparotomía amplia en la que se observa
que el tumor ha roto la cápsula. Existe además una importante cantidad de líquido
ascítico con células malignas. No hay afectación de otros órganos pélvicos. ¿En qué
estadio se encuentra el tumor?:

1. 1. Ia.
2. 2. Ic.
3. 3. II.
4. 4. IV.
Gráfico de respuestas
Comentario

El estadiaje de tumores de ovario no es muy rentable de estudiar. Simplemente recuerde que el


estadio I es limitado al ovario, II extensión a pelvis III extensión a peritoneo IV metástasis y que la
letra C significa a grandes rasgos Cápsula rota Citología liquido peritoneal positiva. En este caso
tenemos un tumor localizado en el ovario derecho con cápsula rota, ascitis tumoral sin afectación
de órganos pélvicos por lo que el estadio es Ic. Recuerde que en este estadio ya es necesario dar
quimioterapia adyuvante.(R2)

!
!
!
!

!
!
!
!

!
!
!
!

!
!
!
!

!
!
!
!

!
!
!
!

!
!
!
!

!
!
!
!

!
!
!
!

!
!
!
!

!
!
!
!

!
!
!
!

!
!

También podría gustarte